You are on page 1of 401

Matematika a mszaki fiskolk szmra

Matematikai feladatok
Matematika a mszaki fiskolk szmra
Matematikai feladatok
Szerkesztette
Scharnitzky Viktor
Nemzeti Tknknyvkiad, Budapest
FISKOLAI SEGDKNYV
A ktet
Gy. Bartha Gyngyike fiskolai adjunktus (5., 9. fejezet)
Dr. Elbert rpdn fiskolai adjunktus (8., 13. fejezet)
Dr. Hadnagy Andrsn fiskolai adjunktus (3., 4. fejezet)
Lrnt Lszl fiskolai adjunktus (11., 12. fejezet)
Riborics Gyrgy fiskolai adjunktus (6., 7. fejezet)
Dr. Scharnitzky Viktor fiskolai tanr (1., 2., 10., 14. fejezet)
munkja
Lektorlta
D r . Reiman Istvnn fiskolai docens
Fedlterv
VMOS Judit
A m ms kiadvnyban val rszleges vagy teljes felhasznlsa, utnkzlse,
illetve sokszorostsa a Kiad engedlye nlkl tilos!
ISBN 963 18 7424 9
Gy. Bartha Gyngyike, Dr. Elbert rpdn, Dr. Hadnagy Andrsn, Lrnt
Lszl, Riborics Gyrgy, Dr. Scharnitzky Viktor, Budapest,-1989
TARTALOMJEGYZK
ELSZ ........................................................................................................................
FELADA- EREDM-
TOK NYK
1. Halmazok .................................................................. 7 215
2. Vektorgeometria .......................................... ........... 13 219
3. Lineris algebra ........................................................ 31 231
4. Komplex szmok...................................................... 40 239
5. Sorozatok .................................................................. 45 242
6. Egyvltozs fggvnyek ........................................... 62 259
7. Egyvltozs fggvnyek differencilszmtsa........ 76 276
8. Egyvltozs fggvnyek integrlszmtsa.............. 90 298
9. Sorok.......................................................................... 118 323
10. Differencilegyenletek .............................................. 136 342
11. Valsznsgszmts .............................................. 158 370
12. Matematikai statisztika............................................ 172 378
13. Tbbvltozs fggvnyek......................................... 182 381
14. Vektorfggvnyek .................................................... 197 391
ELSZ
A ktet, amelyet az Olvas a kezben tart, azokbl a feladatokbl tartalmaz egy
gyjtemnyt, amelyeket a 4. oldalon felsorolt fiskolai oktatk munkjuk sorn az
utbbi vekben felhasznltak. Ebbl kvetkezik, hogy a feladatok kztt vannak
tbb-kevsb ismertek is, hiszen kr lett volna lemondani nhny frappns feladatrl
csak azrt, mert ezek mr valahol megjelentek. Vannak ismert tmjak, melyek
adataikban eredetiek, s vannak szp szmmal eredeti feladatok is.
A gyjtemny sszelltsa sorn arra trekedtnk, hogy minden tmakrben
lehetleg minl vltozatosabb feladatokat, ugyanakkor egy vltozaton bell tbb
hasonl feladatot is nyjtsunk t az Olvasnak annak rdekben, hogy mind a
gondolkodsra, mind a begyakorlsra alkalma legyen.
A feladatok egy-egy tmn bell ltalban (az ltalunk elkpzelt) fokozd nehz
sgi sorrendben kvetik egymst, de mivel egy-egy fejezeten bell tbb tma is
szerepel, ezrt a feladat sorszma a feladat nehzsgi fokra nem ad egyrtelm
eligaztst.
Annak ellenre, hogy az Eredmnyek cm rszben a legtbb feladatnak csak a
vgeredmnyt kzljk avgbl, hogy az Olvas ellenrizhesse munkjnak az
eredmnyt, s megoldst vagy megoldsokat alig adtunk, mgis arra biztatjuk az
Olvast, hogy a mr egyszer jl megoldott feladatot ksrelje meg megoldani rvideb
ben, mskppen, ms eszkzkkel, prblja meg a feladatot egyszersteni, talakta
ni, esetleg ltalnostani. Szmos feladatban ez a lehetsg, az nll alkots lehets
ge is megvan s nagyon rlnnk, ha ezzel minl tbben lni tudnnak.
Tudjuk, hogy sok ember idt rabl, aprlkos s gondos munkja ellenre a
feladatgyjtemnynek szmos fogyatkossga van. Krjk ezrt a t. Olvast, hogy a
ktet felfedezett hibin ne bosszankodjon, hanem ezeket vagy ms megjegyzseit
velnk kzlni szveskedjk; minden szrevtelt ksznettel fogadunk.
Vgl ksznett mondunk mindazoknak, akiknek lelkiismeretes kzremkdse
nlkl e knyv nem szlethetett volna meg.
Budapest, 1987. november A szerzk
1. HALMAZOK
A feladatgyjtemnyben itt s a tovbbiakban a szoksos halmazjellseket alkal
mazzuk. Ezek a kvetkezk:
N = {a termszetes szmok},
Z = {az egsz szmok},
R = {a vals szmok},
C = (a komplex szmok}.
1.1. Melyek halmazok az albbi sszessgek kzl?
a) A ={pratlan szmok};
b) B ={az x ^ - \ = 0 egyenlet gykei};
c) C=={1, 2, ,tg60, ^};
d) D {a. fiskola jelenlegi hallgati};
e) -{&. fiskola els vfolyamnak leny hallgati};
f ) F ={a fiskola magas hallgati};
g) G ={az angolul beszl magyar llampolgrok};
h) H ={az angol nyelvvizsgval rendelkez magyar llampolgrok};
i) I ={a 35 vnl idsebb nappali tagozatos hallgatk}.
1.2.rja fel elemeikkel a kvetkez halmazokat:
a) A ={a 25-nl kisebb pozitv pratlan szmok};
b) B ={24 valdi oszti};
c) C == {18 sszes oszti};
d) D ={a 6-tal oszthat pratlan termszetes szmok}.
1.3. Hny eleme van az albbi halmazoknak?
a) A ={az x ^ - 4 = 0 egyenlet vals gykei};
b) B ={az x^ 4 = 0 egyenlet egsz gykei};
c) C ={az x^ 4 = 0 egyenlet pozitv gykei};
d) D ={az x ^ - 4 = 0 egyenlet irracionlis gykei};
e) E ={az x ^ - 4 = 0 egyenlet gykeinek halmaza}.
1.4. Hny eleme van az albbi halmazoknak?
a) A ={az x ^ - %x +15 ^ 0 egyenltlensg vals megoldsai};
b) B ={az 8x + 15 ^ 0 egyenltlensg egsz megoldsai};
c) C == {az x ^ - 8x + 15 ^ 0 egyenltlensg racionlis megoldsai};
d) Z) := {az x ^ - 8x + 15 ^ 0 egyenltlensg negativ megoldsai}.
1.5. Hny eleme van a kvetkez halmazoknak?
a) A ~ {az x^ + x 56 ^ 0 egyenltlensg egsz megoldsai};
b) 5 == {az x^ + x - 5 6 ^ 0 egyenltlensg pozitv egsz megoldsai};
c) C ~ {az x^ + x - 5 6 ^ 0 egyenltlensg negatv egsz megoldsai};
d) D ={az x^ + x 56 ^ 0 egyenltlensg pros megoldsai};
e) E ={az x^ + x 56 ^ 0 egyenltlensg 7-nl nem kisebb megoldsai}.
1.6. Van-e az albbi halmazok kztt egyenl?
a) A == {2, 3, 4, 6};
b) B ~ {a hattal oszthat termszetes szmok};
c) C == {a 10-nl nagyobb s 20-nl kisebb pros szmok};
d) D ={\2 valdi pozitv oszti};
e) E-= {^-15n3 + S0n2-180n+144 = 0, neN};
f ) F== {a hrommal oszthat pros szmok}.
1.7. Legyen
A = {a. budapesti jrmvek};
B = {a budapesti autbuszok};
C = {a budapesti villamosok};
D = {a budapesti csukls jrmvek}.
Mit jelentenek az albbi halmazok: a) BnC; b) BkjC\ c) Br\D; d) Cr\D; e) B^ D;
f ) C\Z); g) A \ C .
1.8. Legyen A = | 4, 2, -j-, 5 = { e N : < 5}. Sorolja fel az albbi halmazok
elemeit: A<^B, AnB, A \ B , B^ A.
1.9. Legyen
A = {a 2-vel oszthat termszetes szmok},
B = {a 3-mal oszthat termszetes szmok},
C = {a 4-gyel oszthat termszetes szmok}.
Kpezze kt-kt halmaz egyestst, metszett s klnbsgt! Milyen tulajdonsg
szmok alkotjk ezeket a halmazokat?
1.10. Legyen N = {a termszetes szmok}, P = {a prmszmok}, 5 = {a pozitv
pros szmok}, T = {a pozitv pratlan szmok}. Kpezze minden lehetsges mdon
kt klnbz halmaz egyestst s metszett!
1.11. Legyen = {neN; 60 ^ ^ 70}; A2 {neA^i n pros}; A^ = {neAi :
n hrommal oszthat}; A^ = {neA^:n nggyel oszthat}. Hatrozza meg az albbi
halmazok elemeit: A^', A3 ; A4 ; A2 UA3 ;
A^^^A^y A^^^^A^y A^c^A^y
1.12. Legyen A = {1, 2}, B = [7, 8, 9}. Hatrozza meg a kvetkez halmazokat:
Ax B, BxA, Ax A, B^B.
1.13. Legyen T a tglalapok halmaza, R a rombuszok halmaza. ija fel ezek segtsg
vel a ngyzetek N halmazt!
1.14. Legyen H = {hrngyszgek}, D = {deltoidok}, T= {trapzok}. Hatrozza
meg a Hr\Dr\Thakaaz elemeit!
1.15. Legyen K = {kzppontosan szimmetrikus ngyszgek}, T = {tengelyesen
szimmetrikus ngyszgek}. Hatrozza meg a AToT halmazt!
1.16. Legyen a i / alaphalmaz a magyar nyelv knyvek halmaza, tovbb y4 = {reg
nyek}, B = {matematikai trgjf szakknyvek}, C = {tanknyvek} a H rszhalmazai.
Mit jelentenek az albbi halmazok: a) AkjB; b) BnC; c) A; d) B\ C; e) BkjCI
1.17. Jelentse A a fiskola frfi hallgati, B az elsves hallgati halmazt! Igaz-e, hogy
A' \ B=AnB teljesl?
1.18. Legyenek A, B s C egy H halmaz rszhalmazai. Mutassa meg, hogy ekkor
igazak az albbi sszefggsek:
a) AuA = A; b) AyjB = BkjA;
c) {AkjB)u C = Ayj{ByjQ\ d) Au0 = A;
e ) A u H = H ; f ) AnA = A;
g) AnB = BnA; h) An( BnQ = (AnB)nC;
i ) A n H = A ; j) An0 = 0;
k) A^{AriB) = A-, l) An(A(jB) = A.
1.19. Ha a / / alaphalmaz egy rszhalmaza A, akkor az A halmaz N-ra vonatkoz
komplementer hahnaza: A = H\ A . Ha Ar\B = 0, B H, akkor igazak-e a kvetkez
sszefggsek:
a) H=0; b) 0 = Hj
c) A = Aj d) AuA = H;
e) AnA = 0; /J AuB = AnB;
g) Ar\B = kjB\ h) A^{Br\B) = A;
i) An( BuS) = A.
1.20. Mutassa meg, hogy az A, B, C halmazokra igazak az albbi sszefggsek;
a) A \ { A \ B ) = B\ { B\ A) ; b) { A \ Q \ { B \ C ) = ( A \ B ) \ C;
c) ( A \ Q u ( B \ C ) = ( ^ u 5 ) \ C ; d) { A\ Qr ^{ B\ C) = ( AnB) \ C.
1.21. Igazolja Venn-diagram segtsgvel az albbi (disztributivitsi) szablyokat:
aj Au(Br)C) = (AuB)n(AuC);
b) An{BvjC) = {Ac\B)Kj{Ar\C).
1.22. Mutassa meg, hogy a H halmaz A, B, C rszhalmazaira igaz az albbi sszefg-
gs;
(AuB)n(AuB)n(AuB)n(AuB) = 0.
1.23. Milyen kapcsolat van az A, B s C halmazok kztt, ha
a) AntBnC = A; b) Avj B^C = C;
c) AnBnC = A s AuBuC = C; dj AnBnC = A s AuBuC = A.
1.24. Az albbi halmazok kzl x melyikben elem, melyikben rszhalmaz, melyikben
sem nem elem, sem nem rszhalmaz?
a){{x]^y)'^ b) x; c) 0 nx;
e) {x}ux; / ; {x}u{0}.
1.25. rja fel a H ={l, 2, 3} halmaz hatvnyhalmazt!
1.26. Legyen A = {0, 1} s 5 = {0, 2}. Mutassa meg, hogy P{AkjB) # P(^)uP(5).
(P(^) az A halmaz hatvnyhalmazt jelli.)
1.27. ^ 0 5 jelentse az ( A\ B) u ( B\ A) halmazt, az A s B halmaz szimmetrikus
klnbsgt. Bizonytsa be, hogy ekkor
a ) A 0 = A; b) AB= BA;
c) A{BQ = {AB)C\ d) Ar^{BQ = (AnB)(AnC);
ej A \ B c: AB; f j ^ = 5 akkor s csak akkor, ha AB = 0;
gj AC = B C egyenlsgbl A = B kvetkezik.
1.28. Legyen 0 = 0, 1 = 0u{0}, 2 = lu{l}, 3 = 2u{2}, 4 = 3u{3}. rja fel az 1, 2,
3 s 4 halmazok elemeit!
1.29. Jelentse 0,1,2 az 1.28. feladatban szerepl halmazokat. Az albbi lltsok kzl
melyik igaz s melyik hamis?
a ; i e 2; b j \ c ^ 2 ; cj I n l = 0 ;
dJlKj2 = 2; e j ( 0 n l ) e l ; / ; ln2 = 1.
1.30. Jelentse 0, 1, 2, 3, 4 az 1.28. feladatban rtelmezett halmazokat. Hatrozza
meg a kvetkez halmazokat:
10
a) 2u3; b) 2n3; c) 2u4;
d) 2n4; e) 3u4; f ) 3n4.
1.31. Jelentse 0, 1, 2, 3 a 1.28. feladatban rtelmezett halmazokat. Hatrozza meg
a kvetkez direkt szorzatokat;
a; Ox 1; 1 X 1; c; 1x 2;
^ ; 2 x l ; ^;2x2; / 2 x 3 ;
t; 3X2; / j ; 3 x 3 .
1.32. Az A halmaz 7-nl tbb, klnbz pozitv egsz szmbl ll; a szmok
legkisebb kzs tbbszrse 462, szorzatuk 9-szerese pedig egy egsz szm kbe.
Brmely kt A halmazbeli szm legkisebb kzs tbbszrse kisebb 250-nl. Mely
szmokbl ll az A halmaz?
1.33. Az A halmaz elemei klnbz pozitv egsz szmok, ^-nak 7-nl tbb eleme
van; az elemek szorzata nem oszthat 160-nal s nem negyedik hatvnya egyetlen
egsz szmnak sem. Az A sszes elemnek legkisebb kzs tbbszrse 390, s
brmely kt elemnek van 1-nl nagyobb kzs osztja. Melyek az A halmaz elemei?
1.34. Egy lapkiad vllalat hrom jsgot ad ki, A-i, B-i s C-t. A lapkiad
reklmfnke az albbi szmokkal illusztrlja a lapok npszersgt: Az orszgban
100 ember kzl 26 olvassa az A, 25 a 5, 14 a C, 11 az ^ s B,\Q a B s C, 9az A s
C jsgot, 5 pedig mind a hrom jsg olvasja. Ez sszesen 100. Ezrt az orszgban
mindenki olvassa a lapkiad legalbb egy jsgt! Igazat mondott-e a reklmfnk?
1.35. A fiskoln a lenyok kzl 90-nek a haja barna, 70-nek mind a haja, mind
a szeme barna, tovbb 130-nak a szeme s a haja kzl legalbb az egyik barna.
Hny barna szem leny van a fiskoln?
1.36. Egy szvzemben trsasutazst szerveznek. A szvdben dolgozk kztt
300 n van, a szvdn kvl 40 n s 20 frfi dolgozik. A tavalyi trsasutazsokon
az zembl 50 frfi, a szvdbl 240 dolgoz vett rszt. Az idei trsasutazsokra a
tavalyi rsztvevk nem mehetnek el. Hny n jhet a szvdbl a trsasutazsokra
az idn?
1.37. Egy cg eladja, aki hromfajta cikk eladsval foglalkozik, gy szmol be
napi munkjrl; 40 lehetsges vevvel trgyaltam, kzlk 15 nem vsrolt semmit,
15 vsrolt az A rucikkbl, 12 a .S rucikkbl, s 10 a C rucikkbl. Hatan
vsroltak az ^4-bl s a 5-bl, egy vev a 5-bl s a C-bl, s hrom az .^-bl s a
C-bl. Igazat mondott az elad?
1.38. Egy vllalat 100 dolgozja kzl angol nyelvvizsgja van 28 fnek, nmet
nyelvvizsgja 30 fnek, orosz nyelvvizsgja 42 fnek. Angol s nmet nyelvvizsgja
van 8, angol s orosz nyelvvizsgja van 10, nmet s orosz nyelvvizsgja van 5 fnek.
II
Hrom dolgoznak mind a hrom nyelvbl van nyelvvizsgja.
a) Hny dolgoznak nincs egy nyelvvizsgja sem?
b) Hny dolgoznak van csak orosz nyelvvizsgja?
c) Hny dolgoznak van csak nmet s orosz nyelvvizsgja?
1.39. Valaki azt lltotta, hogy a 100 fbl ll vfolyam fele leny, a kollgistk
szma 30, az igazolt sportolk szma 23. A sportol lenyok szma 20, a sportol
kollgistk 10, a kollgista lenyok 8, vgl a sportol kollgista lenyok szma 5.
Mutassa meg, hogy a kzlt adatok kztt ellentmonds van!
1.40. Mutassa meg, hogy brmely flkr kerletn ugyanannyi pont van, mint az
tmrjn!
12
2. VEKTORGEOMETRIA
Vektorok lineris kombincija
2.1. A kocka egy cscsbl kiindul hrom lvektor a, b, c. Fejezze ki ezek
segtsgvel az ebbe a cscsba rkez laptlvektorokat s az ebbe a cscsba rkez
testtlvektort!
2.2. Legyen adott az ABCD paralelogramma s O egy tetszleges pont. Bizonytsa
be, hogy
l + ^ = C) +d\
2.3. Az ABCD paralelogramma cscsainak a helyvektorai rendre a, b, c, d. Fejezze
ki d-t az a, b s c segtsgvel!
2.4. Egy paralelepipedon egyik cscsbl kiindul laptlvektorok x, y, z. rja fel
ezek segtsgvel a velk azonos cscsbl indul a, b, c lvektorokat!
2.5. Egy szablyos hatszg alap hasb alaplapjainak kzppontjbl az alap kt
szomszdos cscshoz az x s y, a fedlap kzppontjhoz pedig a z vezet. rja fel
a hasb tbbi cscsnak a helyvektorait x, y s z segtsgvel!
2.6. Legyen Vj = 3a + 2b, Vj = - a + 3b, Vj = 2a-4b. Fejezze ki a-val s b-vel a
Wj = 3vj V2 + 2V3 s a W2 = V2 2vi + 3v3 vektorokat!
2.7. Legyen v = 2a + b s w = a + 2b. Fejezze ki a kvetkez vektorokat a v s w
vektorokkal:
1
a; 3a-4b; 5a + 2b; c) - a + h.
2.8. Az ABC hromszg slyvonalai az AA^, BB^ s CC^ egyenesek, slypontja
az S pont, s a tr egy tetszleges pontja P. Fejezze ki a
a) P; c ) p 1 ,-
b) P; d) vektorokat a = a, ^ = b, = c vektorokkal!
13
2.9. Az A pont helyvektora a, a B pont b. Legyen az A pont tkrkpe B-re A',
helyvektora a'. lltsa el a'-t a s b segtsgvel!
2.10. Legyenek A, B, C a tr tetszleges pontjai. A brhol vlasztott P pontot
tkrzze ^-ra, a tkrkpet 5-re, az gy kapott pontot C-re, az gy nyert pontot ismt
A-ra, majd hasonlan folytatva az eljrst tkrzzn B-re s C-re! Bizonytsa be,
hogy a hatodik tkrkp azonos P-vel!
2.11. Az A pont helyvektora a B pont b. Hatrozza meg az AB szakasz
negyedel pontjainak helyvektorait!
2.12. A 2 egysgnyi lhosszsg kockt gy helyezzk el a koordnta-rendszer-
ben, hogy az orig a kocka egyik cscsra illeszkedik, a tengelyek pozitv fele pedig
egy-egy lt tartalmaz. Adja meg a kocka cscsainak a koordintit!
2.13. Az ABC hromszg kt cscspontja A(2', 2; 1), B(6 ; 3; 1), slypontja
5(3; 2; 1). Hatrozza meg a C cscspont koordintit!
2.14. Egy szablyos hatszg kzppontja ^(4;1;4), kt szomszdos cscsa
^(3; 1; 5) s B(3; 2; 4). Adja meg a tbbi ngy cscs koordintit!
2.15. Az ABCD paralelogramma cscsai A(3; 2; 5), 5(0; 1;0), C( 5;2;7).
Szmtsa ki a D cscs koordintit!
2.16. Egy paralelepipedon egyik cscsa az orig, az ebbl kiindul lek vgpontjai
az A(3; 6; 4), B( 4; 7; 0), C(9; 1; 3) pontok. Szmtsa ki a tbbi ngy cscs
koordintit!
2.17. Legyen az ABCDAiB^CiDi paralelepipedon ABCD lapjnak kzppontja
E, a BCB^C^ lapjnak kzppontja F, a BD^ testtljnak felezpontja G s a Dj-hez
kzelebb fekv negyedelpontja H, vgl a C^D^ lnek felezpontja K. Ha
A = a(4; 2;-3), A ^ = b(5; 6; - 2 ) s A^^ = c(l; 4; -3), fejezze ki a kvetkez
vektorokat a, b s c segtsgvel s szmtsa ki koordintikat is!
a)A-, b) AC^\ c)A^- d ) W^ - ,
e) A- f ) A p - g ) A - h)
i) f - j ) F; kJ F-, l) HK.
2.18. Dntse el, prhuzamosak-e a kvetkez vektorprok:
a ; a ( - 3 ; 4 ; 7 ) , b(2;5;l);
Jc(12;9;15), d(8;6;10);
c ; e ( 7 ; - 4 ; 2 ) , f(0;0;0).
2.19. Dntse el, hogy az albbi ponthrmasok egy egyenesen vannak-e:
14
a) A( - 4; 5; 2) , 5(2; 0 ; - 3 ) , C(14; - 10; - 13);
^(4;1;7), F ( 5 ; - l ; - l ) .
2.20. Szmtsa ki a P(3; - 4 ; 8) pontnak az A(3; 7; - 2 ) pontra vett tkrkpe
koordintit!
2.21. Adottak az a ( 3 ; - 2 ; 5 ) , b(- 4 ; 2 ; 0 ), c ( - 2 ; 0 ; 5 ) vektorok. rja fel a
Vi = a + 2b c, V2 = 3 a - - b + 2c, Vj = 7ra+)/5b + c vektorok koordintit!
2.22. Bontsa fel a v(13; 56) vektort az a(2; 7) s b( 3; 0) vektorokkal prhuzamos
sszetevkre!
2.23. Bontsa fel a d(31; -37; 19) vektort az a ( - 8 ; 7 ; 1), b(0; 3; 2) s c(l; - 1 ; 4 )
irny sszetevkre!
2.24. Szmitsa ki az albbi vektorok hosszt:
/ 5 30 6 \
a(8; -14; - 8), b(^- ; ~ ^ c(4; - 9 ; 10).
2.25. Adja meg az albbi vektorok irnyba mutat egysgvektorokat:
a(4;-12;3), b(0 ; 0 ; -7 ), c ( - l ; 4 ; 8 ) , d ( l ; 2 ; - 3 ) , v ( - 2 ; 2 ; l ) .
Skalris szorzat
2.26. Az ABC szablyos hromszg oldalhossza 2 egysg. Szmtsa ki az Ad
szorzat rtkt!
2.27. A szgek kiszmtsa nlkl dntse el, hogy az albbi vektorprok hegyes-,
derk- vagy tompaszget zrnak-e be:
aj ( - 3 ; 2 ; 0 ) , (4, 1,5); b) (1; 1; 9), (2; 1; 3);
c) (1;1 ; 1 ), ( - 1 0 ; 7 ; 3 ) ; d) (5; - 3; 4), (1; - 1; 2).
2.28. Adottak az a(3; 6; 1) s b(12; 4; z) vektorok. Hatrozza meg z rtkt gy,
hogy a s b merlegesek legyenek egymsra!
2.29. Hogyan kell megvlasztani p rtkt, hogy az a + / 7b vektor merleges legyen
a b vektorra?
2.30. Hogyan kell megvlasztani az a s b vektorokat, hogy brmely ph?,q esetn
a pz + q\) vektor merleges legyen a qzph vektorra?
2.31. Legyen a, b, c hrom tetszleges vektor. Megvlaszthat-e p s q gy, hogy
az a vektor'merleges legyen a ph- qc vektorra?
15
2.32. Mutassa meg, hogy az a ( - 2 ; 3; 6), b(6; - 2 ; 3), c(3; 6; - 2 ) vektorok koc
kt fesztenek ki!
2.33. Mekkora szgeket zr be a v(4; 1; 8) vektor a koordintatengelyek pozitv
irnyval?
2.34. Van-e olyan vektor, amely az x, y, z koordintatengelyek pozitv irnyval
a kvetkez szgeket zrja be?
a) 45, 60, 120; b) 45, 135, 60.
2.35. Szmtsa ki a kvetkez vektorprok szgt:
j a ( 7 ; - l ; 6 ) , b(2;20;l);
; c ( 3 ; 6 ; - 2 ) , d(5;4;-20);
c ; e ( - l ; 4 ; 7 ) , f ( 5 ; - 2 ; 0 ) ;
^Jg = *+ 2j + k, h = 5 i - 3 j - 4 k ;
e) = 3i 2j 3k, V2 = 2i + 3j + k.
2.36. Mutassa meg, hogy az ^(3; - 8; 2),5(1;6; 2),C(5; 2; 8) cscspon
t hromszg szablyos!
2.37. Az / i ( - 6 ; 5 ; 5 ) , j 5 i ( - 3; - 1; 3), Q(5;4;0); ^ 2(6; - 5 ; - 2 ) ,
^ 2(4; - 2 ; 3), C2(6; 4; 0) s ^ 3( - 2; 7; 4), 4; - 1), Ca(3; 0; 2) cscspont h
romszgek kzl melyik hegyes-, derk- vagy tompaszg?
2.38. Hatrozza meg az A( l ; 5; 6), B( - 2; - 1; 0), C(2; 2; 1) cscspont hrom
szg bels szgeit!
2.39. Szmtsa ki az ^(1; 5; 6 ), B( - 2; - 4 ; 0), C(4; 2; 2) cscspont hromszg
ben az A cscspontbl indul slyvonalnak az AB oldallal bezrt szgt!
2.40. Szmtsa ki az A(19; - 6 ; 3 ) , 5(5;4; -1), C(3;2; -5) , >(I7; - 8; - 1 )
cscspont paralelogramma tlinak hajlsszgt s M metszspontjnak koordin
tit!
2.41. Egy hromszg cscspontjainak a koordinti: v4(7; 2; 1),
jS(-2; 5; 8), C(4; 7; 10). Legyenek az A^ s A2 pontok a CB oldal harmadol
pontjai, F pedig a CB oldal felezpontja. Egyenl szr-e az AA^A2 hromszg?
Mekkora az AFA2 szg?
2.42. Az ABC hromszg cscsainak koordinti: A{ 3;4;0), B{9; 11;42),
C(1; 2; 4). Mekkora a hromszg kerlete? Mekkora a hromszg A cscsnl fekv
szge?
16
2.43. Egy hromszg cscspontjainak koordinti: J(2; 4; 2), B(30; 44; - 16)
C( 2; 4; 0). Legyen a CB oldalt negyedel, a C cscshoz legkzelebb es pont.
Tompaszg-e A A hromszg?
2.44. Hatrozza meg az a(2; - 5; 1) vektornak a b(3; 0; 4) vektor egyenesre es
merleges vetletnek a hosszt!
2.45. Adott ngy pont: A(l; - 2 ; 3), B( - 4; 2; 1), C(3;2; 1), Z)(-4; - 2 ; 5). Ha
trozza meg az vektornak a vektorra es vetleti vektort!
2.46. Az adott a vektorhoz hatrozza meg mindazokat a b vektorokat, amelyek
nek az a-ra vett merleges vetlete ugyanakkora, mint az a-nak a b-re vett merleges
vetlete! (a^^O).
2.47. Bontsa fel az a(3; 6; 9) vektort a b(2; 2; 1) vektorral prhuzamos p, s
a b-re merleges m sszetevre!
2.48. Szmtsa ki az ^( - 1; 1; - 3), B{2; - 2; 3), C(4; 0; 0) cscspont hromszg
leghosszabb oldalhoz tartoz magassga T talppontjnak a koordintit!
2.49. Szmitsa ki az A(4; 5; - 12), B(3; - 4 ; 5), C (- 7; 14; 1) cscspont hrom
szg A cscsbl indul slyvonal s a vele szemkzti oldal F metszspontjnak
koordintit s a BCA szget!
2.50. Szmitsa ki az A(2-, - 1; 1), B(16; 1; 3), C(6; - 1; - 2) cscspont hrom
szg AC oldalhoz tartoz magassg T talppontjnak a koordintit s a magassg
hosszt!
2.51. Egy hromszg cscspontjainak a koordinti: A(l ; 3; 4), B(5; l ; - 2 ) ,
C(6; - 4 ; 1). Szmitsa ki a C cscspontbl indul magassg T talppontjnak az AB
oldal F felezpontjtl mrt tvolsgt!
2.52. Legyen az A( - l ; 0; 2) , B(3;7; -2 ) , C(l; - 1 ; 0 ) cscspont hromszg
slypontja S, az AB oldalhoz tartoz magassgnak talppontja T. Szmitsa ki az
ST szakasz hosszt!
Vektorilis szorzat
2.53. Vgezze el a kijellt mveleteket a kvetkez kifejezsekben:
aj (a-Fb)x(a-2b); 6; (3a-b)x(b-f3a);
c) (a + 2b)x(2a + b) + (a-2b)x(2a-b).
2.54. Adottak az a(2; - 3 ; 1), b(4; 2; -1), c(l; 0; - 3 ) vektorok. Szmtsa ki a
V = (a Xb) Xc vektor koordintit!
2 Matematikai feladatok 17
2.55. Egy kocka egyik cscsbl kiindul kt lvektora a s b. Fejezzk ki ezek
segtsgvel a cscsbl kiindul harmadik lvektort!
2.56. Egy kockt kifeszt hrom vektor kzl kettt ismernk: a(6; 2; 3),
b( - 3 ; 6; -2). Hatrozza meg a harmadik vektort!
2.57. Egy paralelepipedon egyik cscsa az orig, ebbl kiindul kt lvektora
a(1; 0; 2), b ( l ; l ; 0), a harmadik l merleges az a s b skjra s hossza 9
egysg. Szmtsa ki a feltteleknek eleget tev egyik paralelepipedon cscsainak a
koordintit!
2.58. Egy tglatest egyik cscsa az orig, az innen kiindul testtlvektora
d(9; 5; 5), kt lvektora a(4; - 3; 1) s b(0; 2; 6). Adja meg a harmadik lvektort!
2.59. Adjon meg olyan x vektort, amely merleges az a(2; - 3; 1) s a b(l; - 2; 3)
vektorra s a c ( l ; 2; - 7) vektorral vett skalris szorzata cx= 10.
2.60. Mekkora szget zrnak be egymssal az ABCD tetrader ABC s ACD
lapsikjai, ha a cscsok koordinti: ^(2;3;1), ( 4 ; l ; - 2 ) , C(6;3;7),
D { - 5 - - 4 ; 8)?
2.61. Szmtsa ki annak a paralelogrammnak a terlett, amelyet az a( 9; 0; 9)
s b(7; 2; 5) vektorok fesztenek ki!
2.62. Szmtsa ki az ABC hromszg terlett, ha
a j ^ ( 0 ; 0 ; 0 ) , ( - l ; 4 ; 7 ) , C(5;2;l);
b) A{\-,Q-2), 5(4; 3; 8), C (0 ; -4 ; 6 );
c) A ( 4 ; - l ; - 3 ) , f i ( 3 ; l ; - 2 ) , C(l;5;0);
dJA(0;2;3), B(l;0;2), C ( 3 ; - l ; 0 ) .
2.63. Bizonytsa be, hogy az A( l ; 1), B(5; 1), C(7; 7), Z>(3; 5) cscspont ngy
szg paralelogramma, s szmtsa ki a terlett!
2.64. Ha az a s b vektorok ltal kifesztett paralelogramma terlete , akkor
mekkora a 2a + 3b s a 4a 2b vektorok ltal kifesztett paralelogramma terlete?
2.65. Szmtsa ki az A(1; - 1; 2), B(5; - 6; 2), C(1; 3; - 1) cscspont hromszg
B cscshoz tartoz magassgnak a hosszt!
2.G6. Az hromszg hrom oldalfelez pontja: ^i(2; 3; 1), B^{-2; - 1; 2),
C i ( - 1; 0; 3). Szmtsa ki az ABC hromszg kerlett s terlett!
2.01. Az ^(2; 1; - 3), 5(1 ;0; 2), C(6;2; - 1 ) cscspont hromszget vettse
18
merlegesen a P(1; 0; 1), g(2; - 1; 3), i?(- 4; 2; - 1) pontok skjra! Szmtsa ki a
vetleti hromszg terlett!
2.68. Bizonytsa be, hogy tetszleges a s b vektorra (axb)^ + (ab)^ = a^b^!
2.69. Bizonytsa be, hogy ha a + b + c = 0, akkor axb = bxc = cxa!
Vegyesszorzat
2.70. Dntse el, hogy az albbi vektorhrmasok komplanrisak-e:
a ; ( 2 ; 3 ; l ) , ( 1 ; - 1 ; 3 ) , (1 ; 9 ; - 1 1 ) ;
; ( 3 ; - 2 ; l ) , (2;1;2), ( 3 ; - l ; - 2 ) ;
c ; ( 2 ; - l ; 2 ) , ( l ; 2 ; - 3 ) , ( 3 ; - 4 ; 7).
2.71. llaptsa meg, hogy az a = i 2j 3k, b = 5i + 4j + 7k, c = 6i + 7j + 8k
vektorok jobb- vagy balsodrs rendszert alkotiiak-e.
2.72. Dntse el, hogy a kvetkez pontngyesek egy skban vannak-e:
a ; ( l ; 2; - l ) , (0;1;5), ( - 1; 2; 1), (2;1;3);
bj (0;1;1), (3; 5 ; - 4 ) , ( - 4 ; - 2 ; 6);
c j ( l ; 5 ; 4 ) , ( - 2 ; l ; - 6 ) , ( 0 ; 2 ; - l ) , (2;3;4)!
2.73. Vlassza meg z rtkt gy, hogy az a (4; 1;2), b(l;2;3), c(3;3;z)
vektorok komplanrisak legyenek!
2.74. Van-e olyan, a 0-tl klnbz vektor, amely merleges az a(4;2; 1),
b(l;2; - 2 ) s a c(5; - 2 ; 4) vektorok mindegyikre? Ha van ilyen vektor, akkor
egyet adjon is meg!
2.75. Egy skon vannak-e az ^ ( - 2 ; 2 ; - 2 ) , C(2;-2;2),
D(9; - 6; - 15) pontok? Szmtsa ki az ABD hromszg terlett!
2.76. Mekkora az a(2; 3; 4), b(2; 3; 1), c(l; 2; 3) vektorok ltal kifesztett paralel-
epipedon trfogata?
2.77. Mekkora az ABCD tetrader trfogata, ha cscspontjai:
a) A( l ; - 2 ; 3 ) , ( - 4 ; 2 ; l ) , C(3;0;2), Z)(0;-2;5);
b) A { 3 - , - l ; - l ) , 5 ( 5 ; - 2 ; 3), C (4;0;- 2), /)(5;0;1).
2.78. Az ABCD tetrader trfogata 5 egysg. Mik a D cscs koordinti, ha D az
y tengelyen van s a tbbi cscs koordinti: A(2; 1; - 1), 5(3; 0; 1), C(2; - 1; 3)?
2* 19
2.79. Egy tetrader cscspontjai A( 3; l;2), B{4;2; 3), C ( 2 ; l ; 2) s
D(2; 8; 5). Szmtsa ki a BCD laphoz tartoz testnlagassg hosszt!
2.80. Egy tetrader cscspontjai: ^(2;3;1), 5(4; 1 ; 2), C(6;3;7),
D ( - 5; - 4; 8). Szmitsa ki a tetrader D cscsbl hzhat magassgnak a hosszt!
2.81. Az a, b, c vektorok egy skban vannak. Komplanrisak-e a Vj = 2a + 3b,
Vj = 3b 4c, V3 = 2a+5c vektorok?
2.82. Az a, b, c vektorok nincsenek egy skban. Egy skban van-e a kvetkez
hrom vektor: Vj = 2a + 3b, Vj = 5b 4c, V3 = c a?
2.83. Egy paralelepipedon egy cscsbl kiindul laptli egy jabb paralelepipe-
dont fesztenek ki. Hnyszorosa ennek trfogata az eredeti paralelepipedon trfogat
nak?
2.84. A = 2a + 3b + 4c, V2 = a - b + c, V3 = 2a + 4 b - c vektorok ltal kifesz
tett tetrader trfogata hnyszorosa az a, b, c vektorok ltal kifesztett tetrader
V trfogatnak?
Az egyenes
2.85. rja fel a P ponton thalad, v irnyvektor egyenes egyenletrendszert, ha
a; P ( - l ; 3 ; 7 ) , v ( - 4 ; 2 ; 6 ) ;
b) P(0
c; p{9
d) p{i
- 1 ; 2 ) , v ( l ; 7 ; - 9 ) ;
8 ; - 3 ) , v(6;0;2);
- 2 ; 5), v ( 4 ; 3 ; - 2 ) .
2.86.rja fel a kvetkez pontprokat sszekt egyenesek egyenletrendszert:
a ; P ( - 2 ; 5 ; 6 ) , 0 ( 7 ; - 1 ; 3);
b) P(5;l;2), ( - 5 ; l ; 3 ) ;
c j P ( 0; 0; 0), ( 9 ; i i ; - i ) ;
d) P{ i ; V, - 2) , 2 ( 3 ; - 1 ; 0 ) .
2.87. Egy egyenesre illeszkednek-e a kvetkez pontok?
a; ^ ( - 2 ; 5; 3), (l;2;4), C (3; -7; 7);
b) A( l ; 2; 4) , 5 ( - 2 ; 5 ; 3 ) , C(10;-7;7).
2.88. rja fel annak az egyenesnek a vektoregyenlett, amely illeszkedik a
P ( - 3 ; 2 ; - 1) pontra s prhuzamos az x = 3 + 2, = 8 + , z = 1-7? egyenessel!
2.89. Adja meg annak az egyenesnek az egyenletrendszert, amely illeszkedik a
P(1;2;0) pontra s merleges az x = 2 + 3 t , y = 5 + t, z 3 s az x = 8 + ,
y = - t , z=3t egyenesekre!
20
2.90. rja fel annak az egyenesnek a vektoregyenlett, amely illeszkedik a P(2; 1; 0)
pontra, merleges az ----- = - - = z egyenesre s prhuzamos az x+y = - z
D
skkal!
2.91. Hatrozza meg annak az egyenesnek az egyenletrendszert, amely illeszkedik
a P(0; 5; 2) pontra sazx = \ 2 + /, z = 2 egyenest merlegesen metszi!
2.92. rja fel annak az egyenesnek az egyenletrendszert, amely illeszkedik a
P(3; 1; 1) pontra, az x - 2 j + 3z-4 = 0 sikra s merleges az x = 3 + 2/, >^ = 1+ /,
z = 1 egyenesre!
2.93. Mutassa meg, hogy a 3 3x = 6 y = 4z+8 s az , y = 5 + , z = 16 + 5
egyenesek metszik egymst! Hatrozza meg annak az egyenesnek az egyenletrendsze
rt, amelynek mind a kt adott egyenessel van kzs pontja s merleges mind a kt
adott egyenesre!
2.94. Adja meg a p paramter rtkt gy, hogy az albbi kt egyenes messe
egymst:
x + 2 y z1 x 3 y \ z 1
^ ~ p ~ ~ ~ 4 ~ ~ ~ Y ~ '
2.95. Hatrozza meg annak az egyenesnek a vektoregyenlett, amely illeszkedik
2 x 1
a P(\; - 1; 5) pontra, prhuzamos az x +3 y - z = 4 skkal s merleges a - =
= l +y = - egyenesre!
2.96. rja fel annak az egyenesnek az egyenlett, amely illeszkedik a 2xy+ 5 = 0
skra, ennek P(2;9; 1) pontjra s merleges az x 1+ , y = l t, z = 2+3t
egyenesre!
2.97. rja fel az
x + 9 y + 2 y - 2 z+4
= ----- = z - 4 , x - 3 = -------= ------
- 5 3 5 - 3
metsz egyenesek szgfelezinek az egyenletrendszert!
2.98. Hatrozza meg az albbi skprok metszsvonalnak az egyenletrendszert;
a) 2x+ 5y3z + S = 0, x 2y+z5 = 0;
b) x - 2 y + 3 z - 4 = 0, 3 x + 2 j - 5 z - 4 = 0;
c) 2x y + 3z6 = 0, x+y 2z 9 = 0.
21
2.99. rja fel a P(l;3;2) pontra illeszked s a 2x + j + 3z= 1 s
x - y z + 2 = 0 skok metszsvonalval prhuzamos egyenes vektoregyenlett!
2.100. Hatrozza meg annak az egyenesnek az egyenletrendszert, amely prhuza
mos a 2xy + 6 z 7 s x + y z 4 = 0 skokkal s illeszkedik a P( 2; 1; 2) pont
ra. Igaz-e, hogy ez az egyenes az els skra illeszkedik?
2.101. Hatrozza meg a 2x 3y 6 z = 32 sk s a 2(x 2) = 2y~2 = z + 3
egyenes kzs pontjban a skra merlegesen lltott egyenes vektoregyenlett!
2.102. Az S sk egyenlete 3x + 6;^-z = 8. Az S sk kt pontja A(l; I; 1) s
5(4; 1; 2). rja fel annak az egyenesnek az egyenletrendszert, amely illeszkedik
a S skra, az A pontra s merleges az AB szakaszra!
2.103. rja fel az x = 7 + 3t,y = 4 2 t, z = 4 + 3t s az x = 1+ (,y = 8 + 2/,
z = - 12+ / egyenesek normltranszverzlisnak az egyenletrendszert! Szmtsa ki
a kt kitr egyenes tvolsgt!
2.104. Hatrozza meg annak az egyenesnek az egyenletrendszert, amely prhuza-
12x z + 1
mos az x y = 1 skkal, merleges az - = j ^ egyenesre s illeszkedik a
P(2; 1; 1) pontra! Mekkora a kapott egyenes s a sk tvolsga?
2.105. Adott hrom pont: A(2; 3; 1), B( 2; 1; 1), C(4; 1; 3). rja fel az
A, B, C pontoktl egyenl tvolsgra lev pontok halmaznak az egyenlett!
A sk
2.106. Adott a sk n normlvektora s P pontja. rja fel a sk egyenlett!
a; n ( - 3 ; 2 ; l l ) , P(9;l;0); b) n(9;l;0), P ( - 3 ; 2 ; l l ) ;
c ; n ( l ; 0 ; l ) , P(2;7;5); d) n(3;2;l), P(0;0;0);
^ ; n ( 0 ; l ; 0 ) , P ( 5 ; 2 ; - 3 ) .
2.107. rja fel annak a sknak az egyenlett, amely illeszkedik a P(1; - 2; 3) pontra
s prhuzamos a 3 x - 4 y + 5 z - 3 = 0 skkal!
2.108. rja fel annak a sknak az egyenlett, amely illeszkedik az albbi ponthr
masokra:
a ; ^ ( 2 ; 3 ; l ) , B ( - l - 2
b) A( 3; - l ; 2) , 5 ( 4 ; - 1
ej ^ ( 4 ; - 6 ; 7), B( - 6; 4
5), C ( 2 ; - l ; 0 ) ;
-1), C(2;0;2);
-3), C ( - l ; 8 ; 8 ) ;
dj A ( 2 ; - 6 ; - 5 ) , 5(5;10;2), C(-7;0;10);
^ ; ^ ( 1 ; 5 ; 4 ) , B ( - 2 ; l ; - 6 ) , C ( 0 ; 2 ; - l ) .
22
2.109. Egy skra illeszkedik-e a kvetkez ngy pont?
A(2;3;4), 5(0; 2; - 1), C(-2; 1 ; - 6), Z)(l;5;4).
2.110. rja fel annak a sknak az egyenlett, amely illeszkedik a P( - 3; 2; 5) pontra
s az Xtengelyre!
2.111. rja fel a P{2; - 1 ; 3 ) pontra illeszked s a Vi(l; 2; 4), V2(4; 3;0)
vektorokkal prhuzamos sk egyenlett!
2.112. rja fel annak a sknak az egyenlett, amely illeszkedik az ^(3; 4; 5)
pontra s prhuzamos az a(3; 1; - 1) s b( - 1; 2; 1) vektorokkal!
2.113. rja fel annak a sknak az egyenlett, amely illeszkedik az x = 2 + 3,
y = - 1+ 2, z = 3 - 2 s az X= 1+ 3, = 2 + 2, z = - 3 - 2 prhuzamos egye
nesekre !
2.114. rja fel annak a sknak az egyenlett, amely illeszkedik az A(2; - 3; 1) s
B{3; 1; 2) pontokra s prhuzamos az x = 2 +1 , y = - 2 3, 2z = 5? egyenessel!
2.115. Hatrozza meg annak a sknak az egyenlett, amely illeszkedik a
P(3; 2; 5) pontra s prhuzamos az x = \ 2t , y \ = 2z s az x = , j = 1+ /,
z = 2 egyenessel!
2.116. Hatrozza meg a P(5;2;4) pontra s az x = + 3, y = - 2 t 2, z = 6
egyenesre illeszked sk egyenlett!
2.117. Adott kt pont: ^(0; 1; 3) s B{\; 3; 5). rja fel az A ponton thalad s
az AB egyenesre merleges sk egyenlett!
2.118. Az Ax+3y 5z+ 1 = 0 sk egyenletben hogyan kell az A rtkt megv
lasztani, hogy a sk prhuzamos legyen azx = At +\ , y = 3t~2, z = t egyenessel?
2.119. Illeszkedik-e az x ~ \ + 2t, y = 3 t, z = 2+5t egyenes a
4x + >' z + 3 = 0 skra?
2.120. rja fel annak a sknak az egyenlett, amely illeszkedik az x = 1+ 3/,
2j = , z = \ t egyenesre s merleges az 5x = 0 skra!
2.121. rjafelannakasknakazegyenlett, amely merleges azx2j;+3z 2 = 0
skra s illeszkedik a Pi( - 1; 2; 0) s 2; 2) pontokra!
2.122. rja fel annak a sknak az egyenlett, amely illeszkedik a P(l; 1; 1) s
(2; - 3 ; 1) pontokra s merleges a. 3x + y - z - l = 0 skra!
23
2.123. Hatrozza meg annak a sknak az egyenlett, amely illeszkedik az
A(3; 1; 0) s 5( 1; 2; 1) pontokra s merleges a l x y + z 5 skra!
2.124. rja fel a /*(1;2;3) pontra illeszked s az x + 2y 3z+l = 0,
x + 3y- z+6 = 0 skokra merleges sk egyenlett!
2.125. Hatrozza meg annak a sknak az egyenlett, amely illeszkedik a P(1; 4; 7)
pontra s merleges a 2 x - y + z = 3 s x + j - 3 z - I = 0 egyenlet skokra!
2.126. Mutassa meg, hogy a 3x- 9 = y - S = 3t, z = 3>+ 4t s a
9~y 9~z
4 X= ^ egyenes ugyanarra a skra illeszkedik! rja fel e sk egyenlett!
2.127. ija fel az
x 3 j + 1 2 2 X8 6 z
s =
egyenesekre illeszked sk egyenlett!
x 3 v+1 z~2
2.128. Iqa fel a z = = s az x = 2 + 2t, y = l ~ 3 t , z = l + t,
2 6 4
egyenesekre illeszked sk egyenlett!
2.129. Mutassa meg, hogy az jc3 = 3(1- j ) = - ( z+ 1 ) ; 4 - x = 3y + 6 = z
egyenesek s a P(5; 1; 2) pont ugyanarra a skra illeszkednek! rja fel a sk
egyenlett!
2.130. ija fel annak a sknak az egyenlett, amely illeszkedik az
x+ 1 = 3y = 4 - 2 z egyenesre s prhuzamos az ^ ( - 6; 7; 5) s 5(1; - 5 ; - 6)
pontokat sszekt szakasszal!
2.131. Hatrozza meg a P(l;3; 1) pontra illeszked s az x = 2t, y = - 1 ,
z = 3 t s x 3 = y 2 = z egyenesekkel prhuzamos sk egyenlett!
2.132. Dntse el, hogy egy skban van-e a kvetkez hrom egyenes: x = - 5 + 3,
y = 3 t, z = \ t\ X = 6 3t, y = 3 +1, z = l + ; x = l 6 , y = 2t,
z = 2 + 2t.
2.133. rja fel annak a sknak az egyenlett, amely illeszkedik az ^ ( - 3 ; 2; 1)
pontra, prhuzamos az x = 3 + 2/, y=t , z = - l + 4 egyenessel s merleges az
x 2y+5z3 = 0 skra!
2.134. qa fel annak a sknak az egyenlett, amely illeszkedik az x = l + 3,
24
y = 3 + 2t, z = 1 t egyenesre s prhuzamos a 2xy + z 3 = 0 s
x + 2 y - z - 5 = 0 skok metszsvonalval!
2.135. rja fel annak a sknak az egyenlett, amely illeszkedik a P( - 2; 1; 3) pontra
s az x y+3z 8 = 0 s 2 x+y z + 2 = 0 skok metszsvonalra!
2.136. llaptsa meg annak a sknak az egyenlett, amely illeszkedik az
z +i . . . y
----- = = ------ egyenesre es prhuzamos azx+3 = - = z 1 egyenessel!
2.137. Mutassa meg, hogy az x - 3 y + 4 z - 5 = 0 sk merleges az y - x + z = 1
skra! rja fel annak a sknak az egyenlett, amely merleges az adott skokra s
illeszkedik a P(2; 0; - 5 ) pontra!
2.138. Hatrozza meg a 4x 2 j + 3 z + 13 = 0, 4y5x + 2z12 = 0,
6x-4>' - 5z + 11 = 0 skok kzs pontjnak koordintit!
2.139. Van-e a kvetkez ngy sknak kzs pontja:
5x z + 3 = 0, 2x y 4z+5 = 0, 3>+ 2z1 = 0, 3x + Ay + 5z3 = 0?
2.140. rja fel a 2 x - 3 j + z = 28 s 2jc-3j + z = 0 skokkal prhuzamos, tlk
egyenl tvolsgra lev sk egyenlett!
2.141. Elvlasztja-e a 2x + 2y z 2 = 0 sk az A(2; 1; 1) s a B(2; 1; 3) ponto
kat?
2.142. ija fel az AB szakasz felezmerleges skjnak az egyenlett, ha
^(3; 5; -4 ) , 5(1; - 3 ; 2)!
2.143. Hatrozza meg annak a sknak az egyenlett, amely a 4x + 2y z + 5 = 0
s 4x + 2>- z - 1 = 0 prhuzamos skoktl egyenl tvolsgra van!
2.144. rja fel annak a sknak az egyenlett, amely illeszkedik az yl(6;0;0) s
B(0; 4; 0) pontokra, tovbb a koordintaskokkal egytt 8 egysg trfogat tetra
dert hatrol!
Egyenes s sk
2.145. Mely pontokban metszi a z = 4 - 2 t , y = 3 + 3t,z = 1- egyenes a koor
dintaskokat?
2.146. Adja meg a P( 6; 6; 5) s Q{12; 6; 1) pontokat sszekt egyenesek
nek a koordintaskokkal val metszspontjt!
25
2.147. Mely pontban dfi az x = \ + 2t, y ?>t, z = \ + t egyenes a
2 x + hy + z 0 skot?
2.148. Hatrozza meg a Pj (2; 3; - 3),/*2(3; 2; - 2), P3(4; 5; - 6) pontokra illesz-
4 X y+3 4z + 6 ,
ked sk s a ------= -------= -------- egyenes D dfspontjnak a koordintit!
3 4 5
2.149. Egy hromszg cscspontjai: ^ ( - 2 ; 0 ; - l ) ,
C(l; 5; 3). lltson a hromszg A cscsban a hromszg skjra merleges
egyenest! Melyik pontban dfi ez az egyenes az x y + z = 0 skot?
2.150. Hatrozza meg az origt a P(8; - 2; - 6) ponttal sszekt egyenesnek s
a 3x - 2y + 6 z + 4 = 0 sknak az M metszspontjt!
2.151. A p paramter mely rtkre lesz a z x = - 2 + 3, - 2+pt , z = 3 - 2 t
egyenes prhuzamos az x 3j + 6z + 7 = 0 skkal?
2.152. Mekkora terlet hromszget metszenek ki a koordintaskok a
6 x I0y+ 5z = 30 skbl?
2.153. Mekkora trfogat derkszg tetradert metsz ki a 3x 4j + 6z 12 = 0
sk a koordintaskokbl?
2.154. Adottak az ^ = (0; - 6; -3), - 6), C(5;9; 12), P(7;0; -7),
Q(l; 12; - 7 ) s i ? ( - 5 ; 6 ; 11) pontok. Metszi-e az ABC hromszg skja a PQR
hromszg skjt? Ha igen, rja fel a metszsvonal egyenletrendszert!
2.155. Tkrzze a /*( 2;3;3) pontot az x = 3 + 4, j = 12 + 5, z = 2 + 3/
egyenesre! Hatrozza meg a tkrkp koordintit!
2.156. Tkrzze a P( 1; 2; 3) pontot az x + 1 = 3>> = 3z egyenesre! Hatrozza
meg a tkrkp koordintit!
2.157. Hatrozza meg a/*(3; 2; 1) pont 2x + 5 z + 12 = 0 skra vett tkrkp
nek a koordintit!
2.158. Tkrzze a P ( - 3 ; 4 ; 7 ) pontot az ^(3;8;0), 5 ( - l ; - l ; - 2 ) ,
C(5; - 1; 10) pontok skjra! rja fel a tkrkp koordintit!
2.159. Tkrzze a z x - j - z +8 = 0 skot a P(5; 8; 2) pontra! rja fel a tkrkp
egyenlett!
2.160. Adott 4x = 2(y1) = 12zegyenes, azx + 2>>z = 12 sk, valamint
a P(3; 1; 5) pont. Tkrzze a P pontot az adott skra! rja fel a tkrkppontra
illeszked, az adott egyenessel prhuzamos s az adott skra merleges sk egyenlett!
26
2.161. Tkrzze az x= - 3, j = 3 - 3, z = - \ + t egyenest az ABC skra, ahol
^ ( 5 ; - 2 ; 5), fi(3;5;0) s C ( - 2 ; l ; 3 ) . Hatrozza meg a tkrkp egyenlet
rendszert!
2.162. Tkrzze az x = \ + 2t, y = l - 2 t , z = \ - t egyenest a 3x + = 1 skra!
Mekkora szget zr be az adott egyenes a tkrkpvel?
2.163. Tkrzze a 5x 8 y + 3z 6 = 0 skot az 5x 8j+3z + 4 = 0 skra s rja
fel a tkrkp egyenlett!
2.164. Adja meg az ^(5; 2; - 1 ) pont merleges vetlett a I x - y + 'iz = 23 s
kon!
2.165. Vettse merlegesen az ^ ( l ; - 2 ; - l ) , ~ 2 ) s C ( - 2 ; 6 ; - 6 )
pontok skjra az x 3 = y+3 = 6z 48 egyenest! rja fel a vetlet egyenlet
rendszert !
2.166. Adott az X = 4 +1 , y = \ 2t, z = t egyenes s a 2x 2z = 23 3y
sk. rja fel annak az egyenesnek az egyenletrendszert, amely prhuzamos az adott
egyenesnek az adott skra es merleges vetletvel s illeszkedik a P(7; 1; 2) pontra.
Az gy kapott egyenes illeszkedik-e a 4x+6;^ 4z 26 = 0 skra?
2.167. Adott az e i ' . x = 2 + 4t, y = 2t, z = 2 + s az e2 '.x = 2 t,
y = \ +1 , z = 1+ 2? egyenes. rja fel annak az S sknak az egyenlett, amely
tartalmazza az egyenest s prhuzamos az 6 2 egyenessel! Vettse az 6 2 egyenest
merlegesen az S skra! Hatrozza meg a vetlet egyenletrendszert!
2.168. Szmtsa ki az albbi egyenesek hajlsszgt;
a; 2 x - 6 = - 2 0 + 2) = s 2x + 4 = 2 y-6 = |/2(z + 5);
> J x = 2 + 3, j = 0, z = 3 s x = l + 2, ;^ = 0, z = 3 + ;
c ^ x = 4 + , j ; = 5 , z = 3 s x = 1 7 + 2/, j = 3, z = 9+17;
d) X= 1+ 3/, = 2,5t\ z = - I -3? s x = t, y = 3>+ 2t, z = - 5.
2.169. Hatrozza meg a kvetkez sk s egyenes hajlsszgt:
a) x y z = 1 s 2x= 14, ;^=3,z = 2;
b) x + 9y + 4z = 7 s x = 3 + 4t , y = 6 , z 9t;
c) x + 2y + 2z = 3 s x = 5 3, j = 4 + 6, z= 2;
d) 2x + y z = 3 s x = 5 t, y = 2t, z = 3 t.
2.170. Hatrozza meg a kvetkez kt-kt sk hajlsszgt:
a) l x - 3 y + z - l 9 = 0, x + 2 y - z + 4 = 0;
b) x y z l = 0 , 2 x + y z 5 = 0 ;
c) x + 2y + 2 z - 3 = 0, 1 6 x + 1 2 j - 1 5 z - 1 = 0;
d) X 2j + 2z 8 = 0, x + z 6 = 0.
27
3 z y + 1 z 1
2.171. Adott a P(2; - 1; 1) pont, az : ----- = -------= - - - s az 6 2 '. x = - 1,
Z ^ Zr
= - 3 - 5, z = 4 + 4? egyenes. A P pont s az egyenes skot hatroz meg. Az 6 2
egyenes ezzel a skkal vagy az egyenessel zr be nagyobb szget?
2.172. Mekkora tvolsgra van a P(2; 4; - 6 ) p o n t a 3 x - 6 = 2 ( j + l ) = 9z egye
nestl?
2.173. Mekkora az x = 2+3, y=t , z = 1+ 3? s az x = 2 + 3t, y = 1+ ,
z = - 4 + 3 prhuzamos egyenesek tvolsga?
2.174. Mekkora tvolsgra van az ^4(4; 3; 2) pont a6x ;^+18z= 19 sktl?
2.175. Hatrozza meg az x + 2y~3z = 1 sk s az x = l - l t , y = 2t, z = l - t
egyenes tvolsgt!
2.176. Mekkora tvolsgra van egymstl az albbi kt prhuzamos sk:
2x-3y + 6z-14 = 0, 4x-6j+12z + 21 = 0?
X1 y + 2 z 5
2.177. Szmtsa ki a z ----- = -------= ------ s x = 7 + 3/, j = 2 + 2, z = 1-2?
2 - 3 4
egyenesek metszspontjainak a 2x - 16j - 13z + 31 = 0 sktl mrt tvolsgt!
2.178. Hogyan kell megvlasztani A s B rtkt, hogy a 2x j + 321 = 0,
x + 2 y - z + B = 0, x + A y - 6 z+ 10 = Csikknak aj egy kzs pontja legyen; egy
egyenesre illeszkedjenek; c) hrom prhuzamos egyenesben messk egymst?
2.179. Hatrozza meg az x + j 2z1 = 0 s az x + j^ 2z + 3 = 0 skoktl
egyenl tvolsgra fekv sk egyenlett!
2.180. Az X= - 5 + 3, y = 2t, z = 1+ egyenesnek mely pontja van egyenl
tvolsgra az ^(2; 4; 7) s 5(0; 6; 5) pontoktl?
2.181. Adja meg egy olyan egyenes egyenletrendszert, amely az x - 8 j + 4z = 9
sktl 4 egysgnyire, a 4x + 20y 5z = 42 sktl pedig 3 egysgnyire van!
2.182. rja fel annak a sknak az egyenlett, amely az x + y - z + l = 0 sktl
ktszer olyan tvolsgra van, mint az x + y - z - l = 0 sktl s nem helyezkedik el
kzttk!
2.183. Hatrozza meg a z tengelyen azt a pontot, amely egyenl tvolsgra van
a 12x + 9>20z19 = 0 s a 16xI2y+ 15z 9 = 0 skoktl!
2.184. Hatrozza meg az x +y + z - 2 = 0 s x + 2 y - z - l = 0 skok metszsvo
naln azt a pontot, amely egyenl tvolsgra van az x + 2 y + z +l = 0 s
x + 2 y + z 3 = 0 skoktl!
2.185. rja fel annak a sknak az egyenlett, amely felezi az x + 2 y - z - l = 0 s
a z x + 2 y + z+ 1 = 0 skok hajlsszgt s illeszkedik a /*(1; 1; - 1) pontra!
2.186. Hatrozza meg az x y = z s x 1 = 0, y 2 = 0 egyenesek tvolsgt!
X3 y x + 2
2.187. Szmtsa ki az = - - = - ( z + 2) s = y - 2 = z - 4 egyenesek
tvolsgt!
2.188. Hatrozza meg a kvetkez skprok metszsvonalainak a tvolsgt:
x + y - z 1 = 0s2x + y z 2 = 0, illetve x + 2y - z 2 = 0 sx + 2y + 2z+4 = 0.
2.189. Hatrozza meg az x = 4 + 2 t , y = 2 + , z = 2 + 3 s az x = 4 + 2,
y = l - 5 , z = 12- 4 egyenesek normltranszverzlisnak egyenletrendszert!
2.190. Adott az egyenes az i(2; 4; 5) pontjval s Vi(1; 3; 1) irnyvektor
val s az >2egyenes az E2 (S; 9; 7) pontjval s 2(1; 3; 3) irnyvektorval. rja fel e
kt egyenes normltranszverzlisnak egyenletrendszert, a normltranszverzlis s
a kt egyenes metszspontjainak a koordintit s a metszspontok tvolsgt!
2.191. Hatrozza meg a 2x-t-5y- 3z-I-8 = 0 s az x-2_y-l-z-5 = 0 skok met
szsvonalnak vektoregyenlett! rja fel annak az egyenesnek a vektoregyenlett,
amely prhuzamos az elbbi metszsvonallal s illeszkedik a P(2; 1; 6) pontra. lla
ptsa meg a kt prhuzamos egyenes tvolsgt!
2.192. Egy tetrader cscspontjai: ^ ( - 2 ; 1 ; 3 ) , 5 ( 0 ; - 4 ; 6), C ( - 5 ; - 3 ; 5 ) ,
Z)(15; - 3 ; 3). rja fel annak a sknak az egyenlett, amely prhuzamos a tetrader
ABC lapjval s illeszkedik a D cscspontra! Mekkora a tetrader D cscsbl
kiindul magassga?
2.193. Egy hromszg cscspontjai: A(l; - 2 ; 0), B( - 2; 4; 6), C(2; 0; 2). A h
romszg BC oldalnak felezpontja F, az AB oldalhoz tartoz magassg talppont
ja T. Szmtsa ki az FT tvolsgot s rja fel az FT egyenes vektoregyenlett!
2.194. Hatrozza meg annak a tetradernek a trfogatt, amelyet a
3x-l->- 2 z = 17 s az x-l-2y = 9 skok egysgnyi hosszsg normlvektorai s a
skok metszsvonalnak^egysgnyi hosszsg irnyvektora feszt ki!
2.195. Mutassa meg, hogy az x = 1-?,' y = ~ 4 - 4 t , z = - 3 +1 s
r(t) = (t+ 2)i + (2t 2)j + (2t1)k egyenesek egy skban vannak! Vettse merlegesen
e skra az x = + 3, j = - 2 + 2, z = 6 egyenest! rja fel a vetlet egyenletrendszert!
Mekkora szget zr be az egyenes a vetletvel?
2.196. Az x + 3y + 4z 9 = 0; y 2 = 0; x + 4y + 5zl l = 0; y + 2z 4 = 0 s
kok egy tetradert hatroznak meg. Szmtsa ki a tetrader trfogatt s felsznt!
2.197. Az .4(1;0;2), 5 ( - l ; 2 ; 1), C(3;2;0), D ( - 2 ; 2 ; 1) pontok egy tetrader
cscspontjai. rja fel a tetrader lapjainak egyenlett! Mekkora a tetrader A cscs
bl kiindul magassga?
2.198. Mekkornak kell a p paramtert megvlasztani, hogy az ri() = (1 - +
+ (2 - ?)j - (1 + 3/)k s az T2 = ( - 1+ pt)i + ( 3 - - n j + k egyenesek egy skot hat-
4 ^
rozzanak meg? rja fel a sk egyenlett! Mekkora tvolsgra van ez a sk az origtl?
2.199. Hatrozza meg az r() = (3-t-l)i + (5+l)j-(2-t-l)k egyenesen azt a
P pontot, amely a 2x 2j + z 6 = 0 s a 4x 4y + 2z + 24 = 0 skoktl egyenl
tvolsgra van! Mekkora ez a tvolsg?
2.200. Hatrozza meg az x + 4y - 2z = 0 s az 5 x - j + 6z = 9 skokkal prhuza
mos, a P(1; - 2; 3) pontra illeszked egyenes egyenletrendszert! Bizonytsa be, hogy
19
ez az egyenes s az x = 3t + 6 , y = 2 - 14, z = t egyenes egy skot hatroznak
meg! rja fel a sk egyenlett! Mekkora tvolsgra van a P pont az els adott sktl?
3. LINERIS ALGEBRA
Determinnsok
3.1. Szmtsa ki a kvetkez determinnsok rtkt!
a)
D =
1 1 1
2 2 2
1 1 1
2 2 2
1 1 1
2 2 2
b)
D =
^ - i 1 2
3 1 - / 2
3 1 2 - /
, ahol
2 _ _
1;
c)
D =
j k
-1 4
0 2
, ahol i,j, k tetszleges vals szmok;
d)
f )
3 1 - 1 2
e)
- 1 2 5 4
- 5 1 3 - 4 1 3 4 5
Z) =
2 0 1 - 1
9
D =
- 1 4 3 2
1 - 5 3 - 3 0 5 2 3
- 4 - 1 2 3 1
2 3 0 4 - 1
Z) = 0 - 3 0 5 3
- 1 - 1 0 1 - 1
0 - 4 - 2 - 1 - 2
3.2. Igazolja, hogy
X x + d x + 2 d
y y + d y + 2 d
z z + d z + 2 d
= 0.
31
3.3. Hatrozza meg x rtkt, gy, hogy teljesljenek a kvetkez egyenlsgek:
= 0;
= 0.
a)
x^ 4 9
b)
X 1 1
X 2 3 = 0; 1 X 1
1 1 I 1 1 X
c)
3.4. Igazolja, hogy
X 0 0 d) x^ 3 2
0 X 0 - 729; X - 1 1
0 0 X 0 1 4
minden vals x-re!
3.5. Legyen
sin 2 x cos 2 x 1
sin X cos X cos x
cos X sin X sin x
Mtrixok
= 0
'1 -3 - 2
A = 3 4 s B = 1 - 5
5 6 4 3
Hatrozza meg az A + B mtrixot!
3.6. Adott kt mtrix;
A =
4 3 6
4 3
5 0 1
s B = 5 0
6 1
Vizsglja meg, melyik igaz az albbi lltsok kzl:
a) A = B; b) AB = E, ahol E az egysgmtrix; c) A* = B; d) A = B*.
3.7. Legyen
A =
Hatrozza meg az lA 2B + 3C mtrixot, ha = 1!
'1 1 - / '
; B =
4/ \ + 2 -2/ 1
2i 3 _ 4 3 - i
s C =
_ 1 l + _
32
3.8. Egy A mtrix transzponltja:
A* =
Igaz-e, hogy AB = [7, 4]?
3.9. Legyen
1 2 4
3 1 0
tovbb B = [1, 1, 1],
A =
cos a sin a
sin a cos a
Szmitsa ki A ngyzett!
3.10. Szmtsa ki az A^ mtrixot, ha
A =
1 - 2 - 6
- 3 2 9
2 0 3
Ltezik-e (A^)
3.11. Adott kt mtrix:
A =
Vizsglja meg, hogy az AB mtrix egyenl-e a BA mtrixszal!
3.12. Szmtsa ki a B = AA* + 2A*A mtrixot, ha
A =
'1 2
"1 2 A
3 1 0
s B = 2 3
4 0
1 - 2
2 - 1
3.13. Legyen
Vlassza meg c rtkt gy, hogy az AB szimmetrikus mtrix legyen!
3.14. Hatrozza meg az AB mtrixot, ha
' 1 - 1 r l 2 3
A = - 3 2 - 1 s B = 2 4 6
- 2 1 c 1 2 3
b)
A =
A =
1 2 1'
4 0 2
1 4 - 3 '
2 0 1
- 1 3 - 5
3 - A
, B = 1 5
- 2 2
B =
3 Matematikai feladatok
33
C)
d)
'2 r
A = 3 - 2 B =
1 0^
4 2 - 1 2
A = 3 - 7 1 - 8
2 4 - 3 1
5 1 - 2 3'
0 2 3 1
B =
2 3'
- 3 0
1 5
3 1
3.15. Igazolja, hogy (A + B)^ = A^ + BA + AB + B^, ha
A =
5 - 3 '
B =
l 4
2 1_
s
_ 2 -3_
3.16. Igazolja a mtrixok szorzsnak asszociativitst, azaz az (AB)C = A(BC)
sszefggst, ha
3 1 0' 2
A = [2, - 5 , 4 ] , B = - 2 2 5 s C = - 4
4 1 - 3 7
3.17. Legyen
A =
2 0 7
- 1 4 5
3 1 2
0 - 2 5 '1 0 0'
3 8 - 1 mtrixot a C = 0 c 0
1 - 4 6 0 0 1
Szmtsa ki az Aadj A s az (adjA)A szorzatot!
3.18. Szorozza meg az
A =
mtrixszal balrl, majd jobbrl (c tetszleges vals szm)!
3.19. Szorozza meg az
A =
mtrixszal balrl, majd jobbrl is! Mit tapasztal?
3.20. Legyen
Elvgezhet-e az A A ^ mvelet?
34
1 8 0 '0 1 0'
- 3 5 - 1 mtrixot a P = 1 0 0
1 - 2 2 0 0 1
'6 - 1 4
A = 4 1 2
7 - 2 5
3.21. Legyen
A =
- 3 - 2 0'
0 3 2
- 2 0 1
Szmtsa ki a megadott mtrix inverzt, s ellenrizze is a kapott eredmnyt!
3.22. Legyen A egy ngyzetes mtrix. Mikor igaz, hogy AadjA = E?
3.23. Adott az
A =
I 6 9
0 - 1 - 8
0 0 - 1
mtrix. Fennll-e a kvetkez egyenlsg = A(adjA) = A ^A.
3.24. Vizsglja meg, hogy teljesl-e az AB = BA egyenlsg, ha
2 1 - 2 " 0 1 - r
A = - 2 4 4 , B = 1 3 5
1 3 3 2 2 1
Ltezik-e (AB BA)
3.25. Hromfle nyersanyagbl (N) ktfle flksz termket (F), majd ezekbl
hromfle vgtermket (V) lltanak el. Az egyes folyamatok anyagszksglett az
albbi tblzatok mutatjk:
a) N, N, N,
F.
2 0 1
Fi
1 4 0
b) F, F,
N,
2 3
N, 0 3
N,
2 1
Vi V, F3
Fi
2 0 4
Fi
1 1 0
F, F2
Vi
0 2 .
V2
3 0
V,
2 1
Hatrozza meg a teljes folyamat termelsi mtrixt (az egyes vgtermkek nyers
anyagszksglett) !
3.26. Egy zem ktfle nyersanyagbl (A^i; N2 ) hromfle flksz termket
(Fi; F2 ; F3), ezekbl pedig ktfle vgtermket ( F i ; V2 ) llt el. Az anyagszksglet:
F, F2 F3 Fi F2 Fs
1 4 5
? Fi
1 5 4
N2
2 3 6
^2
3 3 1
Mekkora az egyes vgtermkek nyersanyagszksglete s a teljes nyersanyagszk
sglet, ha Fi-bi 1000 db, Fj-bl 1200 db kszl?
3* 35
3.27. Egy zemben ngyfle alapanyagbl (A^; A2 ; A3 ; A4 ,) hromfle flksz
termket (F^; F2 ; F^), ezekbl pedig ngyfle ksztermket ( ^ 1; K2 , K^ ', lltanak
el. Az anyagszksglet a kvetkez:
Ar
A2
^3
A,
F, F2 ^3
F,
1 5 2 3
K,
3 2 2
F2
2 0 4
3 , K2 3 2 4
F,
4 1 4 0
Ks
2 0 5
K, 3 3 1
Melyik ksztermk tartalmazza a legtbbet az Aj alapanyagbl, illetve melyik a
legkevesebbet az ^ 2bl?
3.28. Egy zemben ngyfle alapanyagbl; Ai-; A2 -', Ay; A^-bl hromfle fl
ksz termket; Fi-et; F2 -t; F^-at, ezekbl pedig ngyfle ksztermket; K2 -t;
A^j-at; ^ 4-et lltanak el. Az anyagszksglet a kvetkez;
A, A2 A,
A4 Fi
Fz
Fs
F,
2 1 3 4 2 4 2
Fi
4 1 1
0 ,
K2
3 5 0
F3
3 5 0 3
K, 1 6 1
K, 2 2 3
Melyik ksztermk tartalmazza a legtbbet az A^ alapanyagbl, illetve melyik a
legkevesebbet az J 3-bl?
3.29. Egy zemben ngyfle alapanyagbl (Ai; A2 ', A^; AJ hromfle flksz
termket (Fj; F2 ; F^), ezekbl pedig ngyfle ksztermket \,K2 ',K^\ KJ lltanak
el. Az anyagszksglet a kvetkez;
Ai A2 A, A^
F2
F,
F,
4 0 2 1
K,
3 3 1
F2
1 3 4 2 , K2 0 4 2
F,
2 6 1 5
K3
4 0 2
^4
2 3 1
Melyik ksztermk tartalmazza a legtbbet az A^ alapanyagbl s melyik a legkeve
sebbet az / 4-bl?
3.30. Hatrozza meg a kvetkez mtrixok rangjt!
a)
A =
1 2 3
b)
0 2 3
- 1 0 2
: B = 0 4 6
0 1 0 0 6 9
36
Cj y d) "1 3 - 3 - 2 - 1 r
C =
5
\ D =
1 - 2 - 1 1 1 2
4 0 2 - 1 1 2 - 1
3 0 1 3 3 1 - 2
Lineris egyenletrendszerek
3.31. Oldja meg a Cramer-szably felhasznlsval a kvetkez lineris egyenlet
rendszereket a vals szmok halmazn:
) Xi + 4x2 + 2x3 = 5,
3xj + 2 X2 ^3 = ~ 15
4 x i - X2 ~ X3 = 2;
c) x ly+TZ Av = 14,
3x+ y - z+ V = 0,
- x + 4y+3z- 2v = 20,
5 x - 2 y - z - 3 v = 4;
e) 5xi + 3x2 + 4x4 = 7,
5X2 + -^3 + 6X4 = 30,
Xi+ X2 + X4 = 7,
4xi + 2x2+ 3x4 = 10.
b) x+ y z = 2 ,
x 2y + 3z= 5,
x + 3;^ + 4z = 6;
dj 2 x+ y - 5 z+ u= 8,
x - 3 y - 6 m= 9,
2 y - z + 2 u = - 5,
x + 4 y - l z + 6 u= 0;
3.32. Oldja meg a termszetes szmok halmazn az albbi egyenletrendszereket;
a) 2 x - y + 3z = 3,
3x + y - 5 z = 0,
4 x - y + z = 3;
b) 2x-4y + 3 z - l = 0,
x - 2 j + 4 z -3 = 0,
3 x - y +5 z - 2 = 0.
3.33. Oldja meg az albbi egyenletrendszert az egsz szmok halmazn:
2x+3y+5z = 13,
x + 2y + 4z = 10,
2x+ y+3z = II,
x+ y+ z = 3,
3x + 2y+ z = 5.
3.34. Oldja meg a racionlis szmok halmazn az
Xi ~ X2 - 2 X3 + X4 = 1,
2 xi + X2 + X3 = 2,
- X i ~2X2 +X4 = - I,
3xi 2x2 + 2x3- X4 = 4
egyenletrendszert!
37
3.35. Oldja meg a kvetkez homogn lineris egyenletrendszereket a vals sz
mok halmazn;
a) x + ;;+ z =
0,
J
6 x 4z =
=0,
X- y + 3z =
0, 4x + 2 z ==0,
2 x - 3y - z =
0;
I x - l y ^ 5z =
=0;
c) X i -
X2 + X3
=
0,
d)
X
- +
1 _
z
0,
X i + X2 + 3 x 3 =
0,
2 3 4
X i -
3X2 ~ ^ 3
=
0;
X z
0,
3 ~ 4 2 ~
X 3z
A
6 ~ 12 T
U.
3.36. Rangvizsglattal dntse el, megoldhatk-e a kvetkez lineris egyenlet
rendszerek :
a ) X i - 2x 2+ ^3 = 2, b)
Xi + X2 + X3 - X4 = 4,
3x i + 8x2 6x3 = 5, X1 X 2 + X3 + X4 = 8,
6x1 + 10x2 + 3 x 3 = 4 ; 3X1 + X2 + 3X3 + X4 = 16;
c)
12xi + 11x2 5x3 + 5x4
8,
- 4x i + 3x2 + 3x4 =
- 1 ,
8x1 + 6x2 + 3X3 + 2x4 =
5,
4X2+ X3 + 4X4 = 2.
3.37. Legyen
X - y + z = 0,
3x + 2y z = Cj,
2 x+ y + CjZ = 1.
Hatrozza meg s C2 rtkt gy, hogy az egyenletrendszernek
a) ne legyen megoldsa;
b) legyen megoldsa! (Adjon is meg egy megoldst!)
3.38. Oldja meg az albbi lineris egyenletrendszert:
tx+ y+ z =
x+t y+ z = t,
x+ y + tz =
(t vals paramter)! Vizsglja a megoldhatsg felttelt is!
3.39. Oldja meg inverz mtrix segtsgvel a kvetkez egyenletrendszereket a
vals szmok halmazn:
38
a ) X - y + 2z = 6, b )
2x+ y - z = 5,
3x + 2y+5z= 3;
c) - 3 x + y + 2z = - 2, d)
A x - 6 y - z = 17,
3x+ y - 6 z = - 14;
e) 2x+3y- 2z = - 5 , f )
3x + 2y+5z= 5,
5x + 3y-Az = - 6;
Xj + 5X2 + 6X3 = 10,
Sxj 2^2 + 3x3 = 29,
1x i + Ax 2 ~ 3 x ^ = -3;
4x -2_k + 3z = 13,
-5x + 4j + 2 z = 12,
6 x - 4 y - 5 z = -11;
3x + 2j 4z = 2,
2 x + 3y + 5z = 5,
l x - 2 y - %z = 10.
3.40. Oldja meg bzistranszformcival a kvetkez lineris egyenletrendszereket
a vals szmok halmazn:
a )
c)
x +2 y + z
= 4, b)
2 x j + X2
=
9,
x + y - z
= 3,
3xj + 2x2
=
14,
x + z
= 2;
Xi + X2 + 3x3 + 4X4 10,
2 x j - X2 + 2X3 + 3X4
11;
3^1
-
X3 + 2x4 =
0, d) x + 3y 3u =
1,
X2 +
X3 + X4 =
4,
x - 2 y + 3z- 4u
4,
2xi -
- X2
+ 2x4 =
1,
r z+ u =
3,
Xi
+ 5x3 + 6x4 = 16,
- l y +3 z + M= - 3.
Xj + 2X2 "2 x 3 - 3x4 =
- 5 ;
3.41. Hatrozza meg az adott lineris transzformci sajtrtkeit, illetve sajt
vektorait :
a ) 2 0 - 3
b )
'1 0 2'
A = 0
2
3 ; A = 0 1 0
- 3
3
- 2 0 0 2
39
4. KOMPLEX SZAMOK
4.1. brzolja a komplex szmsk azon z pontjainak halmazt, amelyekre teljesl:
a) Rz > 2; b) 2 < I mz < 1;
c) Rez + Imz = 1; d) \ < \z\ < 2.
4.2. rja fel trigonometrikus alakban a kvetkez komplex szmokat:
a) 1; b) - 8 ; c) i; d) - 2 / ; e) 1+ /; f ) - 2 + 2/;
g) - 3 - 3 / ; h) j/S-/; i ) 2 + i; j ) - 1 - 2 / ; k) 3 + 4/; l) - 2 + 5/.
4.3. rja fel exponencilis alakban a kvetkez komplex szmokat:
a) A- b ) 2 i; c) - 1 0 ; d) - f 2 - ^ i ; e) - i ;
4.4. Vgezze el az albbi komplex szmokkal kijellt mveleteket algebrai alak
ban:
a) (2- /) + (5-4/); b) (3 + 40 + (3-4/);
c) ( 2 - / ) - ( 5 - 4 0 ; d) (3 + 4/)- (3-4/) ;
e; (4+/)(5-2/); f ) ( - l + 2/)(5 + /);
4 + / - 1 + 2/ 5 + 2/
J) /!; Ar; (2-3/)3;m; (l + / f ;
^( 2- / ) ^( - l +2/ ) , / 1 + 2/V
n ) ------ z - .------ ; o) - :
3- 4? \ 3 - i
4.5. Legyen = 3 + 4/, Zj = - 2+5/ . Szmtsa ki
a; Z1Z2; b)
^ 2
rtkt algebrai alakban!
4.6. Oldja meg a kvetkez egyenletrendszereket a komplex szmok halmazn:
a ) (3- / ) x + (4 + 2/)> = 2 + 6/, b) (2 + )x + ( 2 - )y = 6,
(4 + 2i)x - (2 + 3> = 5 + 4/; (3 + 2i)x + (3 - 2i)y = 8;
40
c) x + i y - 2 z = 10,
x y + 2 iz = 20,
ix+2>iy {\ + )z = 30.
4.7. Milyen a Zj s Zj komplex szm egymshoz viszonytott helyzete, ha
a; Im(ziz2) = 0; 6; Re (zizj) = 0?
4.8. Oldja meg a komplex szmok halmazn a z = 0 egyenletet!
4.9. Hatrozza meg a kvetkez kifejezs rtkt
- ...
1 + / + / ^ + / ^ + . . . +1
4.10. Szmtsa ki a z = 2 + 3/ komplex szm tdik hatvnyt algebrai alakban!
4.11. Vgezze el az albbi mveleteket a megadott komplex szmok trigonometri
kus alakjval:
a) (3 + / ) ( - 2 + 20; b) (2 + 30(1 + 5/);
2 + 5/ 5 + 4/ -----
2- 4/
^ ------------------- , / 3 + /
f ) / r + ^ ; 1/(1 + /) ( - 2 + 5/); /zj ] / : ^
(2 + /)3-
4.12. Vgezze el a kijellt mveleteket:
^ l + / t ga , ^ , a + bi ^(l + 2/ ) ^ - ( l - 0^
l - / t g a ' ^ a - W (3 + 2/)3-(2 + /)^
(1 + 0^ , (1 + /)"
( 1- / ) ^ ^ \ \ - i T
d) T, _ ..-2 (n termszetes szm)!
4.13. A (cos a + / sin a)^ kifejezst vizsglva fejezze ki cos 3a-t s sin 3a-t cos a s
sin a segtsgvel!
4.14. Legyen Zj = 8- 8/; Z2 = |^(sin 225 + / cos 225) s Z3 = -8(cos330-
/ sin 330). Szmtsa ki a kvetkez rtkeket:
Zi
aj z = (Zi - Z 2) (algebrai alakban);
Z2
41
b) Z2 (exponencilis alakban);
3
c) fTj, (trigonometrikus alakban).
cos----- H/ sin
4 4
4.15. Legyen n termszetes szm. Igazolja, hogy
a ) (1+ 0" = 2^
r- nn mi\
j ( p /)" = 2" cos------isin );
V 6 6/
a / na no\
c) (1 + cos a + i sin a)" = 2" cos" - 1cos + / sm j .
4.16. Legyen z = 2 2i. Adja meg z kbgykeinek sszegt!
4.17. Oldja meg az albbi egyenletet a komplex szmok halmazn s a gykket
fejezze ki mind a hrom alakban:
a;z^ + 4 = 0; *;z^+16 = 0; cJ z^ + 8 = 0;
/ ; z ^ + l = 0; ej z^ - 1 0 0 = 0 .
4.18. Szmtsa ki az
a ) z ^ - i = 0 ; b ) z * + \ = 0
egyenletek gykeinek szorzatt!
4.19. Oldja meg a komplex szmok halmazn a z^ + f{n 135 + /cos 135) = 0
egyenletet! Adja meg az eredmnyt trigonometrikus alakban!
4.20. Oldja meg komplex szmok halmazn a kvetkez egyenleteket:
a) z'^ + 4z^ + 8 = 0 (adja meg a gykket algebrai alakban);
b) z^ + 4z^ + 5 = 0 (a megoldst trigonometrikus alakban krjk);
c) z^ + 7z^ 8 = 0 (adja meg a megoldst exponencilis alakban).
4.21. Hatrozza meg
a)T-, b ) { - \ ) ' - c ) f - d) f \ - e ) f ^ i - f ) f i
rtkt!
4.22. Szmitsa ki z rtkt s az eredmnyt fejezze ki algebrai, trigonometrikus s
exponencilis alakban:
Z0Z1- Z 3
z = ----------- ,
ahol
42
a ; z o = l + /; Z i = - 1 + /; Z2 = - l - i ;
b) zo = ] / 2 + i]f2 ; Zi = - ] / 2 + i2 ; Zj =
5n .
4.23. Legyen z^ = 2 2/; Zj Z3 = l'^sin 135 + cos 135) s Z4 = 2. Sz
mtsa ki az albbi kifejezseket, s a vgeredmnyt algebrai alakban adja meg:
1
+ z,
j z = -------- ; ; z = ----------- .
1 , . ^2^3
+Zi
^2
4.24. Oldja meg a ziz2z'^ Z3 = z\ egyenletet a komplex szmok halmazn, s az
eredmnyt trigonometrikus alakban adja meg:
Zi = 4 + 4/; Z2 = cos 135-/sin 135;
,3n
Z3 = 1^ (cos 135 + /sin 135); Z4= - 2e ' ^
4.25. Oldja meg a
zf-z1-z3z^ = 0
egyenletet a komplex szmok halmazn s az eredmnyt adja meg trigonometrikus
alakban, ha
3 . ^ . n
Zi = 2^^e Z2 = 2(cos45- Sn45); Z3 = -2]/ 2e ^
4.26. Oldja meg a komplex szmok halmazn a kvetkez egyenleteket:
, 4)1 , n / 7 _ \ 3
^ / 77T 77t\^
aJ 8(p + 3i)e z + 2e = cos------i sin
V 6 6
( 4
bj (cos225-/sin225)z^ + 3 2 / - 0 , 5 j =0.
(A megoldst trigonometrikus alakban adja meg!)
4.27. Hatrozza meg z rtkt trigonometrikus alakban, ha Zj - (zj - z^)z' = 0
i *X
s = 64e ^ ; Z2 = l/2(cos 315-/ sin 315), tovbb z^= - f e
4.28. Adja meg z rtkt trigonometrikus alakban, ha = z\z^, ahol
. ^
Zi = 2|^(cos 225 + i sin 225); Z2 = i f l e ^; Z3 = 8(cos 240 - / sin 240);
Z4=
43
4.29. Szmtsa ki z rtkt trigonometrikus alakban, ha z\z^ = {z^ z-^z^, s
= 2 - 2 / ; ^2 = 2(cos30 + /sin30); Zj = z^ = 2(cos 270 - / sin 270).
4.30. Oldja meg a
Z'^Z\ + Z2 Z^ = Z\
egyenletet a komplex szmok halmazn, s adja meg a vgeredmnyt exponencilis
alakban, ha
5.
Zj = 2(cos 330 i sin 330); Z2 = 8e ;
z^ = f i - i \ Z4 = 4(cos 240 + i sin 240).
4.31. Hatrozza meg Z-t algebrai s trigonometrikus alakban a kvetkez egyen
letbl:
1 - 1 ^ 1 1
Z R iXi^ iX,
a) R=5; X^=4; =
b ) R = 4; Xj^=5; X=S.
4.32. Egy 65 ohm ellenlls, 0,2 H nindukcis egytthatj lgmagos tekercsen
3 A cscsrtk, 300 krfrekvencij vltram folpk keresztl. Hatrozza meg
a tekercs kapcsain a feszltsgklnbsget!
4.33. Kapcsoljunk sorba egy 4 ohmos ellenllst s egy 0,001 06 F kapacits
kondenztort! Hatrozza meg az ramkrben foly ram erssgt, ha az ramkrre
110 V cscsrtk 50 Hz-es vltakoz feszltsget kapcsolunk!
4.34. Hatrozza meg az albbi ktplusok ered impedancijt!
ru
R
a)
b) 0
44
5. SOROZATOK
Bizonytsa be a teljes indukci mdszervel a kvetkez egyenlsgek helyessgt:
5.1. l + 2 + 3 + 4+ . . . + n = ^ ^ ; = 1, 2 , . . . .
5 . 2 . P + 2=.3a + . . . H . 3 - * ^ l 5 i ) ; - , , 2 , . . . .
6
5.3. l^ + 2^ + 3^ + ... + ^ =
2
; = 1, 2, . . . .
5.4. 1^ + 32 + 52 + ... + (2-l)2 ^ (2_J)n(2jl^ _ ^ ^ 1,2,....
(+1) ( + 2)
5.5. 1 -2 + 2-3 + 3- 4+. . . + (+!) = n = 1,2, . . . .
(/i+l)(/i + 2)(n+3)
5.6. l - 2- 3 + 2- 3- 4 + ... + (+l)(/i+2) = -^^----- , ----- = 1,2,....
4
5.7. 1 l! + 2-2! + 3-3! + ... + n-/i! = ( + l ) ! - l ; n = 1 , 2 , . . . .
5.8. 1 -4 + 2-7 + 3- 10+... + (3/i+l) = (+l)^ /i = 1 , 2 , . . . .
5.9. (1 + 1) ( ^1+^ ^ ^ 1+^ ^ . . . ( ^l +^^ = + l ; 1 , 2 , . . . .
45
5. 11. 1 1 - ^
^ - 6 ^ - T o
1 1 1 1
5.12.-----+ ------ + ------ + . . . + ---------- = -------; = 1 , 2 , . . . .
1-2 2- 3 3- 4 n{n+\) n+\
1 1 1 1 n( + 3)
5 .1 3.-------- + ----------+ ----------+. . . +
1- 2- 3 2- 3- 4 3- 4- 5 " n(n+l)(n + 2) 4(n+l)(n + 2 )
n = 1, 2, . . . .
1 2 3 r>
5.14.--------: + + ------- - + . . . +
2- 3- 4 3- 4- 5 4- 5- 6 ( n+1 ) (n + 2 ) (n + 3)
n(tt+l)
4( + 2)(n + 3)
; n = 1, 2, . . . .
12 2^ 3^ n(+l)
5 . 1 5 .-------1" ~ + -------H... + -------------------= ------------j n 1,2,..
............................... (2/1-1) (2+l) 2(2+l)
1 n
n = 1, 2, . . . .
----- + +
1-3 3- 5 5- 7
1 1 1
+ +
1-3 3- 5 5- 7
1 1 1
----- + +
( 2 n - l ) ( 2 + l ) 2 n + l
1 n
5.17 . 1"-------1 ------ +... + -------------------- ------- ; / = 1, 2,...
1-4 4-7 7-10 (3-2)(3n+l) 3+l
5.18. 2-2^ + 3' 22 + 4-23 + ... + -2"-i = (n-l)2"; n = 2,3,... .
rja fel az albbi {a} sorozatok nhny elemt!
n 2
5.19. ; = 1 , 2 , ..., 6.
In
13n
5.20. f ; = 1 , 2 , ..., 5.
5+ 1
n + 2
5-21. T- ; = 3,4, ...,8.
n^-Zn + 2
n^ + n+l
5.22. a = n = l , 2 , ..., 6.
n+ 12
46
1/4n+ 1
5.23. a = H r - - , =1, 2, . . . , 6.
8n
5.24. ==
16
2 1
; = 1, 2 , 5 .
5.25. a:=2 + ( - l ) + i - ; n = 1,2,...,6.
n
5.26. + = 1, 2, . . . , 7.
5.28. a==l +^ + ^+. . . + ^; n= 1,2, ...,6.
;r
5.29. a == cos(+ 1) - ; = 1, 2,..., 7.
5.30. a =sin(2n- 1) - ; = 1, 2 , ..., 7.
5.31. ==
n
sin - ; n = l , 2 , . . . , 8 .
5.32. a ==
n
cos-
6
In
; = 1, 2, 8.
5.33. a == I
1
. = iA:(Ar+l)
; = 1 , 2 , . . . , 8 .
5.34. a:= X
1
,ei(3Ar-2) (3Ar+l)
ha pratlan,
; n = l , 2 , ...,8.
5.35. ha pros; = 1, 2, . . . , 10.
47
1, ha n pratlan,
5.36. ^ ^ ^ pros; n = 1, 2, ..., 10.
n+ 1
5. 37. a==l g ; = 1,2,...,8.
n + 2
6"
5.38. +x r ; n 0, 1, 2 , ..., 10.
n\
Tekintse az albb felsorolt szmokat egy sorozat egymst kvet els nhny
elemnek. Adjon olyan kpletet vagy utastst, amelyet a megadott sorozatelemek s
az n-edik elem kielgt! Dntse el, hogy a sorozat nveked-e, cskken-e, korltos-e,
van-e torldsi pontja, konvergens-e:
5 8 11 14
1 1 1
5.42, 1; 16; 81; 256; ....
5.43. fi- p-, A-,
5.44. 1; -1; 1; -1; ....
5.45. -13; -18; -23; -28; ....
5.46. 1,5; 1,25; 1,125; 1,0625; ....
ija fel az albbi rekurzv formulval megadott {a} sorozatok nhny elemt!
llaptsa meg, hogy a sorozatok konvergensek-e:
5.48. i = - l , a := -5-4a_i; n = 2, 3, 4, 5.
48
5.49.01 = 1, 02 = 3, a == 2 a_i + 5 _2; = 3, 4, 5, 6.
5.50.01 = 2, 2 = 3, := a_i-a_2; = 3, 4, 5, 6, 7, 8.
5.51.01 = 3, == 2-------- ; n = 2, 3, 4, 5.
n- l
5.52.01 = 1, 02 = 2, = 3,4,5,6.
5.53. i = l, 02=1. +i == + n = 2, 3, 4, 5, 6.
n - l + 5
5.54.01 = 4, a:= ; n = 2,3,4,5.
5.55.01 = 1, a = ^* + 1; n = 2, 3, 4, 5.
^n-1
5.56. i = l, a+i==a^-l; =1,2, 3, 4, 5.
( - 1 ) "
5.57.01 = 0, a + i - = a + ; n = l , 2 , 3,4,5.
Igazolja, hogy az albbi {} sorozatok konvergensek! Szmtsa ki a sorozatok
hatrrtkt is!
1 1\
^n + l
2
+
j
1 a \
^11+ 1
a +
a j
==
1 /
, ( 2-i+ 2
3 V n
a - - -
-
4
- + 3
5
a =
- l
3
+ 2;
; = 0 , 1 , 2 , . . . .
; a>0; n = 0 , 1 , 2 , . . . .
; a>0; n = 2 , 3,
4 Matematikai feladatok
49
1
5.63.01 = 2 s a-=2-------- ; n = 2, 3, . . . .
,-
n - l
5.65. = 1^6, 2 = / + l/, j = / + l/ + l^,.... a == l^6+ a_i;
n = 2, 3, . . . .
5.66.01 = 1/^6+1), 2 = ^(Z>+l) + l/ft(Z)+l),..., == 1^6(6+l) + _i;
Z>eN; n = 2 , 3 , . . . .
Hatrozza meg a kvetkez {a} sorozatok legkisebb elemt:
5.67. a = = 2 - 9 n - 100; = 1 , 2 , . . . .
25
5.68. a- -= n-\ ; n = 1, 2,... .
n
5.69. a ~ 7 8 ; n = 1 , 2 , . . . .
5.70. ==3 + ^ + ^ ; =1 , 2 , . . . .
in
Hatrozza meg a kvetkez {a} sorozatok legnagyobb elemt:
T
5.71. := ; = 0 , 1 , 2 , . . . .
n\
5.72. a := 2-|/ - ; = 1 , 2 , . . . .
l/
5.73. a: = 20+ 19 - 2; = 1, 2, . . . .
9
5.74. a'= 2 - n - n = 1, 2,... .
rja fel az albbi {a} sorozatok legkisebb s legnagyobb rtkt:
n^ 2nn
-----rcos
l + 2 3
5.75. 7-;j c os = 1 , 2 , . . . .
50
n . nn
5.76. a == sm^ ; = 1 , 2 , . . . .
1+n 4
2nn
5.77. a =cos" ; = 1, 2,
nn
5.78. == sin" ; n = l , 2 , . . . .
6
Mutassa meg, hogy az albbi {a} sorozatok konvergensek! Hatrozza meg azt a
v(e) kszbindexet, amelynl nagyobb index elemek a sorozatban az elrt e>0-nl
kisebb hibval kzeltik meg a sorozat hatrrtkt!
5.79. ; = 1,2,3,...; =10 ^
2rt'- 1
5.80. a==2"; = 1, 2, 3, . . . ; =10-^.
" 4: 3^
; n = 1, 2, 3,...; =10 .
1 /
i.82. == V-
'2n+i
2 n
; = 1,2,3,...; =10^
l + 3
5.83. a==- ; n = l , 2 , . . . ; =10-3.
5+ 1
5.84. a ~ ^77+ 11/; = 1 , 2 , ...; =10
- 1
31^+2
5.85. a = - ^ ----; = 1 , 2 , ...; =10^
2|/n+ 1
5.86. a:= ; = 0, 1, 2, . . . ; =10-.
5.87. a ==
" 2 + 42
; 77= 1, 2, . . . ; = 10
5.88. a == Ig^q^ ; = 1 , 2 , ...; =10 ^
4*
51
Hatrozza meg az albbi {a} sorozatok hatrrtkt, amennyiben az ltezik;
10"
5.89. a== ; = 0 , 1, . . . .
100
5.90. a =----- ; n = 0, 1,... .
6 ni
T
5.91. a ~ -----------; = 0, 1, ... .
2"+1000
3+2_ 1
5.92. ; = 0, 1,....
5.93. a =- ; OO; = 0, 1,... .
10"+10^
5-9-' . - 5 . + 2-+10 '
T - l ~
5.95. == ; h = 0, 1,....
0 + 5)(1 + 5^)(1 + 53)...(1 + 5")
5.96. a==------------------ --------------------- ; = 1 , 2 , . . . .
5 ' 9 7 ' " " ~ ( i + c)(i + c^)(l + -3)...(l + c") . . . .
6" + c"
5.98. ; ,c>0; = 1 , 2 , . . . .
72 + 3+10
5.99. fl== ; n = l , 2 , . . . .
100n'- 2 7 + ;r
39n3 + 252-16 + 9
5.100. ==------------------------- j-; =1 , 2 , . . . .
263 36^18+ -
43 + 52 + 6m+ 7
5101-""'"^1-92-273 + 34
52
81n-85-122 + 3
9-172 + 3n3-9^ " ~
l/^ + 5^ - 8
5.103. a== 3-------------- ; = 2 , . . . .
4n^ + 2
^42 + 2+100
5.104. a==^^---------------- ; n = l , 2 , . . . .
^57j2-6n-10
pn^ + 2 + 2 n
5.105. a==j^^--------^---; = 1,2,....
|/3 + + 16-3/1
l/n^ +
5.106. ==j^^---------- ; = 1 , 2 , . . . .
l/2 + 3n-2
l/n^ + 3n + 36
5.107. ==^^--------------- ; =1 , 2 , . . . .
1/3-22 + 25
3 3
5.108. == l/TT-|/; = 1 , 2 , ....
5.109. a== 3------ ^j ; = 1 , 2 , . . . .
p n + \ - f n
12
5.110. == 3 ^-------; n = l , 2 , . . . .
|/7^-(|/n + 2 - l / n - 2 )
. . . . (2+l)(3 + 2)(4 + 3)(5n + 4)
5.111.= = ------------------------ ; =1 , 2 , . . . .
(2n + 3r(18+17V5
" (6 + S r "
(22 + 3 + 4)i (32-4 + 5)i
(62 + 7n-8) '
5.113. = ^ '">20 ^ 1, 2, . . . .
53
(+l)( + 2)( + 3)(/ + 4)
5.114. ==------- 2------;---------------- ; n = l , 2 , ....
n^ + 2n^ + 3fi + 4
Vizsglja meg, hogy mely eset(ek)ben konvergens az {a} sorozat, s konvergencia
esetben hatrozza meg a sorozat hatrrtkt is! ( ke N*==Nu{0}.)
1+ 2 + 3 + ... +
5.115. a==-------- -------- ; n = l , 2 , ....
l^ + 2^ + 3^+... + 2
5.116. ==------------ ------------; = 1 , 2 , ... .
rr
l3 + 23 + 3^ + ... + n3
5.117. a==------------ ------------; =1, 2, . . . .
rr
12 + 32 + 52 + ... + (2k - 1)2
5.118.0,==---------------- ^ n = l , 2 , . . . .
rr
[1 + 3 + 5 + . . . + ( 2n-l)]2
5.119. a==^-------------- ; =1, 2, . . . .
tr
(2 + 4 + 6 + . .. + 2)3
5.120. ==^^-------------^ = 1 , 2 , . . . .
rr
1-2 + 2-3 + 3- 4+. .. + (+!)
5.121. a==-------------------r------ n =\ , !,
rr
l - 4 + 2-7 + 3-10+. . . + n(3+l)
5.122. ==-------------------- --------------------; =1, . 2, ....
/
(1 + 1)
1\
v " 2 ,
1\ / 1
, . , 3 .
5.123. ==------------------- ^ ------------------- ; = 1 , 2 , ... .
Szmtsa ki a kvetkez {a] sorozatok hatrrtkt, amennyiben az ltezik:
1 1 1
1+ - + - +. . . +
2 4 2"-i
5.124. ==--- j---j-------- ; = 1, 2,....
1+ - + - + . . . + ---- -
3 9 3"-i
54
1 l! + 2-2! + 3 - 3! + ... + !
--------------------------- ^ .............................................
5.126. a- =
3 !
( +! ) ! + !
; n = 1, 2 , . . . .
9- (+l ) ! + 2-!
5.127. a =----------------------- ; n = 1, 2,... .
10-(+l)!-7!
6.1 2 *. 1 - 3,
/
1-
/
1 .
1 - - ) ; = 2 , 3 , . . . .
5.129. = (1 + 1)1 " = ^>2, . . . .
1 1 1
6/ V 10/ \ n{n-\-\)
; = 2, 3, . . . .
Igazolja, hogy az albbi {a} sorozatok konvergensek! Szmtsa ki a sorozatok
hatrrtkt is;
5.131. a : = l J ^ - l , b>l - n = l , 2 , . . . .
5A32. a:=hn-, =1 , 2 , . . . .
n+ l
5.133. a : = - ; = 1 , 2 , . . . .
] [ n !
5.134 a == 1 +
fTi
+12
\ n+ 12
; = 1, 2, . . . .
/
5.135. a 1), >0; ^ l ; = 1 , 2 , . . . .
5.136. a ==
n fi
^ + l/c'
, b,c>0; b ^ l ; c ^ l ; = 1 , 2 ,
55
5.137. a =
\ { r ,
1+ - \]fb-^c 6, c >0; c # l ; = 1, 2,
Szmtsa ki az albbi {a} sorozatok hatrrtkt, amennyiben az ltezik:
ln + ]/n + ]/n
5A38. a:=^ ^ = ; n = 1, 2, . . . .
5.139. a-=^n + ^n + ^n ]fn', n = l , 2 , . . . .
3 4 5
i/m+ |/+ |/+ |/
5.141. a n^(|/+ 12|/ + )/1); = 1 , 2 , . . . .
5.142. := + 1|/2^; n = l , 2 , . . . .
n\]j2n^ + %n-\ - l/2n^+16n+11)
5.143. a== ; = 1, 2, . . . .
5.144. a ~ 19 |/3n^ 25|/3+3^; n = 2, 3,... .
2 1^
5.145. a:= 7 ., , , ; n = l , 2 , . . . .
^n2+15n-10-|/2-6n + 5
5.146. a == + 5 7 2tj) ; = 1, 2, . . . .
5.147. a := 28 - + 28 - 71; n = l , 2 , . . . .
5.148. a == + i4 _ 21 - + i 4 + 71; n = 2, 3, . . . .
13l/2n+l
5.149. == , , , =; n = 1, 2,
1/I3n^+12-1/132-12
5.150. a--= pn^ + 2n- \ - 3n- , = 1 , 2 , ....
56
5.151. a = + = 1 , 2 , . . . .
5.152. --------- ; = 2, 3, . . . .
5.153. := n\]ln^ - A - n y , n = 2, 3, . . . .
5.154. a =sin [n i]/4n^ + + 1- + 2n)] ; = 1 , 2 , ..
5.155. a ==
5.156. a ==
5.157. a =
5.158. a ==
5.159. a ==
tg
= 1, 2,... .
pn^ + 4 n ^ - 2 n - l - 2 n ^
1+2 + 3+ ... +
1+ 3 + 5+ + ( 2 - l )
pn^ + 202 - l On - ]/5n* - 10 - 5
= 1, 2, . . . .
pn^ + 2n + 3 - p n ^ + 6 n + 5
pn^ + 5 n +l - p n ^ + 7 n - \
5.160. a =
36"
^3 + 7^ ^+3
!
; = 1, 2,... .
5.161. a==
5.162. a =
5.163. ==
5.164. :=
\3n + 4/
12 + 22 + 32+ ... +2
|/4-53 + 9-23
- 3 \ "
; = 1, 2,... .
? - 5
; = 1, 2,... .
n 5
'2 + 3\
2 ^ )
/4n + 5Y^"
v4-3/
; = 1,2,... .
; = 1,2,... .
57
5.165. a- =
5.166. a ==
5.167. =
5.168. a :=
V3n + 5y
2 + 4\"+2
3-6y
/^5n-6Y" +^
\4n + 3 /
n^ + l V ^ ^
\n^ + 3
J
5.169. a ==
5.170. a ==
5.171. ==
2^ - + 1
n = 1, 2, . . . .
n = 1, 2, . . . .
n= 1, 2, . . . .
= 1, 2, . . . .
= 1, 2, . . . .
n = 1, 2, . . . .
; = 1, 2, . . . .
/" + 2 + 4V^"'
5.172. ) ; = 1 , 2 , . . . .
V^- n+ 1)
n\ \ - n)
5.173. Legyen a =-----------. A Amely rtkre konvergens a sorozat? Adjon
5 n n
meg olyan Artket, amelyre lim = 1, illetve lim = 0.
5.174. Hatrozza meg a As egytthatk rtkt a
f n^+l \
lim -----; Xn-f i =0; = 1 , 2 ,
00 Y/? +1 y
felttel alapjn!
5.175. Hatrozza meg a X egytthat rtkt a
a + 5\"+
lim
n-^GOV 3
=
/
felttel alapjn!
58
5.176. Legyen
Mely b rtkekre konvergens a sorozat? Hatrozza meg a sorozat hatrrtkt is!
5.177. Legyen
a = \'n^ - Xn+\ - 2 + 3; n = 1 , 2 , ... .
Mely rtkekre konvergens a sorozat? Adjon meg olyan 2. rtket, amelyre
lim a=l, illetve lim a = 0.
->oo ->oo
Szmitsa ki a kvetkez hatrrtkeket;
5.178. lim
II->00
5.179. lim
n-* 00
5.180. lim
00
f n ^ - 1 )
V + l j
0
' r*'+fn + j n^ f n\
n^ + n + + n
; n= 1 , 2 , . . . .
e^n^ + n \ n^ + 2
+ iyn^-6n+ 5 ^4 _ 2 + g + - 1'
\ n ^ - 2 n + 4j n^+l
n= 1 , 2 ,
5.181. lim
->00
2 + 5^^
; = 1, 2, . . . .
/f? + 3n- n
5.182. lim 4------- TT-ir; = 1, 2, ....
4n + 9\^-^"
\4n + 5/
rja fel a sorozatok -edik elemt zrt alakban, majd llaptsa meg a sorozatok
hatrrtkt:
5.183. lim
n-*00
1 1
+ -----+ ... +
1
1-2 2-3 n(n+l)_
; n = 1 , 2 , ... .
59
5.184. lim
n00
5.185. lim
1 1
+ -----+...+
1
1-3 3-5
1 1
+ -------- + ... +
1-2-3 2 - 3 - 4 (+!)(+ 2)
; = 1, 2, . . . .
; = 1, 2,
5.186. lim
1
; = 1, 2, . . . .
5.187. lim X n= 1,2,....
k = 1
5.188. lim ( ^ ^ ^ - 2 l / ^ + 2 + l / ^ + I ) ; 1,2,....
k= 1
k = 2
5.189. lim X lg ( 1 - ^ ) ; = 1, 2, . . . .
5.190. lim X ; n= 1 , 2 , ....
1 2-3 + 2 - 3 - 4 + . . . + (+l)( + 2)
5.192. lim
00
2 + l
i \ /
(1 + 1) 1+ -
2
n
1+ -
3y - - i ) '
12 2^
+ ----- + . . . +
1-3 3-5 , ( 2 - l ) ( 2 n + l )
5.193. lim----------- ----------- ^
n00 1+2 + 3 + ... +
n = 1, 2, . . . .
n = 1, 2, . . . .
Hatrozza meg, mely x-ekre konvergensek a kvetkez {a} sorozatok:
5.194. a = = ( ^ ^ V = 0 , 1 , 2 , . . . .
5.195. a-=
2 Y
V^^-9
; = 0, 1, 2, . . . .
60
5.196. ==
\ "
x ^ - x + 3
; n = 0 , 1 , 2 , ... .
5.197. a :=
x ^ - x - 2 '
6 + x - x ^
; n = 0 , 1 , 2 , . . . .
Tekintse az albb felsorolt kifejezseket egy sorozat egymst kvet els nhny
elemnek! Adjon meg olyan kpletet vagy utastst, amelyet a megadott sorozatele
mek kielgtenek! Konvergensek-e az albbi {a} sorozatok? Adja meg a konvergens
sorozatok hatrrtkt is:
5.198.
5.199.
1 1+ 3 1+ 3 + 5 1 + 3 + 5 + 7
3 5 + 7 7 + 9+11 9+11 + 13+15
1 1 + 2 1+ 2 + 3 1 + 2 + 3 + 4
2 3 + 4 4 + 5 + 6 5 + 6 + 7 + S
1 1 - 2 1 - 2 + 3 1 - 2 + 3 - 4
2 3 - 4 4 - 5 + 6 5 - 6 + 7 - S
61
6. EGYVLTOZS FGGVNYEK
Hatrozza meg a vals szmok krben azt a legtgabb rszhalmazt, amelynek
elemeihez az albb felsorolt kpletekkel fggvnyrtk rendelhet:
x + 2
x ^ - 4 x +3 '
x^ + x
x ^ - x
6.3. X H. 1/9- x^ .
6.4. X
i
f T .
+ x.
6.5. X^ / - 1- x ^
6.6. X^ lg (x").
\ + x \
6.7. X!- In
\ - x )
6.8. X 1^ |/^lg cos X.
6.9. X n- sin lg X.
6.10. Xn- j/lg sin (x^ 5x + 6).
6 .11. X i-> arctg tg (x - 1).
6.12. X ctg - .
2
62
Dntse el, hogy az albbi fggvnyek kzl melyik pros, melyik pratlan, s
melyik nem tartozik egyik elz csoportba sem:
6.13. X H- 5.
6.14. X |x|.
6.15. X X.
6.16. X [x].
6.17. X h- x][^.
cos 3x
6.18. X
x^
6.19. X sin lx| .
6.20. X Isin x| .
6.21. Xw- sin (sin x).
Dntse el, hogy az albbi fggvnyek kzl melyik periodikus s melyik nem!Ahol
lehet, hatrozza meg a peridus hosszt:
6.22. X sin 2x.
V
6.23. X !-cos - .
6.24. X [x],
6.25. X ^ {x} .
{
0, ha Xegsz szm;
2, ha az x-et kzvetlenl megelz egsz szm pros;
- 2, ha az x-et kzvetlenl megelz egsz szm pratlan.
6.27. Xn- cos (cos x).
6.28. X !-Arctg x.
6.29. X^ 2*
63
6.30. Kpezze az albbi fggvnykapcsolatok inverz kapcsolatt:
= b ) y = 2>x^ + 2 -, c) d) y =
6.31. llaptsa meg, hogy az albbi fggvnyeknek vagy valamely leszktsknek
van-e inverz fggvnye; ha igen, rja fel az inverz fggvnyt!
a ; / ( x ) = 5; ;/(x) = 21n(3-+2) + 5. c) f{x) = x \ D, : = [ - \ - \ ]
1 , , 1, ha Xracionlis
= TT ; = 0, ha irracionlis '
/ ) / W = h a x > f ~
71 n
4 4
6.32. A koordinta-rendszer transzformcija segtsgvel brzolja az albbi
racionlis fggvnyeket;
aj y = - 3 x + 2; bj y = - x ^ + 4x~ 5; cj y = ^ ;
3 2x
d) y = ---- r ; e) y =
x - 1 3 - x
6.33. A koordinta-rendszer transzformcija segtsgve! brzolja az albbi
fggvnyeket:
3^___
a) y = [/2 x; b) y = 2 ^x 2 - 1.
6.34. Oldja meg az albbi trigonometrikus egyenleteket:
a) |/sinx f l cosx + 2 = )/3;
.J 2 - l o j | / 3 ;
2 cos 2x r~
c ) 1- 2 sm x + 1/2 = 0;
sin X+ cos X
- , \ n\ n
d) cos x + cos I - + x j 21 cos -1 (cos x) cos I -
1 '
e) tgx+ - c t g x + 1 =
4
COS^ X
- 1.
6.35. Igazolja az albbi azonossgokat:
1+ cos 2x 1- cos 2x
a) cos X= ------------; b) sin x = ----- ------
64
l - t g 2-
c) COS X = d) sin X=
6.36. A koordinta-rendszer transzformcija segtsgvel brzolja az albbi
trigonometrikus fggvnyeket:
1+ cos ( 2 x - 2 )
a) y = ; b) 2 y = h sin { 2 x + 2 ) - 2 ;
1
c) y = ?>- - t g x ; d) 2 y - 2 = ctg
6.37. A linearizl formulk felhasznlsval, a koordinta-rendszer transzform
cija segtsgvel brzolja az albbi fggvnyeket:
a) y =
2 sn^ ( x - l) + 2
1+ 2 cos^ ---- X3
3 ' ^ 2
6.38. rja fel egyszerbb alakban az albbi fggvnyeket!
y) X1^ cos (arcsin 2x);
cj X tg (arcsin x);
bj X sin(Arccos 2x);
dj X >->sin[arctg(2x 1];
X
e) X cos^ I arctg - 1.
6.39. Igazolja az albbi azonossgokat:
n
a) Arcsin x + Arccos ^ 2
n
c) arctg X + arcctg ^ ^ 2
eJ Arcctg( - x) = ti- Arcctgx.
b) Arcsin |/x + Arcsin |/l - x = - .
d) arccos ( - x) = n arccos x ;
6.40. Szmtsa ki az albbi kifejezsek rtkt segdeszkz hasznlata nlkl:
1
a) Arcsin -p ;
P-
b) Arccos c) Arctg 1/3;
r r
d) cos
Arcsm 1~ 2 j
tg
Arccos -
\ ^ )
6.41. Oldja meg az albbi egyenleteket:
n
a) arcsin ( x + 27t) ^ ~ 2
c) 7TArcsin X= Arccos x;
5 Matematikai feladatok
b) 3 Arcsin ][x n = 0;
d) Arcsin x = Arccos x.
65
6.42. A koordinta-rendszer transzformcija segtsgvel brzolja az albbi
ciklometrikus fggvnyeket:
a) y = 2 Arcsin ( - 2 x + 2) + n; b) y = ~ Arccos
W Y v
d) 3y = 6 Arcctg ( 2x) 2n.
6.43. Oldja meg az albbi egyenleteket, illetve egyenletrendszereket:
a) 4- 5^*-3 5^+1 = 25;
c) log3^3 + 41ogg,3 = 6;
e) = 81;
jlg(x + >;)2-lgx = 2 lg 3.
b) ^ I g (271 + 3 ^ ) = 1;
d) = 2;
I 5lg(2y-x- 1) _ j .
6.44. A koordinta-rendszer transzformcija segtsgvel brzolja az albbi
exponencilis s logaritmus fggvnyeket:
; j = 2 3 - 2 - + l ; b ) y = ~ e ' ~ ^ + l ;
c) y = 2 log2 (4 - 2 x ) - l ; d) y = 2\g(3x + 2 ) - l .
6.45. rja fel egyszerbb alakban az albbi fggvnyeket:
a j X H>- sh (arch 2 x) ;
ej XHth [arsh (x+ 1)];
b ) X ^ e h (arsh x);
j XHe sh (2 arch x).
6.46. A koordinta-rendszer transzformcija segtsgvel brzolja az albbi
hiperbolikus fggvnyeket s inverzeiket:
a ) y = ^sh ( ^ V i ;
1 / x'
c) y - \ = ~ 2 V 2
e) y = 2 arsh (2x + 2) + 1;
1
g) y = 2 arth( - ~ x - -
- 2;
/
+ 1:
.;>; = 3 c h ( l - - J - 2 ;
3 2x'
1+ cth
d) y = -------
1
f ) y ^ ~ arch
2
/ 1- x
2
/ 2x +2
h) y ^ 2 arcth I
- 2;
6.47. A linearizl formulk felhasznlsval, a koordinta-rendszer transzform
cija segtsgvel brzolja az albbi fggvnyeket:
66
a) y = sh^ {-2x);
b) 3y = 2ch^ ( - 14.
6.48. Igazolja a fggvny hatrrtknek defincija alapjn az albbi lltsokat:
. 2 _
a) lim (2x + 5) = 7;
x^l
b) lim
1
^-1 x+ 1
= - 2; c) lim - = 1.
X-O X
6.49. Hatrozza meg - amennyiben ltezik - 2a f fggvny hatrrtkt x ^ a
esetn! Ha ltezik az A vges hatrrtk, akkor hatrozza meg, hogy az x=a hely
milyen sugar krnyezetben kzeltik meg a fggvnyvrtkek a hatrrtket e-nl
kisebb hibval:
2 x
/ W = ; r v : > a = 0, e=10
b)f {x) =
2 x^ + X
x^ + 5x + e
x ^ - 9
a = 3, =10
6.50. Hatrozza meg - amennyiben ltezik - az f fggvny hatrrtkt x->oo
esetn. Ha ltezik az A vges hatrrtk, akkor hatrozzon meg adott e > 0-hoz olyan
ct)(e) rtket, amelyre |/(x) - ^4i < e, hacsak x > &(e):
a) f ( x) = ^ ^ , e = 10-^-
2 - 3 x
b) f ( x) = 3-^ e=10
6.51. llaptsa meg, hogy az albbi racionHs trtfggvnyek szakadsi helyein
milyen jelleg szakads van;
a)f (x) =
X
2 x^ + x
b) Xx) =
x ^ - \
x ^ - x - 2
6.52. Vizsglja meg, hol nem folytonosak az albbi fggvnyek, s llaptsa meg,
hogy ezeken a helyeken milyen jelleg szakadsuk van:
" x ^ - 9
x ^ - 9 x '
ha x^O, 3;
1
3
ha - 3 ;
0, ha x = 0 ;
1,
ha x=3.
ha x^O;
- 2, ha x = 0 .
ha
ha
x # 0 ;
x = 0.
67
Hatrozza meg az albbi fggvnyek jobb, illetve bal oldali hatrrtkt az adott
a helyen:
Ix 3|
6.53./(x) = ----- a=3.
x - 3
l/x + 4 - 2
6.54./(x) = ^^----- ;---- , a = 0.
X
6.55./ ( x ) = ^ . a = 0.
x ^ - X
2, ha x ^ - 1;
6.56./ ( x ) = ^ 0, ha - K x < 5 ; a = 5.
- 2, ha 5^x.
1, ha x < 0;
6.57./(x) = ^ 7, ha x = 0; = 0.
1, ha x > 0.
6.58./(x)
= ^ 1 K = 0-
S i n - , ha x>0.
X
6.59./(x) = Arctg-, a = 0.
X
x + 1
6. 60. f (x) = 2 ^ - \ a=l .
Hatrozza meg az albbi hatrrtkeket:
x^ 5x^ + 7x+ 1
6.61..lim ^..--------- -.
*-0 2 x ^ - 9 x - 5
6.62. lim
xe
(eh x) |/x + 4
1\
6.63. lim ( XIn
a:00 \ Xj
68
6.64. lim
In^
\ x j
6.65. lim
00
e"
6.66. lim
x-6 In (x^)
6.67. lim In^ (x^).
X0
6.68. lim sin - .
x^O X
Hatrozza meg az albbi racionlis trtfggvnyeknek az adott a helyen vett
00
tpus hatrrtkt:
x ^ - l x + 4
6.69. l i m ----- r------ .
x-oo 3x^ + 5
2x^-3x + 2
6.70. hm
x-co -5x^ + 2x 1
. . 12x2+ 5x4-1
6.71. hm ------- ;------------- .
-V- - 00 X + 5x 1
(x+l)io + (x + 2) + (x+3)i + ... + ( x + 100)
i . ------------------------ .v>o+10--------------------------
0
Hatrozza meg az albbi racionlis trtfggvnyeknek az adott a helyen vett -
tipus hatrrtkt:
x^ - 4x + 4
6.73. lim
-2 x^-5x + 6
x^ + 2x 3
6.74. lim
x^- 3 X +2x + 15
69
, ( l + x ) ( l + 2x)(l + 3 x ) - l
6.75. hm--------------------------------
x-O X
x " - l
6.76. lim------ , eN.
x^l X1
x^ x ^ ~ x +l
6.77. lim---- r------------
x^i x^ + x - 2
x*' 16
6.78. lim -------- .
x^ - 2 X + 2
8 x ^ - 1
6.79. lim
6.80. lim
x-i'i
16 x^ 5x + 1
x ^-3
2x^-3
00
Hatrozza meg az albbi irracionlis trtfggvnyeknek az adott a helyen vett
tpus hatrrtkt;
6.81. lim
a:- 00 X
pc^+2 x ^ - 7 x
6.82. lim -
]/x^ +3x^ + 2
4 5 ______
\/x^ + 2 -]2 x ^ - l
6.83. lim ------- p = = -----
2j/x +3x
]lx^ + x^ + ^x^ + 3x^
6.84. lim ------ j ----- -----------.
0
Hatrozza meg az albbi irracionlis trtfggvnyeknek az adott a helyen vett -
tpus hatrrtkt:
70
4
x - 2
6.85. lim V
x - 1 6 | / x - 4
4
6.86. lim -p= .
l/l+x^- 1
6.87. lim ------------.
L'x+13-21/x+1
6.88. lim ^ ^ ------ .
x-3 x ^ -9
3^x^-5-2x
6.89. lim .
x-3 X +X12
l/llx+3-t'4x+17
6.90. lim ^ -----
x-2 X - 5 x + 6
x^ + x - 2
6.91. lim
- - 2 [/x^ + x + 2|/x^ + 4x + 8
l'l + 3 x - l / l - 4 x ^
6.92. lim ^
x-O Ix
y.Y-3
6.93. lim -V-
6.94. lim
^-9 p + l x - S
X- + X- 2
|/x^ + x - l - l / x
3
i/r+z^ - 1
6.95. lim -------- r-----.
Ix
6.96. lim T----------
x-O
l/l + x - j / l - x
71
6.97. lim
|/x2-3x + 6- ) / - x ^ + 5x-2
^-2 |/-x^ + 5 x + 3 - ] l - 2 x^ + 9 x - \ '
Hatrozza meg az albbi fggvnyeknek az adott a helyen vett od oo tpus
hatrrtkt:
6.98. lim ( ---------------------------r
x^2 \x(x 2) X 3x + 2
6.99. lim
x->00
6.100. lim (l^x^ + x + l - ^ x ^ - x + l ) .
X->00
6.101. lim ( j l x + x - p x - 0 j.
6.102. lim ([/(x + a){x + b) x).
6.103. lim \ x ^ + \ - ^ x ^ - l ) .
A lim ( 1H 1 = e hatrrtk felhasznlsval szmtsuk ki a kvetkez hatrr-
X->00 y ^ )
lkeket:
/3x+l\2^ -i^
6.104. lim -------
j:00\ 3X + 7
/ 2x - l
6.105. lim -------
x->oc y2x+ 3
6.106. lim
6.107. lim
/
\ -x-7
/
2 - 5 x
V- 1- 5Xy
3;,2_ 2X2x^ + 20
3x^-5
6.108. lim {x[ln(3x + 4) - l n(3x-2)]}.
72
6.109. lim 1
rjc4-s
6.110. lim f 1+ , k , l , r , t e Z\{0}, m, seZ,
jc-Qo \ Ix + mJ
Hatrozza meg az albbi hatrrtkeket:
6.111. lim x(|/x2 + 1-a:).
/ 3 x + l l V "
6.112. hm .
*^00 \ 2 x + 4 J
S.113.
*-oo\2x 1/
1^1 + a: sin X - cosx
6.114. lim^^-----------------------.
x-o sm 2x
i^i+tg2x - / r ^
6.115. hm----------- :--------------
x-o sm X
X
6.116. hm
*-0 |/l+ sinx-|/cosx+ sinx
Ksztsen bra vzlatot az albbi racionlis trtfggvnyek grbjrl! (Az adott
fggvnyeket mindig a legtgabb rtelmezsi tartomnyban tekintse!)
6.117. xh^
6.118. X
x2 - 3 x+ 2
1
6.119. X
l - x ^ '
2 x ^ - x - 3
x ^ - x - 6
x^ + 3x^- 4x
73
( x + l ) ( x + 3 ) ( x - 2 )
6.121. jc t- -----------------
i x - l ) i x + 2)(x + 3)
. Szmtsa ki a kvetkez hatrrtkeket Hm----- = 1 felhasznlsval:
*-0 X
sin^ X + 2 sin a:
6.122. lim------------------.
x-o Xcos X
(sin 5x) tg 2x
6.123. l i m -
*-0 x^
6.124. lim x^ ctg^ 2x.
x^O
6.125. lim
cos X- COS^ X
x-0 X^
2 1
x"^ sm -
6.126. lim
x-o sm X
. sin X
A trtkifeiezs bvtse utn szmtsa ki az albbi hatrrtkeket lim----- = 1
x-^O X
felhasznlsval;
6.127. lim
x-0 sm 4x
l/l - sin^ 3 x - 1^1+ sin^ 3x
6.128. lim ----------
x^
1- l/cos 2x
6.129. lim---- ^-7---- .
x-o 3x
1cos l / ^
6.130. lim----------'
x-O X
x(/l + tgx - ^1- tgx)
6.131. lim -------- ----------------
x-o sm"^ X
74
Hatrozza meg az albbi kifejezsek hatrrtkt (clszer lehet a
1 cos 2 x sin X
sin X = r----- azonossag s hm------ = 1 felhasznlsa):
2 *-0 X
. cos2x - l
6.132. hm----- ^------.
6.133. lim
x-o Xsin X
cos X- 1
6.134. lim
x-O X
1cos 3x
X-O x^ cos x
4x* + x^
6.135. lim
6.136. lim
*-0 x(l cos x)
t g x sinx
x->o x^ cos X
2 sin X - sin 2x
6.137. lim
x-O X^
Hatrozza meg az albbi kifejezsek hatrrtkt gy, hogy helyettestst alkalmaz.
majd felhasznlja a hm----- = 1 hatrrtket:
x-O X
sin/nx
6.139. lim ------, m, neN.
x^n smx
75
7. EGYVLTOZS FGGVNYEK
DIFFERENCILSZMTSA
Az ebben a fejezetben szerepl fggvnyek mindig a legtgabb rtelmezsi tartomnyukban tekinten
dk.
7.1. Hatrozza meg az x helyhez tartoz klnbsgi hnyados hatrrtkeknt az
albbi fggvnyek x helyhez tartoz differencilhnyadost:
a)f {x) = '(x-, b)f {x) = - s x = 2.
Hatrozza meg az albbi fggvnyek x szerinti els derivltjt:
7.2./(X) - ^ + f x .
7.3./(x) = (1- x3)(x2 - 2 x).
2x^ + 3x
7.4./(x) =
1+ 2x
, 7.5./(x) = sin^ x-sin(x^).
7.6./(x) = arctg|/x.
7.7. /(x) = xe^'^cos 2x.
1
cos n
arsh
7.9./(x) = e K
76
7.10./(x) = arctg
x^+l
ln(x + l/x^-l).
In (archj/x).
7.13./(x) = th
cos^ 2x\
x ^ - l
7.14./(x) =
7.15./ ( X ) =
7.16./(x) =
7.17./ ( X ) =
/sh 3x\
In - n - .
\sin' X /
Arsh(j/x e~^).
sin^ (Arccos x).
n cos^(x^) - sin 2.
7.18./(x)
7.19./(x)
7.20./ ( X )
7.21./(x) =
= sh^
X
tg
sh^x
sin 371.
l^x^+l
sin^(l - x)
71
2COS^X 4 '
7.22. f i x)
7.23./(x)
7.24./ ( X )
7.25./(x)
= In In^ (x^).
= cos^ (arctg e^.
= 10g2
^ch 3x\
\sm"- X/
3
= log2(l/ ? sh x).
77
sin^ 2 x ^ 7 1
7.26./W = ^ + e ^ t g - .
2*
7.27. f{x) = log2 (x*^) - arctg e*+
l-ch^x
Hatrozza meg az albbi, paramteres alakban adott fggvnyek x szerinti els
derivltjt:
7.28. x{t) = ctg ; y{t) = tg^.
7.29. x(0 = ^ ; jW = ln(e<=').
7.30. x(0 = lg (tg 0; j ( 0 = cos 2.
7.31. x{t) = cos f, y{t) = e sin t.
+1 2/
7.33.
7.34. x(t) = Arccos(cos t); y(t) = In^.
7.35. x(0 = Arcsin(/^); >>(0 = ln|/l - 1 ^.
V 1
7.36. x(0 = arctgl/; j ( 0 = In .
7.37. x(0 = ln(l + /^); y(t) = -arctg t.
7.38. x{t) = n 3* = On 3) (sin t f - c o s n.
7.39. x(0 = 2* y(t) = tg^ + t g ^ .
7.40. x(0 = y(t) = Ig^-e^
7.41. x(0 = Inln^/; y(t) = 3^'.
78
7.42. x() = lg(ch 0; j (0 =
sh t
eh t
7.43. x() = Ch2/; y{t)
7.44. x(0 = ch^(^); y{t) = arctg
( z l )
v + l j
7.45. x(t) = y(t) = (1 - t^)aTi t.
Hatrozza meg logaritmikus differencilssal az albbi fggvnyek x szerinti els
derivltjt:
7.46./(jc) =
3
7.47./(x) =
7.48./(x) = (2x)*.
7.49./(x) = x ^ .
7.50./(x) = (^^)'"*.
7.51./(x) = (l^)*\
7.52./(x) =
7.63./M = ( i ) .
7.54./(x) = (In x)'^.
7.55./(x) = (c o s x f \
' l + x>"
7.56./ ( X ) =
1 - x
79
7 . 5 7 . / W = (1^)"
7.58. f{x) = arctg (r*)- (arctg x)*.
Hatrozza meg az albbi, implicit alakban adott fggvnyek x szerinti els
derivltjt:
7.59. x^y = sin x.
7.61. dixclgy-xy^ = 3.
m
7.62. V Un y + tg y - y ^ x ^ = n^+e.
7.63. sin;;-3*V^ + x^ln y =
7.64. arctg (x;^)+ log2 ( x - j ) = 0.
7.65. Arccos = 5.
7.66. lg(x-:F)-4''''^* = 0.
7.67. j/l - j - cos(xj) + tg 1 = 2.
7.68. l/l + 2 v - t g - = arsh 1.
7.69. >' cos l/xX In sh j =
x^
cos n
ctgx
7.70. (cos 1)*---------+y^ = x.
y
Hatrozza meg az albbi implicit fggvnyekben >-nak x szerinti differencilhnya
dost az (xq; >o) pontban:
7 . 7 1 . x ; ; - / - 3 = 0, ^(4; 3).
80
n
7.72. sin {x + ly) = e'+ 1, Pq (^0; - j .
7.73. Hatrozza meg az x = 5 cos t , y = A n t egyenletrendszerrel megadott gr-
T
hs q = - -hez tartoz rintjnek egyenlett!
7.74. Hatrozza meg az = ]jx1egyenlet parabolag s az x^ + y^ = 5 egyen
let kr metszspontjban a krhz hzott rint egyenlett!
7.75. Mekkora szg alatt metszi az y tengelyt az y = |/3 sin x+ \ nf e grbe? rja
fel a metszspontban a grbhez hzott rint egyenlett!
7.76. Hatrozza meg az x = cos t, y = sin t egyenletrendszerrel megadott
n
grbe t = - -hez tartoz rintjnek irnytangenst!
s i ^ n
7.77 rja fel az j = 4 2x sin - egyenlet grbe rintjnek egyenlett annak
.v = 0 abszcisszj pontjban!
3
|/jC sin(7r-.v)
7.78. rja fel ?izy = e + 2^ cos^ n egyenlet grbe rintjnek egyen
lett annak x = 0 abszcisszj pontjban!
7.79 Hatrozza meg az j = ^2 -ix |.g jg jq egyenlet grbe x =l abszcissz-
j pontjhoz tartoz rintjnek az egyenlett!
7.80. rja fel az j; = log3(^Ix + 1ch^ x) egyenlet grbe normlisnak egyenlett
annak x = 0 abszcisszj pontjban!
Sh X 71
7.81. Hatrozza meg az v = -----+ t g - egyenlet grbe x = 0 abszcisszj
2 ch^ X 4
pontjhoz tartoz normhsnak az egyenlett!
3
^4-
X
7.82. rja fel az y = tg^ ~ 2~I ctg- ln(l x) egyenlet grbe rintjnek s
V V
normlisnak egyenlett annak x = 0 abszcisszj pontjban!
7.83. Adott az x^ + y^ = 5 egyenlet kr s az j = x^l egyenlet parabola.
Hatrozza meg a kt alakzat metszspontjait! rja fel a kisebb abszcissza rtk
metszsponton tmen, a krt rint egyenes egyenlett!
6 Matematikai feladatok 81
7.84. Adott az y = 2^"-4(Arctg |/x+ 1) eh x+cos^ x egyenlet grbe. rja fel az
X= 0 abszcissza rtk pontban a grbhez hzhat rint egyenlett! Prhuzamos-e
a kapott rint az y + 3 = 1x egyenlet egyenessel?
(x-3)2 0 + 1)^ / 1\
7.85. rja fel a z -----------h ---------= 1 egyenlet ellipszis F 2; - 1pontjban az
4 3 V 2/
rint egyenlett! Hatrozza meg a felrt rint s az y = - 2x+ 3 egyenes hajlssz
gt!
7.86. Hatrozza meg az y x 2 egyenlet egyenes s az (xl)^ + (y+ 1)^ = 8
egyenlet kr hajlsszgt a metszspontokban!
7.87. rja fel a kvetkez polrkoordints alakban megadott grbk adott pont
beli rintjnek egyenlett;
aj r = |/cos l(p, >o = 4 ;
71
b) r = (p, n = 2 '
7.88. Szmtsa ki az albbi fggvnyek grbjhez az adott pontban illeszked
simul kr sugart, grblett s kzppontjnak koordintit!
a ; ; ; = x ^ - l , Xq=1; b) y=e^, Xq = 0; c) y = ~ , Xq = 2\
X
IV 2 r, 1 1 . / Jx=COS^]
d ) y = x \ xo = 0; e ) y =\ n x , Xo=l; y = an^ t \ ' =
7.89. Mutassa meg, hogy az/(x) = |/9x^ 6x+2arsh (3x1) fggvnynek van
szlsrtke!
7.90. Hol van szlsrtke s hol lehet inflexis pontja az albbi fggvnyeknek?
a) f ( x) = ln(x^ + x + l ) + ^ a r c t g ^ ^ ^ ;
b) /(x) = ln( x^ + 2x)+ arcth(x1) s i n - .
Szmtsa ki az albbi hatrrtkeket a LHospital-szably segtsgvel:
7.91. l i m - ^ .
x- 0 Sin 5 x
sin 5 x sin 3x
7.92. lim
82
sin Xjc cos X
7.93. lim------ r^r---- .
x^o Sin X
xc osx sinx
7.94. lim----------------- .
x^o X Sin X
xsin(l x) cos(l x) + x
7.95. lim-----------------------------------
x-^i 2 sin (1 x)
l/2 + 2th2x-l/2chx
7.96. lim ----------- ;------------- .
x ^ o sh X
1/1+ t h 2x - l / l - t h 2x
7.97. lim ----------------------------
x-^o sh X
2 ctg 3x
7.98. lim---- ^ ,
x-O X
7.99. lim
7.100. lim
*-0 sm X
sh 2x + sin X
, x^o 2x
, arctgx
7.101. hm--------
x-O X
Xsm X
7.102. lim--------------
1 - X - y
l/l +x'sh X eh 3x
7.103. lim^-------- ^-------------
x->o sh 2x
x3(x-3)
7.104. lim
sin2x 2 sinx
sm X
7.105. lim
x-o Arcsin x
6* 83
7.106. lim
COS^ X
n n
'^2 X - -
2
7.107. lim
sm Xx
-0 1sin Xcos X
sin^ x ^ - sh ( - 2 x)
7.108. lim--------------- ^ ^
x-o ( x - l ) l n ( l + x )
x ^ - 4
7.109. lim---------------.
x-2 , n
sm -
7.110. lim
eh 5x cos X
x^o ln(l x)
7.111. lim
sin 3x - sin X
In
7.112. lim -
x-o ln(l + x)
1+x^
l - x j
x-o sm Xcos X
In
1
7.113. lim
e + - ] - l
xj
X
- 1
In
sm X
7.114. lim
-v^0+ X
7.115. lim
1
I n -
X
X00 X
7.116. lim
In X
x^O* ctg X
84
lg, l/cos X
7.117. lim -----
.V--0+ JC^
7.118. lim XIn^ ( 1+ - ).
x^cc \ Xj
7.119. lim l/2xctg(x2).
v^0+
7.120. lim xHnx.
x->0*
7.121. lim
7.122. lim -e"*.
x^-oo X
7.123. lim x^ sin - .
JC-00 X
7.124. lim
x - i \ x - l InxJ
7.125. lim - -V
r-0+\smX X/
7.126. lim
jc-O
1 1
\ x^ sin^ X
7.127. lim (ctgx)' *.
7.128. lim
x^O
1
/ \ 3c^
^sin
\ ^
1
7.129.
JC-l
Vizsglja meg albbi fggvnyek menett:
7.130./(x) = x^-4x^-^4x.
85
7.131./ W - (x+I)(j:+3y.
7.132./ W = (2- 1) |5+4.
7 . 1 3 3 . / W -
7.134.
6x
7.135./(x) = ^ ^
4 4x
7.136./ w =
x + 3
7 . 1 3 7 . =
( x - l ) ^
7 . 1 3 8 . / W - ^ .
1x - 3 x - I S
7 . 1 4 0 .
7.141.
x - 3 x - I 0
7.143./ W = 4jr^+ ;
7.144. /(x)
/1 + x
7.145./(x) = In 1 ^
7.146. f{x) = In^ X.
7.147. f {x) = x\n{x^).
7.148. f {x) = x^ In (x^).
7.149. f i x) = x \ n - .
X
7.150./(x) = xMn-.
X
1
I n -
7.151. /(x) = .
7.152. f i x) =
7.153. f i x) = ] { ^ \
7.154. f i x) = x?-^
1
7.155. f i x) = -g-".
7.156. f i x) = x^e-".
7.157. f i x) = xe~^\
_
7.158. f i x) = xe ^
7.159./ ( X ) = y .
7.160. f i x) = (X + 2K+2
7.161./(x) = x^A
7.162. Ossza fel a 4-et kt rszre gy, hogy az egyik rsz ngyzetnek s a msik
rsz kbnek sszege maximlis (minimlis) legyen!
87
7.163. Az 1000 cm^ felszn, fell nyitott hengerek kzl a maximlis trfogat
nak mekkora a sugara? Mekkora a maximlis trfogat?
7.164. Hatrozza meg a 108 dm^ felszn, fell nyitott ngyzetalap egyenes
hasbok kzl a maximlis trfogat hasb mreteit s trfogatt!
7.165. Azok kzl a ngyzetalap tglatestek kzl, amelyeknek egyik cscsban
sszefut lek sszege s, melyiknek lesz legnagyobb a trfogata? Mekkora a maxim
lis trfogat?
7.166. Hatrozza meg az adott a alkotj legnagyobb trfogat kp m magass
gt s alapkrnek r sugart!
7.167. 4 db 1 m hossz rdbl ngyzetes alap, egyenes gla alak storvzat
ksztnk. Mekkora alapterlet esetn lesz a stor trfogata maximlis?
7.168. Egy ablak egy a, b oldal teljes tglalapbl s egy flje rajzolt flkrbl
ll; kerlete 4 m. Hogyan vlasszuk meg a mreteket, hogy az ablak terlete maxim
lis legyen?
7.169. Az r sugar krbe rhat derkszg ngyszgek kzl melyiknek legna
gyobb a terlete?
7.170. Az r=4 cm sugar flkrbe az tmrn fekv, maximhs terlet trapzt
szeretnnk rajzolni. Hogyan kell a trapzt mretezni?
7.171. Hatrozza meg a 2a alap, m magassg egyenl szr hromszgbe
rajzolhat maximlis terlet tglalap M magassgt!
7.172. Egy tglalap oldalai 5 s 4 egysgnyiek. Krje olyan egyenl szr hrom
szget szerkesztnk, amelynek alapja az 5 egysgnyi oldalra esik, szrai pedig tmen
nek a tglalap msik kt cscsn. Mekkora legyen az egyenl szr hromszg
magassga, hogy terlete a legkisebb legyen? Mekkora a minimlis terlet?
B
88
7.173. Adott egy a tfogj egyenl szr derkszg hromszg. A hromszg
egyik befogjn jelljn ki egy tetszleges P pontot, amelyen keresztl prhuzamos
s merleges egyenest hz az tfogra. A keletkez trapz mikor lesz maximlis
terlet? Mekkora ez a maximlis terlet? Ez a maximlis terlet hny %-a a
hromszg terletnek?
7.174. Egy szablyos ngyoldal gla alaple 18 cm, magassga 40 cm. Szablyos
ngyoldal hasbokat runk bele gy, hogy a hasb alaplapja a gla alaplapjn, a
hasb fedlapjnak cscsai pedig a gla oldallin legyenek. A hasbok kzl melyik
a legnagyobb trfogat?
7.175. Hatrozza meg az adott R sugar gmbbe bert maximlis trfogat kp
sugart s magassgt!
7.176. Egy adott R sugar krcikkbl tlcsrt formlunk. Mekkora kzpponti
szg mellett lesz ennek trfogata a legnagyobb?
7.177. Szmtsa ki sin 30 = ^rtknek felhasznlsval sin 31 kzelt rtkt,
10 ^ pontossggal!
7.178. Egy kocka lei 10 cm hosszak. Mekkora lesz kzeltleg a trfogata, ha
leit 0,2 cm-rel megnveljk?
7.179. Hny cm-rel kell nvelni egy V= 1000 1trfogat gmb sugart, hogy a
trfogat kzeltleg 5 literrel nvekedjk?
7.180. Egy ngyzetalap hasb mretei; = 0,5 dm, m = 4dm. a kismrtk vl
toztatsa m mekkora megvltozst eredmnyezi, ha a trfogat lland marad?
89
8. EGYVLTOZS FGGVNYEK
INTEGRLSZMTSA
Az
F(,ax+b)
f {ax + b) dx = -------------l-c[, 6 gR; a=0 ] alkalmazsval hatrozza meg
az albbi fggvnyek hatrozatlan integrljt!
8.1. .V 3 ( 1 - x ) 5 + -----+
2 (3 + x)'
5
3 *
8.2. X 2^+]/ l + 2x+ l { ~ ^
8.3. X ^ -
f x
|/2x - l
j/ x^ + 5 _ + + T x^ + l x + \ 2
8.4. X i- ^ 1= + ^ + --------- 1--------+ ---------------
x^][x |/x x + 4
8.6. X!-3 cos X- 2 sin - + -a - +
2 |/3 cos^x s i n ^ ( l - x ) '
8.7. X
cos 2x
sin^ Xcos^ X
+ cos X sin X.
sh X , X 5 5
8.8. X!--------heh-----------;----- 1- ---------- ,
2 2 2 ch^ 2x , 3 - X
sh^-----
90
8.9. X
eh 2x 1
sh^ I x sh^ X ch^ X
3 x +1 1 5
8.10. XI- + -------------- +
2x x^l (x + 1) In 2 5 3x
- - 1 2 3 1
8.11. X H> =H .... . +
8.12. jc
] / l - 2 x^ ] [ ^ + \ 4 + 4x^ l/2x^ - 2
1
3|/5x2-25
'+i 9 |A-
2 2x^+1
8.13. X
p{Ax^ + Ax + 2) l ^ 3 px^ + ( ) x-^
/ - x 2 + x + -
A
8.14. X H-
x^ + 2x + 2 1 5
+
( x + i r x^ + x + l 4 x - x ^ - 3
Adja meg az albbi fggvnyek primitv fggvnyeinek halmazt s ellenrizze a
megoldst az sszetett fggvny differencilsi szablya segtsgvel:
8.15. /(x) = xsinx^.
8.16. /(x) = - (1 + x) cos (1 + x f .
4 + 4x
8.17. / ( X ) =
cos (x + 2x+ 1)
8.18./ ( X ) =
sh^ x^
8.19. f i x) = 4xe-^\
8.20. / ( x ) = 2"''* cos X.
8 .21./ ( X ) =
1
COS^ X
91
8.22. / ( x ) = - ,
8.23. f i x) =
2|/x sin^ f : '
8.24./ ( x ) =
2 x
Hatrozza meg a kvetkez j f (x)f'(x) dx (a# 1) s
integrlokat:
8.25. ^ x ^ ] / ^ 2 d x .
8.26.
(x+1) j/jc^ + 2jc-3 +
2 -
x.
8.27. e~^dx.
r 4x
8.28.
8.29.
8.30.
8.31.
8.32.
8.33.
8.34.
dx.
dx.
r ch 2jc
sh"^ 2x
/x.
(Arctgx)^
1+ x^
/x.
2x^-1
o2 x
dx.
r w
. /(^)
dx tpus
92
8.35. dx.
sin 2x
1+ sin^ X
8.36. 2 tg 3x dx.
8.37.
8.38.
8.39.
8.40.
8.41.
8.42.
8.43.
- cth 2x dx.
2
dx.
1) arcth x
1
dx.
th(l-jc)
X
dx.
ctg
dx.
X cos x^
|/sin x^
sin 2x
dx.
-dx.
]j\ + sin^ X
8.44. f Xtg Xdx.
8.45. J sin^ Xdx.
8.46. eh - dx.
2
8.47. j sin^ Xcos^ Xdx.
fch'^x
8.48. - rr- d^-
shx
93
Parcilis integrlssal szmtsa ki a kvetkez integrlokat:
8.49. J 2 x^ sin 2x dx.
8.50. J(2x+1)s1ia:/x.
8.51. (x"-l)e*/x.
8.52. J sin xx:.
8.53. 3^"' cos - dx.
2
8.54. J Xe* cos X dx.
8.55. J 4 COS^A' dx.
8.56.
8.57.
X
- In^ Xdx.
2
In X
dx.
X
8.58. j Arccos x dx.
8.59. J 2x Arctg x dx.
8.60.
' / I
- x^ ) arth Xdx.
8.61. f 2x^ sin (x^) dx.
8.62. x^ Arctg - dx.
X
8.63. f 2Ml+x)^/x.
8.64. :dx.
X
8.65. j sin In x dx.
94
8.66. Jch Xcos I x dx.
8.67. J Arcsin x dx.
8.68. Ix^e-^^ dx.
8.69. J x^ cos^ Xdx.
8.70. J(l-x)sh2jc/x.
j fggvny bevezetsvel hatrozza meg a kvetkez integrlokat:
8.71.
8.72.
8.73
8.74.
i f c
l - x f c
^thx+ 1
dx.
ch^x
dx.
^ ( i +] f x y
dx.
' sin + 1
dx.
^x+l
8.75. J cos ^2 x 1dx.
3
8.76. j sin f x dx.
8.77. J eh ^2 x dx.
8.78. l / T ^ dx.
x +l
8.79.
8.80.
dx.
1 + dx.
x^
95
8.81.
X
dx.
^x + 2
8.82. f dx.
8.83.
1
dx.
\ + ][x
8.84. J sin ]jx dx.
A racionlis trtfggvnyek integrlsra vonatkoz mdszerek alkalmazsval
szmtsa ki a kvetkez integrlokat:
8.85.
8.86.
8.87.
8.88.
8.89.
8.90.
8.91.
8.92.
8.93.
+ +
2 x ~\ (l-3x)2 4 x ^-4 x+ l
/
X X- 1 1
+ ---------- +
1- 2x + 5 x^ - 2x + 2
2 x - 3 2x + 5 ,
+ ^ ---------- 1dx.
dx.
dx.
x ^ - 2 x + 2 x^ + 2x + 5
2x
( x + l ) ( x + 2) ( x+3)
1
dx.
dx.
x^ + x
6x - 2
x^ + 2 x - 3
dx.
l+x^
3x- 3x^
2
dx.
( x +l ) ( x^ + l)
12X+18
dx.
x^ + x ^ - l x
dx.
96
8.94.
8.95.
8.96.
8.97.
8.98.
8.99.
8.100.
8.101.
8.102.
8.103.
8.104.
8.105.
8.106.
8.107.
dx.
dx.
4x + 2
(jc^ - 2jc + 1) (x^ + 2x + 3)
'x^ + 2x^- 5x + l
-------- ;-----------dx.
x^ + 4
j c ^ - 10jc^ + 24jc^
jc^-lOx + 25
3x^ + 4x + 3
( x- l ) ^( x^ + 2 x + 2 )
x^(x+l )
8 x + 2 2
(x+l ) ( x^ + 6 x + l 2 )
:---------dx.
2 x^ + x - l
2 x
2 x - X - 1
' x^ + x^ + x +l
3x^ + x - 2
Cx^ + x^
2 . dx.
x ^ - 4
' 2x^+llx+16
+ 4x^ + 8x
' x^+ lOx+18
dx.
dx.
dx.
x^ + 6 x^ + 9x
x^ + x^ + 2x + 3
x^ +5x^ + 6
dx.
fx^ + 5x2 + 4x + 8
1 dx.
J x^ + Ax^
7 Matematikai feladatok
97
8.108.
8.109.
' - x ^ + 3x^ + 5x^ + 6
+ 3x^
dx.
<'2 x^ + 5x^ + 2 x^ + 2 x +\
x^ + x^
dx.
Az irracionlis fggvnyek integrlsra alkalmas helyettesitssel szmtsa ki a
kvetkez integrlokat:
8.110. 1/3-9x^/x.
8.111. p + 5x^dx.
Z A M . \ p x ^ - A d x .
8.113. J l/- 2 4 - 1 0 x - x 2 Jx.
8.114. J|Z5?+4/x.
8.115. l/4x2 + 4x + 7/x.
Z ^ ^ % . l p - 2 x - x ^ dx.
8.117. 1^x2 + 8x + 20 /x.
8.118. l/x2-4x + 6/x.
8.119. / x2 - 6 x- 7 /x.
1
8.120.
8.121.
8.122.
)/4- 16x^
5
Jx.
l/x^-5
2
1^2x2+1
/x.
/x.
8.123.
^-4^2 + 4x + 3
/x.
98
8.124.
8.125.
8.126.
8.127.
8.128.
8.129.
dx.
dx.
^Ax^ + Ax + 2
dx.
\ + f c
fx
2- | ^
[ l x +\
x+l
dx.
dx.
' x^'x+1
dx.
\ - p ^
8.130. flOx|/^+Ip</x.
8.131.
8.132.
8.133.
8.134.
dx.
l / - 3 - 8 x - 4 x ^
x - l
dx.
1/2jc2 + 4jc + 3
1
dx.
(2x- 1) (|/2jc- 1 - 1)
dx.
Adja meg az albbi trigonometrikus fggvnyek primitv fggvnyeinek a halma
zt:
8.135. f i x) =
smx
1+ sin X
1*
99
cos X
1+ sin X
8.140. f i x) - 5jjjj((n-cosx)
1
8- 1 - / W ' r f T 5 ^ '
COS X
cosx
8.144./(x) 2 - 2 cosx-sinX
X
8.145. f i x) = sin^ 2x + 2 cos"' - .
8.146. f i x) = tg^ 3x + ctg-
8.147. f i x) = sin X.
8.148. f i x) = cos^ 2x.
8.149./(x) = sin^.
8.150. f i x) = cos^ 3x.
100
8.151./(x) = t g^lx.
8.152./(x) = ctg^.
8.153. f i x) = sin^ X cos^ x.
8.154. f (x) = sin^ Xcos^ x.
8.155. f ( x) = sin'^ xcos^ x.
8.156. f (x) = sin X cos x.
X
8.157. f (x) = sin2xcos-.
8.158. m = cos 2x cos 3x.
Szmtsa ki az albbi exponencilis s hiperbolikus fggvnyek hatrozatlan integ
rljt:
eX ^2x
X 1
8.160. jc !-sh^ - +ch^ 3xH-----z-------ch^ 2 x.
2 sh^ 2 x
X 2
8.161. X cth-------------- l-e*shx + e'*ch^x.
2 th 5x
8.162. X
8.163. X
8.164. X 1^
\ + e^'
1
2 e^
1
1
8.165. X
l+e^
101
8.166. X
8.167. X
l + e^
l+e^^'
1
8.168. jc
eh X
5
8.169. X
2 sh 2x
1+ shx
sh x(l +ch x)
Hatrozza meg az albbi fggvnyek hatvnysort az x = 0 krnyezetben, s a
kapott hatvnysor segtsgvel adja meg. a fggvnyek primitv fggvnyeinek a
halmazt:
sm X
sh2x
8.171. X ------ .
X
8.172. X
8.173. X 2~^\
In( l-x )
8.174. X -------
8.175. X
Arctg X
Szmtsa ki az albbi hatrozott integrlokat:
8.176.
8.177.
+ 2x I dx.
1/2X+2
102
8.178. - (sin 2x + eh 3x) dx.
1
0
2
8.179.
1 \
ch^l x l +x^J
dx.
0, 5
8.180.
8.181. J xe^^dx.
-1
8.183.
sm X
-dx.
8.184. In Xdx.
2 x
8.185.
3 ( 2 - ^ )
dx.
8.186. Szmtsa ki az x h* x^ fggvny alatti terletet az x = 0-tl x = 2-ig gy,
hogy
a) az intervallumot n egyenl rszre osztja, s vizsglja a tglalapterletek sszeg
nek hatrrtkt, midn -oo;
b) hatrozott integrllal, a Newton-Leibniz-formula alkalmazsval!
8.187. Hatrozza meg az j = grbe alatti terletet - 1-tl 2-ig
a) &hatrozott integrl defincija alapjn,
b) &Newton-Leibniz-formula alkalmazsval!
103
8.188. Mutassa meg, hogy
1
a) J X*dx = dx;
-1
b) ^si nxdx = 2 j sin x dx.
0 0
Hatrozza meg az albbi egyenletekkel megadott grbk alatti terletet a megadott
intervallumban:
8.189. j 3, [3; 4].
8.190. >^= COS - ,
8 . 1 9 1 . / = 9 - jc, [0;8].
10
, [0;5].
^x + 4
8.193. j = In X, [ 1 ; 4
8.194. y = sin
8.195. y = tg X,
8.196. y = arthx,
8.197. y = sin^ x, [0; n].
8.198. Hatrozza meg az j 7x+ 12 egyenlet grbe s a koordintatenge
lyek ltal bezrt vges terlet mrszmt!
8.199. Hatrozza meg az Xq rtkt gy, hogy az y - - egyenlet grbe alatti
X
terlet az [1; e] s az [e; Xq] intervallumban egyenl legyen egymssal!
4 13
8.200. Szmtsa ki az y = egyenlet hiperbola s az j - x egyenlet
egyenes ltal bezrt vges terlet nagysgt!
104
8.201. Mekkora az + = 1 s az + = 1 egyenletekkel megadott
ellipszisek kzs rsznek a terlete?
8.202. Az y = ^ egyenlettel megadott grbe alatti terlet az [1; b] intervallumban
1
- . Szmtsa ki b rtkt!
2
8.203. Hatrozza meg annak a vges terletnek a mrszmt, amelyet az y
tengely, az y = f x egyenlet grbe, valamint a grbnek az Xq = 4 abszcisszj
pontjhoz hzhat rintje zrnak be!
8.204. Szmtsa ki annak a vges terletnek a mrszmt, amelyet az x + y = 1
Q&afx+][y = 1 egyenletekkel megadott fggvnyek grafikonjai zrnak kzre!
8.205. Szmitsa ki az y = x^, az y = ^x^ s az y==3x egyenletekkel megadott
grbk ltal hatrolt vges skrsz terlett!
8.206. Szmitsa ki az albbi egyenletekkel megadott grbk ltal bezrt vges
tartomnyok terlett:
aj y = x^ + 2 x 4 s az y = - x ^ + 2 x + 2 ;
b j y = x^ + 2 x +l s az y = x 1 ;
cj y )/x + 4 s az y = |/4 x!
8.207. Milyen arnyban osztja kett az = 2x egyenlet parabola az = 8
egyenlet kr terlett?
8.208. Szmtsa ki az
x l = r cos , j = 2 + r sin
paramteres egyenletrendszerrel megadott kr terlett!
8.209. Szmtsa ki a kznsges ciklois egy ive alatti terlet mrszmt, ha annak
paramteres egyenletrendszere
x = 2(- sin ), y = 2(1-cos 0, 0^ ^ 2.
8.210. Szmtsa ki az
X = a(sh t t), y = a(ch t t)
105
paramteres egyenletrendszerrel megadott grbe alatti terletet a O ^ ^ l interval
lumban!
8.211. Szmtsa ki az
X= 2 cos - c o s 2, j = 2 sin - s i n 2?
paramteres egyenletrendszerrel megadott kardioid terlett, ha 0 ^ ^ 27t .
8.212. Mekkora az
X = cos t, y = sin 2 t
paramteres egyenletrendszerrel megadott grbe alatti terlet mrszma a
0 ^ ^ - intervallumban?
8.213. Hatrozza meg az
X= cos t, y =
sin^ t
2 + sin t
paramteres egyenletrendszer grbvel krlhatrolt terletet!
Szmitsa ki a kvetkez egyenletekkel megadott grbk vhosszt a megadott
intervallumban:
4 -
8.214. = [0;4],
8.215. = In cos X,
^. 2 ^Q. y = f :, [1; 2],
8.217. 7 = In( l-x^),
71
^4
r
x^
8.219. j = In X, [l;e].
1 - x
2
r
106
8.221. y = 1In sin X,
% n
3 2
8.222. J; = - x ^ [0; 1],
8.223. y = { 2 + 2xf ,
1 r
6 3
2 2 2
8.224. Szmtsa ki az + egyenlet asztrois teljes vhosszt!
Szmtsa ki a kvetkez paramteres egyenletrendszerrel megadott grbk vhosz?
szt az adott intervallumban:
8.225. x = 2 t , y = 3^ 0 ^ ^ ^ .
8.226.
8.229. X = r(cos t + t sin t), y r(sin t t cos t),
n
8.230. X = sin t, y = co^ t, 0 ^ .
5n
M 7T _
8.231. X = a[ cos + In tg - , y = a sin , ~ ^ t ^ .
\ 2 J 2 6
8.232. X = ch^/,>' = sb^/, O^^l.
8.233. X= - t h O ^ ^ l .
eh t
8.234. X = a(ch t ~t ) , y = a(ch t +t ) , O ^ / ^ l .
8.235. X= sh eh , j = 2 eh , O ^ ^ l .
107
8.236. jc = ^ ch^ , = sh t, 1.
8.237. X = {t^ 2) sin + 2/cos t, y = (2 ^)cos t + 2t sin t, O^ t ^ n .
8.238. Mekkora az j 1 s az x tengely kztti tartomny x tengely krli
megforgatsakor keletkez forgstest trfogata?
2 2 2
8.239. Mekkora az x^+y^ = egyenlet asztrois megforgatsval keletkez
forgstest trfogata?
8.240. Forgassa meg az x tengely krl az 5y = x^-4x + 9 s az
5y = - x^ + 6x + 1 egyenlet parabolk ltal kzrezrt vges tartomnyt! Mekkora
a keletkezett forgstest trfogata?
Forgassa meg az albb felsorolt grbeveket az x tengely krl, s szmtsa ki a
keletkezett forgstestek trfogatt:
n n
2 2
8.242. >^= eh X, [-1;1].
I ,
8.243. y ~ ^ *^ ^]-
8.244. y = 2xe^, [0; 1],
8.245. j = In X,
8.246./ - x ^ = I, [0;3],
8.247. y = xl n X, [l;e]
8.248. y = ]/2x+l, [0; 13],
4
8.249. y = xj/r+x^, [0; 1].
108
8.251.;; = ^------- , [0; 1],
2 x - \
8.252. y =
, [0; 1].
8.253. y = e ^ |/sin x, [0; n].
8.254. >= tg 2x,
n
8.255. y = eh x, [0; In 2].
Forgassa meg a felsorolt grbeveket az y tengely krl! Szmtsa ki az gy keletke
zett forgstestek trfogatt:
8.256. ][i+][y = l, l ^ y 4 .
2
8.257. y =
8.258. y =
l + 2x^
1
l ^ y 2 .
l ^ y S 2 .
1
8.263. Forgassa meg az y tengely krl az y = az 3; = - x ^ egyenlet parabolk
s az > = 4 egyenlet egyenes ltal kzrezrt vges tartomnyt! Mekkora a keletkezett
forgstest trfogata?
8.264. Tekintse azy = e^, y=e~^ s az x= 1 egyenlet grbk ltal hatrolt vges
tartomnyt! Forgassa meg ezt a tartomnyt az x tengely, illetve az y tengely krl!
Szmtsa ki a keletkezett kt forgstest trfogatt!
109
8.265. Az y = |/x egyenlet grbe s az x + j = 2 egyenlet egyenes kztti
vges tartomnyt megforgatjuk az x tengely, illetve az y tengely krl. Szmtsa ki
a keletkezett kt forgstest trfogatt!
8.266. Tekintse az y = 4 s az y = 9 x^ egyenlettel megadott fggvnyek
grbi kztti sikrszt az els sknegyedben! Forgassa meg ezt elbb az x, majd az
tengely krl! A keletkezett forgstestek kzl melyiknek nagyobb, s mennyivel
a trfogata?
8.267. Az y = x^ egyenlet grbe O^ x ^ a intervallumba es darabjt megforgat
juk az X, illetve az y tengely krl. Az a mely rtke mellett lesz az gy keletkezett kt
forgstest trfogata egyenl?
Forgassa meg a kvetkez paramteres egyenletrendszerrel megadott grbket az
Xtengely krl, s szmtsa ki a keletkezett forgstestek trfogatt az adott intervallu
mon:
8.268. X= eh , y = sh , l ^ / ^ 4 .
n
8.269. X= cos , j = sin 2, 0 ^ ^ .
n
8.270. X = sin t, y = t, 0 ^ t ^
8.271. x = ^ J = 2/^ l ^ / ^ 2 .
8.272. Forgassa meg az
X = r(cos / + / sin t), y = r(sin t t cos t)
paramteres egyenletrendszerrel megadott fggvny grafikonjt az y tengely krl!
Szmtsa ki a keletkezett forgstest trfogatt, ha 0 ^ / ^ ^ !
8.273. Szmtsa ki az x = a cos t, y = b s\n t paramteres egyenletrendszerrel
megadott ellipszis megforgatsbl nyert forgsi ellipszoid trfogatt!
8.274. Forgassa meg az x = t n t , y = 1- cos paramteres egyenletrendszer
rel megadott ciklois intervallumba es vt mindkt tengely krl! Szmtsa
ki a keletkez forgstestek trfogatt!
*____
8.275. Tekintse az x= ; y ^9 egyenletrendszerrel megadott zrt grbt!
Forgassa meg az x tengely krl, s szmtsa ki a keletkezett forgstest trfogatt!
110
Forgassa meg az albbi grbk megadott darabjt az x tengely krl, s szmtsa
ki a keletkez forgstestek palstjnak a felsznt:
8.276. j = eh X, - l ^ x ^ l .
8.277. j = ^Jc^ O^x^l.
8.278.7 = 1^ ^ , - 3 ^ x ^ 3 .
8.279. y = p - 2 x , 0 ^ x ^ 2 .
8.280. y = sinx, O^ x ^ n .
8.282. = 1,
B.283. y = ] f 2 ^ , 2 ^ x ^ 3 .
8.284. Mekkora azx^ + {y-2)^ = 1egyenlet kr jc tengely krli megforgats-
val nyert trusz felszne?
2 2 2
8.285. Mekkora az egyenlet asztrois x tengely krli megforgatsa-
kor keletkezett forgstest felszne?
x^ y^
8.286. Szmtsa ki az + = 1egyenlet ellipszis x tengely krli megforgat-
sval nyert ellipszoid felsznt!
8.287. Forgassa meg az y^ = 8x egyenlet parabola 0 ^ x ^ 6 intervallumba es
darabjt az x tengely krl, illetve az y tengely krl! Szmtsa ki a keletkezett
forgstestek palstjnak a terlett!
Forgassa meg az albbi paramteres egyenletrendszerrel felrt grbk megadott
darabjt az x tengely krl, s szmtsa ki a keletkezett forgstestek palstjnak a
terlett:
8.288. x = t ^ , y = t ,
I l i
8.289. X ^ a cos^ t, y = a sin^ t, 0 ^ ^ ^ .
8.290. X = cos t, y = sin t, 0 ^ ^ .
t n
8.291. X = cos + ln t g - , y = sin , - ^ t ^ n.
8.292. Mutassa meg, hogy az x = r cos , j; = r sin paramteres egyenletrend
szerrel megadott kr megforgatsbl nyert gmb felszne \r^n.
8.293. Forgassa meg az jc = (-sin t), y = (1-cos t) paramteres egyenlet
rendszer ciklois O ^ t ^ n intervallumba es vt az y tengely krl! Mekkora a
keletkezett forgstest palstjnak a terlete?
Hatrozza meg az albbi homogn tmegeloszls s egysgnyi felleti srsg
sklemezeknek
a) az X s az j tengelyre vonatkoz elsrend (statikai) nyomatkt;
b) a. slypontjnak koordintit;
cj az Xs az y tengelyre, valamint az origra vonatkoz msodrend nyomatkt:
8.294. Az y = x^ egyenlet parabola s az x tengely kztti tartomny, ha
0^ x ^ 2.
8.295. Az y = f x egyenlet parabola s az x tengely kztti tartomny, ha
0 ^ x ^ 4 .
8.296. Az y = 4 x^ egyenlet parabola s az x tengely kztti sikrsz.
x^ y^
8.297. Az + = 1 egyenlet ellipszis x tengely feletti ive s az x tengely
kztti skrsz.
8.298. Az y = \ nx egyenlet grbe s az x tengely kztti tartomny, ha 1^ x ^ e .
8.299. P\2 .y = sh Xegyenlet grbe s az X tengely kztti tartomny, ha O ^ x ^ l .
8.300. Az = cos x fggvny s az x tengely kztti skrsz, ha 0 ^ x ^ ^ .
112
Hatrozza meg az albbi sklemezek slypontjt:
8.301. Az X = r cos , j = r sin t paramteres egyenletrendszerrel megadott
krcikk, ha 0 ^ ^
8.302. Az X = a(/-sin i), y = a(l - cos t) paramteres egyenletrendszerrel meg
adott ciklois ve s az jc tengely kztti vges skrsz, ha 0^ ^ 2;t.
8.303. Szmtsa ki az x^ + (j + 1)^ = 4 egyenlet kr x tengely feletti rsze sly
pontjnak a koordintit!
Hatrozza meg a kvetkez egyenletekkel megadott homogn tmegeloszls s
egysgnyi vonalmenti srsg grbknek
a) az X s az j; tengelyre vonatkoz elsrend (statikai) nyomatkt;
b) a slypontjnak a koordintit;
c) a z x s az y tengelyre, valamint az origra vonatkoz msodrend nyomatkt:
8.304. x^ + y2 = ^2^ [0;r],
8.305. 7 = 2 eh , [0;2],
3
8.306. J = x^ [1;2].
2 2 2
8.307. x^ + / = [ 0 ; 4
3n
8.308. X = a cos^, y = a sin^, n ^ t ^ ~ .
8.309. X= 3(/-sin t), y = 3(1 - cos t),
8.310. X= 4 cos /, 3; = 4 sin , 0 ^ t ^
Hatrozza meg az albbi egyenletekkel megadott grbk x tengely krli megfor-
gatsval keletkez homogn tmegeloszls s egysgnyi srsg forgstesteknek
a) az (y;z) skra vonatkoz elsrend (statikai) nyomatkt;
bj a slypontjt;
cj az X tengelyre vonatkoz msodrend nyomatkt:
8.311. j = eh X,
8.312. = O^AT^l.
2 x
8.313. j = x^, - l ^ j c ^ O .
8 . 3 1 4 . / _ ^ 2 = i o^ x^3.
8.315. j = 1/5^, OS x ^ l .
v2
8.316. ^ + y = 1 - 4 ^ a:^4.
8.317. x = d j = 2/^ O/^1.
8.318. X= 2 cos /, j = sin /, 0 ^ ^ ^ .
8.319. X= a(-sin t), y = a ( l - c o s /),
8.320. Forgassa meg azy = x^ egyenlet grbe 1 intervallumba es darab
jt az X, illetve az y tengely krl! Szmtsa ki a keletkezett homogn tmegeloszls
s egysgnyi srsg forgstestek
a ) elsrend (statikai) nyomatkt;
b) slypontjt;
c) msodrend nyomatkti
8.321. Forgassa meg az x ~ t, y = - paramteres egyenletrendszerrel megadott
grbe 1 intervallumba es darabjt az y tengely krl! Szmtsa ki a keletke
zett homogn tmegeloszls s egysgnyi srsg forgstestnek
a) az (x; z) skra vonatkoz statikai nyomatkt;
b) &slypontjt;
c) az y tengelyre vonatkoz msodrend nyomatkt!
Hatrozza meg az albbi grbk x tengely krli megforgatsval keletkez homo
gn tmegeloszls s egysgnyi srsg forgsfelletnek
a) az (y;z) skra vonatkoz elsrend statikai nyomatkt;
bj a slypontjnak a koordintit;
cj az X tengelyre vonatkoz msodrend nyomatkt:
8.322. y = l , l ^ x ^ 4 .
114
B. 323. y = x, O ^ x ^ l .
8.324. y = i f x, 0^jc^3.
8.325. = 1,
8.326. X = a sin^ t, y a cos^ t, 0 ^ ^ .
8.327. x = 3cost, >^ = 3 sin/,
8.328. X = / - s in /, = 1 - cos /, O^ t ^ n .
Hatrozza meg az albbi grbk y tengely krli megforgatsval keletkez homo
gn tmegeloszls s egysgnyi srsg forgsfelleteknek
a) az (x; z) skra vonatkoz elsrend (statikai) nyomatkt;
b) a slypontjt;
c) az y tengelyre vonatkoz msodrend nyomatkt:
8.329. j = 0^jc^2.
8.330. X = cos^ t, y = a sin^ t, 0 ^ t ^ ~ .
8.331. Szmtsa ki a kvetkez improprius integrlokat:
- 2
1
b)
a)
I
~^dx;
1
GO
d)
x f + x
:dx;
00
00
e)
1
x ^ - 5 x + 6
c)
x^ + 4x + 5
dx\
dx; f ) dx;
g) ^4xe-^^dx;
j )
dx;
-'21/ ^
h) j In x,dx;
0
k) dx;
i)
l)
1
dx;
x ^
8*
115
m) J In X dx\
0
n)
3x
dx;
o)
1
x ^ - 2 x - 3
dx.
8.332. Szmtsa ki az
cos X
2 n x
dx integrlt a [0; tt] intervallum 6 egyenl rszre
osztsa esetn
a) SLtrapzformula felhasznlsval;
b) a. Simpson-formula felhasznlsval!
8.333. Szmtsa ki az J e dx integrlt
0
a) a [0; 1] intervallum 10 egyenl rszre val felosztsa esetn, a trapzformula
felhasznlsval;
b) a [0; 1] intervallum 10 egyenl rszre val felosztsa esetn, a Simpson-formula
felhasznlsval;
c) ha az integrland fggvnyt a Maclaurin-sornak tizedfok polinomjval
helyettestjk!
d) Becslje meg az elkvetett hibt!
8.334. A Simpson-formulval szmtsa ki kzeltleg az albbi tblzattal meg
adott fggvny grafikonjnak az x tengely krli megforgatsval keletkezett test
trfogatt:
X 0 2 4 6 8 10 12
y
2 6 7 7 3 4 5
8.335. A Simpson-formulval szmtsa ki kzeltleg annak a szabsmintnak a
terlett, amelynek a szlessge az elejn 10 cm, majd 5 cm-enknt mrve a szlessget
12; 20; 18; 16; 20; 16; 25; 28; 31; 34 cm!
8.336. Simpson-formulval szmtsa ki kzeltleg annak a forgsszimmetrikus
gpalkatrsznek a trfogatt, amelynek az egyik, szls tmrje 10 cm, majd
4 cm-enknt mrve az tmrket, 14; 18; 20; 22; 18; 16 cm-t kapunk!
8.337. Egy 80 mm hossz, homogn anyagbl kszlt forgsszimmetrikus gpal
katrsz tmrje 20 mm-enknt mrve:
/o = 62mm; /i = 64mm; / 2 = 62mm; <3= 55mm; 4 = 42mm.
A gpalkatrsz slya 1,71 kp. Hatrozza meg az alkatrsz anyagnak fajslyt 2
tizedesjegy pontossggal
a) ^ trapzformula felhasznlsval;
b) a Simpson-formula alkalmazsval!
116
8.338. Egy homogn sklemez egyik szle 60 mm hossz egyenes szakasz, kt
oldala erre merleges szakasz, msik szle grbe vonal. Az egyenes szakaszon balrl
jobbra haladva a lemez szlessgt 15 mm-enknt megmrve, a kvetkez rtkeket
kaptuk; Wo = 30mm; mi = 35mm; W2 = 25mm; m3=16mm; m4=12 mm.
A Simpson-formula alkalmazsval hatrozza meg
a) 2Llemez terlett;
b) a. lemez slypontjnak a tvolsgt a lemez egyik szltl!
8.339. Egy 120 mm hossz forgsszimmetrikus gpalkatrsz tmrjt 30 mm-
enknt megmrve, a kvetkez rtkeket kaptuk: 40 mm; 46 mm; 48 mm; 50 mm;
g
44 mm. A gpalkatrsz alumniumbl kszlt, srsge o = 2,3 r . Szmtsa ki a
cm^
gpalkatrsz tmegt
a) a trapzformula alkalmazsval;
b ) a. Simpson-formula alkalmazsval!
8.340. Szmtsa ki az y=f {x)\ y = 0-, x = 0; x=12 egyenlet grbkkel hatrolt
szimmetrikus homogn sklemez slypontjnak a koordintit, ha tudja, hogy
/(0) = 2;/(2) = 3;/(4) = 4;/(6) = 5
a) a. trapzformula alkalmazsval;
b) a. Simpson-formula alkalmazsval!
8.341. Hatrozza meg azy = f{x) grbe s az x tengely ltal hatrolt szimmetrikus
homogn sklemez slypontjnak a koordintit, ha tudja, hogy /(0) = 0; /(3) = 5;
/(6) = 6;/(9) = 8;/(18) = 0
a) a trapzformula alkalmazsval;
b) a. Simpson-formula alkalmazsval!
117
9. SOROK
A szmsor konvergencijnak defincija alapjn vizsglja meg az albbi szmso
rok konvergencijt! Ha konvergensek, akkor szmtsa ki a szmsorok sszegt is!
T
l + 3* + 5
3^+1
A' =0
9-6- I
00 4^ + 3
9-7-
A=0 ^
9.8. ^ - j T ) , ahol a megadott pozitv szm.
Vizsglja meg, hogy a kvetkez pozitv tag szmsorok konvergensek vagy diver-
gensek-e?
118
a. J
CC t
9.10. y ------------------------------ .
, ^ , i k + l ) i k + 2 ) ( k + 3 )
9.11. I
9 * 4 * 3 2
1
, % { c + k ) { c + k + \ )
, ahol c adott nemnegatv szm.
9.17. X In
i = 2 \
\
llaptsa meg, hogy az albbi szmsorok a paramterek mely rtknl konvergen
sek! Hatrozza meg a szmsor sszegt is!
9.19. Y, (sin 2a), ahol aeR.
k =0
119
^ 2b Y
9.22. f ( - 3 c ~ Y , ahol c e R \ { 0 } .
co ^k +2
9.23. ahol JeR.
k =0^ ^
A Cauchy-fle konvergenciakritrium segtsgvel vizsglja meg az albbi szmso
rok konvergencijt!
co I
9.24.
k=\ /t
co l
9.25. X
bk
9.26. X - T ahol 0^>t<10.
A= 010
Valamelyik konvergenciakritrium segtsgvel dntse el, hogy az albbi pozitv
tag szmsorok konvergensek vagy divergensek:
* 1
9-27. Z p .
t=l ff
1
00 3
1
co 1
9.32. y ^ .
120
* 3
1
2 k +\
GO I
9-38. S
if^ 2 In k
* 5A; + 3
9-40. I
^ 7
^ 8A:^+1
00
9 . E ^ .
k=\ ^
k=\
00 I
9 . 4 5 . 1 ^ .
*=i K
121
9.46. Yj ^hol 0 < / j < l .
*=i
00 k
00 1
9.47. I .
'^=^kik
1
9.48. S
k =0 !
00 j
9.49. I
00 2 *
9.50. I
Jk= 0 ^
00 2^
c
/t!
9-54. I - .
fc = 0 ^
V ( ^ + l ) ( ^ + 2)
' ^ I
fc = 0 ^
9-56. I
f c=l K
9-57. Z
oc J^k
T
122
9-59. I ^
jfc=l ^
00 1^2
2 k^
k~i
sin- l
\/k + 2 + \ / k-2
9.62. y --------- ^ ------.
k =2 3]/k
00 4^+1
9 - 6 3 . 1 ^ .
V3>
9.64. I
Jt = 3 / ^ \
123
9.71. X
1
ut, { k +l ) i k + 4)
k +l
00 J
9.74. X
t=i k + ^
oo
\ V
9.75. E .
k = 2
\ ^ /
s i \ ^
9.77. E * k
fc=l \ ^ j
9-78. E
/c=i e"
1
9.80. X
k In^ k
(arctg kY
9.81. X
, ^ , { k + 2 )2 ^-^'
(th k f
k - l
9.82. X ( /^ - 2 1 ^ T T + | / ^ ) .
k = 1
9.83. X
1
fc=i ^/fc+I+|/^+2 '
124
9.84. X [ 2 k - ^ - k ^ ) .
k =l
k = l
Dntse el, hogy az albbi vltakoz eljel szmsorok konvergensek vagy diver
gensek :
00 J
k+l __i_
9.86. X ( - 1 )
.=1 2 k - l
( - I f
k
00 2 k \
9.89. X
k =i ^
9.90. X (-O'-O,!.
fc = 0
9.92. X ( - l ) ' -0,l*+i.
k = 0
2 k +l
kt'i ' ' k^ + k
9.93. I ( - 1 ) \ 2
00 fc + 1
9.94. (-1)' 1^^.
k = 0
9.95. f; 3(-0,l)^
k= 0
125
Hatrozza meg a kvetkez vals fggvnysorok konvergenciatartomnyt s
sszegfggvnyt (ahol x e R):
.96. I (
k = i \ x -
*=o(l+Jc)*
; x # l .
t =0 (x 1)
9.100. 10-V.
i = 0
9.101. X
(x+ 1)*
,t-iA:(A:+l)(A: + 2)
k
,t'i(^+l)(A: + 2)(A:+3)
4)
'
*!?
[=1
* 2A:+1 / j c - l V
9.106. f v **.
t = 0
9.107. X (tg^)
t=o
126
2x
9.108. y ------.
00
9.110. X (3x)^
k =0
llaptsa meg az albbi vals fggvnysorok konvergenciatartomnyt, ahol
x e R:
9.111. X k\x!.
k = 0
9.112. X kx^.
k = l
00 2*~^
^ k { X V
9.115.
f c=l ^
00 2 ^
9.116. I
jfc= 0
9.117. X
jfc=l ^
X
X
9.120. I - ~ x \
k=0 K '
127
9.121. I - x * .
k=l
00 + 9
9.122. Z
k=l ^
00 ^2 I 1
9.123. Z
00
1= 0 k +l
9.124. y
/1-xN
jj = i 2A: \1 +x j
x ^ - l .
Hatrozza meg az albbi vals fggvnysorok (xeR) konvergenciatartomnyt!
Dntse el, hogy a fggvnysor a konvergenciatartomny mely pontjaiban abszolt
konvergens, s melyekben felttelesen konvergens:
fc= i (l+x)(2 + x)(3 + x)...(A: + x)
; X# - 1, - 2 , - 3 , - k .
t=i
kx
JC
Hatrozza meg az albbi fggvnyek Maclaurin-sort! llaptsa meg, hogy ez a
hatvnysor mely vals szmokra lltja el az f ( x) fggvnyt!
9.131./(x)==sh 2x.
9.132./(x):=ch3x.
128
9.133./(x):=2".
9.134. /(x):=sin^ X.
9.135./(x):=cos^ X.
9.136./(x)==sin 3x.
9.137. /(x)!=cos 3x.
9.138./(x)==e-^\
9.139./(x)==xe^
9.140./ ( x ) : = : ^ ; x # - l .
1 + x
9 . 1 4 1 . / ( x ) : = - ^ ; x # l .
1 - x
9.142./(x) ^
l+x^
9 . 1 4 3 . / ( x ) = = : ^ ; x^3.
3 - x
9.144./ W ^ ;
9.145. / ( x ) ~ arctg x.
9.146./(x)==ln(x^+l).
9-147./(x) = = ^ ^ ; x ^ l .
9.148./(x)==ln(x+1) ; - K x .
Hatrozza meg a kvetkez fggvnyek Maclaurin-sornak els hrom tagjt:
9 . 1 4 9 . / ( x ) : = ^ .
COS X
9 Matematikai feladatok 129
9.150./(x) == tg X.
9.151./(x)== th X.
9.152. /(x) == arcsin x.
9.153. /(x) := arccos x.
9.154. /(x) := arth x.
9.155./(x) := XIn ( x + 1); l<x.
9.156./(x) != In |cosx|.
9.157./(x)==e^sh X.
Hatrozza meg az albbi hatvnysorok konvergenciatartomnyt s sszegfggv
nyt :
00
k = \
, ^^k( k+l )
' 2 k - l
k =C
X+l
9-162. y .
k=o 4k+ l
9.163. Szmtsa ki sin x harmadfok Taylor-pohnomja segtsgvel sin kzelt
rtkt! Becslje meg a kzelts hibjt!
n
9.164. Szmtsa ki cos x negyedfok Taylor-polinomja segtsgvel cos kzelt
rtkt! Becslje meg a kzelts hibjt!
130
9.165. Szmtsa ki e kzelt rtkt hatodfok Taylor-polinomja segtsgvel.
Becslje meg a kzelts hibjt!
9.166. Hatrozza meg Inx Taylor-sort 1 krl. Szmtsa ki In 1,1 kzelt
rtkt gy, hogy harmadfok Taylor-polinommal kzelt! Becslje meg a kzelts
hibjt!
9.167. rja fel tg x Maclaurin-sornak els hrom tagjt! Szmtsa ki tg 20 kzel
t rtkt gy, hogy harmadfok Taylor-polinommal kzelt! Becslje meg az elkve
tett hibt!
llaptsa meg, hogy a kvetkez fggvnyeket az adott intervallumban mekkora
hibval kzelti meg msodfok Taylor-polinomjuk:
9.168./(x)==cosx; 0 ^ x ^ 0 , l .
9.169./"(x) != eh x; 0 , l ^ x ^ 0 .
9.170./(x)==e*; 0 ^ x g 0 , l .
Rendezze t az albbi fggvnyeket x - a hatvnyai szerint! (rja fel az/(x) fggv
nyek Taylor-polinomjt az a hely krl!)
9 . 1 7 1 . / ( j c) : = x ^ - 3x3+ 2x^-5;
9.172./(x)==x'^-x+l; a=\ .
9.173./(x)==x3-I-4x2 + 7x+2; a = - 2.
9.174./(x)==x3 + 7x^+18x+22; a = - 3 .
Oldja meg kzeltleg a kvetkez transzcendens egyenleteket! (Kzeltse az egyen
letekben szerepl transzcendens fggvnyeket Maclaurin-poUnomjuk nhny, alkal
masan vlasztott szm tagjval!)
9.175. 6 sin x + x^ = 2x2.
9.176. x^ = cosx.
9.177. = x^ + 8x+7.
9.178. 5 - 2 x = 2chx.
9* 131
Szmtsa ki a kvetkez hatrozott integrlok rtkt a megadott pontossggal, az
integrandus Taylor-polinomjnak alkalmasan vlasztott szm tagja segtsgvel:
9.179. = 0, e=10-.
0
9.180.
6 ^
dx\ a = 0, =10"*^.
9.181.
sm X
/x; a = 0, fi =10
9.182.
cos X
dx\ a = 0 ; e=10
9.183.
r shx
dx; a = 0 , e= 10
0,1
9.184.
0,5
' arctgx
dx; a = 0 , e=10
0,1
Hatrozza meg a kvetkez feladatokban megadott periodikus fggvnyek (x e R)
Fourier-sort:
9.185./(x) ~ |x|, ha n<x ^ n ,
s /(x) = f {x + 2 kn) minden x-re s minden egsz ^-ra.
f 2x+l, ha 7T<x^0,
9.186./(x)== <
2x+l, ha 0<x^7T,
s /(x) = f {x + l ki ^ minden x-re s minden egsz A:-ra.
9.187./(x) := 3x^, ha - n<x- ^ n,
s /(x) = f {x + 2 kn) minden x-re s minden egsz A:-ra.
132
j e ha 7T<x^0,
9.188./(x)==<
e"", ha 0 <x ^ n ,
s f{x) f ( x + 2 kn) minden x-re s minden egsz k-ra..
X
9.189./(x) == eh - , ha n<x-^ii,
s f{x) = f {x + 2 kn) minden jc-re s minden egsz k-xa..
9.190./(x) := c o s - , ha - n < x ^ n ,
s f (x) = f ( x + 2 kn) minden x-re s minden egsz k-ra.
3, ha 7T<x^0,
9.191./(x)==
- 3 , ha 0<x^;r,
s /(x) = f ( x + 2 k7ij minden x-re s minden egsz k-ra.
9.192./(x) := X, ha n<x Sn,
s/(x) = f ( x + 2 kn) minden x-re s minden egsz k-ra.
9.193./(x) == s h - , ha - n<x- ^ n,
s /(x) = f {x + 2 kTt) minden x-re s minden egsz ^-ra.
9.194./(x) := sin , ha n<x-^n,
s /(x) = f {x + 2 kn) minden x-re s minden egsz k-ra.
9.195./(x)
J
X
- + 1, ha ;r<x^O,
1, ha 0<x^7T,
s /(x) = f {x + 2 kn) minden x-re s minden egsz A:-ra.
9 . 19 6. /(x )-
0,
2 sin X, ha 0 < x ^ n,
s /(x) = f ( x + 2 kn) minden x-re s minden egsz ^-ra.
133
r u
ha n<x ^ O,
9.197./(x) ==
X
c o s - , ha 0 <x ^ n,
s f(x) = f {x + 2 kn) minden x-re s minden egsz k-ra.
1, ha n<x^O,
0, ha 0 <x ^ n ,
s /(x) = f {x + 2 kn) minden x-re s minden egsz k-ra.
X, ha ;r<x^O,
x^, ha 0<x^7i,
s/(x) = f ( x + 2 kn) minden x-re s minden egsz k-ra.
ha 7T<x^0,
2x
ha 0 < X^ 7t,
s /(x) = f { x +2 kn) minden x-re s minden egsz k-ra.
- x + 2, ha - K x ^ O ,
9.201./(x)== ;
x + 2, ha 0 < x ^ l ,
s /(x) = f ( x + 2 k) minden x-re s minden egsz k-ra.
9.202. f ( x) := x \ ha - K x ^ l ,
s /(x) = f {x + 2 k) minden x-re s minden egsz k-ra.
9.203. f i x) := eh X, ha - 1< x ^ 1,
s /(x) = f ( x + 2 k) minden x-re s minden egsz ^-ra.
{
2, ha 1< X^ 0,
2, ha 0 < x ^ l
s/X^) ~ f ( x+2k) minden x-re s minden egsz A:-ra.
9.205./(x) := 7tx, ha 2 <x ^2,
s /(x) = f ( x+4k) minden x-re s minden egsz k-ra.
9.206./(x) := shX, ha - 1 < x ^ 1,
s /(x) = /(x+2A:) minden x-re s minden egsz fe-ra.
134
9.207./(x)==
0, ha - K x ^ O ,
X, ha 0 < X^ 1,
s /(x) = f ( x + 2 k) minden x-re s minden egsz k-ra.
9.208./(x):=
1, ha l<x^O,
e*, ha 0 < x ^ l ,
s /(x) = f {x + 2 k) minden x-re s minden egsz k-ra.
135
10. DIFFERENCILEGYENLETEK
Sztvlaszthat vltozj differencilegyenletek
Oldja meg az abbi differencilegyenleteket! rja fel a differencilegyenlet ltalnos
megoldst! Ha kezdeti felttel is adott, akkor rja fel a kezdeti felttelt kielgt
partikulris megoldst is!
10.1. / =
1
X
XD = 2.
1 0 . 2 . / = / .
10.3. x y ' - y = 0 y{- 2) = 4.
10.4. xy' + y = 0; y( - 2) = 4.
10.5. ;;/ + x = 0; y( - 2) = 4.
10.6. x^dx + i y +l ^ d y = 0 .
10.7. 4ydx + xdy = 0.
^0.S. y ' = 4xf y, y(l)=l-
3
10.9./ = y(2) = 0.
10.10. { l +y) dx- ( l + x)dy = 0.
10.11. j - 2 + (jc + 3 ) / = 0.
10.1 2 . ( l - x^) dy + xydx = 0.
10.13./ - j = 0; y( - 2) = 4.
136
1 0 . 1 4 . / = , 2
4 1 - / )
y{ l - x^)
10.15. x( y^- l )dx + y{ x^ - l ) dy = 0.
10.16. (l + 2y ) dx - { 4- x ) dy = 0.
10.17. x + = 1.
dt
10.18. y ^ d x - x ^ d y = 0.
10.19. x{y- 3) dy = 4y dx.
10.20. / = (tgx) tgy.
10.21. / = 2"-^
10.22. / = 10"+^
W. 23. e>- ^dx + e^-dy = 0- j;(0) = 0.
10.24. X dy + 2y dx = 0; y(2) = 1.
10.25. xydx+( x+l ) dy = 0.
10.26. ;; = / In j ; y(2)=l.
10.27. xy dx + {l + x^) dy = 0.
10.28. (x + xy^) dy = 3 dx.
^0.29. xy' + y ^ y^; Xl) = 0,5.
10.30. l - y ^ = {\ +x^) ( y' f .
10.31. Axdy ydx = x^ dy.
10.32. x (x - \ ) y' +y{y- 1) = 0.
10.33. dy + {xy- xy^) dx = 0.
10.34./ - x / = 2xj.
137
10.35. 2 x V + r = 2.
1 0 . 3 6 . - ^ + - * ^ . 0 .
x ( y - l ) y(x + 2 )
10.37. xy' = e^ + 2/ .
10.38. (1 + dy- e^dx = Q\ y(0) = 0.
Ay
10.39. / =
x(y-3)
10.40. {I - x^)y' - 2xy^ = xy.
10.41. ^y^ + 1dx xy dy.
10.42. y(9 + 4x^)y' = 1.
10.43. 4xy^ dy 3y^ dx = xy^ dx + I x dy.
10.44. xy d x +^ \ - x ^ dy = 0.
10.45. (x^-2x)y' = 2(xy + x - y - l ) .
10.46. (2 y + xy)y' = / - l .
10.47. ( 1 + x^) dy- x^y dx = 0; y{\) = 2.
10.48. xydy = {y-^ \) { \ - x ) dx.
10.49. y \ x ^ - \ ) c i g y = 1.
10.50. (\ + 2y) dx + {A x^) dy = 0.
10.51. j / + xe^ = 0; y(l) = 0.
10.52. xy^ dx = x^ dy Ax dx.
10.53. x V + tgy = 3x^ tg;;; XI) = 7 -
6
10.54. yy' + y' -y' ^ sin^ 2 x = 0 ', y(0) = 1
138
10.55. 5yy' - 4t gy^ = 2xyy'.
10.56./ c t g x + > ; = 2; j ; ( 0 ) = - l .
71
10.57. xy' - cos 2y = 1 ; lim j .
4
10.58. ( x^ - l ) y' + 2xy^ = 0; X0)= 1.
10.59. 3 1 n ( x - l ) + 2 / = 2x y ' - 9y ^ l n( x - l ) .
10.60. x^( y+l ) dx + y \ x - l ) d y = 0 .
10.61. cos y dx + {\ + e sin j /j = 0; y(0) = - .
10.62. {xy^~ x) dx-^{y +xy) dy = 0.
10.63. 2 / = x(cos^ x) (1 +y) In (1
10.64. (x sin x ) dx ydy = {ly^x sin x) dx.
10.65. |/3 x^(sin y)y' = xcos^y.
10.66. cos ydx + {l+ e*) sin ydy = 0; XO) = 0.
10.67. 2rcos t drtg t dt = 0.
10.68. = Ixylny.
10.69. ( 1 + / ) / = J-
10.70. (2x+ l )y' - 3y = 0; y(0) = 5.
1 0 . 7 1 . / = / - 8 j + 1 2 .
10.72. 2/ = y ^ - 6y ^ +l l y - 6.
10.73. y - y x \ n x = 0.
10.74. x0;2+l)-_>;(x2+l) = 0.
10.75. x^yy' + xy^ + 9yy' - 4x = 0.
139
10.76. / sin^ x - y \ n y = 0.
X yy'
1 0 . 7 7 . ^ = 0; P( l ; - l ) .
\ +y 1+x
10.78. yy^yy' ^- ^ = 0; P(l; 1).
10.79. (x^y-^(iy')y' + xy^ x = 0; P(3; 1).
10.80. ( x+l ) yy' - x^( y+l ) = 0.
10.81. j j ' = x^y + 4 y - x ^ - 4 .
y + 2 y^ + 4y + 8
10.83. (sin x)cos^_y + /(cos x - l)sin j = 0.
10.84. (x^ + x^y)y' + y^ + xy^ = 0.
10.85. (e^^-4e^-5)y' = e\
1
10.86. l/25-x^ / =
COS^J
10.87. (l+x^)yy' = - ------ .
arctg X
10.88. /(5-4COSX)/ = 1.
10.89. 2 ^ ^ / = ( x + l ) ( / + 4).
10.90. (2 y + 3)y' = (e^+l) (j^ + 2y+5).
10.91. / = xy + 3 y - 2 x - 6 .
10.92. /cos y = (sin y)tg^ x.
10.93. p x - i y = y \ n y .
10.94. (y^ sin^ x)dx = 6(ctg^ x) (y^+l)dx.
140
10.95. y^( \ + 2 sin x + cos x)y' = y+l .
10.96. (2e^^-3e^+l) {2y + 4)dy = ( 2 / + 8;;) (3 - 4e^)dx.
10.97. (x^+ 1)/ sin(ln y) = xy.
10.98. y' sin X= j In y.
10.99. x]/ l -y^ + yy' ]/l~x^ = 0; P(0;1).
10.100. x ( l - x V = (x^ + x+l);;.
10.101. 5 gramm cukrot a kvnkba szrunk s lland keverssel biztostjuk a
cukor egyenletes olddst. Megfigyeltk, hogy a cukor fele 20 msodperc alatt
olddott fl. Mennyi cukor lesz oldott llapotban 1 perc mlva a kvban?
10.102. A rdium bomlsi sebessge minden idpillanatban egyenesen arnyos a
jelen lev tmegvel. Hatrozzuk meg, hogy az tmeg rdium hny szzalka
bomlik el 100 v alatt, ha tudjuk, hogy a rdium felezsi ideje 1590 v!
10.103. A vasalprsbl kivett ruhadarab 100 C-rl 60 C-ra 10 perc alatt hl le.
A terem hmrsklete 20 C. Hny perc alatt hl le a ruhadarab 25 C-ra?
10.104. A kemencbl kikerl kenyr hmrsklete 30 perc alatt a kezdeti
120 C-rl 60 C-ra cskken. A leveg hmrsklete a trolban 30 C. A hls
kezdettl szmitva mennyi id alatt cskken a kenyr hmrsklete 40 C-ra?
10.105. Egy 1kg 20 C hmrsklet vizet tartalmaz ednybe egy 0,5 kg tmeg,
/
75 C hmrsklet alumniumtrgyat dobunk az alumnium kzepes fajhje:
\
J \
0,2------ . A vz hmrsklete 1 perc alatt 2 C-t emelkedik. Mikor lesz a test s a
kgC/
vz hmrskletnek a klnbsge 1C? (A hvesztesgektl eltekintnk.)
10.106. Egy tartlyban lev 100 1soldat 6 kg st tartalmaz. A tartlyba 3 l/perc
sebessggel tiszta vz folyik be, s ugyanennyi oldat folyik ki. Mennyi lesz az oldat
startalma egy ra mlva? (A s egyenletes eloszlsrl lland keverssel gondosko
dunk.)
10.107. Egy 100 literes tartlyban 10 kg s van oldott llapotban. A tartlyba
percenknt 5 liter vz folyik be s ugyanannyi oldat folyik ki. Mennyi s lesz a
tartlyban egy ra mlva? (A s egyenletes eloszlsrl lland keverssel gondosko
dunk.)
141
10.108. Egy 200 trfogat teremben a leveg szn-dioxid tartalma 0,15%.
A szellzberendezs percenknt 20 m^ 0,04% szn-dioxidot tartalmaz levegt juttat
be a terembe s ugyanennyi levegt szv ki a terembl. Hny perc alatt cskken a
teremben a leveg szn-dioxid tartalma az eredeti koncentrci ktharmadra?
10.109. Egy baktriumtenyszetben a baktriiunok szmnak nvekedse egyene
sen arnyos a mindenkor jelenlv baktriumok szmval. Egy ksrletben megfigyel
tk, hogy a baktriumok szma 4 ra alatt duplzdik meg. a) Mekkora a baktrium
llomny a 12. ra vgn? b) Ha egy msik alkalommal a 3. ra vgn 10"^, az 5. ra
vgn pedig 4-10"'^ szm baktriumot figyeltnk meg, akkor hny baktrium volt
a ksrlet elejn?
10.110. Egy telepls lakosainak a szma 50 v alatt megktszerezdik. Hny v
alatt hromszorozdik meg a lakossg, ha a szaporods teme vltozatlan?
10.111. Tegyk fel, hogy a lakossg szmnak gyarapodsa a lakossg szmval
egyenesen arnyos. Hatrozzuk meg a lakossg L szmnak s a idnek L(t)
sszefggst, ha feltesszk, hogy ez a fggvny folytonosan differencilhat, s
tudjuk, hogy valamely, ltalunk kezdetinek vett idpontban a lakossg szma Lqvolt,
s ez egy v alatt p szzalkkal ntt!
10.112. Egy R cm bels sugar hengeres tartlyban H cm magassgig ll a vz.
A tartly fenekn egy r cm sugar, kr alak nyls van (r < R). Mennyi id alatt rl
ki a lartly? (A vz kifolysi sebessge 0,6 fgh.)
10.113. Egy 60C-os kpszg, tlcsr alak edny cscsnl 0,5 cm^ terlet
nyls van. Az ednyben 10 cm magassgig vz ll. A nylst kinyitjuk, a) Hatrozza
meg a vz h magassgt a t id fggvnyeknt! b) Milyen magasan ll a vz az
ednyben 4 s mlva? c) Mennyi id alatt folyik ki az egsz vzmennyisg? d) Mennyi
id mlva lesz a vz magassga 4 cm? (A vz kifolysi sebessge 0,6 fgh.)
10.114. Egy 10 cm sugar, henger alak csvezetket 6 cm vastagsg szigetels
bort, amelynek hvezetsi egytthatja k = 3 10'^. a) Mekkora az 1 m csre jut
hvesztesg rnknt, ha a csben raml gz hmrsklete = 200 C, a kls
hmrsklet Tj = 30 C? Mekkora a szigetels hmrsklete a cs kzpvonaltl
szmtott x(> 10) tvolsgban?
10.115. Amikor az ejternys ernyje kinylik, sebessge 30 . A lgellenlls
2 S
mv
kg, ahol m az ejternys (teljes) tmege, v a pillanatnyi sebessge. Hatrozzuk
16
meg az ejternys v sebessgt az ejterny kinyitstl szmtott id fggvnye
knt! (g~ 10 m/s^.) Mi trtnik akkor, ha az ejternys igen magasrl ugrik (ha t elg
nagy)?
142
10.116. Bizonytsa be, hogy ha egy test a nehzsgi er hatsra gy mozog egy
ellenll kzegben, hogy a kzegellenllsbl ered er a mozgs irnyval ellenttes
irny s a test sebessgnek ngyzetvel arnyos nagysg, akkor elg hossz id
elteltvel a test mozgsa egyenletesnek tekinthet!
10.117. Tekintsnk egy ramkrt, amelyben az Eq elektromotoros er, az R
(ohmikus) ellenlls s az L nindukcis egytthatj tekercs sorba van kapcsol
va. Legyen a bekapcsolt ram Eq feszltsg egyenram. rjuk fel az ramerssget
az id fggvnyben!
10.118. Statisztikai megfigyelsek igazoljk, hogy egy j termk irnti kereslet
kezdetben lassan, bizonyos id elteltvel egyre gyorsabban emelkedik, majd a kereslet
lanyhul s vgl a telitettsgi szint kzelben kzel lland. Ha y'(t) jelenti a kereslet
vltozst a t idpontban s A a teltettsgi szintet, akkor y' arnyos a pillanatnyi
kereslettel s a teltettsgi szinttl val eltrssel: y' = ky(A y).
a) Oldja meg a differencilegyenletet!
b) Mi a megolds, ha kezdetben ( = 0) a keresletet egysgnyinek tekintjk?
c) Ab ) felttel mellett mikor ri el a kereslet a teltettsgi szint 90%-t?
10.119. Melyek azok a grbk, amelyeknek brmely pontjban a pontbeH rint
meredeksge arnyos a pont abszcisszjval?
10.120. Melyek azok a grbk, amelyeknek brmely pontjban a pontbeH rint
meredeksge fordtottan arnyos a pont abszcisszjval?
10.121. Melyek azok a grbk, amelyeknek brmely pontjban a pontbeli rint
meredeksge arnyos a pont ordintjval?
10.122. Melyek azok a grbk, amelyeknek brmely pontjban a pontbeH rint
meredeksge fordtottan arnyos a pont ordintjval?
10.123. Hatrozzuk meg annak a grbeseregnek az egyenlett, amelyiknek min
den grbjre fennll, hogy a P( x; y) pontjhoz tartoz normHsnak az x tengelyig
terjed darabja ugyanakkora, mint a P pontnak az origtl mrt tvolsga!
10.124. Melyek azok a grbk, amelyekre az a s x abszcisszk kz es grbe
alatti terlet mrszma megegyezik a megfelel vhossz mrszmval?
10.125. Hatrozza meg annak a grbnek az egyenlett, amely illeszkedik a
/ iO; 0) s ^ 2(2; 8) pontokra, s brmely pontjhoz tartoz rintje, ordintja s az
Xtengely ltal hatrolt vges hromszg terlete k lland!
143
y' = f{ax + by+ c) tpus differencilegyenletek
10.126. / = X + ;;.
^ Q ^ 2 ^ . y ' - y = 2x-3.
10.128. / = { y - Ax f .
1 0 . 1 2 9 . / = -2(2x + 3 # .
10.130. / = (4x + j - 3 ) ^ .
1
1 0 . 1 3 1 . / = .
x + y
10.132. {x + 2y)y' = 1;X0) = -1-
10.133. / =
1
+ 1.
x - j
10.134. (JC+ / V = 1-
10.135. / = ^y- 2x.
10.136.
10.137.;;
10.138. y
10.139. y
10.140. y
= l/4x + 2 y - l .
=
= e^++L
= c o s ( y - x ) .
= t g ( y - 2 x).
1 0 . 1 4 1 . / = tg^(x + y).
10.142. (x + y)dx + (3x+3y-4)dy = 0.
10.143. y' + 4x + 6 y = 4x^+ 12xy + 9y^+ 1.
144
10.144. xdy = {x + y)dx.
10.145. (,x + 2y) dx- x dy = 0.
2x+y
1 0 . 1 4 6 . / = ---------
x + l y
10.147. {lx + y)dx-^{y-^x)dy = 0.
y + x
Homogn differencilegyenletek
10.148. / =
10.149. / =
10.150. / =
y - x
x+y
x - y '
x + y
x + 2 y
10.151. x V = y ( x + y ) .
10.152. ( x ^ + / ) / = 2 xy.
10.T53. (y^ - x^)dx + xy dy = 0.
10.154. (x^ + 2xy)dx + xy dy = 0.
10.155. (2 x + 3y)dx + ( y- x) dy = 0.
10.156. y In - - x y ' = 0.
10.157. xy + y^ = (2x^ + xy)y'.
10.158. ( 3x^- y^) y' - 2xy = 0.
10.159. x j / = y^ + 2 x \
10.160. j;2 + x V =
10.161. ( y ^ - 2 xy)dx + x^ dy = 0.
10 Matematikai feladatok 145
10.T63. {x^ + l x y - 2 y'^)dx + {y^ + 2xy-2x^)dy = 0; XO) = 3.
10.164. 2xV = y{2x^-y^).
10.165. {x^ + y^)dx + ^xy^ dy = 0.
10.166. {x^ + 6 x^y^ + y*)dx-^4xy{x^-^y^)dy = 0; j ( l ) = 0.
10.167. {y + ^ ) d x = Xdy.
10.168. xy' = 2 { y - f ^ ) .
10.162. {x^+y^)dx + xy dy = 0; P(l; -1).
10.169. xy = i x ^ - y ^ + y.
10.171. xy' = y- xe^.
10.172. xy' = xe^ + y; y(l ) = 0.
y V n
10.173. / = - + s i n - ; XI) = -
X X 2
10.174. [ x - y cos - j j x + x cos - dy = 0.
y
10.175. xy' = jcos In
X
10.176. x y ' - y = (x+j)ln
x+y
10.177. I
x j
y -
sin - = e*.
X
146
- \ - /
10.179. (^l + 2e>J/jc+2e^
X
1---- ]dy = Q.
yj
10.180. Keresse meg azoknak a grbknek az egyenlett, amelyekre igaz a kvet
kez llts; a grbk tetszleges P{x\ y) pontjnak az origtl mrt tvolsga ugyan
akkora, mint az a szakasz, amit a P pontban a grbhez.hzott rint az y tengelybl
(az origtl mrve) lemetsz.
Elsrend lineris differencilegyenletek
10.181./ - j ' = 1.
10.182. y ' - y = 2 x + 2 .
10.183. y ' - y = x^ + 2 .
10.184. = 4e~\
10.185. y ' - y = e^^ + x+l .
10.186. y ' - y = sin x.-
10.187. y ' - y = eh 2x.
10.188. y ' - y = 3e^^+cos x.
10.189. y ' - y = 2 e^^ + sin 2x + x - l .
10.190. y ' - y = e^.
10.191./ - j = eh X.
10.192. y' + 3y = cos x+6.
di
1C.193.----- 6/ = 10 sin It.
dt
10.194. / + 2j; = sh 2x + 2x^
10.195. A sorosan kapcsolt R (2) ellenlls, L (H) nindukcis egytthatj
ramkrben U sin co bels feszltsg hat ( a krfrekvencit, t az idt, U a bels
10* 147
feszltsg maximumt jelenti). Hatrozza meg az I ramerssg rtkt tetszleges t
idpontban, feltve, hogy a kezdeti / = 0 idpontban az ramerssg q= 0.
10.196. x / + y = 1; P(l;2).
10.197. x / - 2 y = 2x\
10.198. ( x - 2) / = y+2( x- 2) ^
10.199. y + 2xy = 4x.
10. 200. x / + x^ + x y - y = 0.
10.201. xy' + xy + y = 3x.
10.202. / = 2x(x^+y).
10.203. x y + ( l - 2x) y =
10.204. x y + xy+l = 0; i>(l; 1).
10.205. x{x- \ )y' + 2xy = 1.
10.206. Z + = 0; P(2;).
10.207. x y ' - y = x^ + 3x^-2x.
10.208. (2x+ 1)/ = 4X + 2;;.
10.209. x (x- l ) / - j = x(x- 1).
10.210. {l + x^)y' -2xy = l + x \
1
10.211. x / + 2j = -----p ; XTO.
l + |/x
10.212. (jcj; + e^) dx- xdy = 0 .
10.213. / + 2xj = jce "\
10.214. ( e^+l ) y' - ey = e*(e*+l)^.
7 - -
10.215. X / - - = e
X
148
10.216. x / - 2 y = (x-2)e^.
10.217. x / - x y = (1 + x V -
10.218. xy' + (x+l )y = 3x^e-\
10.219. 4xy' = l l y + x^e-^.
10.220. xe^^(xy' + 2y) = \ + x^fce^^.
_ 1
10.221. j c V - j = xe Mnx.
10.222.x y ' \ nx +y = In jc.
10.223. ( j c / - 1) In X = 2y.
10.224. xy' + y + x In x = 0.
10.225. xy' +y = In^ x.
10.226. {2y + x)dx = x </_)/+4(ln x) /x .
10.227. (x In x) y' - y = (x In x)^; y(e) = .
10.228. x W + ( l - 2x) y = x^.
10.229. / sin x - y cos x = l; y = 0.
10.230. y' sin x y cos x = cos x.
10.231. x y ' - y = x^ cos x.
10.232. / cos x + j = 1- sin x.
10.233. y'cos^x+y = tg x; j(0) = 0-
10.234. y' +y ctg x = x 4 cos x.
10.235. / - j tg X= cos^ X.
10.236. dr + C2r ctg 0 + sin 20)d0 = 0.
149
10.237. 2y' +y ctg x = 2 cos x.
10.238. / ----- = cos X.
1-X
10.239./ - y tgx = 1 - x .
10.240. x / = (1 -X tg x)y + x^ cos x.
10.241. j = x ( y ' - x cos x); F(0; 0).
10.242. (1 +x^) / + 2x> = X cos 2x.
10.243. xy' + 2y = sh (l-x) ,
10.244. y' + y ctg x = y = - 4 .
10.245. y' + 2xy = cos^ x.
10.246. {l + x^)y' + y = Arctg x.
10.247. (x+ 1)/ = 2 y + { x + l f Arctg x.
10.248. 2 x / _v = 2x Arctg 3x.
10.249. (x + / ) / j = y dx.
10.250. (sin^;; + x ctg y)y' - 1.
Hinyos msodrend differencilegyenletek
10.251. / ' = x + cos X.
10.252. / = + 1; Pi(0;0), ^^(l; 1).
1 0 . 2 5 3 . / = - ;X1) = 0 , , / ( l ) = 0.
X
10.254. (1 + sin x)^j" + cos x = 0.
10.255. / ' = lnx - P ( l ; 0 ) , 0(2; 1).
150
10.256. f n ? y " = 1; j;(0) = tt, /(O) = ^ .
10.257. y"(x^ + x y + 2 x +l = 0.
10.258. x V + ln X= 1; ^1) = 1, / ( l ) = 1-
10.259. A szabadon es test mozgst ler differencilegyenlet
(fs
= g
alak, amelyben g a nehzsgi gyorsuls. Hatrozza meg a differencilegyenlet ltal
nos megoldst s azt a partikulris megoldst, amikor egy q idpillanatban a Vq
sebessg test addig megtett tja Sq!
10.260. A lejtn rgztett test a rgzts megsznse utn mennyi id alatt r a
lejt aljra, ha a lejt hossza /= 50 m, hajlsszge a = 25, a srldsi egytthat pedig
A:=0,4?
10.261. x / ' - / = 0.
10.262. x / ' - y ' =
10.263. xy" - y ' = + P(0;0), Q(l; 1).
10.264. = x (x- 1); y(2) = 1, / ( 2) = - 1.
X - 1
10.265. x V = / 2 .
10.266. y"(e^+l) + y' = 0.
10.267. (y"y = / .
10.268. 2 y" = 2 x / - 3 .
10.269. 2 xy'y" ^ y ' ^ - 1 .
10.270. y"(x^+l) = 2x/ ; y(0) = 1, /(O) = 3.
10.271. 2 / ' - / 2 + 4 = 0.
10.272. / ' = / + e\
151
10.273. 2 xy" + y' = 0; y(l) = 0, / ( l ) = 1.
10.274. xy" = / I n - .
X
10.275. ( l - x^) y"- xy' = 0.
10.276. xy"- y' = x^ sin x.
10.277. / y = X.
10.278. / = x(y'f-, y(0) = 1, y'(0) = - 1.
10.279. ( l + J c V + / ^ + l = 0.
10.280. / ' = y'^ sin x.
10.281. = 0(1; ")
10.282. yy"+y'^ = 1; XO) = 1, /(O) = 0.
10.283. yy" = 2/^; j;(0) = 1, /(O) = 1.
10.284. 2yy" - y' ^- l = 0.
10.285. y"(y- l ) = 2/^.
10.286. y Y = l .
10.287. yy" = y' ^-y' ^.
10.288. y"(y^ + 2y)-2yy' ^-2y' ^ = 0.
10.289. / ' = 4 y Y - 8 y / \
1 0 . 2 9 0 . / > In J + / 2 = 0.
1 0 . 2 9 1 . 7 / = 20'^ + / ) .
10.292. yy"+y'^ = y^.
10.293. / = / ^ + / .
152
10.294. / ----- = 0.
1 - y
10.295. / ( l + / ^ ) - 3 / ' = 0.
10.296. = 1.
10.297. j(l - In y)y" + (1 + In y ) / ^ = 0.
10.298. y"\[y = / .
10.299. / =
10.300. Fggessznk fel kt ponton egy homogn, lland keresztmetszet, hajl
kony s nyjthatatlannak felttelezett ktelet, amelyet csak a sajt slya terhel.
Hatrozza meg azt az y{x) fggvnyt, amely a ktl egyenslyi alakjt rja le!
Msodrend lineris differencilegyenletek
10.301. / - 3 / = 0.
10.302. / + 5 / = 0
10.303. y " - 4 y = 0.
10.304. y" + 9y = 0.
10.305. y" + 2 y ' - 15y = 0.
10.306. y " - 6 / + 8y = 0.
10.307. 4 y " - 4 y ' - y = 0.
10.308. y" + 2y' + y = 0.
10.309. 9/'+ 2 4 / + I6y = 0.
10.310. y" + 4y' + 29y = 0.
10.311. / ' + 4 / + 9v = 0.
10.312. y"-2]l5y' + 9 = 0.
153
10.313. y" + 2f2y' + 5y = 0.
10.314. A szabad harmonikus rezgmozgs differencilegyenlete:
+coV(0 = 0,
ahol co>0 lland. Hatrozza meg a mozgst ler y(t) fggvnyt! Mely fggvny
elgti ki az j(0) = 0, j(0) = Vqkezdeti feltteleket?
10.315. A csillaptott harmonikus rezgmozgs differencilegyenlete
cPy(t) dy{t) -
ahol k > 0 a csillaptsi tnyez, (o>0 a krfrekvencia.
a) Hatrozza meg a mozgst ler y{t) fggvnyt! b) Mely fggvny elgti ki az
v(0) = 0, j(0) = Vq> 0 kezdeti feltteleket?
10.316. y"5 y' + 6 y = 12.
10.317. y" + 5y' + 6 y = I2x.
10.318. y " - y ' - 2 y = 2x2.
10.319. 9 / ' - 1 2 / + 4j = 4x \
10.320. y" + 6 y' + 34y = 17x2-62x + 23.
10.321. 4 / ' + 2 / = jc^.
10.322. / - j = - 4 cos X.
10.323. / ' - 6 / + 9 = 12 sin 3c.
10.324. y" + 4y = 3 sin x 6 cos x.
10.325. y" 4y' + 5y = 26 cos 2x+ 17 sin 2 x.
10.326. y " - 6 y' + 25y = 55 cos 4x.
10.327. / ' + j + sin x = 0.
10.328. y" + 9y+\ = sin 3x.
154
10.329. / ' + 4y = sin^ .
10.330. - y " + 2 / - 1 7 y = 20 sin ^2x.
10.3,31. y " +l 0 y ' + l 6 y = 27e \
'10.332. y" + y ' - 6 y =
1 0 . 3 3 3 . / ' + / - 6 ; ; = \0e^^' +5e-^\
1 0 . 3 3 4 . 2 / + ;; = ~4e\
10.335. y " - 2 y' + y = x^ + 4e^\
10.336. I6 x = 4e^^- I 6 y.
10.337. y" + 2y' + 5y = 8>-"+10x+1.
10.338. y" + 4y' - I2 y = 16 eh 2 x.
10.339. y " - 2 y' + y = 2 eh x.
10.340. 2y " - 3y ' - 2y = 2x+ 10 sh 2x.
X
10.341. 4 / ' - 4 / + j = 2 sh - +e ^ + x.
10.342. 6y ' - y " - 25y = 16 sh^ y .
10.343. y"- 2y' = 32 ch^ x.
10.344. 2 / ' + 3/-2>; = 4 sh^x + 2x + 2.
10.345. / ' = 4(sh2x + >;+l).
10.346. 2 / ' - 4 / - l = 2 sh^x + x.
10.347. y" - 4;;' + 4;; + X- 2e^ eh x = 0.
10.348. y" + y ' - 20y = xe~^\
155
10.349. y" + 2y' - 3y = 16xe~^\
10.350. = (2x+3)e^.
10.351. 8 / + lj' =
1
10.352. y" + 4y = sin^x + 4x+
10.354. y"- 4y' + 5y =
X X ^ ^
sin - + cos -
2 2
10.355. 4y"- 16/+ 17j; = 10e^* + 20 cos ^ +34x.
10.356. y" + y ' - l 2 y = cos x.
10.357. / ' - y ' - 2 y = e^^(l + sin x).
10.358. y" + 2y' + y = 2 x sin x.
10.359. y " - 3 y' + 2 y = xe^sin x.
10.360. Egy csillaptatlan rezgmozgst vgz m tmeg pontra periodikus erha
tst gyakorolunk; ekkor a pont knyszerrezgst vgez. Ha a gerjeszt er
F = Fq sin (Ogt, akkor a mozgst ler differencilegyenlet
^ + coy(t) = sm 0 ) t,
dr m
ahol (g> 0 a gerjeszt er frekvencija. Hatrozza meg a mozgst ler y(t) fggvnyt,
ha y(0) = 0, j(0) = Vq.
A 10.361.-10.375. feladatokban az adott partikulris megoldst felhasznlva
szmtsa ki a differencilegyenlet ltalnos megoldst!
10.361. (1 + x^)y"- 2 xy' + 2y = 0; y^ = x.
10.362. 3 x Y + x / - y ^ 0' ,y, = x.
156
10.363. (l + x^)y" + x y ' - y = 0; j i = x.
10.364. x^y"- 2xy' + 2y = 0; yi = x^.
10.365. x ^ y " - x y ' - 3 y = 0; =
10.366. x \ x + l )y"-2y = 0; = 1+ -.
X
10.367. {e^+l )y"-2y' -e^y = 0 ; y i ^ e^- \ .
10.368. xy" + 2y' - xy = 0 ; y , = ~ .
X
10.369. / ' + (tg x - 2 ctg x ) / + (2 ctg^x)y = 0; = sin x.
10.370. tg x + 2 j = 0; = sin x.
10.371. xy- y ' + 4x^y = 0; yi = cos x^.
10.372. / ' sin^ = 0; = ctg X.
10.373. ( l - x ^ ) / ' - x / = 0; yi = Arcsin x.
1
10.374. x / ' + 2 / + xy = 0; y, = - sin x.
X
10.375. y" + 4 x / + (4x^ + 2)y = 0; y^ = e~^\
10.376. / ' + j = t g X.
10.377. y"- 2y' +y =
- v'>2
(1-x)
2 x
1 0 . 3 7 8 . 4 / + 4j; = ^
10.379. y"-4y' + 3y =
10.380. / ' + 2 / + 5y =
] ! \ - x^'
e^+\
1
e"" cos 2x
157
11. VALOSZINUSEGSZAMITAS
11.1. Hnyflekppen helyezkedhet el a sakktbln a vilgos-s a stt kirly? (A-
kirlyok ltal elfoglalt mezk sem lben, sem cscsban nem rintkezhetnek egyms
sal.)
11.2. Egy trsasgban 8 n s 8 frfi van. Ha mindannyian - klnnem prokat
alkotva - tncolnak, hnyfle lehet a prok sszettele?
11.3. llaptsa meg, hogy az 1, 2, 3, 4, 5, 6 szmoknak hny olyan klnbz
permutcija van, amelyben pontosan hrom szm a termszetes sorrendben kveti
egymst!
11.4. Egy labdarg-mrkzsen az A csapat 3 : 2-re nyert a B csapat ellen.
llaptsa meg, hogy hnyfle lehetett a mrkzs lefolysa a pillanatnyi lls alakul
sa szempontjbl!
11.5. Hny klnbz tvonalon juthatunk el a sakktbln a bal fels sarokban
lv mezrl a jobb als sarokban lvre, ha brmely rintett mezrl csak az alatta
lv, vagy pedig a jobb oldaln ll mezre lphetnk?
11.6. Egy rendezvnyen a belpjegy egyttal tombolajegy is. A tomboln 3
ajndktrgyat sorsolnak ki a 300 rsztvev kztt. Hnyfle lehet a hzs eredm
nye, ha
a) az ajndktrgyak mind klnbzk;
b) ha az ajndktrgyak egyformk?
11.7. Oldja meg az albbi egyenletet a 0 s 23 kz es egsz szmok halmazn:
\ x ) \ x j
11.8. Szmtsa ki a binomilis ttel felhasznlsval 101^ rtkt!
158
11.9. Keravili-vsr alkalmval 4 (nem felttlenl klnbz) alkatrszt tartalma
z csomagokat rustanak. Az alkatrszek lehetnek: villsdug, kapcsol, eloszt,
kbel. Hny olyan klnbz sszellts csomag lehetsges, amelyben van kap
csol?
11.10. Egy 34 fs szervezet 5-tag vezetsget vlaszt: 1 titkrt s 4 vezetsgi
tagot. Hnyfle kimenetele lehet a vlasztsnak? (A titkrt a vezetsgi tagoktl
megklnbztetjk, de a ngy vezetsgi tag kztt nem tesznk klnbsget.)
11.11. Egy 25 mteres szvetvget maradk keletkezse nlkl 2 m-es s 3 m-es
darabokra kell vgnunk. Hnyflekppen tehetjk ezt meg, ha a klnbz mret
darabok sorrendje is szmt?
11.12. Hnyflekppen lehet tz, egyms utn kvetkez egsz szm kzl tt
kivlasztani gy, hogy a kivlasztott szmok kztt pontosan hrom egyms utn
kvetkez legyen? (A kivlaszts sorrendjre nem vagyunk tekintettel.)
11.13. t szelet tortt rendelnk a cukrszdban; az sszelltst a felszolglra
bzzuk. A cukrszda kszlete: 11 szelet csokoldtorta, 27 szelet citromtorta, 3 szelet
puncstorta s 8 szelet dobostorta. Hnyflekppen teljesthet a rendels? (Az azonos
fajtj tortaszeleteket nem klnbztetjk meg.)
11.14. llaptsa meg, hogy hny olyan ngyjegy szm van, amelyben az 1-es
szmjegy pontosan ktszer fordul el! (A ngyjegy szm els jegye termszetesen
nem lehet nulla.)
11.15. Egy rasztalnak ngy fikja van. Az asztal tulajdonosa kt nagy- s hrom
kismret bortkot dugott a fikokba. Hnyflekppen helyezkedhet el az t bortk
az rasztal fikjaiban? (Az azonos mret bortkokat nem klnbztetjk meg, a
fikokat viszont igen.)
11.16. Az oxignnek hrom, a hidrognnek kt stabil izotpja van. Hnyfle
vzmolekula kpzdhet ezekbl?
11.17. Ngy dobkockval dobunk egyszerre. A dobsnak hny olyan klnbz
kimenetele lehet, amelyben a dobott szmok kztt legalbb 2 hatos fordul el? (Az
esetek sszeszmllsakor a dobkockkat klnbztesse meg!)
11.18. Ngy, egyforma mret kocka minden lapjra egy-egy szmot rtunk.
A felrt szmok kztt nincs kt egyforma. Hny klnbz (2,2) tpus mtrix
olvashat le a kockk fels lapjairl, ha a megfelel alakzatban minden lehetsges
mdon kirakjuk ket?
159
11.19. Egy terem mennyezetn 5 sorban, 6 oszlopban sszesen 30 lmpa van
felszerelve. Kzlk 4 vilgt. Nincs olyan, sor, sem olyan oszlop, amelyben egynl
tbb lmpa gne. Hnyflekppen lehetsges ez?
11.20. Bizonytsa be, hogy az adott esemnytr tetszleges A, B, C esemnyei
sszegnek valsznsgre fennll:
P(A + B+Q = P(A) + P(B) + P(C)- P( AB) - P( AQ- P{BQ + P{ABQ.
11.21. Egy nemzetkzi tallkoznak 48 rsztvevje van; kzlk mindenki beszl
magyarul vagy nmetl. Harmincan vannak, akik magyarul, ugyancsak harmincan,
akik nmetl tudnak. Mekkora a valsznsge annak, hogy kt rsztvev tolmcs
nlkl tud beszlgetni egymssal, ha brmely pr kivlasztsa egyenl valsznsg?
11.22. Egy 50 tag trsasgban 15-en csak magyarul s oroszul, 10-en csak magya
rul s nmetl, 10-en csak oroszul s angolul, 10-en csak magyarul, 5-en csak angolul
beszlnek. Mekkora a valsznsge annak, hogy a trsasg kt tagja tolmcs nlkl
tud beszlgetni? (Brmely kt tag kivlasztsa egyenl valsznsg.)
11.23. A 0, 1, 2, 3, 6 szmjegyeket vletlenszeren sorba lltjuk. Szmtsa ki a
valsznsgt annak, hogy ily mdon egy 4-gyel oszthat tjegy szmot kapunk!
11.24. Vletlenszeren kivlasztunk egy hatjegy szmot (brmelyiket ugyanak
kora valsznsggel). A klnfle szelvnyekre nyomtatott sorszmokhoz hasonl
an, ez a szm 0-val is kezddhet, st, mind a hat szmjegy lehet 0. Szmtsa ki annak
valsznsgt, hogy
a) a. kivlasztott szm szmjegyei mind klnbzk,
b) a. kivlasztott szm szmjegyei mind klnbzk s nvekv sorrendben k\ tik
egymst,
cj a kivlasztott szm szmjegyei nemcskken sorrendben kvetik egymst,
dj a kivlasztott szmban minden szmjegy pontosan ktszer fordul el,
ej a kivlasztott szm els hrom jegye megegyezik a msodik hrommal, esetleg
ms sorrendben!
11.25. Villamoson, autbuszon az utasok ltal mkdtetett jegykezel gp a
jegyen lev kilenc szmozott mezbl nhnyat - tbbnyire hrmat - kilyukaszt.
Ezzel kapcsolatosak a kvetkez krdsek.
aj A gpnek hny olyan klnbz belltsa lehetsges, amellyel ajegyen hrom
lyuk keletkezik?
bJ Mekkora annak a valsznsge, hogy az t lyukra belltott gp hrom, elre
megadott mezt is kilyukaszt?
cJ Mekkora annak a valsznsge, hogy a hrom lyukra belltott gp a jegynek
ugyanazokat a mezit lyukasztja akkor is, ha a jegyet lapjval fordtva tesszk a
gpbe?
160
Ab ) s di c) krds esetben feltehet, hogy a gpnek brmely megfelel belltsa
egyformn valszn.
11.26. A 32 lapos magyar krtybl egy jtkosnak kiosztanak 4 lapot. Mekkora
annak valsznsge, hogy a 4 lap kztt pontosan kt piros lap s egy sz van?
\
11.27. A 32 lapos magyar krtybl az osztsnl 5 lapot kapunk. Szmtsa ki
annak a valsznsgt, hogy az 5 lap kztt mind a ngy szn elfordul!
11.28. Hrom kockval dobunk egyszerre. Szmtsa ki annak valsznsgt,
hogy legalbb egy hatost is dobunk!
11.29. A totszelvny egyik hasbjt gy tltttk ki, hogy egyik mrkzsre sem
tippeltnk 2-est. Mekkora annak a valsznsge, hogy az els 13 mrkzsbl
legalbb lO-es tallatot rtnk el ezzel a tipposzloppal, ha egyik csapat sem nyer
idegenben?
11.30. Szmtsa ki annak valsznsgt, hogy egy lotthzs alkalmval kihzott
t szm kztt
a) nincs pros szm,
b) SLprosak s a pratlanok szma is legalbb kett!
11.31. A lott tippszelvnyen a 90 szm 6 sorban, 15 oszlopban helyezkedik el.
Mekkora a valsznsge egy olyan lotteredmnynek, amelynl a kihzott t szm
kzl hrom ugyanabban a sorban, hrom pedig ugyanabban az oszlopban van?
11.32. Mekkora annak a valsznsge, hogy egy lotthzs alkalmval kihzott
t szm
a) mrtani,
b) szmtani sorozatot alkot?
11.33. Valaki minden hten 1szelvnnyel lottzik. Mennyi annak a valsznsge,
hogy egy v alatt (52 hzs sorn) egyszer sem nyer?
11.34. Ngyen megajndkozzk egymst gy, hogy mindegyikk hoz valamit,
majd a hozott ajndktrgyakat egyms kztt kisorsoljk. Mekkora a valsznsge
annak, hogy egyikk sem a sajt ajndkt kapja?
11.35. Hat kocsibl ll metrszerelvnyen ngy olyan utas tallhat, aki meg van
hlve.
a) Mennyi annak a valsznsge, hogy mind a ngyen ms-ms kocsiban utaznak?
b) Mennyi annak a valsznsge, hogy legfeljebb kt kocsiban tartzkodik meg
hlt utas?
11.36. Egy 20 fs tanulcsoportbl ketten hinyoznak. Szmtsa ki annak valsz
nsgt, hogy a hinyzk a nvsorban egyms utn kvetkeznek! (A hallgatk kzl
brmelyik kett ugyanakkora valsznsggel hinyozhat.)
11.37. Egy raktrban egyforma alapmret, klnbz magassg (20 cm, ill.
30 cm) kartondobozokbl 6 darabot helyeznk egyenknt egymsra. Szmtsa ki
annak valsznsgt, hogy az oszlop magassga nem haladja meg az 1,5 mtert, ha
minden lpsben egyenl valsznsggel vlaszthatunk kisebb vagy nagyobb dobozt!
11.38. Egy raktrban polcokra dobozokat helyeznk el. Egy-egy polcra az egyms
fl rakott dobozok mg ppen befrnek, ha egyttes magassguk 1 mter. A dobo
zok magassga hromfle: 20 cm, 30 cm, s 40 cm. A dobozok egy szlltszalagon
rkeznek, mindhrom mretben egyforma valsznsggel. Mekkora annak a valsz
nsge, hogy egyms utn rkez dobozokat egyms fl rakva, azok a polcra ppen
befrnek?
11.39. Egy 20 m-es szvetvgbl olyan darabokat vgunk le, amelyek hossza
2 vagy 3 m. Ezt addig folytatjuk, mg az egsz vget fel nem daraboljuk. Szmtsa ki
annak valsznsgt, hogy nem keletkezik 1m-es maradk, ha a 2 s 3 m-es darabo
kat vletlenszeren, egyforma valsznsggel vgjuk le!
11.40. Az zletben 10 rdi van abbl a tpusbl, amelyikbl vsrolni akarunk,
de a 10 rdi kzl 3 hibs. Mennyi annak a valsznsge, hogy a kiprbls sorn
csak a harmadik rdi lesz j?
11.41. 10 villanykrtbl, amelyek kzl kett rossz, visszatevs nlkl, tallomra
kivesznk hrom darabot. Mennyi annak a valsznsge, hogy ezek kzl
a) az elsre kivett rossz, a tbbi j,
b) legfeljebb egy rossz,
c) nem mind j,
d) legalbb egy j?
11.42. Darabru-szlltmnyban az ru 20%-a selejt. A szlltmny tvtelekor
vsszatevssel 10 elem mintt vesznek. Mekkora annak a valsznsge, hogy a
mintban pontosan kt selejt lesz, de ezeket nem egyms utn hzzk?
11.43. Egy zemben ktfle technolgival is gyrtanak egy bizonyos termket.
A termkek 60%-t olyan technolgival gyrtjk, amelynl a selejtarny 10%, a
msik technolgival gyrtott termkeknl a selejtarny 15%. A kszruraktrban,
ahol a ktfle technolgival gyrtott termkeket nem kezelik kln, vsszatevssel
kivlasztunk egy 10 elem mintt. Szmtsa ki annak valsznsgt, hogy a mintban
legfeljebb 1 selejtes termk lesz!
162
11.44. Egy dobozban 14 csavar van. Az els s a msodik fajtbl 5-5, a harmadik
fajtbl 4 darab. A dobozbl 5 csavart tallomra kiemelnk. (Brmelyik csavar
hzsnak ugyanannyi a valsznsge.) Szmtsa ki annak a valsznsgt, hogy az
els s a msodik fajtbl is legalbb kett lesz kzttk!
11.45. Darabru-szlltmnyban 40% az I. osztly, 50% a II. osztly s 10% a
III. osztly termkek arnya. Szmtsa ki annak valsznsgt, hogy egy 10 elem,
visszatevssel kivlasztott mintban 4 db I. osztly, 5 db II. osztly s 1 db III.
osztly termk lesz!
11.46. Egy villamosmegllban, ahov reggel 7 s 1/2 8 kztt gyalogosan rke
znk, ktfle villamosra szllhatunk fel. Az egyik jrat 5, a msik 12 percenknt
rkezik a megllba. 7 rakor mindkt villamos jn. Szmtsa ki annak valszns
gt, hogy nem kell 3 percnl tovbb vrakoznunk a megllban! (rkezsi idpon
tunk a megadott intervallumban egyenletes eloszls valsznsgi vltoz.)
11.47. Vletlenszeren kivlasztunk kt, 0 s 1kz es vals szmot. A kivlasz
tott szmok eloszlsa a (0, 1) intervallumon egyenletes. Szmtsa ki annak valszn
sgt, hogy
a) a. kivlasztott sszeg kisebb, mint - ,
b) a kivlasztott szmok klnbsge nagyobb, mint ^ ,
c) a kivlasztott szmok szorzata kisebb, mint - .
4
11.48. A (0, 1) intervallumban vletlenszeren kivlasztunk kt, klnbz pon
tot. A kivlasztott pontok eloszlsa az intervallumon egyenletes. A kt pont az
intervallumot hrom szakaszra bontja. Szmtsa ki annak valsznsgt, hogy e
hrom szakaszbl hromszg alkothat!
11.49. A 32 lapos magyar krtyt ngy jtkos kztt egyenlen sztosztjuk.
Jellje A azt az esemnyt, hogy egy kivlasztott jtkosnak 3 piros lap jutott, B pedig
azt, hogy az utna kvetkez jtkosnak 3 piros lap jutott. Szmtsa ki a P{B\)
valsznsget! Fogalmazza meg szavakban, hogy ez minek a valsznsgt jelenti!
11.50. Ismertek a kvetkez valsznsgek: P{A\B) = P{B\)
P{A I) = - . Szmtsa ki a P{A), P{B), P{AB) s P{A + B) valsznsgeket! (Feltesz-
8
szk, hogy Pi^A) > 0 s 0 < P{B) < 1.)
11* 163
11.51. Egy termk 50 darabos ttelben 15 a selejt. Az adott ttelbl vissza tevssel
5 elem mintt vesznk. Szmtsa ki annak valsznsgt, hogy a mintban 3 j s
2 hibs termk lesz, feltve, hogy harmadikra jt vlasztunk!
11.52. Egy rasztal ngy fikjban sszesen 3 piros ceruza van. Egy fikot tal
lomra kihzva mennyi annak a valsznsge, hogy ha benne tbb piros ceruza is van,
akkor mindhrom piros ceruza ebben a fikban tallhat? (Mindegyik piros ceruza
brmelyik fikban ugyanakkora valsznsggel lehet.)
11.53. Egy zemben gyrtott darabru 60%-a els osztly, 40%-a msodoszt
ly. A termk gyri minstse sorn 1%-os valsznsggel els osztly termket
msodosztlynak, 5%-os valsznsggel msodosztlyt els osztlynak minste
nek. Mennyi a valsznsge annak, hogy egy els osztlynak minstett termk
valban els osztly?
11.54. Egy folytonos eloszls ^ valsznsgi vltoz srsgfggvnye:
4
-(1 + x), ha - l < x < 0
f ( x) = <
0 egybknt,
rja fel s brzolja ^ eloszlsfggvnyt!
11.55. Egy folytonos eloszls ^ valsznsgi vltoz srsgfggvnye:
f
9
j/3x+1, ha O^ x ^ l ,
0 egybknt.
Szmtsa ki annak valsznsgt, hogy a valsznsgi vltoz - -nl nagyobb rt
ket vesz fel!
11.56. Egy folytonos eloszls ^ valsznsgi vltoz srsgfggvnye:
1
/
a Arccos x, ha 0 < x < - ,
2
a) Mekkora az a rtke?
b) Szmtsa ki vrhat rtkt!
164
11.57. Egy folytonos eloszls ^ valsznsgi vltoz srsgfggvnye:
ae~'^\ ha 0<x<4,
m = <
10 egybknt.
a) Mekkora az a rtke?
b) Szmtsa ki ^ vrhat rtkt!
11.58. Egy folytonos eloszls ^ valsznsgi vltoz srsgfggvnye:
{
axlnl(x, ha l <x<e .
0 egybknt.
a) Mekkora az a rtke?
b) Szmtsa ki ^ vrhat rtkt!
11.59. Egy valsznsgi vltoz azxi = 217,x2 = 218,x3 = 2l9, ^4 = 220 rtkeket
rendre = 0,05, P2 = 0,25, ^3 = 0,40, />4 = 0,30 valsznsggel veszi fel. Szmtsa ki
a valsznsgi vltoz vrhat rtkt s szrst!
11.60. Egy folytonos eloszls ^ valsznsgi vltoz srsgfggvnye:
"4 Inx
f i x) = <
ha l ^x<oo,
0 egybknt.
Szmtsa ki a valsznsgi vltoz vrhat rtkt s szrst!
11.61. Egy folytonos eloszls ^ valsznsgi vltoz srsgfggvnye:
0 egybknt.
Szmtsa ki a valsznsgi vltoz szrst!
11.62. s tj folytonos eloszls valsznsgi vltozk egyttes eloszlsfggv
nye
' ' 0, ha X^ 0 vagy j ^ 0,
xy
1, ha
es
X y
x > 0, j;>0 s - + - ^ 1.
Z0 J
a) Szmtsa ki a P( 0^^< 1, 0^/ / < 1) valsznsget!
b) Hatrozza meg a ^ valsznsgi vltoz f ( x) srsgfggvnyt!
165
11.63. Fejezze kia <^, tj, Cvalsznsgi vltozk H(x, y, z) egyttes eloszlsfggv
nynek segtsgvel a P(0 ^ ^ <a, 0 ^rj<b, valsznsget, ahol a, b, c
megadott pozitv vals szmokat jellnek!
11.64. k ^ , r i valsznsgi vltozk egyttes srsgfggvnye
h{x,y) =
0 egybknt.
/ I I 1\
Szmtsa k i a P l - - ^ ! ^ ^ - *1 ^ ~j valsznsget!
11.65. Hatrozza meg az f{x\ y) feltteles srsgfggvnyt, ha a s;/ valszn
sgi vltozk egyttes srsgfggvnye:
Kx, y)
X y , X V
^ > 0 , j ; > 0 es 2 ^ 3 ^ ^
0 egybknt.
11.66. Kt kockval dobunk egyszerre. A dobs eredmnye a kt dobott szm
klnbsgnek ngyzete. Szmtsa ki a dobs eredmnynek vrhat rtkt!
11.67. Egy trsasjtkban, melyet dobkockval jtszanak, a szably szerint hatos
dobsa esetn jbl dobhatunk, de csak egyszer. Mennyi az egy menetben elrhet
eredmny vrhat rtke, ha az elrt eredmny a dobott szm, vagy a kt dobott szm
sszege?
11.68. Darabru-szlltmny 75% els osztly, 25% msodosztly termket
tartalmaz. Egy els osztly termk rtke 100 Ft, egy msodosztly termk 80 Ft.
A szlltmnybl visszatevssel 4 elem mintt vesznk. Szmtsa ki a minta rtk
nek vrhat rtkt! Mennyi a minta rtknek feltteles vrhat rtke azzal a
felttellel, hogy a mintban szerepl termkek nem mind els osztlyak?
11.69. Egy folytonos eloszls valsznsgi vltoz srsgfggvnye:
3
j/lOO-x, ha 0^jc<100
f (x) = 2000
0 egybknt.
rja fel az f (x\ B) feltteles srsgfggvnyt, ahol B azt az esemnyt jelli, hogy a
valsznsgi vltoz 75 s 100 kztti rtket vesz fel! Szmtsa ki az
feltteles vrhat rtket!
166
11.70. A folytonos eloszls s;/ valsznsgi vltozk egyttes srsgfggv
nye:
( l - x ^ - ; ; ^ ) , ha x>0, y>0, x^ + y^<l ,
h(x, y) = S 3;t
/ l/3\
Szmtsa ki az MK17 =
0 egybknt.
feltteles vrhat rtket!
11.71. Fejezze ki a ^ valsznsgi vltoz harmadik centrlis momentumt a
A:-adik (k = 1, 2, 3) momentumokkal!
11.72. Egy folytonos eloszls ^ valsznsgi vltoz srsgfggvnye:
2
- ( x - 1 ) , ha \ <x<4,
0 egybknt.
Hatrozza meg ^ eloszlsfggvnyt s szmtsa ki az eloszls medinjt!
11.73. Egy darabru-szlltmnyban a termkek 60%-a els osztly, 40%-a m
sodosztly. A szlltmnybl visszatevssel 8 elem mintt vesznk. Mi a valsz
nbb : az, hogy a mintban 6 db els osztly s 2 db msodosztly termk lesz, vagy
az, hogy a minta legfeljebb 1 db msodosztly termket tartalmaz?
11.74. Egy darabru-szlltmnyban az ru 10%-a srlt, a tbbi p. A szllt
mnybl visszatevssel 25 elem mintt vesznk. Mi a valsznbb: az, hogy a
mintban pontosan 2 srlt termk lesz, vagy az, hogy a mintban legfeljebb 1 srlt
termk lesz?
11.75. Egy kevs gyakorlattal rendelkez munks ltal ksztett alkatrszek 15%-a
hibs. Egy gyakorlott szakmunks ltal ksztett alkatrszekbl visszatevssel 3 elem
mintt vve ugyanannyi annak a valsznsge, hogy kivlasztott munkadarabjai
mind jk, mint az elbbi munksnl annak, hogy egy 3 elem vsszatevses mintban
legfeljebb egy munkadarabja hibs. Hny szzalka hibs a gyakorlott szakmunks
ltal ksztett alkatrszeknek?
11.76. Egy 50%-nl kevesebb selejtet tartalmaz darabru-szlltmnybl vissza
tevssel 2 elem mintt vve, 0,24 annak a valsznsge, hogy a mintban pontosan
egy darab hibtlan termk lesz. Mekkora annak a valsznsge, hogy a mintban
mindkt termk hibtlan lesz?
11.77. Darabru-szlltmnybl visszatevssel 2 elem mintt vve 0,32-dal na
gyobb annak a valsznsge, hogy a mintban nem lesz selejt, mint annak, hogy a
167
mintban pontosan egy darab selejtes termk lesz. Hny szzalk selejtet tartalmaz
a szlltmny?
11.78. Prbagyrts sorn 20 gp kszlt el, amelyek kzl 5 javtsra szorul.
A teljes mennyisgbl 4 tallomra kiemelt gpet kldenek fellvizsglatra. A gyrts
akkor indulhat meg, ha a fellvizsglt gpek kzl legfeljebb egy szorul javtsra.
Mennyi a valsznsge annak, hogy megindulhat a gyrts?
11.79. 25 szk kzl visszatevs nlkl 2 elem mintt vve, 19-szer nagyobb a
valsznsge annak, hogy a mintban lesz hibtlan szk, mint annak, hogy nem lesz.
Hny hibs szk van?
11.80. Egy dobozban azonos^szm, egyforma mret rz- s aclcsavar tallhat.
A dobozbl tallomra kiemelnk kt csavart. (Brmelyik csavar kivlasztsnak
ugyanakkora a valsznsge.) Hny csavar van a dobozban, ha 0,24 annak a
valsznsge, hogy mindkt kivett csavar rzbl kszlt?
11.81. Egy zembehelyezs eltt ll gpteremben 16 gp van. Kt szerel vizsgl
ja a gpeket. Egyidejleg kt gp vizsglata folyik. Annak a valsznsge, hogy
tallomra egy-egy j gpet vlasztanak, ugyanakkora, mint annak a valsznsge,
hogy egy j s egy hibs gpet vlasztanak ki. Hny hibs gp van a teremben, feltve,
hogy nem mind hibs?
11.82. Egy textilzemben 100 szvgp van. Annak valsznsge, hogy egy gp
egy adott pillanatban ppen nem dolgozik: 6%. Mennyi annak a valsznsge, hogy
egy adott pillanatban nem dolgoz gpek szma
a) 4-nl kevesebb,
b) 6-nl tbb?
(Feltehet, hogy egy adott pillanatban nem dolgoz gpek szma Poisson-eloszlst
kvet.)
11.83. Egy 30 mter hossz, msfl mter szles szvetvgben sszesen 117 szvsi
hiba tallhat. Szmtsa ki annak valsznsgt, hogy a szvetvg tallomra kiv
lasztott 1m^-es darabjban legfeljebb 1 szvsi hiba fordul el, ha az 1 m^-es
szvetdarabban elfordul szvsi hibk szma Poisson-eloszlst kvet!
11.84. Egy 288 oldalas knyvben sszesen 72 sajthiba tallhat. Az egy-egy
oldalon tallhat sajthibk szma Poisson-eloszlst kvet. Szmtsa ki annak val
sznsgt, hogy egy tallomra kivlasztott oldalon tbb sajthiba is van!
11.85. Elemes bresztrban az elem kimerlsnek idpontja (ra, perc) az
egsz nap folyamn egyenletes eloszls, folytonos valsznsgi vltoz. Az elem
kimerlse a ht brmely napjn egyenl valsznsggel bekvetkezhet. Az elem
kimerlse csak akkor okoz problmt, ha az este 10 ra s reggel 6 ra kztt
168
kvetkezik be; klnben az elemet kicserljk. Feltve, hogy az elem kimerl, mekko
ra annak a valsznsge, hogy ennek kvetkeztben valamelyik munkanap reggeln
nem jelez az ra?
11.86. Mazsols slt tsztt 20 db egyforma mret szeletre vgunk. Legalbb
hny szem mazsolt kell a tsztba tennnk, ha azt akarjuk, hogy
a) legalbb 90%-OS,
b) legalbb 99%-os valsznsggel minden szeletben legyen legalbb egy szem
mazsola?
(Az egyszersg kedvrt feltesszk, hogy a szeletels sorn nem srlnek meg a
mazsolaszemek.)
11.87. Szmtsa ki annak valsznsgt, hogy a valsznsgi vltoz a vrhat
rtk szrs sugar krnyezetbe esik, azaz hatrozza meg a
valsznsget a kvetkez folytonos eloszlsok esetn:
a) egyenletes,
b) exponencilis,
c) normlis!
11.88. Igazolja, hogy ha ^ exponencilis eloszls valsznsgi vltoz, s jc>0,
Xo>0, akkor
i>(<r > X + Xol< > Xo) = > x)\
11.89. Egy elektronikus berendezs olyan egyforma alkotelemekbl ll, amelyek
mindegyiknek mkdse szksges a berendezs hibtlan zemelshez. Az egyes
elemek lettartama 2000 ra vrhat rtk, exponencilis eloszls, egymstl fg
getlen valsznsgi vltoz. Hny elem alkotja a berendezst, ha az 99%-os valsz
nsggel fl rn bell meghibsodik?
11.90. A rdium felezsi ideje 1600 v.
a) A rdium tmegnek hny szzalka bomlik el 800 v alatt?
b) Hny v alatt bomlik el a rdium tmegnek 1%-a?
(Egy Ra atom elbomlsig eltelt id exponencilis eloszls valsznsgi vltoz.)
11.91. Egy szabad neutron elbomlsig eltelt id exponencihs eloszls valsz
nsgi vltoz, amelynek vrhat rtke 260 s.
a) Mekkora annak a valsznsge, hogy egy szabad neutron 10 percen bell nem
bomlik el?
b) Mennyi id alatt bomlik el egy neutronnyalbban a neutronok fele?
11.92. A Fldn tlagosan hromvenknt okoz slyos katasztrft fldrengs.
Mekkora annak a valsznsge, hogy egy vben kt slyos katasztrfval jr
169
fldrengs is bekvetkezik? (Feltehet, hogy kt fldrengs okozta katasztrfa kztt
eltelt id exponencilis eloszls valsznsgi vltoz.)
11.93. Egy taxillomson ppen nincs szabad taxi s taxira vr utas sem, amikor
odarkezik egy 8 tag trsasg, akik 3 klnbz irnyba akarnak menni. Mekkora
a valsznsge annak, hogy 5 percen bell a trsasg valamennyi tagja taxiba tud
lni (ehhez 3 kocsira van szksgk), ha kt szabad taxi rkezse kztt eltelt id
exponencilis eloszls, 2 perc vrhat rtk valsznsgi vltoz?
11.94. Egy zemrszben deszkkat ksztenek. Ezek hossza normlis eloszls
valsznsgi vltoz 400 cm vrhat rtkkel s 3 cm szrssal.
a) Mekkora annak a valsznsge, hogy a deszkk hossza nagyobb, mint 406 cm?
b) Mekkora annak a valsznsge, hogy a deszkk hossza legfeljebb 1,5 cm-rel
tr el 400 cm-tl?
11.95. Egy szerszmgppel fogaskerekeket gyrtanak. Feltehet, hogy a legyrtott
fogaskerekek tmrjnek tnyleges rtke normlis eloszls valsznsgi vltoz.
A fogaskerekek tmrjnek vrhat rtke 40 mm, szrsa 0,2 mm. Egy fogaskerk
akkor felhasznlhat, ha tmrje 39,95 mm s 40,05 mm kz esik. A legyrtott
fogaskerekekbl egyet tallomra kivve, mennyi a valsznsge annak, hogy az azrt
nem hasznlhat, mert mreten felH?
11.96. Egy gyrtmny mrethibja - azaz eltrse a nvleges mrettl - normlis
eloszls valsznsgi vltoz, amelynek vrhat rtke 0. Annak valsznsge,
hogy a termk mrethibjnak abszolt rtke nagyobb 10 mm-nl: 0,2112. Mekkora
a valsznsgi vltoz szrsa?
11.97. Egy gyrtmny mrethibja 0 vrhat rtk, 8,2 mm szrs normlis
eloszls valsznsgi vltoz. A gyrtmny selejt, ha a mrethiba meghaladja a
15 mm-t. A gyrtmny els osztly, ha 5 mm-nl kisebb a mrethiba. Mennyi annak
a valsznsge, hogy egy gyrtmny nem selejt, de nem is els osztly?
11.98. Egy munkapadrl kikerlt termk hossza normlis eloszls valsznsgi
vltoz, amelynek vrhat rtke 200 mm, szrsa 2 mm.
a) Szmtsa ki annak valsznsgt, hogy a termk hossza 197 mm s 203 mm kz
esik?
b) Mekkora pontossg biztosthat 0,95 valsznsggel a munkadarabok hossz
ra?
11.99. Adott a lognormlis eloszls ^ valsznsgi vltoz M{^) vrhat rtke
s D{Q szrsa. Szmtsa ki az ;/ = In (normlis eloszls) valsznsgi vltoz m
vrhat rtkt s a szrst!
170
11.100. Egy granultum szemcsi megkzeltleg gmb alakak. A szemcsk
tmrje lognormlis eloszls valsznsgi vltoz, amelynek vrhat rtke 3 mm,
szrsa 1 mm. Szmtsa ki annak valsznsgt, hogy egy vletlenszeren kivlasz
tott szemcse tmrje
fy) 3 mm-nl kisebb,
b) mm s 4 mm kz esik!
11.101. Egy ^ valsznsgi vltoz relatv szrsa, azaz------rtke 0,02. A Cse-
bisev-egyenltlensg alapjn becslje meg, hogy a valsznsgi vltoz rtke legfel
jebb mekkora valsznsggel tr el a vrhat rtktl legalbb 5%-kai!
11.102. A parlament egy trvnyjavaslatrl orszgos npszavazs tjn kvn
dnteni. A szavazatok megszmllsakor az els rszeredmnyek azt mutatjk, hogy
300 000 szavazatbl a javaslat mellett szl 147 000. Van-e legalbb 1% esly arra,
hogy a javaslatot a szavazk tbbsge elfogadja?
171
12. MATEMATIKAI STATISZTIKA
12.1. Egy zem napi villamosenergia-fogyasztsa 100 egymst kvet munkana
pon az albbi tblzat szerint alakult:
napi villamosenergia-fogyaszts
(kWh)
ki (gyakorisg)
750-850 2
850-950 8
950-1050 15
1050-1150 38
1150-1250 21
1250-1350 12
1350-1450 3
1450-1550 1
Ksztse el a napi villamosenergia-fogyaszts gyakorisgi hisztogramjt!
12.2. Egy varroda napi szvetfelhasznlsa 50 egyms utni munkanapon az
albbi tblzat szerint alakult:
napi szvetfelhasznls (m) ki (gyakorisg)
160-200 1
200-240 0
240-280 4
280-320 10
320-360 16
360-400 11
400-440 5
440-480 2
480-520 1
Ksztse el a napi szvetfelhasznls srsghisztogramjt!
172
12.3. Egy lelmiszeripari termk nvleges tmegre 100 g. A tmeg ellenrzsekor
100 db termknl a grammban kifejezett tmegre a kvetkez (kerektett) rtkek
addtak:
93, 101, 102, 106, 99, 99, 96, 91, 106, 107, 97, 96, 105, 100, 102, 103, 105, 98, 100, 92,
107, 100, 104, 110, 103, 98, 98, 96, 102, 93, 100, 104, 103, 100, 102, 91, 103, 98, 92,
97, 95, 108, 94, 104, 103, 104, 101, 101, 103, 86, 102, 101, 106, 100, 94, 98, 96, 96, 94,
109, 103, 101, 105, 99, 95, 99, 98, 93, 97, 96, 88, 96, 99, 94, 94, 95, 92, 89, 107, 100,
100, 102, 101, 101, 99, 106, 105, 101, 97, 103, 95, 106, 109, 102, 108, 108, 102, 98, 99,
95.
Ksztse el a termk tmegnek srsghisztogramjt! Szmtsa ki a mintakzp
rtkt, tovbb hatrozza meg a tapasztalati medint!
12.4. Egy hdon egy rn t figyeltk az egyik irnyban percenknt thalad
gpkocsik szmt. A megfigyelt adatok a kvetkezk:
8, 12, 9, 15, 7, 10, 14, 20, 15, 7, 10, 22, 14, 13, 17, 9, 14, 8, 7, 13,
10, 15, 10, 8, 14, 17, 8, 10, 4, 9, 11, 5, 15, 10, 11, 6, 8, 13, 10, 19,
16, 9, 9, 16, 16, 11, 10, 8, 12, 13, 16, 18, 17, 14, 12, 13, 12, 9, 13, 10.
Szmtsa ki az adatokbl (statisztikai mintbl) a mintakzp, az empirikus szrs
ngyzet s a varicis tnyez rtkt!
12.5. Egy radioaktv sugrforrson flperces idtartamok alatt berkez rszecs
kk szmt Geiger-Mller szmllval 30 esetben megmrve, a kvetkez adatokat
kaptuk:
886, 865, 856, 833, 867, 887, 824, 852, 829, 864,
834, 887, 853, 831, 820, 855, 837, 862, 796, 872,
840, 834, 876, 808, 895, 868, 803, 907, 809, 824.
Radioaktv prepartumbl meghatrozott idtartam alatt kisugrzott rszecskk
szma elmletileg Poisson-eloszlst kvet. Ez alapjn becslje meg annak valszns
gt, hogy a prepartumon vgzett jabb 30 mrsbl ll sorozatban (a szmllcs
s az idtartam vltozatlan belltsa mellett) a legnagyobb rtk legalbb 910 lesz!
12.6. Italpalackoz automata ltal az egyes palackokba tlttt dtital trfogat
nak 40 elem minta alapjn megllaptott mintakzepe 246 cm^. Hosszabb idn t
vgzett megfigyelsek azt mutattk, hogy a gp 8 cm^-es szrssal dolgozik. Feltehet,
hogy az egyes palackokba kerl ital mennyisge normlis eloszlst kvet. Adjon
95%-OS megbzhatsgi szinten konfidenciaintervallumot az egyes palackokba tlttt
ital mennyisgnek vrhat rtkre!
12.7. Egy zemben a gyrtott ksztermk ellHtshoz naponta felhasznlt
nyersanyag mennyisge a nyersanyag minsgtl, a gyrtsor aktulis paramterei
tl s egyb tnyezktl fggen vletlenszeren vltozik; vgeredmnyben normlis
173
eloszlsnak tekinthet. 64 egymst kvet munkanap adatai alapjn az tlagos napi
nyersanyag-felhasznls 4,4 tonna. A szrs tbbves tapasztalat szerint 0,3 tonna.
A nyersanyagbl negyedvenknt egyszer rkezik szlltmny. Mekkora legyen a
negyedvre (64 munkanapra) megrendelt nyersanyag mennyisge, ha azt akarjuk,
hogy ne kvetkezzk be 2%-nl nagyobb valsznsggel nyersanyaghiny? (Az
zemnek nincsenek nyersanyagtartalkai.)
1 2.8. Bizonyos klnleges izzlmpk tlagos lettartama meghatrozott feltte
lek (napi 16 ra folyamatos zemid, stabil feszltsg) mellett 80 elem minta alapjn
1150 ra. Az izzlmpk lettartama normlis eloszlsnak tekinthet, melynek
szrsa 150 ra. 1db izzlmpa ra 48,- Ft. Egy vllalatnl egyidejleg nhny szz
ilyen tpus izzlmpa vilgt. Egy adott idszakban az izzlmpk zemrinak
sszege 1,62 milli ra. Feltehet, hogy a kigett izzk cserlse folytn a lmpallo
mny forintrtke az adott idszak vgn lnyegben ugyanakkora, mint az elejn
volt.
Adjon az adott idszakra vonatkozan a szksges izzlmpakltsgre 99%-os
megbzhatsgi szint fels becslst, ha az izzlmpkat a lert felttelek mellett
zemeltetik!
12.9. Bizonyos fajta nvnyi mag csrakpessgt vizsglva megllaptottk, hogy
a rendelkezsre ll kszletbl 1000 magot szmll mintt vve, az 1000 mag kzl
273 csrzott ki. lltson fel 98%-os megbzhatsgi szinten konfidenciaintervallumot
annak valsznsgre, hogy a kszletbl szrmaz mag kicsrzik!
12.10. Valamely rucikkbl egy 10 000 darabos szlltmny rkezett. A szllt
mnybl visszatevs nlkl 200 elem mintt vve, a mintban 5 db hibs termket
talltak. llaptsa meg 95%-os megbzhatsgi szinten, hogy milyen hatrok kztt
lehet a szlltmnyban a hibs darabok szma!
12.11. A magyarorszgi lott trtnetben az 1957 ta 1987. oktber 15-ig bezr
lag megtartott 1652 sorsols eredmnyekppen legtbbszr a 3-as szmot hztk ki
(125-szr), legkevesebbszer az 5-s szerepelt a nyertes szmok kztt (70-szer). Egy-
1
egy lottsorsolskor brmely szm kihzsnak elmleti valsznsge = 0,0555
18
llaptsa meg a fenti adatok alapjn, hogy milyen megbzhatsgi szinten esik az
elmleti valsznsg a 3-as, ill. 5-s szm hzsnak ismeretlen valsznsgre
vonatkoz konfidenciaintervallumba!
12.12. 18-21 v kztti letkor, 171-175 cm kz es testmagassg frfiak egy
25 fs csoportjnak mellkerlet adatai cm-ben mrve a kvetkezk:
87, 91, 92, 92, 93, 93, 93, 93, 94, 94, 95, 96, 96, 96, 97, 97, 99, 99, 99, 101,
101, 102, 103, 104, 105.
174
Feltehet, hogy az adott testmret normhs eloszlst kvet. lltson fel az adatok
alapjn a mellkerlet szrsra 90%-os szint konfidenciaintervallumot!
12.13. Az elz feladatban szerepl csoportban a derkkerletek cm-ben mrt
rtkei nagysg szerint rendezve:
73, 74, 74, 74, 74, 75, 76, 77, 79, 81, 81, 82, 82, 84, 84, 85, 85, 85, 86, 87,
87, 88, 89, 89, 91.
Feltve, hogy a derkkerlet normlis eloszlst kvet, adjon az adatok alapjn a
derkkerlet szrsra 90%-os szint konfidenciaintervallumot!
12.14. Automata szerszmgp ltal gyrtott alkatrsz egyik jellemz mrete az
elrs szerint 2,10 mm. A szban forg mret normhs eloszlst kvet. Alapos
vizsglatok azt mutattk, hogy a szrs 0,033 mm. 10 db vletlenszeren kivlasztott
alkatrsz megfelel mretnek kzprtke 2,14 mm. Az u-prba alapjn dntsn
99%-os szignifikanciaszinten arrl a hipotzisrl, hogy az alkatrsz jellemz mret
nek vrhat rtke 2,10 mm!
12.15. A konyhast tartalmaz csomagok tmege normlis eloszls valszns
gi vltoz. A szrst nagyszm mrsi adat alapjn ismertnek tekintjk, rtke 25 g.
Egy csomag s nvleges tmege 1 kg. Ellenrizni kvnjuk, hogy a csomagok tlagos
tmege megfelel-e az elrt rtknek. E clbl 20 db vletlenszeren kivlasztott
csomag egyttes tmegt lemrjk. A mrs eredmnye 19,82 kg.
a) Az M-prba szerint 95%-os szignifikanciaszinten elfogadhat-e az a feltevs,
hogy egy csomag s tmegnek vrhat rtke 1 kg?
b) Mekkora a msodfaj hiba elkvetsnek valsznsge 95%-os szignifikancia
szinten, ha 40 mrs alapjn alternatv hipotzisnk a vrhat rtkre 0,99 kg?
Mekkora ebben az esetben a prba ereje?
12.16. Ktmints w-prbval kvnjuk ellenrizni, hogy kt csomagolgp ltal
ellltott, konyhast tartalmaz csomagok tmegnek vrhat rtke megegyezik-e,
vagy sem. Az zemben mkd egyik gp 25 g, a msik (egy rgebbi tpus) 35 g
szrssal dolgozik. Az egyes csomagokban lv s tmege normlis eloszls valsz
nsgi vltoz. Vletlenszeren kivlasztunk 20-20 darabot az egyik, ill. a msik
gpen ellltott csomagok kzl, majd kln-kln meghatrozzuk a 20-20 csomag
tmegnek tlagrtkt. 95%-os szignifikanciaszinten a mintatlagoknak legfeljebb
mekkora eltrse esetn fogadhat el az a feltevs, hogy a kt gpen ellltott
csomagok tmegnek vrhat rtke megegyezik?
12.17. Egy csomag teavaj nvleges tmege 100 g. A tnyleges tmeg normlis
eloszls valsznsgi vltoz. 30 db vletlenszeren kivlasztott csomag grammban
kifejezett tmege a kvetkez;
103, 101, 105, 99, 95, 99, 98, 93, 97, 96, 88, 96, 99, 94, 94, 95, 92, 89, 107, 100, 100,
102, 101, 101, 99, 106, 105, 101, 97, 103.
175
aj A 30 adat alapjn -prba segtsgvel dntse el: 95%-os szinten elfogadhat-e
az a feltevs, hogy egy darab vaj tmegnek vrhat rtke 100 g!
b) Vizsglja meg ugyanezt a krdst az els 15 adat alapjn!
12.18. Bizonyos tpus 30 perces magnkazettk (egyik oldalnak) lejtszsi idejt
vizsglva, 10 db kazetta adatai a kvetkezk:
30:10, 29:46, 29:37, 30:01, 30:06, 29:53, 30:04, 29:29, 29:32, 30:03.
(30:10 jelentse: 30 perc 10 msodperc.)
Feltehet, hogy egy kazetta lejtszsi ideje normlis eloszls valsznsgi vltoz.
A felsorolt adatok alapjn dntsn -prbval 95%-os szinten arrl a feltevsrl, hogy
egy magnkazetta lejtszsi idejnek vrhat rtke - a vizsglt fajtbl - 30 perc!
12.19. 19-21 v kztti letkor frfiak kt csoportjban vgzett mrsek a cm-
ben mrt mellkerletre a kvetkez rtkeket eredmnyeztk:
I. (Az els, 26 fs csoportban a frfiak testmagassga 161 s 165 cm kz esik.)
89, 90, 90, 91, 91, 91, 91, 92, 93, 93, 95, 95, 96,
96, 96, 96, 97, 99, 99, 99, 100, 103, 103, 105, 105, 105.
II. (A msodik csoport ltszma 25 f. A testmagassg itt 171 s 175 cm kztti rtk.)
87, 91, 92, 92, 93, 93, 93, 94, 94, 94, 95, 96, 96,
96, 97, 97, 99, 99, 99, 101, 101, 102, 103, 104, 105.
A mellkerlet mindkt csoportban normUs eloszls valsznsgi vltoznak te
kinthet.
a) Ktoldali F-prbval ellenrizze, hogy szrsaik 98%-os szinten-megegyeznek-e!
b) Az a) feladat eredmnytl fggen, igenl vlasz esetn vizsglja meg ktmin-
ts -prba segtsgvel 95%-os szinten azt is, hogy a kt csoportban a mellkerletek
vrhat rtke egyenl-e!
12.20. Fizika mrsi gyakorlaton egy 18 fs csoport tagjai mindannyian elvgeztk
ugyanazt a mrst; egy ismeretlen erssg fnyforrs fnyerssgt kellett meghat
rozni kt klnbz berendezssel.
A Ritchie-fle fotomterrel kapott eredmnyek a kvetkezk voltak (candelban):
105, 123, 112, 110, 127, 135, 121, 101, 115,
108, 112, 129, 107, 124, 140, 128, 115, 120.
A Lummer-Brodhun-fotomterrel kapott eredmnyek (ugyancsak candelban):
115, 125, 124, 119, 126, 130, 120, 123, 118,
116, 124, 125, 107, 121, 126, 126, 117, 121.
Az els s a msodik mrsi eljrssal kapott eredmny is normlis eloszls, kt,
egymstl fggetlen valsznsgi vltoznak tekinthet. Ellenrizze 95%-os szinten
176
azt a feltevst, hogy a ktfle mdszerrel kapott mrsi eredmnyek vrhat rtke
megegyezik!
12.21. Ugyanazt a fajta mosport kt gyregysgben is ellltjk. A dobozokba
csomagolt termk nvleges tmege 600 g. A kt gyregysg termkeibl 25-25 elem,
vletlenszeren kivlasztott mintnk van. Az els gyregysgbl kikerlt termk
tmegnek empirikus szrsa 12 g, a msodik gyregysgben ellltottak 38 g. Az
egyik, ill. a msik gyregysgben gyrtott termkek tmege kt, normlis eloszls,
fggetlen valsznsgi vltoznak tekinthet. Welch-prbval kvnjuk ellenrizni
95%-os szinten azt a feltevst, hogy a kt gyregysgbl szrmaz termkek tmeg
nek vrhat rtke megegyezik. A rendelkezsnkre ll mintk alapjn kaphat
mintakzepeknek legalbb hny grammal kell eltrnie egymstl ahhoz, hogy a
feltevst elvessk?
12.22. Bizonyos fajta darabru 500-as ttelekben rkezik. Az tvev 40 elem
minta alapjn dnt a ttel elfogadsrl. Ha a mintban legfeljebb kt hibs termket
tall, akkor a szlltmnyt tveszi, ellenkez esetben visszautastja.
a) Szmtsa ki, hogy ily mdon az tvev legfeljebb mekkora valsznsggel utast
vissza olyan ttelt, amelyben a hibs darabok szma nem haladja meg a 10-et!
b) Legfeljebb mennyi egy olyan ttel elfogadsnak valsznsge, amelyben a
hibs darabok szma legalbb 75?
12.23. A kvetkez tblzat 1652 lotthzs adatait tartalmazza arra vonatko
zan, hogy melyik szmot hnyszor hztk ki.
1 2 3 4 5 6 7 8 9 10 11 12 13 14 15
91 76 125 92 70 97 97 89 86 107 79 108 107 92 99
16 17 18 19 20 21 22 23 24 25 26 27 28 29 30
81 90 108 97 96 89 95 103 99 95 78 75 80 114 71
31 32 33 34 35 36 37 38 39 40 41 42 43 44 45
77 90 88 105 89 92 87 90 74 79 96 101 87 83 91
46 47 48 49 50 51 52 53 54 55 56 57 58 59 60
101 100 87 105 91 105 83 92 89 85 114 83 76 84 100
61 62 63 64 65 66 67 68 69 70 71 72 73 74 75
84 79 75 104 97 91 100 81 106 76 93 94 93 87 120
76 77 78 79 80 81 82 83 84 85 86 87 88 89 90
89 109 89 86 82 97 83 87 103 86 107 91 76 91 94
Ennek alapjn ;^^-prbval vizsglja meg 95%-os szinten azt a hipotzist, hogy az
1-tl 90-ig terjed szmok brmelyike kihzsnak 1/90 a valsznsge!
12.24. A 12.1. feladatban egy zem napi villamosenergia-fogyasztsra vonatkoz
adatokat tall. Vgezzen ;f^-prbval tiszta illeszkedsi vizsglatot 95%-os szinten
12 Matematikai feladatok
177
arra a H'qhipotzisre vonatkozan, hogy az adatok olyan normhs eloszlst kvet
nek, amelynek vrhat rtke 1100 kWh, szrsa pedig 166 kWh!
12.25. Pamutcrna varrkpessgnek huroklts gyorsvarrgpen 25 cm-es var
rsszakaszokkal vgzett vizsglata sorn a crna szakadsmentes hosszrtkeire a
kvetkez adatokat kaptk.
Szakadsmentes hossz
(m)
Gyakorisg
Szakadsmentes hossz
(m)
Gyakorisg
0-10 13 80-90 3
10-20 16 90-100 4
20-30 15 100-110 2
30-40 12 110-120 0
40-50 13 120-130 1
50-60 8 130-140 0
60-70 5 140-150 0
70-80 6 150-160 1
Vgezzen ^-prbval becslses illeszkedsvizsglatot 95%-os szinten arra a hipotzis
re vonatkozan, hogy a vizsglt pamutcrna szakadsmentes hossza exponencilis
eloszls valsznsgi vltoz!
12.26. Egy textilgyrban a ktgpen dolgozk egyidejleg 7 gpet kezelnek.
A gp zemelshez szksgess vl beavatkozsok esetn (pl. csvecsere, vglevtel,
tllts) a gpet idnknt le kell lltani. Szlszakads, ttrs, kelmefeltekereds s
egyb hibk miatt is bekvetkeznek gplellsok. Egy mszak folyamn percenknt
megfigyeltk, hogy az egyik dolgoz ltal kezelt gpek kzl ppen hny gp ll. Az
egyttll gpek szmt s ezek elfordulsnak gyakorisgt az albbi tblzat
tartalmazza.
Egyttll gpek
szma
Megfigyelt
gyakorisg
0 108
1 171
2 122
3 53
4 19
5 7
178
Ellenrizze ^-prbval 95%-os szinten azt a hipotzist, hogy a mszak folyamn
egy adott pillanatban ppen nem dolgoz gpek szma Poisson-eloszlst kvet!
12.27. 175 cm-nl alacsonyabb, 18-21 v kztti frfiak egy 68 fs csoportjnak
testmagassg-testtmeg adatai a kvetkezk:
Testmagassg (cm) Testtmeg (kg)
159 57, 62, 63
160 66
161 64
162 70, 71, 77, 82
163 60, 61
164 58, 63, 64, 65, 66, 68
165 57, 73, 75
166 62, 65, 65, 70, 70
167 65, 66, 75, 80,
168 63, 67, 69, 71, 79, 84
169 61, 64, 66, 70, 74, 75, 76
170 55, 58, 61, 62, 71, 74, 75, 75, 86
171 66, 67, 68, 69, 75, 77
172 61, 65, 67, 70, 84
173 56, 77
174 58, 68, 75, 80
Vizsglja meg az adatok alapjn ^-prbval 95%-os szinten azt a hipotzist, hogy
a testmagassg s a testtmeg fggetlen valsznsgi vltozk!
12.28. Gyrtmnyellenrzs sorn tzetesen tvizsgltak 2500 db kazetts magne
tofont. A hibs kszlkeknl a hibk jellegt is felmrtk. A hibkat alapveten kt
csoportba lehetett sorolni: a mechanikai, ill. az elektronikai jelleg hibk csoportjba.
118 olyan kszlket talltak, amelynek csak mechanikai jelleg hibja volt. 77
kszlk csupn elektronikai szempontbl volt hibs. 5 db kszlk mindkt szem
pontbl hibsnak bizonyult.
;(^^-prbval 99%-os szinten ellenrizze azt a feltevst, hogy a ktfle hiba elfordu
lsa fggetlen egymstl!
12.29. Egy mszaki fiskolra jelentkezk kzl a fizikbl felvtelizk fizika
rsbeli dolgozatainak eredmnye- 1985-ben, ill. 1987-ben a kvetkez volt:
12*
179
Az elrt pontszm
(vizsgapont)
Dolgozatok szma
1985. 1987.
0 26
12
1 35 16
2 32
44
3 33 53
4 24 20
5 17 19
6 13 23
7 8 20
8 13 14
9 8 15
10 8 3
11 4 4
12 4 5
13 2 0
14 2 0
15 1 1
Elfogadhat-e 1e = 0,95 szinten az a hipotzis, hogy 1985-ben s 1987-ben elrt
fizika felvteli rsbeli eredmnyek egyforma eloszlsnak?
12.30. A 12.27. feladatban tallhat adatok alapjn szmtsa ki a testmagassg s
testtmeg kztti korrelcis egytthat becslt rtkt!
12.31. 18-21 v kztti letkor frfiak egy 25 fs csoportjban vgzett mrsek
a derkkerlet s mellkerlet sszetartoz rtkeire a kvetkez adatprokat eredm
nyeztk. (A csoportban a testmagassg 161-165 cm.)
Derkkerlet (cm) 73 72 70 73 74 75 78 75 77 80
Mellkerlet (cm) 88 90 91 90 91 91 91 92 93 93
76 84 75 82 83 85 80 78 80 91 85 86 87 86 95
95 95 96 96 96 96 97 99 99 99 100 103 104 105 105
A mellkerlet rtkt a derkkerlet lineris fggvnyvel kvnjuk kzelteni. rja fel
a regresszis egyenes egyenlett a minta alapjn becslt paramterekkel!
180
12.32. A vz mechanikai srsgnek klnbz hmrskleteken mrt rtkeit a
kvetkez tblzat tartalmazza.
t f C ) (kg/m^)
0 999,867
5 999,926
10 999,727
15
999,126
20 998,23
25 997,07
30 995,67
35 994,05
40 992,24
45 990,24
1 kg vz trfogatt a C-ban mrt hmrsklet msodfok polinomjval kvnjuk
kzelteni. rja fel a regresszis parabola egyenlett a rendelkezsre ll adatok
alapjn becslt paramterekkel!
181
13. TBBVLTOZS FGGVNYEK
13.1. Ksztsen rtktblzatot az f{x' ,y) = 3 y - 2 x - l ktvltozs fggvnyhez
a [0; 3] intervallum egsz rtkeinek felhasznlsval!
13.2. Hatrozza meg az albbi hozzrendelsi utastssal megadott ktvltozs
fggvnyek rtelmezsi tartomnyt:
a) (x;y) b) (x;j) ^ ^ -
c) (x;y) I- ln(x+j); d) (x; j ) i- Arctg - ; e) {x\y) i- |/sn(x^+P) !
X
13.3. rja fel az albbi egyenletekkel megadott hromvltozs fggvnyek rtelme
zsi tartomnyt:
a) = x'^+y'^ + z ^ - \ \ b) u = -j= \ ; c) u =
p ^ - x ^ - y ^
13.4. Hatrozza meg a kvetkez fggvnyek szakadsi pontjait:
2 xy 2
a) f ( x; y) = ; b) f ( x; y) = sm ;
x-ry xy
c) f i x; y) = ln(x^ + y^ - 4) ; d) f ( x; z) = !
xyz
13.5. Adott a z = |/5x^ + 5y^ egyenlet ktvltozs fggvny.
a) Hatrozza meg az (x; y) skkal prhuzamos skmetszeteinek (szintvonalainak)
egyenleteit!
b) Adja meg az (x;y) skkal val metszsvonalnak az egyenlett!
c) Definilja a felletet!
13.6. Adja meg a kvetkez egyenletekkel megadott ktvltozs fggvnyek s a
koordinta-skok metszsvonalainak az egyenleteit, tovbb az adott fggvnyrt
kekhez tartoz skmetszetek (szintvonalak) egyenleteit:
182
6x + 4y + 3z-24 = 0; z = 2; b) + + =A\ ^ = 2
c; z = 4-(x^ + / ) ; z = l !
13.7. Szmtsa ki a kvetkez hatrrtkeket:
sin xy x + y
a) lim------- ; b) lim ----- - ; c) lim
- * 0 X x-*oo x^ac y )
1 y * 00 y 00
13.8. Mutassa meg, hogy a kvetkez hatrrtkek nem lteznek:
x-\-y nx
a) lim------; b) lim sin-------- !
:c-^ox-y x-00 6 x + y
0 y 00
13.9. Folytonosak-e az albbi fggvnyek az adott pontban:
a) f {x\ y) = Arcsin - ; />o(0; 1); b)f ( x- y) =- Arctg - ; Po(2; 2);
y x
c ; / ( x ; y ) = z - - ^ ; P o ( l ; l ; l ) ?
2 x-^ + y^
13.10. Hatrozza meg a kvetkez ktvltozs fggvnyek elsrend parcilis
derivlt fggvnyeit:
a) f ( x; y) = x ^ - 6 x^y + y^; b) f (x; y) = In
X
c) f i x; y) = (sin x)ln + (In x)cos y; d) f{x; y) = e) f{x; y) = Arctg - ;
J)f (x-, y) = 2 ^ ; g) f ( x; y) = cos y^; h)f (x; y) = Arcsin !
x + y
13.11. Hatrozza meg a kvetkez hromvltozs fggvnyek elsrend parcilis
derivlt fggvnyeit:
a) f ( x ; y ; z ) = x^ + y^ + z^ 3xyz;
b) f ( x ; y ; z ) = xy + xz+yz;
c)f {x-, y-z) = P + 7 T ? ;
d) f ( x; y; z) = In xyz;
ej f ( x; y; z ) ==(xy^;
f ) f ( x ; y ; z ) ^ z ^ y ;
g) f ( x; y ; z ) = x^!
13.12. Hatrrozza meg az albbi egyenletekkel megadott fggvnyek x s szerin
ti parcilis derivltjnak rtkt a megadott Pqpontban;
| / s m V V 2 /
183
b) z ^ i g{x^-l yy, Poi2 ,l)
c) z = Arctg-^-^ ; Po(l; 1)!
1xy
13.13. Szmtsa ki az m^ + m^ + w' sszeget a / *o(l ;l;l) pontban, ha
u = ln(l + x + / + z^).
13.14. Hatrozza meg a kvetkez ktvltozs fggvnyek teljes (totlis) differen
ciljt :
a) f{x; y) = ; b) f{x; y) = x^'; c) f (x; y) =
]/x-]/y
x+y
d) f {x\ y) = e) f {x\ y) = xy\n^{x+y)-, f ) f {x\ y) = sin^x + x c o sj ;
9 ) f(x', y) = Arccos .
13.15. Hatrozza meg a kvetkez hromvltozs fggvnyek teljes differencil
jt:
xy
a) u(x;y; z) = ; b) u(x; y; z) = si n; c) u{x\ y, z) = \n{x + y +z)\
z
df
13.16. Adja meg a derivltfggvnyt, ha
dt
a) fix\y' ) = Arcsin(x-;;) s x = 3; y = 4?^;
b) f ( x; y) = 6 xy cos(x + j ) s x=t ; y=t^;
c) f {x\ y) = s x = cos^t; y = sin 2 t;
d) f{x; y) = In xy s X = tg t; y = ft.
du
13.17. Adja meg a derivltfggvnyt, ha
dt
a) m(x; y, z)= sin (x + y + z) s x= t^; y = 2 t 1; z = ;
b) m(x; y; z) = Arccos s x = sh ; = eh ; z = 1-
xy
13.18. Adja meg az u{v\ w) fggvny x , y s z szerinti elsrend parcilis derivlt
jait, ha
vw
a) u{v; w) = s v(x; y) = Arctg xy; w(x; y) = x sin ;;;
v+w
b) u(v; w) = s v(x;y; z) = ln{x +y - z ) ; w{x;y) = x^\
13.19. Hatrozza meg az albbi, implicit alak ktvltozs fggvnyek x s j
szerinti elsrend parcilis derivlt fggvnyeit:
184
aj x c o s j + j c o s z + zcosx = 2; b) e + = 0;
c) xVz" = 1; d) (2x + y)"<2^+^> = e.
13.20. Szmtsa ki az albbi ktvltozs fggvnyek irnymenti derivltjt az
adott Virnyvektor egyenes mentn az adott Pq pontban;
1
c ) f { x \ y ) =
a) f ( x ; y ) = - ^
cos^(x-y)
b) f ( x ; y ) = sin(x^ + j;2);
In X ny
ny In X
3 __________________
d) f { x ; y ) = lg ^2 xy + y^ + y^\
ny
e) f { x- y) = c o s ^ ^ ^ ;
f ) f ( x; y) = Arctg ;
v(-l/3; - 1);
v(/3; 1);
v ( - 3 ; 4 ) ;
|/2 ]/2 ^
/n
2
v | - y ;
g) f ( x' , y) = InxV+ln
h ) f ( x ; y ) = ] / x ^ y + x y ^ ;
y - x
i ) f i x ; y ) = arsh
e ' ^
j ) f ( . x; y) = y -
y + x
X"
13.21. Hatrozza meg az
v(l;l);
v(l '; - / 3 ) ;
v ( - l ; l ) ;
v(l; l);
v(-2/3;2);
v(3; - 4 ) ;
(x;y) I-* 2 y s i n x - e ^ + y^
Po
\ f
Po(e',e^);
PoiO; 1);
n ( 2; 2);
^ o ( l ; 2);
Po(]f2 -,^);
^ o ( l ; 3 ) ;
Po(5-, 12);
^o(2;5)!
ktvltozs fggvny
a) irnymenti derivltjt a Po(0 ; 1) ponton tmen, az x tengely pozitv irnyval
a = 225-os szget bezr egyenes mentn;
b) gradienst a Po(0; 1) pontban!
13.22. Hatrozza meg az
(x; v) i- In x y + l n -
ktvltozs fggvny
a) irnymenti derivltjt a Po(l; 1) ponton tmen, v( - 1; - 1) irnyvektor egye
nes mentn;
b) 1) pontbeli gradienst!
185
13.23. Hatrozza meg az
ktvltozs fggvny
aj irnymenti derivltjt a /*o(l; 2) ponton tmen, az x tengely pozitv irnyval
60-os szget bezr egyenes mentn;
bJ gradienst a Fq( 1; 2) pontban!
13.24. Szmtsa ki az f{x\ y) = Arctg (y') fggvny
a) irnymenti derivltjt a 1) pontban s az j = (1 - 2x)^ + ln 2 egyenlettel
megadott fggvny Xq = - abszcisszj pontjban hzott rintjnek az irnyban;
b) gradienst a Po(l; 2) pontban!
13.25. Hatrozza meg az
Xx; y; z) = sin xyz
hromvltozs fggvny
/ 1 1\ ^
a) irnymenti derivltjt a Poi n ; - ponton tmen, v
\ 2 2 J
tor egyenes mentn;
1 1\
b) gradienst a Pq pontban!
\ 2 2/
f{x\y) = ]jx^y^ - xy^
1 2 2\
irnyvek-
13.26. Szmtsa ki a kvetkez egyenletekkel megadott z = f (x;y) ktvltozs
fggvnyek irnymenti derivltjt az adott a irnyszg egyenes mentn az adott Pq
pontban, s adja meg ebben a pontban a fggvny gradiensvektort:
a) z =
y
Arctg - ;
X
a=150;
- 1);
b) z = e*'+^ + ln^x+v; a = 315;
c) z ^ ^x^y^ - xy^; a = 120;
i^o(2; 1);
\fZ 1
/ 4 2\
d) z = 2^ "^sin-; a=150;
-
e) z = a = 60;
x + y
f ) ^ =
Arctg ; a = 120;
^ o ( l ; 2);
x - y
g) z =
ln|/2x^ + / ; a = 60;
- 21^)
h ) z ^
tgZ +^ln (,+ D.
a = 225;
i o(i ; 0).
186
13.27. Szmtsa ki az / ( x ; j ) = In (xj^) + In (x^) fggvny tiszta msodrend
parcilis derivltjainaic rtkt a Po{l\ 2) pontban!
13.28. Mutassa meg az albbi fggvnyekre, hogy vegyes msodrend parcilis
derivltjaik egyenlek;
a) f { x \ y ) = e^cos j - e ^ s i n x ; b) f ( x ; y ) =
c) f ( x ; j ) = x^ + - 3 cos x^y^!
13.29. Mivel egyenl az
x / L- y f ^ y
kifejezs, hay(x;y) = sinxj?
13.30. Mivel egyenl az
kifejezs, haj {x, y) = f ^ - \ - x y >
13.31. Adja meg az
z
kifejezst, ha f {x\ y) = e^.
13.32. Bizonytsa be, hogy
f ' L - f nyy
f" + f"
J yx J yy
= 0,
ha /(x; y) = In
13.33. Adja meg a
dx^ dy^
kifejezs rtkt, ha J{x; y) = cos ay, a e R.
13.34. Bizonytsa be, hogy
ox ay
- X ^ y
ha /(x \y) = sin I- e* cos - .
y X
187
X y
13.35. Mivel egyenl az xf ^+yf ' kifejezs, ha/x; v) = sin - cos - ?
X
13.36. Mivel egyenl az xC + yf' kifejezs, ha/(x; v) = sin - ?
y
2
13.37. Adja meg a ---- f'^ + kifejezs rtkt, ha f {x; y) = y^ In
13.38. Szmtsa ki az f "x~f yy kifejezs rtkt a Pq
J{x;y) = In sin (x^ + / ) .
- ; f i n ) pontban, ha
X
13.39. Szmtsa ki az f(x; y) = cos - +/*"' ^ ktvltozs fggvny vegyes msod-
rend parcilis derivltjainak rtkt a Po(0 ; 1) pontban!
8 X
13.40. Mivel egyenl ^ ~ f'xx ' f'yy kifejezs, ha f ( x; y) = ]/2xy^ + xy>
3 5
13.41. Adja meg az f (x;y) = | / ^ c h - x fggvny tiszta msodrend parcilis
derivltjainak klnbsgt!
13.42. Mutassa meg, hogy az f ( x; y) ^ f ^ n h y fggvny megoldsa az
1
4'
13.43. Mennyi a z / x ; j ) = 4 sn 27t y ---- fggvny tiszta msodrend parcilis
\ 2/
derivltjainak arnya?
13.44. Mekkora a rtke, ha az/(x; j ) = x^ + axy^ {a e R) fggvny megoldsa a
(p-f d^f
~ H-----r = 0 differencilegyenletnek?
dx^ dy^
d^u
13.45. Adja meg a fggvnyt, ha (x; v; z) = |/x^ + - 2xz.
dydz
13.46. Szmtsa ki az + fggvny rtkt a P ( l ; l ; l ) pontban, ha
m(x - , y , z ) = In (x^ + + z^ + 3x j z) .
188
13.48. Mutassa meg, hogy az f { x \ y , z ) = fggvny kielgti az n.
^x^+y^ + z^
Laplace-fle differencilegyenletet, azaz u"^+uyy + u"^ = 0.
13.47. Adja meg az u'"^^ fggvnyt, ha u{x\y, z) =
13.49. Hatrozza meg a kvetkez fggvnyek loklis szlsrtkeit:
a) f ( x ; y ) = x^ + xy + y ^ - 5 x - 4 y + l ;
b) f { x \ y ) = 2 + 2x + 4 y - x ^ - y ^ ;
c j / ( x ; y ) = 2 x^ + 2 y^ + ( x - i y + ( y - i y ;
1 1 xy
d) f ( x ; y ) = - + - +
X y 21
e) f { x \ y ) = sin Xcos (x + y) + sin
13.50. Egy tglatest egy pontba sszefut leinek sszege 24 egysg. Mekkork az
lek, ha a tglatest trfogata maximlis?
13.51. Adott r sugar s m magassg egyenes krkpba rjunk maximlis trfo
gat tglatestet! Mekkora a tglatest trfogata?
13.52. A 15 egysgnyi kerlet hromszgek kzl melyiknek legnagyobb a ter
lete?
13.53. Adja meg a z = |/x^ 2y^ egyenlettel megadott fellet Pq(3; 2) helyhez
tartoz pontjban
aj rintskjnak egyenlett;
bj normlisnak paramteres egyenletrendszert!
13.54. Adja meg a z = xy egyenlettel megadott fellet Po(2; 6 ) helyhez tartoz
pontjban
aj rintskjnak egyenlett;
bJ normlisnak paramteres egyenletrendszert!
13.55. Hatrozza meg a z = cos {x~2y) egyenlettel megadott fellet rintskj-
n n\
nak az egyenlett a Pq ~ ^ helyhez tartoz pontjban!
V2 A)
13.56. Egy egyenes henger magassgt (4 0,2) mternek, sugart pedig (2,5 0,1)
mternek mrtk. Becslje meg, hogy milyen hibval szmthat ki a trfogat!
13.57. Egy hromszg kt oldalt (2002), illetve (300 5) centimternek, a
kztk lv szget pedig (60 1) foknak mrtk. Mekkora abszolt hibval szmtha
t ki a hromszg harmadik oldala?
189
13.58. Szmtsa ki az albbi integrlokat:
i 1 1
a ) ] ( ^ T + b)
2x
0 0
-dx
i+y"
dy, c)
^ dy] dx\
9 f ^ \ 1
d) j l j dy)dx; e) j
0 \ 0 /
' 2 - 2 x

2 x - 2
/ 4 - x 2 - y 2
dx.
13.59. Hatrozza meg az albbi/(x;ktvltozs fggvnyek integrljt az adott
H tglalapon:
a) f { x- y) = xy- / / = {(x;; ; ) 1; 0 ^ ^ 2};
b) f { x \ y ) = 2x^^3xy + Ay^-, i / = {(x; j ) e 11 0 ^ ^ 3};
c ) f { x \ y ) = X sin
1
d)f{x-, y) =
(x + y+ I)^
/ / = | ( x ; j ; ) e R ^ | l ^ x ^ 2; 0 ^ ^
H = { ( x - y ) e R^ \ OS x ^ l ; 0 ^ j ^ 1}.
13.60. Szmtsa ki a kvetkez ketts integrlokat az adott sorrendben, majd az
integrls sorrendjnek felcserlsvel:
r ? \ / ]fy
a ; ( J ix^ + y^) dx) dy;
o \ - f J
b ) \ ( ^ i y + y)dx^dy-
d) \ ( l xysi n{x^ + y^ydy dx.
13.61. Szmtsa ki az albbi ktvltozs fggvnyek integrljt az adott A, B,
C cscspont hromszgtartomnyon:
a) f ( x ; y ) = 3x^ + 2x>';
Ai - \ ; Q) ,
5(1 ; 0), C(0; 1);
b) f ( x ; y ) = x^ + xy; ^(0; 0), Bi2 ; 0 ),
c) f i x ; y) =
2x + 3 j + l ;
^ ( - 1; - 1),
B{2; -4) , C(l;3);
d) f ( x ; y ) =
^ .
5
X
Bi2; 3), C(3;3);
e) f ( x ; y ) =
Zy^
?
X
Ai \ ; \ ) , 5(2; 3), C(3;3);
f ) f i x- , y ) =
sin(x+j);
AiO; 0), 5(0; 3),
C ( - l ; 2);
g ) f i x ; y ) =
^(0; 0), 5(1; - 2 ), C(2; - 2 ) ;
h) f ( x ; y ) = xy^; ^(0; - 2), Bi6 ; 1),
C(3; 1);
i ) f ( x ; y ) = 5(3; 1), C(3;4);
j ) f i x ; y) =
cos(x+j;);
^ ( - 1; - 1),
5(1; 3),
C(7;3);
k) f i x ; y) = j c o s ( x + j ) ; ^(0; 0), B{2n; n). Cin; n).
190
A{- 2;0), B(4; 0), C(3; 1),
n( 0 ; 1);
A{0; 0), 5(2; 2), C(4; 2), i5(5;0);
A(0; 0), 5(1; 0), C(l;l), Z)(0; 1);
A(0; 1), 5(1 ; 1),
C(e; e). >(0;e);
^ ( - 1; 1),
B(0; 1), C(0; 2), D{-2-2).
13.62. Szmtsa ki az albbi ktvltozs fggvnyek integrljt az adott A, B, C,
D cscspont ngyszgtartomnyon:
a) f { x \ y ) = x+y,
b) f { x \ y ) = f c ^ - ,
c) f{x-, y) = y{\ + x^+y'^) 2;
2 x
d)f{x-, y) = ;
yix^ + y'^)
2 xy
e)f {x-, y) =
13.63. Szmtsa ki a kvetkez ktvltozs fggvnyek integrljt azon a vges
skrszen, amelyet az albbi egyenletekkel megadott egyenesek zrnak kzre:
a) f { x \ y ) = 3>y^-x\ x = 0, ^ y + x = 2,\
b) f { x \ y ) = A x y - 3,y^-, y - 2 x = 2, y - x = 3, j = 6;
xy
c ) f { x \ y) = ---- ; y = x, j + x = 2, x = 0;
.------- 1
d) f{x-, y) = p x + y, x = 0, y = 2x, j = - - x + 5 .
13.64. Integrlja az albbi ktvltozs fggvnyeket az adott egyenes s parabola
ltal bezrt vges tartomnyon:
a) f{x-, y) = 2 xy\
b) f { x \ y ) - =x + 2 xy,
/ ( x ; j ) = x - 2j ;
d) f{x-, y) = x ~y \
e) f { x : y ) ^ x - y ,
f ) f { x \ y ) = x- Ay \
g) f { x \ y ) = x + 2 y,
h) f {x- , y) - =2 x + 2 y\
J.
(x + 2)
y = x - 2 .
/ = x;
y = x + 2 . j = x^;
y - 0 , ; ; = 1- x ^
y = U
y = 2 x ^ - 1 ;
y ^ x , y = 4x- x^ ;
y = 4x+5, y = (2 x+l )^;
y + x = 3, y = ( x- l ) ^;
j + x = 0, y + 2 = x^;
y = x + 2 , y = x^.
13.65. Hatrozza meg a kvetkez ktvltozs fggvnyek integrljt az adott
parabolk ltal bezrt vges skrszen:
a) f {x\ y) = x^ + Ay^\ x = y^, y = x^\
b) f {x\ y) = x + 2 yco?,x', y + A = x^, y + 2 = - x ^ .
13.66. Szmtsa ki ketts integrllal az albbi egyenletekkel megadott grbk ltal
hatrolt vges skrsz terlett:
191
a) / = 2x; y = x - 4 ;
b) y = x ^ - 2 x - l ; y = - x ^ + 4x~l ;
c) xy=5; xy = 7; y = 2x; y = 6 x.
13.67. Szmtsa ki a ff (3y^x + 2x^_y)/rintegrlt, ha a vges Ttartomny a z = -
(T) X
egyenlet hiperbola s az j = x, x=5 egyenlet egyenesek ltal hatrolt sikrsz!
13.68. Szmtsa ki a Jf (x+y)dT integrlt, ha a vges T tartomny az = sin 2x
egyenlet grbe s az j = 0, x = 0, x = ^ egyenlet egyenesek ltal hatrolt skrsz!
az 13.69. Szmtsa ki a \ \ {2y+\ ) dT integrlt, ha a vges T tartomny
y = 1+ th Xegyenlet grbe s az j = 0, jc=0, x = In 2 egyenlet egyenesek ltal
hatrolt skrsz!
13.70. Szmtsa ki a JJ { x - y ) dTintegrlt, ha a vges T tartomny az j = cos - x
egyenlet grbe s az j = 0, x = 0, x = tt egyenlet egyenesek ltal hatrolt skrsz!
13 71. Hatrozza meg az albbi f {x\ y) ktvltozs fggvnyek integrljt az adott
H tartomnyon:
a ) f { x \ y ) = -
sin 2x
b)f{x-,y) = T ,
Sin 2x
c) f {x\ y) =
cos^x
H =
H =
d) f {x\ y) = 2j + sin^x, H =
cos X 2y
e)Rx-, y) = ^ ^ f , H -
sin X 3
X
g) fi.x-,y) = y s i n - +x , H =
h) f(x-, y) = 2 cos X - 2;^, H =
i) f i x; y) = xy,
192
(x; y) eR^
(x; y) R^
( x ; j ) e R 2
n n
6 - ^ - 4
- < X < -
6 - 4
n
0 ^ X ^ -
4
n
0 ^ X ^ -
6
sin X^ ^ cos X ^;
sin^ X^ ^ cos^ x^;
sin X ^ j ^ cos x | ;
0 ^ j ^ 1+ sinxi;
1
(x;j )eR2 I O ^ x ^ l ; - - ^ y ^ ^ ) - .
2
(x; j ) g R2 I O^ x ^ l ;
(x;>)eR^
n
O ^ x ^ - ; cos X
H
k) f{x;y) = -. T S n i +(^~ H = {(x; j ) eR^ | 2 ^ x ^ 3 ; ( x - 1)^ ^ ^ x+ 1}.
(X !_)
rcy 2
13.72. Mivel egyenl a - dT integrl, ha a vges T tartomnyt az j = - ;
JJ X X
(T)
X^
j = egyenlet grbk s az j = 4 egyenlet egyenes hatroljk?
13.73. Mivel egyenl a JJ dT integrl, ha a vges T tartomnyt az y = e^
egyenlet grbe s az = ex + 2e;y = 4 egyenlet egyenesek hatroljk?
13.74. Mivel egyenl a J"} In xydT integrl, ha a vges T tartomnyt az j^ = x
(T)
egyenlet parabola saz_v = X;y=l;;^ = 2 egyenlet egyenesek hatroljk?
13.75. Szmtsa ki a n^ y dT integrlt, ha a vges T tartomny az
y = Arccos x egyenlet grbe s a z j = - , j = - egyenlet egyenesek ltal
hatrolt skrsz!
13.76. Szmtsa ki a j j 2xy^ i x^y dT integrlt, ha a vges T tartomny az
X= fy-, X= |/y egyenlet grbk s az y = 1egyenlet egyenes ltal hatrolt skrsz!
13.77. Szmtsa ki a dT integrlt, ha a vges T tartomny az
xj
<T)
a) y = f x egyenlet grbe s az y = - , x = e egyenlet egyenesek ltal hatrolt
e
skrsz;
b) y = x, y = 2x, x = 2, x = 1 egyenlet egyenesek ltal hatrolt skrsz!
13.78. Szmtsa ki a
16
2 dT integrlt, ha a vges T tartomny az y =
(T)
X
egyenlet grbe s az j = - , x =2 egyenlet egyenesek ltal hatrolt skrsz!
13 Matematikai feladatok 193
13.79. Hatrozza meg az albbi f{x' ,y) ktvltozs fggvnyek integrljt azon
a vges tartomnyon, amelyet az adott egyenlet grbk zrnak kzre:
2|/2
a) f {x\ y) = + y = (x>?,x, y = x, x = 0;
71
b) f ( x; y) = 2 xco<iy; y = x^, y + x = 6 , j = 0;
c) f ( x; y) = x^y eh xy^; y = fx, y = - f x , x = l ;
1 ^
d) f ( x- y) = 3 x^y^ cosxy^- y^ = x, j = - j/x, x = fz;
2ch 2 y 1
e ) f i x ; y ) = ^
f ) f (x- , y) = (^2 7 ^;2)2 ; ^ + >^ =0, ; ; = x - 2, x = ^ ;
g) Rx\ y) = x+ ; ;; = (x+l)^ y = - x + \ , x = - 3 , x = - 2 ;
- 1)
+ j- = ^ + 2;
n
i) f ( x; y) = x + 2 y; = sm 2x, y = 0 , ^ = 2
j ) f {x\ y) = 2 y + 2>-, y = sinx, y = 0 , x = ^ ;
o z
k) f ( x; y) = X sin (x^ + y); y=x^, x =
n
l) f{x-,y) ^ x - Ay , j = sinx, y = 0 , ^ = 2 '
13.80. Szmtsa ki a kvetkez integrlokat
1 1 - x - x - y
(* /
a)
b)
V
0 0 0
1 - y 1 - x - y
r / r / r
{\ + x + y + z f
:dz dy dx;
1
-dz dx dy.
{\ + x + y + z f / /
0 0 0
c) Hatrozza meg az integrlsi tartomnyt!
d) Szmtsa ki az integrlt gy, hogy elszr x szerint integrl!
13.81. Szmtsa ki a
2x
dV integrlt, ha a
(V)
F = {(x; j ; z ) e R ^ I x ^ O , z^O, x + ;; + z ^ 1} zrt trrsz, s
194
a) elszr x szerint integrl;
b) elszr y szerint integrl;
c) elszr z szerint integrl!
13.82. Szmitsa ki a x^y^zdV integrl rtkt, ha
V = {( x; y; z) eR^ \ x =l , z = 0, y = x, z=xy}!
13.83. Hatrozza meg a kvetkez/(x;j; z) hromvltozs fggvnyek hrmas
integrljt azon V vges tartomnyon, amelynek hatrai az x^ + y^ + z^ = 1egyenlet
gmbfellet, az x = 0, j = 0, z = 0 egyenlet skok, tovbb x^O, z^O:
aj f ( x; y; z) = xyz; b) f {x\ y; z) = x^yzl
13.84. Szmtsa a fff dV integrl rtkt, ha
(V)
V = {(x; y; z) eR^ I x =l , y=0, z = 0, y = x, z = x + y}!
13.85. Szmtsa ki a
' 1
;---- r dT integrlt polrkoordintkra val ttrssel, ha
x^ + y^
(T)
T= {(x;j;)eR" | { x - l f + y^ ^ 1}!
13.86. Szmtsa ki JJ xy dT integrlt polrkoordintkra val ttrssel, ha a
(T)
vges T tartomny hatrai az ( x\)^ + y^ = 1 s az (x 2)^ + y^ = 4 egyenlet
krk!
13.87. Szmtsa ki azt a kbtartalmat, amelyet az egysgnyi tmrj hengerbl
az egysgnyi sugar gmb metsz ki, ha a gmb kzppontja a hengerpalston van!
13.88. Szmtsa ki annak a trrsznek a trfogatt, amelyet a z = 4 y^,
z = y^ + 2 egyenlet parabolikus hengerek, az x= - 1 s x = 2 egyenlet skok hat
rolnak !
13.89. Szmtsa ki annak a trrsznek a trfogatt, amelyet a z = 4 x^ y^
egyenlet forgsi paraboloid s az (x; j ) sk hatrol!
13.90. Szmtsa ki annak a trrsznek a trfogatt, amelyet a z = x^ + y^,
z = + 2 y^ egyenlet paraboloidok s az j; = x, = 2x, x = 1egyenlet skok hat
rolnak !
13.91. Hatrozza meg a z = 4 x^ 3^^egyenlet fellet s a z=0 egyenlet sk
ltal hatrolt homogn test slypontjnak a koordintit!
13* 195
13.92. Szmtsa ki a z = y ^ - x egyenlet fellet, s az x = 2, j ; z = 0 egyenle
t skok ltal hatrolt test slypontjnak a koordintit!
13.93. Szmtsa ki azx + y + z = 8, x = 0, j = 0, z = 0 skok ltal hatrolt homogn
tetrader
a) slypontjnak a koordintit;
b) a koordintatengelyekre vonatkoz tehetetlensgi nyomatkait!
13.94. Hatrozza meg az x^+y^ + z^ = 64, z^O egyenlet homogn flgmb
a) slypontjnak a koordintit;
b) az x , y , z tengelyekre s a slyponton thalad tengelyre vonatkoz tehetetlens
gi nyomatkait!
13.95. Szmtsa ki az x = 0, z = 0, s az x + + 2z = 4 egyenlet skok ltal hatrolt
homogn gla
a) slypontjnak a koordintit;
b) axx, y, z tengelyekre s a slyponton thalad tengelyre vonatkoz tehetetlens
gi nyomatkait!
13.96. Szmtsa ki a kvetkez improprius integrlokat:
1 ( 1 d)^dy\
0 \ 0
00 / 00 \
b) \ \ \ xye-^^"-^" dx]dy,
0 \ o /
00 / 00 \
c) j Jx dy\
196
14. VEKTORFGGVENYEK
Ebben a fejezetben a fggvnyek - hacsak nem mondunk mst - a legtgabb
rtelmezsi tartomnyukban tekintendk.
14.1. Hatrozza meg a kvetkez fggvnyek paramter szerinti derivltfggv
nyt;
a) r() = sin )i + + Arctg |/ k ;
"W = j ^ i + (tg50j + l n / + ? k ;
/ 1+
c) r() = i + ( Arcsin )j sh y - j ^ k .
14.2. Hatrozza meg az albbi fggvnyek paramter szerinti msodik derivlt
fggvnyt:
a) r() = M + (sh 3)j + ln (+l)k;
b) r() = 2^i - (cos 4)j + 1^1- k;
c) r(0 = Y ^ i - ( A r c t g 20j + (2 + 3 A .
14.3. Hatrozza meg az
r(0 = 35'i + [ln(l + 30 ]j - l / T k
fggvny paramter szerinti -edik derivltjt!
14.4. Hatrozza meg az
1 +1 ,
) - T ' ^ '
trgrbe q=\ paramterrtkhez tartoz pontjban az rint irny egysgvektort!
197
14.5. rja fel az
r(0 = (/-3)i + (2+l)j + ^k
trgrbe rintjnek egyenletrendszert sl 1 ^ = 2 paramterhez tartoz pontjban!
14.6. Hatrozza meg az
t + 1
r(0 = ( + l ) ^ i + j + - ^ k
grbe q1 paramter pontjban hzhat rintjnek vektoregyenlett!
14.7. Keressen az
1 2 3
r() = ----- H--------jH--------k
\ + t 2 - r 3 + t
2 x
trgrbn olyan pontot, amelyben a grbe rintje prhuzamos a ----- =
6
j - 5 1- z
= ----- = ------ egyenessel!
14.8. Keresse meg az
r(0 = ^ i + y j + y k
trgrbe azon pontjait, amelyekben a grbe rintje prhuzamos az
x + 3>+ 2z - 10 = 0 skkal!
14.9. Van-e olyan pont az
r(0 = (2 + 2)i + ^j + ( l - 0k
trgrbn, amelyben a grbe rintje prhuzamos az x + y + z = 1989 sikkal?
14.10. Hny olyan pont van az
r() = (sin t t cos )i + (cos + sin )j + (^ + 1) k
trgrbn, amelyben a grbhez hzhat rint prhuzamos az (y, z) koordintask
kal?
14.11. Mekkora szget zrnak be egymssal az
r(0 = (^ + l)i + ( + l ) 2j + (-l)3k
trgrbe j = 0 s 1 paramterrtkhez tartoz rinti?
198
14.12. rja fel az
r() = (sin - / c o s )i + (cos +isin )j + ( + 1)*^
trgrbe rintjnek az egyenletrendszert az a) = 0 s b) ^2 = 2 Paramter
pontban! Mekkora szget zr be ez a kt rint?
14.13. Hatrozza meg az
1
r() = (sin )i + (cos t)\ H------- k
cos t
trgrbe to = 0 paramter pontjban az rint, a fnormlis s a binormlis irny
egysgvektort!
14.14. rja fel az
r(0 = (+3)i+ yj + (2-5)k
trgrbe rintjnek, fnormlisnak s binormlisnak egyenletrendszert a tg = 1
paramter pontban!
14.15. Hatrozza meg az
r() = (/3-2^)i + (3/ + 2)j + (2-5)k
trgrbe rintjnek, fnormlisnak s binormlisnak egyenletrendszert a t(,= l
paramter pontban!
14.16. rja fel az
r(0 = ( + 3)i+ yj + (2-5)k
trgrbe q = 1 paramter pontjban a simul-, a norml- s a rektifikl skjnak
az egyenlett!
14.17. Hatrozza meg az
r(0 = (3-2/2)i + (3/ + 2)j + (^-5)k
trgrbe 1 paramterrtkhez tartoz pontjban aj a simulsikjnak, bj a nor-
mlsikjnak, cj a rektifikl skjnak, egyenlett!
14.18. Szmtsa ki az
r(0 = 32i + (2+3)j + 3^k
trgrbe grblett s torzijt annak to = - 1 paramter pontjban!
199
14.19. Szmtsa ki az
r(0 = ( l - 2^)i + (2- 2)j+ yk
trgrbe grblett s torzijt annak q = \ paramter pontjban!
14.20. Szmtsa ki az
X = A cos t, y = l n t , z = 2t
n
trgrbe grblett s torzijt annak ^ ^ Paramter pontjaban!
14.21. Szmtsa ki az
r(t) = (eh )i + (sh )j + k
trgrbe grblett s torzijt annak paramter pontjban! Jobbcsavarod-
s-e a grbe ebben a pontban?
14.22. Hatrozza meg az
r() = (cos^)i + (sin^Oj + (sin 2)k
trgrbe grblett s torzijt annak q = - paramter pontjban!
6
14.23. Mutassa meg, hogy az
r(0 = (3 sin )i + (3 cos )j + 4k
csavarvonal grblete is, torzija is lland!
14.24. Mutassa meg, hogy az
1+ 1 t
grbe skgrbe!
14.25. Mutassa meg, hogy az
r() = { a i t ^ + b i t + + ^ b x t ^ c - ^ + + c ^ \ i
egyenlet grbk, ahol Ui, bi, Ci, i = l, 2, 3 tetszleges vals szmok, skgrbk!
14.26. Hatrozza meg, hogy az
r(0 = (a eh 0*+ {a sh t)\ + btk
grbe tetszleges pontjban mely a s rtkekre lesz egyenl a grblet a torzival!
200
14.27. Szmtsa ki az
r(0 = (e' cos ) + (^ sin i)\ + e'k
egyenlettel megadott trgrbe grblett s torzijt, tovbb rja fel az rintje s
a normlskja egyenlett a 1 ^ = 0 pontban!
14.28. Hatrozza meg az
r(0 = (32-20i+/^j + ( l - / ) k
egyenlet trgrbe q = 2 paramter pontjban a) az rintje vektoregyenlett; b)
a normlskja egyenlett; c) a grblett; d) a torzijt!
14.29. Hatrozza meg az
r(0 = ( ^ - l ) i + ( + 2)j + ( / ^ - l ) k
egyenlet trgrbe = 1 paramter pontjban az rint, a normlis, a binormlis
egyenletrendszert, a simulsk, a normlsk, a rektifikl sk egyenlett, a grbletet
s a torzit!
14.30. Vizsglja meg az
r(0 = ei + e~*l + tk
trgrbt annak paramter pontjban! (rja fel az rint, a fnormlis, a
binormlis vektoregyenlett, a simul-, a norml- s a rektifikl sk egyenlett,
szmtsa ki grblett s torzijt!)
14.31. Szmtsa ki az
t
.2
x = t+3, y = ~ , z =
2 ' ~3 ~
trgrbe vhosszt, ha
14.32. Szmtsa ki az
r(0 = H + l t ^ + ^t'^k
trgrbe vhosszt, ha - l ^ / ^ 2.
14.33. Szmtsa ki az
r(0 = (2 cos /)i + (2 sin )j + 4/k
csavarvonal vhosszt, ha 0 ^ ^ 2;t.
201
14.34. Szmtsa ki az
r() = (e* cos )i + sin )j +
trgrbe vhosszt, ha 0 ^ ^ 3 .
14.35. Szmtsa ki az
r(/) = [t cos (3 In )]i + [t sin (3 In )]j + j/6k
trgrbe vhosszt, ha 0 ^ ^ 27t.
14.36. Szmtsa ki az
(0 ^ ift cos )i + {ft sin i)\ + k
trgrbe vhosszt, ha l ^ ^ 9 .
14.37. Szmtsa ki az
cos . sin t
r(0 = 1+ j + ( l - t h ) k
eh eh
trgrbe vhosszt, ha a) 0^ ^ l 00; j O^^c, ahol c pozitv vals szm!
14.38. rja fel annak a sknak ktparamteres vektoregyenlett, amely illeszkedik
az A{1', 3; 5), (4; 5; 6), C(l; 2; 3) pontokra!
14.39. rja fel annak a hengerfelletnek a vektoregyenlett, amelynek vezrgrbje
az (x,y) skban az x^ + y^ = 4 egyenlet kr, alkoti pedig prhuzamosak a z
tengellyel!
14.40. rja fel annak a hengerfelletnek a vektoregyenlett, amelynek vezrgrbje
az (x, y) skban az x ^ - y ^ = 4 hiperbola, alkoti pedig az a(4; 2; 5) vektorral prhu
zamosak !
14.41. rja fel annak a kpfelletnek a vektoregyenlett, amelynek cscsa a
F(2; - 1; 3) pont, vezrgrbje az (x, y) skban az y^ = 4x egyenlet parabola!
14.42. rja fel annak a gyrfelletnek (trusznak) a vektoregyenlett, amely az
(x, z) skban fekv (x - a)^ + z^ = krnek (a, b pozitv llandk s a > 6) a z tengely
krli forgatsval keletkezik!
14.43. Hatrozza meg annak a grbnek az egyenlett, amely illeszkedik az
r(w, v) = (u + 2v)i - vi + (u^ + 3t?)k
felletre s eleget tesz az u^ = v felttelnek!
202
14.44. Tekintse az
r(u, v) = (4 cos u cos r)i + 4 cos u sin t?)j + (4 sin w)k
gmbfellet u = 2t, v = 3t egyenletrendszerrel adott felleti grbjt! Hatrozza meg
a felleti grbe = paramter pontjhoz tartoz rintjnek egyenlett!
14.45. Hatrozza meg az
r(w, v) = - 2 v^)\ + {uv^)\ + (u^v - w)k
fellet rintskjnak egyenlett annak Uq = 2 , Vq = - l paramter pontjban!
14.46. rja fel az
r(u,v) = + +
fellet rintskjnak egyenlett annak Uq = 2 , To= 1 paramter pontjban!
14.47. rja fel az
r(u, v) = (u cos r)i + (u sin p)j + 3rk
csavarfellet rintskjnak egyenleti annak P(l; |/3; n) pontjban!
14.48. Hatrozza meg az
r(u, v) = (u+ r)i + (u^ + v^)j + (u^ + t;^)k
fellet rintskjnak egyenlett annak P(2; 2; 2) pontjban!
14.49. Szmtsa ki az
u = x^+y^- 3xy
skalrmez gradiensvektort a P(2; - 1) pontban!
14.50. Hatrozza meg a kvetkez skalrmezk gradienstert:
a) u = 2 yz;
shx
b) u = e z cos y +
2 y + z
x^ + y^ y^ + z^
y^ + z^ x^ + y^'
14.51. Hatrozza meg az albbi skalrmezk gradiensvektort a P pontban:
x^ y^ z^
203
b) u = 2 ] f ^ + \n-^, ^ Q ; 2; i )
c) u = ^ ^ , P(2; 2;l).
14.52. Hatrozza meg az albbi skalrmezk gradiensvektort a P pontban:
a) u = xy+yz + xz, P ( 2 ; - 3 ; l ) ;
b) u = xysix\z + xz\ r\ y+^y,
c ) u = Ifi, P(3; - 4 ; 1).
14.53. Hatrozza meg az albbi skalrmezk gradienstert:
a ) u { r ) = \ t \ - , b ) u ( r ) = \ r \ ^ ;
c) M(r) = In |r|; d) (r) = |r|^ +
ir|2 '
14.54. Szmtsa ki az
u = x^ + 2y^ + 3z^ + xy + 3x 2y6z
skalrmez gradiensvektornak hosszt az 0(0; 0; 0) s a P(2; 0; 1) pontban! Melyik
pontban nullvektor a gradiensvektor?
14.55. Szmtsa ki, hogy mekkora szget zrnak be az
X
Ml = x^+y^ + z^ s 2 = arcsin------+z
X y
skalrmezk gradiensvektorai a P(1; 1; 0) pontban!
14.56. Hatrozza meg az albbi v vektormezk divergencijt:
aj V = (x+y^ + z^)i + (x^+y + z^)j + (x^+y^ + z)k;
b) Y = 3xi + (x-2y) j + ( z- x) k;
c j v = e^^i + e^^j + e^^k.
14.57. Hatrozza meg az albbi v vektormezk divergencijt az adott F pontban:
aj V = (x^-y^)i + (y^-z^)i + (z^-x^)k; P(2; 1;0);
bj y = - i + ^ i + - k , P( l ; 2; 3);
y z x
cj V= ( x^+ - x)j + x y z \ P(l; - 1; 2).
14.58. Hatrozza meg az albbi v vektormezk divergencijt:
aj Y = ; bj Y= rj r; c ; v = l r p r .
204
14.59. Forrsmentes-e a
V= 0^ + z^)i + (z^ + x^)j + (x^+ y^)k
vektormez?
14.60. Forrsmentes-e a
V= (x+y' ^-z^)\ + { - x ^ +y + z^)\ + {x^- y^ + z)]i.
vektormez?
14.61. Szmtsa ki az albbi vektormezk rotcijt:
a) V = (x+y^ + z^)i + (x^+y + z^)} + (x^+y^ + z)k;
b) y = 3xi + (x-2>)j + (z-x)k;
c ) \ = + +
14.62. Szmtsa ki a kvetkezk vektormez rotcijt az adott P pontban:
a) v = (x^ - y^)i + (y^~ z^)} + (z^ - x^)k, P(2;l;0);
b) y = - i + - j + - k , P(l;2;3);
y Z X
; V= (x^ + y^)i + 3i4xy^-x)j + xyz % P(l; - 1; 2).
14-63. Szmtsa ki a kvetkez vektormezk rotcijt:
aj V = ; 6 j v = l r i r ; c;v=|r |^r .
14.64. rvnymentes-e a
V= f)-^2y^ + (y^+\)k
vektormez?
14.65. rvnymentes-e a
V= + xzj + + 2z)k
vektormez?
14.66. Szmtsa ki a
V= (3x + 2y)i-{5x + z^)j + (x^-y^) k
vektormez divergencijt s rotcijt!
14.67. Szmtsa ki a
V= x^yzi + xy^zj + xyz^k
vektormez divergencijt s rotcijt!
205
14.68. Szmtsa ki a
/ , \ / v-,\ /
V=
1
In i + ( l n y ) j + i n ^ l k
2
V X j
vektormez divergencijt s rotcijt!
14.69. Szmtsa ki a
V= (sin x^yz)i + (sin xy'^z)} + (sin xyz^)k
vektormez divergencijt s rotcijt!
14.70. Szmtsa ki a
V= (sh^ x)i + (eh xy)j + (In >z)k
vektormez divergencijt s rotcijt annak Tq = 2j - - k helyvektor pontjban!
14.71. Szmtsa ki a v = - ^ vektormez divergencijt s rotcijt az
Irr
fo = i - 2 j + 3k helyvektor pontjban!
14.72. Szmtsa ki a
y
V = ------- i-2xy^i + xyzk
Arctg X
vektormez divergencijt s rotcijt annak Tq = i + 2j + 2k helyvektor pontj
ban!
14.73. Hatrozza meg a rt rt v vektort, ha
V = en +e^^i +e^^k.
14.74. Szmtsa ki a rt v s rt rt v vektorokat, valamint div rt v rtkt a
P(1; 1; 1) pontban, ha
V= ( x - 2y)i + xyz j + xz^k.
14.75. Szmtsa ki div grad u rtkt az Fq = 2j + k helyvektor pontban, ha
u = ye^ + xe^ + yz^.
14.76. Szmtsa ki grad div v rtkt a F(l; - 1; 1) pontban, ha
V= ( x^ - 2 y z y + (xy^-z^)j + (z^-xyz)k.
14.77. Hatrozza meg div grad rtkt s a rt grad vektort!
206
14.78. Szmtsa ki a
V = (3x^ + y^)i +(x^ - y^)i
vektormez vonalmenti integrljt az y = 2 3x egyenes mentn, ha 0 ^ x ^ 1.
14.79. Tekintse a
Y = ------------- I -4- -------------- I
x^ + y^
vektorfggvnyt s az >4(-l;0), 5(0; 1), C(1;0), D{1; 1) pontokat. Szmtsa ki a
fggvny vonalmenti integrljt, ha az integrls tja az '
a) AB egyenesszakasz; b) A, B pontok hatrolta, orig kzppont negyedkrv;
c) AC egyenesszakasz; d) A, C pontok ltal hatrolt, orig kzppont flkrv;
e) BD egyenesszakasz; f ) DC egyenesszakasz;
g) x = l , x = \ , y = \ , y = 1 egyenesek ltal meghatrozott ngyzet.
14.80. Szmtsa ki a
V= yzi + xzj + xyk
vektorfggvny vonalmenti integrljt az
r() = (2 - l ) i + ( + 2)j + 3k
egyenlet grbe mentn, ha t e [0; 1].
14.81. Szmtsa ki a
V = (j^ x^)i + 4yz] x \
vektorfggvny vonalmenti integrljt az
r(0 = ri + t^j +
trgrbe mentn, ha 0 ^ ^ 1.
14.82. Szmtsa ki a
V= (2 xz + y^)i + {2 xy + z^)j + (x^ + 2 yz)k
vektorfggvny vonalmenti integrljt a P(0; 0; 0) pontbl a g(3; - 1; 2) pontba
vezet egyenesszakasz mentn!
14.83. Szmtsa ki a
V= (x + yz)i + {x^ - z^)j + (xy + z)k
vektorfggvny vonalmenti integrljt a P i ( l ; 1; 1) s P2 (0 ', 3; 5) pontokat sszekt
egyenesszakasz mentn, Pj-tl Pj haladva!
207
14.84. Szmtsa ki a
V= ( - x ^ + y+z)i + ( x - y ^ + z)} + ( x+y- z ^) k
vektorfggvny vonalmenti integrljt a P(2; 0; 0), Q(0 ; 2; 0), /?(0; 0; 2) pontok ltal
meghatrozott hromszg kerlete mentn, ebben az irnyban haladva!
14.85. Szmtsa ki a
V= 2jc- - j + ( x - z ) k
z
vektormez vonalmenti integrljt az
r(0 = ( l - O i + (^-l)j + ^k
egyenlet grbe mentn, ha 1
14.86. Szmtsa ki a
V= x H- yi + - k
vektormez vonalmenti integrljt, ha az integrcis t az
r(0 = H-----jH--------- k
1+ / 1+ 2'' l + 3
grbe s 1.
14.87. Szmtsa ki a
V = (x->)i + (x+>')j + xj^k
vektorfggvny vonalmenti integrljt az
r(0 = (cos )i + (sin t)\ + /k
egyenlet grbe mentn, ha
14.88. Szmtsa ki a
V = ------- + 2 yzj + (y^+ l)k
1 X
vektormez vonalmenti integrljt az
r(/) = yi + l/TTj + /k
egyenlet grbe mentn, ha 1
208
14.89. Szmtsa ki a
vektormez vonalmenti integrljt az
r ( 0 = + t ^ } + t \
grbe mentn, ha I ^ ^ 4.
14.90. Szmtsa ki a
V= (y + z)i + (x + 2)j + (x + j)k
vektorfggvny vonalmenti integrljt az
r() = (sin )i + (cos )j + k
egyenlet grbe mentn, ha 0^ ^ 2;t.
14.91. Szmtsa ki a
V= (y+z)i+(z + x)j + (x + j;)k
vektormez vonalmenti integrljt az
r () = (sin^ )i + (sin 2 i)\ + (cos^
egyenlet grbe mentn, ha 0 ^ ^ .
4
14.92. Szmtsa ki a
V =
xz
- j +
y
vektormez vonalmenti integrljt az
r ( ) = (sin^ )i + (cos^ Oj + ^^k
n n
grbe mentn, ha - ^ ^ .
4 3
14.93. Szmtsa ki a
V = - - 2 xy+ -
X X)
i+l
y y ) y
vektormez vonalmenti integrljt, az
14 Matematikai feladatok 209
r(/) = (sin^ /)! + 4j + (/ sin 2/)k
. . . . . ^
egyenlet grbe menten, ha - ^ ^ .
4 3
14.94. Egy konzervatv elektromos ertr potencilfggvnye:
y
u{x,y) = y^+ Arctg-.
X
Adja meg az erteret! Mekkora munkt vgznk, ha az egysgnyi tltst a P(l; 1)
pontbl a Q(2; 2) pontba visszk?
14.95. Egy konzervatv vektormez potencilfggvnye
u(x, y, z) = x( y +z y~ In .
z
Adja meg a vektormezt! Mekkora a vonalmenti integrl rtke, ha az tvonal a
P(1; 1; 1) pontbl a g ( l ; 2; 2) pontba vezet egyenesszakasz?
14.96. Egy konzervatv vektormez potencilfggvnye:
u = ]f xy^.
Hatrozza meg a vektormezt! Mennyi a vonalmenti integrl rtke, ha az tvonal
a P i ( l ; 0; 1) pontbl a PjCl; 1; 0 pontba, majd a Pj pontbl a P j l ; 1; 0) pontba
vezet szakasz?
14.97. Egy konzervatv vektormez potencilfggvnye:
u = sin yz + cos xy + tg xz.
Adja meg a vektormezt! Mennyi a vonalmenti integrl rtke, ha az integrls tja
az az orig kzppont, egysgsugar krvonal, amely az x, z koordintaskon
fekszik?
14.98. Van-e az albbi vektormezknek potencilfggvnye? Ha van, hatrozza
meg!
aj V= {2 xy - z ^ - yz)i + {x^ + z ^ - xz)j + (2 yz - 2 xz - xy)k;
z z l
'' " x\ j x^y^-z^^^ y ] / x Y - z ^ ^ ~
14.99. Konzervatvak-e az albbi vektormezk? Ha igen, hatrozza meg a poten
cilfggvnyket!
a) V = {2 xy In z + ye^)i + (x^ In z + xe^^)j + - + 3z^^ k ;
210
w V- ( ^ + | 5 + | / 5 ) | - h( ^ + l f i + | S ) i + ( ^ + ^ ) k ;
c j V = ( f y + f z ) \ + { ^ + \ f z ) \ + { f y ^ f z ) V i .
14.100. Hogyan kell megvlasztani a p paramter rtkt, hogy a
V = (6xyz+4j^z^)i + (3x^z + 12xy^z^)l + (3x^y+pxy^z)k
vektormez konzervatv legyen? rja fel a potencilfggvnyt is!
14* 211
EREDMENYEK
J. fejezet
1.1. Az F s G kivtelvel valamennyi.
^. 2. a) A = {1,3,5,7,9,11,13,15,17, 19,21,23};
b) - 8, - 6, - 4 , - 2 , 2 , 4 , 6, 8,12};
c; C = { -18, - 6, - 3 , - 2 , - 1, 1, 2, 3, 6, 18}; d) i) = 0.
^ . 3. a) 2; b) 2 ; c) l; d) 0 ; e) l.
1.4. a) Vgtelen sok, a 3 ^ x ^ 5 intervallum vals szmai; b) hrom, mgpedig
5 = {3,4, 5};
1.5. J16; ; 7; ej 8; /; 8; e) \.
^. B. A = D = E, B=F.
1.7. rtj BnC = 0; 5uC = {a budapesti autbuszok s villamosok};
c) BnD = {a budapesti csukls autbuszok}; d) Cr>D = {a budapesti csukls
villamosok}; e) B \ D = {a budapesti nem csukls autbuszok};
f ) C \ D = {a budapesti nem csukls villamosok}; g) A \ C = {a budapesti nem
csukls jrmvek}.
1.8. ^ u 5 = | - 4 , 0 , 1 , ^ , 2 , 3 , 4 | ; AnB = {2}; . 4 \ = | - 4 , ^ | ;
B \ A = {0, 1, 3, 4}.
1.9. A' uB = {a pros termszetes szmok s a 3-mal oszthat pratlan termszetes
szmok}; AkjC A; B^ C = {a 3-mal vagy 4-gyel oszthat termszetes szmok};
AnB = {a 6-tal oszthat termszetes szmok};/nC = C;Br\C = {a 12-vel osztha
t termszetes szmok}; = {a 6m-4 alak termszetes szmok}; B \ A = {a
3(2/1) alak termszetes szmok}; A \ C = {a 2(2A:1) alak termszetes sz
mok}; C \ A = 0; j 5 \ C = {a 12A:-3, 12^-6s 12A:-9 alak termszetes szmok};
C \ B = {a 12A:-4 s 12A:-8 alak termszetes szmok}.
215
1.10. NvjP = N-, Nu S = N; N' uT = N; PuS = {azok a szmok, ame
lyek vagy prmek vagy prosak}; Pu T = T\j{2}\ S u T = N; NnP = P;
NnS = S; Nr\ T = T; PnS = {2}; P n T = P \ { 2 } ; S n T = 0.
1.11. ^2 = {60,62,64,66,68, 70}; ^3 = {60, 63, 66, 69}; ^^4 = (60, 64, 68};
A2 ^A^ = {60, 62, 63, 64, 66, 68, 69, 70}; ^ 2X ^3 = {62, 64, 68, 70};
A2 ^ A^ = {60, 64, 68}; A^kjA^ = {60, 63, 64, 68, 69}; A^T^A^ = {64, 68};
A2 C^A3 = {60, 66}; Aj^A^. = {60,62,64,66,68,70} = A^; = {62,66,70};
A ^ \ A 2 = 0; A^nA^ = {60}; A^^^A^ = {63, 66, 69}.
1.12. ^ X5 = {(1, 7), (1, 8), (1, 9), (2, 7), (2, 8), (2,9)};
Bx A = {(7,1), (8, 1), (9,1), (7, 2), (8, 2), (9, 2)};
A x A = {(1, 1), (1, 2), (2, 1),(2, 2),};
B x B = {(7, 7), (7, 8), (7, 9), (8, 7), (8, 8), (8, 9), (9, 7), (9, 8), (9, 9)}.
1.13. Af = Tr\R.
1.14. HnDr\ T = {ngyzetek}.
1.15. Kn T = {tglalapok s rombuszok}.
1.16. a) AkjB = {regnyek s matematikai knyvek};
b) Br\C = {matematika tanknyvek}; c) = {azok a magyar nyelv knyvek,
amelyek nem regnyek}; dj B \ C = {olyan matematikai trgy knyvek, ame
lyek nem tanknyvek}; ej BuC = {a matematikai trgy knyvek s az sszes
egyb knyv a tanknyvek kivtelvel}.
1.17. Igen, mert A \ B is a fiskola nem elsves frfihallgatinak halmaza s Ar\B
is az.
1.19. h) A^iBr^B) = Au0 = A; i) An(BKjB) = Ar^H = A.
W /A l.,
216
1.20. a) A \ ( A \ B ) = AnB = B \ ( B \ A ) . (brzolja a halmazokat!)
m ; A \ B , B\A,[ lA\C, B\C.
1.21. Pldul az aj igazolsa:
A : vzszintesen, BnC: fgglegesen vo- jobbra dl, ^uC: balra dl fer-
nalkzott rsz;
Au(BnC): ahol van vonal;
dn vonalkzott rsz; (AuB)n(AuC):
ahol ktfle vonal van.
1.22. Mivel
{AyB)r^{AvjB) = A^ ( Bn B) = A kj0 = A_,
{AyjB)n{KjB) = ^{Br^B) = / u 0 = A,
ezrt az adott kifejezs bal oldala: Ar\ = 0.
^.23. a) AczB s Ac C; b) AczC s BczC; c) A(^B<=C; d) A = B=C.
1.24. Az X elem a. d), e), f ) pldkban; az x rszhalmaz ab) s e) pldkban; az
A- sem nem elem, sem nem rszhalmaz aza) s c) pldkban. Mivel a feladat szvege
az x-rl kzelebbit nem mond, vlaszainkban felhasznltuk, hogy x.
1.25. P(H) = {0, {1}, {2}, {3}, {1, 2}, {1, 3}, {2, 3}, {1, 2, 3}}.
217
1.26. Az {1; 2} halmaz eleme a P(y4u) halmaznak, de nem eleme a P(J)uP()
halmaznak.
1.27. A bizonytsokat Venn-diagramok vagy azAB = egyen
lsg felhasznlsval vgezhetjk el.
1.28. l = {0}; 2 = {0,{0}} = {0, 1}; 3 = {0, {0}, {0, {0}}} = {0, 1, 2};
4 = {0, {0}, {0, {0}}, {0, {{0}, {0, {0}}}} = {0, 1, 2, 3}.
1.29. Csak a c) llts hamis, mert ln2 = 1; a tbbi llts igaz.
1.30. 3; b) 2; c) 4; d) 2- ej 4; f ) 3.
1.31. aj 0; j{(0, 0)}; ej {(0,0); (0, 1)};
d) {(0, 0), (1,0)}; e) {(0, 0), (0, 1), (1, 0), (1, I)};
f ) {(0, 0), (0, 1), (0, 2), (1, 0), (1, 1), (1, 2)};
g) {(0,0), (0, I), (1,0), (1,1), (2,0), (2, I)};
h) {(0, 0), (0, 1), (0, 2), (1, 0), (1, 1), (1, 2), (2, 0), (2, 1), (2, 2)}.
1.32. A = {1,2, 3, 7, 11, 14,22,77}.
1.33. A = {15, 30, 39, 65, 78, 130, 195, 390}.
1.34. A reklmfnk nem mondott igazat, mert a felsorolsban valjban csak
40 ember van. Ugyanis ABC=5, ezrt A B \ C = 11 5 = 6, A C \ B = 9 5 = 4,
B C \ A = 10-5 = 5. Ebbl kvetkezik, hogy A \ A C ^ A B = 26-(4 + 5 + 6) = 11,
B\AB<j BC = 25-(6 + 5 + 5) = 9, C\ ACu BC = 14-(4+5 + 5) = 0 (1. az brt).
1.35.110.
1.36. Ha N az zem ndolgozinak, S a szvd dolgozinak s T a tavalyi
218
utasoknak a halmaza, akkor a TnS'niV halmaz elemeinek a szmt kell megllapta
ni; ez 90 f.
1.37. Nem. Ha x jelli azoknak a vevknek a szmt, akik mind a hrom rucikk
bl vsroltak (x^O), akkor a vevk szma
1 5 + 1 2 + 1 0 - 6 - 1 - 3 + x = 27 + jc,
ez pedig nem lehet egyenl az elad ltal megadott 25-tel.
1.38. a) 20; b) 30; c) 2.
100
1.39. Az ellentmonds pldul gy mutathat ki: Mivel a sportol kollgista
lenyok szma 5, ezrt a sportol kollgista fik 10 5 = 5, a sportol nem kollgis
ta lenyok pedig 20-5 = 15. A sportol nem kollgista fik szma gy
23- (15 + 5 + 5 ) = - 2 volna, de ez lehetetlen.
1.40. tmutats: Vettse az tmrre a flkr pontjait, az tmrre merlegesen!
2 . fejezet
2.1 . - ( a + b); - ( b + c); - ( a + c); - ( a + b + c).
2.2. Az s ^ 3 - ? a paralelogramma kt szemkzti lvektora, ezrt
ezek egyenlek. Ebbl az egyenlsgbl trendezssel kvetkezik a bizonytand
egyenlsg.
2.3. d = b + c a.
x + y - z
2.4. a = ----------; b =
x - y + z
c =
- x + y + z
219
2.5. Az klaplap cscsainak helyvektorai: x, y, y - x, - x, - y, x - y; a fedlap cs
csai x + z, y + z, y - x + z, - x + z, - y + z, x - y + z.
2.6. Wi = 14a-5b; = -(a+13b).
2.7. aj 3a-4b = i(IOv-llw); j 5a + 2b = ^ ( 8v-w); cj -a+:j^b =
1
= ( - 21v + 12w).
2.8. a ) J = 2c-a; b ) T d = 2b-a; c ) = b + c-a; d j T =
2b + 2c a
2.9. a' = 2b-a.
2.10. Ha a szban forg pontok helyvektorai a, b, c, p, akkor a tkrkpek hely
vektorai rendre 2a-p; 2b-2a + p; 2c-2b + 2a-p; -2c + 2b + p; 2c-p; p.
2.11.
a + 3b a + b 3a + b
4 2 4
2.12. (0 ; 0 ; 0); (2; 0 ; 0); (0 ; 2; 0); (0; 0; 2); (2; 2; 0); (0 ; 2; 2); (2; 0; 2); (2; 2; 2).
2.13. C(l; - 1 ; 1 ) .
2.14. C(4; 2; 3); I>(5; 1; 3); (5; 0; 4); l(4; 0; 5).
2.15. D(-2; - 1; 12).
2.16. ( - 1 ; 13; - 4 ) ; (5; 8; - 3 ) ; (12; 7; - 7 ) ; (8; 14; -7).
2.17. a ) ^ = a + b = (9; 8; - 5 ) ; b) = a + b + c = (10; 12; - 8);
c) = a + c = (5; 6; - 6); d) ~A>^ = b + c = (6; 10; - 5 ) ;
- ^ 1 / 11^
-4 ) ; / ; Z ^ = a + - ( b + c) = ( ^ 7 ; 7 ; - y ^
1
e ) A = - (a + b + c) = (5; 6
n
T
9\ 1 / 13^
-j; / j ; Z ^ = - a + b + c = 8 ; l l ; - - c j ) A = ^ i - ^ { h + c) =
i ) F= - ^ a = f-2;-l;^); jV ^ = ^(-a + b + c) = (1; 4; - 1);
2 \ IJ 1
= ^(-3a+b+c) = 1; i ) ; 1 ) h ^ = ^(a+b+c) =
220
2.18. a) nem; b) igen, d = - c; c) igen, mert a zrusvektor brmely vek
torral prhuzamos.
2.19. a) igen; b) nem.
2.20. P'(3; 18; - 12).
2.21. V i ( - 3; 2; 0); V3((3; t - 2- 4/ 5; 2(/5- 7t); 5(7t + 1)).
2.22. v = 8a + b; Vj = 8a(16; 56), V2 = b(-3;0).
2.23. d = - 3 a - 3 b + 7c; , = -3a(24; - 21; -3), dj = -3b(0; - 9 ; - 6),
d3 = 7c(7; - 7 ; 28).
2.24. |al = 18; |b| = 1; ld = j / m = 14,036.
^ / 4 12 3\ / 1 4 8\
2.25. e J - ; - ; e,(0; 0; - l ) ;
2
13 13 13
1 2 3 \ / 2 2 1
= (0,2673; 0,5345;-0,8018); e.
/4 /4 /4,
2.26. 2.
2.27. Az adott kt-kt vektor hajlsszge: a) tompaszg; b) hegyesszg; c) derk
szg; d) hegyesszg.
2.28. z = - 1 2 .
ab
2.29. p = - , ha b#0. Ha b zrusvektor, akkor p tetszleges.
2.30. Az a s b vektorokat gy kell megvlasztani, hogy a -L b s la| = |b|
egyszerre teljesljn.
2.31. Igen, mgpedig ha p : ^ = ac : ab.
2.32. Az a, b, c vektorok kockt fesztenek ki, mert ab = bc = ca = 0,
|a| = |b| = i c l = 7.
2.33. Az X, y, z tengellyel bezrt szg rendre 63,64; 96,38; 21,IT.
2.34. a) van; b) nincs.
221
2.35. a) 90; b) 57,49; c) 107,29; d) 106,78; e) 148,7.
2.36. AB=BC=CA = \a][2.
2.37. Az hromszg derkszg; az A2 B2 C2 hromszg hegyesszg; az
A^B^Ci hromszg tompaszg.
2.38. a = 32,31;)?= 38,33; y= 109,36.
2.39. 16,6.
2.40. 30; M(ll; - 2; - 1).
2.41. Az AA1 A2 hromszg egyenl szr; az AFA2 szg derkszg.
2.42. k = 49 + / ^ = 89,31; 59,74.
2.43. Az AAi B hromszg tompaszg.
2.44. 2 egysg.
/ 77 44 44\ '
2.46. A feladat feltteleit kielgtik mindazok a b vektorok, amelyek vagy merle
gesek a-ra (ekkor a merleges vetletek hossza 0), vagy amelyekre |b| = ]a!.
2.47. p(6; - 6 ; 3 ) ; i n ( - 3 ; 0 ; 6 ) .
2.48. r ( l ; - 1; 1) .
2.49. F ( - 2 ; 5; 3); BCA szg = 33.
2.50. r ( 10; - 1; - 5 ) ; m = 2 ^ = 10,198 egysg.
2.51. 7F=0.
1/ ^
2.52. ST = = 2,52 egysg.
2.53. a) - 3(a x b); b) 6(a x b); c) 0.
2.54. v(18; 19; 6).
222
a Xb
2.55. c = ----- .
Ia|
2.56. c(2; 3; 6) vagy - c(- 2 3 ; - 6).
2.57. Az alaplap cscsai; (0; 0; 0), ( - 1; 0; 2), (l ; 1; 0), (0; 1; 2), a fedlap cscsai:
( - 6; 6; -3 ) , ( - 7 ; 6; 1), ( - 5 ; 7; - 3), ( - 6; 7; - 1).
2.58. c ( 5 ; 6 ; -2).
2.59. x(7;5;l).
2.60. = 55,68.
2.61. t = 18|/3 = 31,18 terletegysg.
2.62. a) t = ^l/8^ = 21,68 terletegysg; b) t = = 20,62 terletegy
sg; ej =0 terletegysg; d) t = = 2 , \ 2 terletegysg.
2.63. A\\c3\ A\\B\ =20 terletegysg.
2.64. = 161bxa| = 16.
2.65. m = 5 egysg.
2.66. k = 2{][^+p2 + 'l) = 24,33 egysg; t=\Op. = \A,\ terletegysg.
m f i
2.67. t = = 10,42 terletegysg.
2.68. (axb)^ + (ab)^ = a^b^ sin^(a, b) + a^b^ cos^(a, b) = a^b^.
2.69. bxc = b x ( - a - b ) = b x ( - a ) = axb;
c x a = ( - a - b ) x a = - b x a = axb.
2.70. a) igen; b) nem; c) igen.
2.71. Balsodrs rendszert alkotnak.
2.72. a) igen; b) nem; c) igen.
223
2.73. z = y
2.74. Mivel abc = 0, ezrt pldul azaxb = ( - 2 ; 7 ; 6 ) vektor merleges a hrom
adott vektor mindegyikre.
2.75. A pontok nincsenek egy skban. = 42,7 terletegysg.
2.76. V= 3 trfogategysg.
2.77. a) V= 6 trfogategysg; bj V= 1 trfogategysg.
2.78. Z)i(0;8;0).
?l/3
2.79. m = = 4,04 egysg.
2.80. m = 11 egysg.
2.81. Igen.
2.82. V1V2V3 = 22abc 0, s ezrt nincsenek egy skban.
2.83. A kapott paralelepipedon trfogata az eredeti paralelepipedon trfogatnak
a ktszerese.
2.84. Mivel VjVjVj = 27abc, ezrt Ki = 27F.
x +1 y 3 z 1
2.85. 0; X = - l - 4 , j = 3 + 2, z = 7 + 6? vagy ^ ^ '
b ) x t , y 1+ 7, 2 = 2 9;
c) X = 6 + 9, 7 = 8, z = 23; d) x = \ + At, y = 2 + 3 / , z = 5 2t.
2.86. a) x = - 2 + 3t , y = 5 - 2 t , z = 6 - t ; b) x = 5-10/, y= 1, z = 2 + t;
c) x = 9t, y =l \ t , z= - 1 ; d) x = 3 + t, y = - l ~ t , z = t.
2.87. a) Nem illeszkednek egy egyenesre, b) Egy egyenesre illeszkednek.
2.88. r(0 = ( - 3 + 2t)i + (2 + /)j + ( - 1- 7/)k.
2.89. X = - 1 + 3/, j; = 2 - 9 / , z= - 4 / .
224
2.90. r(0 = (2-4)i + (l + 30j + k.
2.91. x = - 2 t , y = 5-6, z = 2.
2.92. X = 3 3, j = 1+ 6, z = 1+ 5/.
2.93. A metszspont: M( 3; 2; 1). Az egyenes egyenletrendszere: x = 3 + 7,
j = 2 + 23, z = l - 6 t.
2.94./7 = 3.
2.95. r(0 = (l + 13)i-(l + 60j + (5-70k.
2.96. X= 2 + 3/, j = 9 + 6/, z = 1+ .
X1 X1 2 - z
2 . 9 7 . ^ = ^ = z- 2 s - ^ = >^+8 = .
2.98. a) x= f, y = - 7 5;z = - 9 - 9 f , b ) x = 2 + 2t , y = - l + 7, z = 4;
c) X= 5 - t , y = 4 + 7/, z = 3/.
2.99. r(/) = (1 + 2/)i + (3 + /)j + (2 + /)k.
2.100. X= - 2 - 5 / , j = 1+ 8/, z = 2 + 3/. Igaz, illeszkedik.
2.101. r ( / ) = (l + 2/)i -3/j-( 5 + 6/)k.
2.102. X = 1+ I 0t , y= 1-3/, z = 1+ 12/.
2.103. X = - 7 - 2 / , j = 4 + 3/, z = 4 + 4/; / = 4 | / ^ = 21,54 egysg.
2.104. X = 2 + 3/,y = 1+ 3/, z = 1+ /. d=0.
2.105. A ponthalmaz a lOx14 = (5;^ +4) = 2z egyenletrendszer egyenes.
2.106. a) - 3 x + 2 j + l l z + 25 = 0; b) 9x+y + 25 = 0; c) x + z - 1 = 0;
d) 3x 12y + z = 0; e) y - 2 = 0.
2.107. 3 x - 4 j + 5z-26 = 0.
2.108. a) I7x -3v+12z = 37; b) 3x + 3>; + z = 8; c) 5x + 2>^-3z+13 = 0;
d) l l x 6j + 9z13 = 0; e) 2 x + y z = 3.
15 Matematikai feladatok 225
2.109. Igen, a sk egyenlete: 2 x +y - z = 3.
2.110. 5y~2z = 0.
2.111. 12x+16y+5z-23 = 0.
2.112. x + 4y + 7z+16 = 0.
2.113. 6 x - 2 0 y - l l z + l = 0 .
2.114. 1 6 x - 3 j - 1 0 z - 3 1 = 0.
2.115. x+5y + 6z-23 = 0.
2.116. 2 x + y + 4z- 28 = 0.
2.117. x + 4y + 2z - 2 = 0.
2.118. A = - l .
2.119. Nem illeszkedik, mert pl. nv = 2 # 0.
2.120. 2 x +l 0 y +l l z - l 3 = 0.
2.121. 4x+5y + 2 z - 6 = 0.
2.122. 2x + 7y+13z + 4 = 0.
2.123. 2 x + 3 y - z - 3 = 0.
2.124. l x - 2 y + z + %= 0.
2.125. 2 x + l y +3 z - 5 \ = 0.
2.126. - 7 X + J + Z+1 0 = 0.
2.127. 8 x-22j + z-4 8 = 0.
2.128. 3 x + y - 3 z = 2.
2.129. 5 x + 3y + 4z = 14.
2.130. 2x+3;;-2z = 0.
226
2.131. JC+ J + 2 Z - 6 = 0.
2.132. A hrom egyenes egy skban van.
2.133. 1 3 x - 6 j - 5 z + 56 = 0.
2.134. 13x-14;;+llz+51 = 0.
2.135. 3j-7z+18 = 0.
2.136. 19x- 2y- 9z - 66 = 0.
2.137. 7x + 57+2z- 4 = 0.
2.138. K( - 2; l ; -1).
2.139. Van. A kzs pont: Pl -
- 4
TT
A .
n I I
2.140. 2 x - 3 j + z - 1 4 = 0.
2.141. A megadott sk az s 5 pontokat nem vlasztja el.
2.142. x + 4 y - 3 z - 9 = 0.
2.143. 3 x + 2 y - z + 2 = 0.
2.144. A feltteleknek kt sk felel meg, egyenletk: 2x + 3y 6 z 12 = 0, illetve
2x+3y + 6 z 1 2 = 0.
2.145. MJ 2 ; 6; 0), 0; 2), M,,(0; 9; - 1).
2.146. MJ 9 ; - 4 ; 0), 2; -3), M(3; 0; -2).
/12 3 15^
2.149. D(4;6;2).
2.150. M(4; - 1 ; - 3).
15*
227
2.151./J= - 3 .
2.152. t = ^|/1449 = 19,03 terletegysg.
2.153. r = 4 trfogategysg.
2.154. Metszi; a metszsvonal egyenletrendszere ; x = 1+ , j = 3 ' it,z = 2+1.
2.155. P'{0; 11; -13).
/19 12 23^
2.157. F ( l ; 1;6)
2.158. P'(9;0; 1).
2.159. x - j - z + 2 = 0.
2.160. 2 x - 3 y - 4 z + n = 0.
2.161. x= - 1 , 7 = 5-3, z = 5 + .
2.162. 0.
2.163. 5x - Sy +3z +U = 0.
/53 5 20^
2.164. A'
7 7 7
1
2.165. X= \ + 2t , y = 2t, z = 2 - - .
2.166. X = l +3t , y = l - 4 , z = 2 - 3 ; igen.
2.167. S\ x-3>y + 2z + 6 = 0. e' 2 : I4x = 25-14, 14;; = 23+14,
14z= -2 0 + 28?.
2.168. a; 60; 45; cj 90; dj 43,4.
2.169. aj 0; b) 24,22; c) 26,56; d) 30.
228
2.170. a) 90; b) 61,87; c) 82,34; d) 45.
2.171. Az 6 2 egyenes az egyenessel zr be nagyobb szget.
2A12. d = = 6,568 egysg.
122
9
= 2,53 egysg.
44
2.174. d = = 2,32 egysg.
2.175. d = 7= = 0,8 egysg.
] j \ A
2.176. d=2A,5 egysg.
2.177. d=0.
2.178. a) A=^l\ b) A ^ l , 5 =3 ; c) A = l, 5 ^ 3 .
2.179. x + y - 2 z + \ = 0.
2.180. Az AB szakasz felezmerleges-skjnak s az adott egyenesnek a dfs
pontja a keresett pont: /*( 2; 2; 2).
X8 V+ 4z 4x + 20y 5z
2.181. Egy keresett egyenes a z ----- ------ = 5 s a ------- -------= 5 skok met-
215 25
szsvonala: x = ------- 40; v = 21, z = ----- H52.
7 7
2.182. x + ; ; - z - 3 = 0.
/ 2\
2.183. A feladat felttelnek kt pont felel meg, a 0 ; 0 ; - -
V
es a
( 28\
^ 2(^0; 0; - Y j pont.
2.184. A keresett P pont az x = 3 - 3, >>= - l + 2t, z = t egyenlet metszsvonal
s az x + 2_y+ z - 1 = 0 egyenlet, az adott skok tvolsgt felez sk dfspontja:
^ ( - 1 ; 3 ; 0 ) .
229
2.185. Kt szgfelez sk van, ezek egyenlete z+1 = 0 , illetve x + 2y = 0.
A P pontra az els sk illeszkedik.
f i
2.186. d = = 0,707 egysg.
2.187. d=][\% = 4,24 egysg.
2.188. Az els skpr metszsvonala: x = \ , y = t , z = f, &msodik skpr metszs
vonala: x=2t, y= - t , z= - 1. \ tvolsg d= 1 egysg.
.
2.189. jc = 6 - 3 t , y = ~4 + 2t, z = 8 - 4 .
2.190. A t normitranszverzlis egyenletrendszere; x = 3 + 3, y = \+2t,
z = 4 - 3. A s metszspontja: Mi(3; 1; 4), a ts ej metszspontja: Mji; 3; 1).
Mi M2 = ]/n = 4,69 egysg.
2.191. A metszsvonal vektoregyenlete: r() = ri (7 + 5)j 9(1 + )k; az egye-
nes vektoregyenlete: r() = ( 2 - )i + (l - 50j + (6-9)k; d = / = 0,453 egysg.
86
2.192. 2x13j + 7z 90 = 0, m - j = = 5,77 egysg.
/222
/ 7 14 14\ ^
2.193. F(0; 2; 4); r - - ; - ; - j ; F r = 2[/3 = 3,46 egysg;
r(/) = - r i + (2+ll)j + (4+110k.
2.194. ni(3; 1; - 2); n^d; 2; 0); m(4; - 2 ; 5); V = = 0,134 trfogategy
sg. A vektorok irnytstl fggen nyolc klnbz helyzet, de azonos trfogat
tetrader van.
2.195. A kt egyenes skja: 2x + y 2z = 4. A vetleti egyenes egyenletrendszere:
17 2 10
.v = + , j = - - - 2/, z = , a kt egyenes prhuzamos.
2.196. A tetrader cscspontjai: ^4(1; 0; 2), 1; 2; 1), C(3; 2; 0), D( - 2; 2; 1).
A trfogat: V = -trfogategysg; a felszn: F = ~{l + 1^104+ [^168 + 1/5) = 13,2 ter
letegysg.
230
2.197. A lapok skjnak egyenlete: x + 3j + 4z = 9; y = 2; x + 4y+5z.= 11;
y + 2z = 4; az J cscsbl hzhat magassg hossza; m = l egysg.
2.198. A ~ 0 egyenletbl p = 2. A sk egyenlete x + 8>>3z 20 = 0.
" 20
/ = - ^ = 2,32 egysg;
2.199. P ( l ; l ; - 1); /= 3 egysg.
2.200. Az egyenes egyenletrendszere; x = l + 22, y = - 2 - 1 6 , z = 3-21.
A kt egyenes metszi egymst, a metszspont M(12; 10; 7,5). A sk egyenlete;
13l/^
26x107j + 92z 692 = 0. A tvolsg d = -------= 2,84 egysg.
3. fejezet
3.1. a) Z) = 0;
c) D= - 2i +nj +Zk- ,
e) Z) = 28;
21
b) D= - 6 + i;
d) Z) = 40;
f ) D = - 4 .
3.2. Az els oszlopot a msodikbl s a harmadikbl levonva, az llts igazolhat.
3.3. a) Xj = 2, Xj = 3;
c) x = 9;
Xi = X2 = l, X3= - 2 ;
d) Xj = 0 s X2 = 2.
3.4. sin^ 2x cos^ 2x+sin^ x + cos^ X = 0.
- 2 0
3.5. A + B = 4 - 1
9 9
3.6. c) s d) igaz.
3.7.
-13/ 2 - 3 i
- 7 - 3 + 8/
3.8. Nem, mivel A s B nen szorozhatk ssze.
3.9, A^=
sm 2a
231
1 - 2 - 6
3.10. A^= - 3 2 9
2 0 - 3
(A^) ^ nem ltezik, mert sorvektorai linerisan sszefggk. (A^ els sort a
msodikhoz adva, az j mtrix kt sora megegyezik, igy det A^ = 0.)
3.11. Nem, mert AB egy (2 x 2)-es, BA pedig egy (3 x 3)-as mtrix.
3.12. B =
15 4
4 15
3.13. AB =
0 0 0
0 0 0
c 2c 3c
s ez c = 0 esetn szimmetrikus.
- 12_
(sszegzvektor) ;
10 4 - 1 T
15 - 1 - 12 7
5 1 - 2 3
7
4
3.14. a) AB =
b) AB =
c) AB =
d) AB =
3.15. (A + B)2 =
A^ =
BA =
A^ + BA + AB + B^ = 4
3.16. AB = [32, - 4 , - 37] ; (AB)C = - 179;
' 6 r
2
= 4
~io r
_4 -2_ _ 4 2_
"19 - 1 8 '
; B^ =
9 - 8'
_12 - 5_ _ - 4 17_
13 r
- 1 29
; AB =

_ 4 -9_ 4 5_
10 r
4 2
BC =
2
23
- 17
; A(BC) = - 179.
232
3.17. adj A =
3 7 - 2 8
17 -1 7 -17
- 1 3 - 2 8
' - 8 5
0 0'
A adj A = (adj A)A
=
0 -85 0
_ _ {
85E.
0 0 - 8 5
0 - 2
5' 0 - 2c 5
CA = 3c 8c - c ; AC 3 8c - 1
1 - 4 6 1 4c 6
-3 5 - r ' 8 1 0 '
3.19. PA = 1 8 0 ; AP = 5 - 3 - 1
1 - 2 2
- 2 1 2
Balrl szorozva a szorzott mtrix els kt sora cserldtt fel, jobbrl szorozva pedig
az els kt oszlopa.
3.20. Nem, mert det A = 0.
3 2 - 4
3.21. adj A = - 4 - 3 6
6 4 - 9
det A = - 1, gy
- 3 - 2 A l 0 0'
A-i = 4 3 - 6 s AA ^ = 0 1 0
- 6 - 4 9 0 0 1
= E.
Megjegyzs: az inverz mtrix elemi bzistranszformci tjn is meghatrozhat.
(L. a Vektorgeometria c. fiskolai tanknyv 2.3.6. pontjt!)
3.22. det A = 1esetn, ill. ha adj A = A ^(az inverz mtrix elemi bzistranszform-
cival is meghatrozhat).
3.23. det A = 1, gy az egyenlsg fennll.
- 3 1 r -3 1 r
3.24. AB = 12 18 26 ; BA = 1 28 25
9 16 17 1 13 7
AB#BA; AB-BA =
gy a keresett inverz nem ltezik.
0 0 0'
11 . .
233
3.25. a)
Vi V2 V3 , b) V, V2 V,.
N,
5 1 8
N,
6 6 7
N, 4 4 0 N, 6 0 3
N3
2 0 4
N3
2 6 5
3.26.
Vi
teljes anyagszksglet
N,
41 20 65 000
N, 41 21 66 200
3.27.
A, A2
^3 ^4
Fi
Fi
^2 ^3 \
1 5 2 3
2 0 4 3
4 1 4 0
K,
3 2 2 15 17
K, 3 2 4 (g) 19
K, 2 0 5 22 0
K,
3 3 1 13 16
K2 tartalmazza a legtbbet ^i-bl, 23-at, pedig a legkevesebbet Aj-
3.28. A^ Aj Aj A^
Fi
2 1 3 4
F 2
4 1 1 0
F ,
3 5 0 3
Fi F, F,
K, 2 4 2 26 10
K2
3 5 0 26 14
K, 1 6 1 (g ) 9
^4
2 2 3 21 0
Kj tartalmazza a legtbbet A^-bcA, s a legkevesebbet ^ 3-bl.
3.29. Ki tartalmazza a legtbbet ^ 3-bl, 19-et, s pedig a legkevesebbet az
^ 4-bl, 13-at.
3.30. a) e(A) = 3 (ugyanis det A 0);
b)Q{K)=\- c ; e ( C ) = l ;
d) e(D)=4.
3.31. a) Xi = 1,
234
X2 = 0, ^3 = 2;
b) x =\ , J = - l , z = 2\
c) x =l , y = l, z = 3, v = - 2 \
d) x = 3, y = - 4 , z = - l , u=l ;
e) = 0, de pl. gy nincs megolds.
3.32. a) x= 1, y = 2 s z= 1; b) nincs megolds (mivel x= 1).
3.33. x = 2; y = - 2 ; z = 3.
3.34.
1 - 1 - 2 1 1 '1 - 1 - 2 1 1
2 1 1 0 2 0 3 5 - 2
i
0
- 1 - 2 0 1 - 1 0 - 3 - 2 2

1
1
0
3 - 2 2 - 1 4 0 I 8 - 4
I
i
1
1 - 1
- 2
1 1
r '1 - 1 - 2
1 1
r
0 3
5 - 2 i
0 0 3 5

2 i
0
0 0
3
0 i
0 0 0 3
0 !
0
19 10 1 10
0 0
T
3 1
1 0 0 0
1
3 1
1
Xi = l, l; X2=-0, 2; X3 = 0; X4=-0,3.
3.35. a) Z)^0, gy x=y = z = Q-,
b) Z) = 0, gy X: : z = 3 : ( - 2 ) : ( - 5 ) vagy x = 3, j^= - 2 t , z = - 5t , ahol / tetszle
ges vals szm;
c) D = 0, gy Xi: X2 : X3 = ( - 2 ) : ( - I ) : 1, vagy x^ = - 2t , y= - t , z^ = t, ahol t
tetszleges vals szm;
d) D^O, gy x = y = z = 0.
3.36. Legyen A az egytthatmtrix, B a kibvtett mtrix, n az ismeretlenek
szma.
a) e(A) = (B) = n = 3, gy van egyrtelm megolds:
2 I 5
" = 3 "^ = 6 -
b) (A) = (B) = 2, gy van megolds, mivel azonban = 4, 2 ismeretlen vlaszt
hat szabadon.
c) e(A)#e(B), gy az egyenletrendszernek nincs megoldsa.
3.37. a) 1, de C2 = 0, nincs megolds.
Cl = 1 s 2 = 0 esetn vgtelen sok megolds van: z = t, y = 2 + 1, x = t +l
{t paramter). Egyb Cj, Cj prvlaszts esetn egyrtelm megolds van, pl. C2= 1
s Cl = 0 esetn : x = !,>= 2, z= 1.
235
3.38. Z) = ( - l ) ^ ( + 2); O, = - ( - 1)^ ( + 1);
Z), = s D, = ( - l ) 2( / + l ) 2.
a) Ha t^I s # 2, akkor
+1
y =
1
+ 2
es z =
{t +\ ?
+ 2
j Ha t =\ , akkor az egyenletrendszer linerisan sszefgg, x = l y z; kt
ismeretlen szabadon vlaszthat.
cj Ha = - 2 , az egyenletrendszernek nincs megoldsa.
3.39.
a) A ^ =
b) A"' =
46
1 9 In
4
=
1
4
11 5
28
; X
= A-' b = [3, 1, 2]*.
1 1 3
- 4
1
- 6 39 21 2'
=
45 -45 45 ; X= A ^b = - 2
485
46 31 - 4 2 3
'37 8 i r
21 12 5
22 6 14
X= A b =
2
-2
3
- 12 - 22 - 1 6
-13 -38 -23
- 4
e ) x = [l, -1, 2]*.
X= A =
4 6
/ ; X= [2, - 2 , 1]*.
- 2
1
- 1
236
3.40.
a)
v = - 4 ; >-=5; z = - 2 .
b )
A' i = 4 ; ^ 2 = 1 ; ^ 3 = -
Xi
^2 ^3
X4 b
'2
0
0 0 9
3 2 0 0 14
1 1 3 4 10
2 - 1 2 3 11
2 1 0 0 9
- 1 0 0 0 - 4
- 1 0 3 4 1
4 0 2 3 20
0 1 0 0 1
1 0 0 0 4
0 0
1
4 5
0 0 2 3 4
0 1 0 0 1
1 0 0 0 4
0 0 1 4/3 5/3
0 0 0
1/^
2/3
0 1 0 0 1
1 0 0 0 4
0 0 1 0 - 1
0
l s
0
i X,
0
^= 2.
1 2
237
c) (Itt mr egyms mellett vannak a tblzatok, rvidtve!)
Xi ^2
^3
X4 1
X2
^3
X4I
^3
X4I
^4 !
3 0 - 1 2 1 0 0 - 1 6 - 1 6 | -48 -1 6 -48 0 0
0 1 1 1 1 4 1 1| 4
1
1
4 0 1
2 - 1 0 2 1 1 - 1 -10 -1 0 | -31 9
- 9I
-27 0 0
1 0 5
6 1
16 0 5 6| 16 5
6|
16 1 1
-
1 2 - 2 - 3 1- 5 2 3 3| 11 I
1|
3 1 3
q{A) = 3, n = 4, gy egy ismeretlen, pl. X4 szabadon vlaszthat. Ekkor: Xj = 1-
- .V4 = 1- , X2 = 1, X3 = 3 - X4 = 3 - , x^=t. (t tetszleges vals szm.)
eV
y
u u
-6 ' 10
4 j- 12
l l - 3
I
8 | - 2 4
1
1
0
0
3
-2
1
- 7
0
3
I
3
1
4
- 3
- 3
3
- 5
- 7
0
3
- 1
3
- 3
- 1
1
1
0 I -8
-2 I 6
I
- 1
- 4
- 1
0 1 0
x= 8, y = 3 + m, z = 6 + 2m, ahol u szabadon vlaszthat.
3.41.
a) Ai = 2, A, = 4, A,= - 4.
Si = [ l ; l / 3 ; 0 r ; = [ - 3 ; 2]*; S3 = [-|/3; 1; -2^3]*.
A normlt vektorrendszer;
s? =
f i 1^ 1
*
~ 3 r
*
oO_ r ^ . 1 .
- ^ 1 S2 -
4 T 2_
, S3 -
4 4 2_
b) Si = s? = [1; 0; 0]*; = s = [0; 1; 0]*;
S3 = [2; 0; 1]*, normiv: s=
2 1
238
4. fejezet
4.1.
4.2. a) cos 0 + / sin 0;
c) cos 90 + /sin 90;
e) f i (cos 45 + i sin 45);
g) 3/2 (cos 225+ sin 225);
i) j/5 (cos 26,6 + isin 26,6);
k) 5 (cos 53,2 + / sin 53,2);
l) 5,39 (cos 111,8 + /sin 111,8).
4.3. a) 4e''^; b) 2e^\
4.4. a) 7 - 5 ;
b) 8 (cos 180 + /sin 180);
d) 2 (cos 270 + /sin 270);
f ) 2l/2(cosl35 + /sinl35);
h) 2 (cos 330 + /sin 330);
j) fS (cos 243,4 + i sin 243,4);
, 5 n . 3 n Aln
6
b) 6;
18 13
f ) 7 + 9/; g ) -----1-----/;
y/ 29 29
21 20
c) lOe'^; d) 2e^ , e) e ^ ; f ) 2e
d) 8/; e) 22-3/; c) - 3 + 3/;
3 11
k) 1; l) - 4 6 - 9 / ;
41 38
m; - 4 ; o; 0,21-0,14/.
14 23
4.5. -26 + 7/; b) - i .
4.6. a) X 1+ i s j;= /; b) x = 2+ /, y = 2~i:
c) X = 3 - Ili, y = 3- 9/, z = 1- 7/ .
4.7. A z^-et s a Z2-t szemlltet vektorok a) egymssal prhuzamosak; b) egyms
ra merlegesek.
239
1 l/3
4.8. Legyen z = a + bi, a = 0 vagy a = l s b = 0; a = - s b = .
4.9. 5-5i.
4.10. 122-597.
4.11.
a; 2 -4,47 (cos 136,4 + /sin 136,4); b) 18,4 (cos 135+/sin 135);
c; 1,3 (cos 144,2 + /sin 144,2); d) 1,01 (cos 327,1 + /sin 327,1);
e) l,33[cos(85,4 + A:90) + /sin(85,4 + A:90)], A; = 0, 1, 2, 3;
f) 1,17 [cos (12,7+ A:72) + /sin (12,7+ A:72)], /t = 0, 1, 2, 3, 4;
g) l,97[cos(52,3 + A:120) + /sin(52,3 + A:120)], A: = 0, 1, 2;
h) 0,942 [cos (13,6 + A:60) + /sin(13,6 + /t60)], A: = 0, 1, 2, ..., 5;
i) 0,85[cos(2 + m ) + /sin(2 + /t72)], k = 0, l, 4;
J) 0,77 [cos (25,1 + A:90) + /sin (25,1+ A:90)], ^ = 0, 1, 2, 3.
4.12.
a) cos 2a + / sin 2a;
44-5/
c)
318
d) 2; e) 2/"-.
4.13. cos 3a = cos^ a 3 cos a sin^ a;
sin 3a = sin^ a + 3 cos^ a sin a.
4.14. j z = 56 + 72/; =
3
c; ^ = 2[cos(70 + A:120) + /sin(70 + A:120)], /t = 0; 1; 2.
. ^
+ i sin -
4.15.
a) 1+ / = 1/2 ( cos - ___
\ 4 4
llTT '117C\
, ezrt (1 + /)" = 2 ( cos-----1-/ sin
V
nn nn
p- . / llTT 117C\ ^ ^
0; 1/3 - / = 21 cos---- + / sm----- = 2 cos---- / sin -
V 6 6 J V 6 6/
^ nn
cos / sin
6 6
(l/3-O" = 2"
/
a
ej 1+ cos a + / sin a = 2 cos^ - + 2/ sin - cos - = 2( cos -
2 2 2 V 2
s ebbl addik a bizonytand llts.
a a
cos - . / 5n -
4.16. 0.
240
4.17. a) Zq = |^(cos 45 + / sin 45) = | / 2 e = 1+ /;
Zi = l/2(cos 135 + isin 135) = |/2e ^ = - 1 + /;
22 = ^(cos225 + /sin 225) = ]/2e'^ = - l - i ;
Z3 = |/2(cos315 + /sin315) = |/2e ^ = 1- / .
b) zo = 2(cos 45 + i sin 45) = 2e^ = |/2 + /|/2;
. ^
zi = 2(cos 135 + sin 135) = 2e * = -]/2+i2;
Z2 = 2(cos225 + /sin225) = 2 e ^ = - ^ - i ] / 2 ;
Z3 = 2(c0s315 + sin315) = 2e' = ]/2~i]l2.
c) Zq = 2(cos 60 + / sin 60) = le ^ = 1+ /|/3;
zi = 2(cos 180 + isin 180) = l ^ = - 2;
. ^
Z2 = 2(cos300 + /sin300) = l e ^ = \-i]JZ.
a) Zq = cos - + 1sin - = e - i;
3;t 'in i ^
Zi = cos + 1sin = e = - 1;
ej Zq = 10(cos0 + /s in0) = 10e' = 10;
Zj = 10(cos rt + isinrt) = lOe = -10.
4.18. a) ZoZiZ2 = i; b ) ZqZ^Z2 Z^ = - 1.
6
4.19. Zo = [/2(cos 45 + 1sin 45);
6
Zi = |/2(cos 165 + /sin 165);
6
Z2 = l/2(cos 285+isin 285).
4.20. a) Zo = 0,64+ l,55i; z^ = - 0 , 6 4 - 1,55/;
Z2 = -0,64+ l,55i; Z3 = 0,64- l,55i.
b)zQ = l,31(cos 51,14 + isin 51,14);
zi = l,31(cos 171,14 + isin 171,14);
Z2 = l,31(cos 291,14 + isin 291,14);
Z3 = l,31(cos 68,85 + isin 68,85);
Z4 = l,31(cos 188,85 + / sin 188,85);
Z5 = l,31(cos 308,85 + /sin 308,85).
>- E ( ; in
Jn.
Z4 e , Z5 c) Zo = 2; Zi = 2e ^; Zj = 2e ^; z^ = e Z4 = e''; Zj = ^
16 Matematikai feladatok 241
4.21. aj 0,77 + 0,64/; j 0,04; c) 0,2; d) l; e) 111,3; / j 4,81.
4.22. a) z = - i = cos 270 + / sin 270 = ;
3n
b) z = - i = cos 270+ sin 270 = e \
. , / - 0 , 2 4 2 - 1,37/, , , / - 0 , 7 0 7 + 0,957,
4.23. a; z = < 0,242+1,37/; ^ = <-0:707 + 0;457/. ~
4.24. z - 0,84[cos(ll,25 + A:90) + /sin(ll,25 + A:90)], A: = 0, 1,2, 3.
4.25. z = 2[cos (80 + A: 120) + /sin (80 + A: 120)], k = 0, 1,2.
4.26. a) z = 0,5[cos(60 + A:120) + /sin(60 + A:120)], A: = 0, 1,2,3.
b) z = 2[cos (45 + kn) + / sin (45 + A:72)]. A: = 0, 1, 2, 3, 4.
4.27. z = 2[cos (52,5+ A:90) + /sin (52,5 + A:90)], A: = 0, 1, 2, 3.
4.28. z = cos (80 + k 120) + / sin (80 + k 120), k = 0,1,2.
4.29. z = 2[cos(48 + A:72) + /sin(48 + A:72)], A: = 0, 1, 2, 3, 4.
4.30., z = 2,114e k jF 0, 1, 2, 3.
4.31. a) Z = 3,5 + 2,24/ = 4,24(cos 3153' + /sin 3153');
b) Z = 3, 1+\ , \ i = 3,83(cos 16,7 + /sin 16,7).
4.32. U = 265 cos (300/+ 0,74) volt.
4.33. I = 22 cos (314/- 5,6) amper.
1 1 1 1
4.34. a; Z = ----------- ; b) Z = ------------ + c) Z =
1 1 1 1 /coC 1 .
I------- I------- ------------1- /coC
R icoL R icoL R + icoL
5. fejezet
5.1. Az = 1, 2, 3 termszetes szmokra az llts igaz, mert
1-2 2-3 3-4
1 = -----; 1+ 2 = ------; 1+ 2 + 3 = ------
2 2 2
242
Feltve, hogy n ^ k rtkre igaz az sszefggs:
1+ 2 + 3+ ... +k =
k(k+l )
bizonythat, hogy n = k +l esetn is igaz. Ugyanis az elz egyenlsg mindkt
oldalhoz {k+ l)-et adva, a jobb oldalt talaktva;
k(k+l ) ( k+l ) i k + 2)
1+ 2 + 3+ ... +k + {k+l ) = ^ +(yt+l) = ------Y-----
Lthat, hogy az utols kifejezs ugyanolyan alak, mint a bizonytand llts jobb
oldala, azaz minden pozitv egsz szmra rkldik az sszegkplet helyessge.
5.2. l2 + 2" + 3^+ ... +k^ + {k+l)^ = =
(A:+l)(A: + 2)(2A: + 3) ^
k(k 1^^
5.3. l^ + 2^ + 3^+ ... +k^ + ( k +i y = -------- +(k+l )^ =
' {k+l ) {k + 2 r ^ ^
5.4. 12 + 32 + 52+ ... +(2A:-1)2 + (2A:+1)2 = 1)A:(2A:+I) =
(2A:+l)(A:+l)(2A: + 3) ^
5.5. 1 -2 + 2-3 + 3- 4+ ... +k{k+\ ) + ( k+\ ) {k + 2) =
k{k+\ ) {k + 2) (A:+l)(A: + 2)(A: + 3)
---------------------- l-(A:+1) (k+2) = -------------------------
5.6. 1- 2- 3 + 2 - 3 - 4 + 3 - 4 - 5 + . . . +A:(A:+1) (/t + 2) + (/r+1) (/t + 2) (A: + 3)
A:(A:+l)(A: + 2)(A: + 3)
= ----L +(A:+l)(A: + 2)(/t + 3) =
4
(:+l)(A: + 2)(A: + 3)(A: + 4)
5.7. 1 l! + 2-2! + 3- 3! + ... +A:-/t! + (/r+l)(A:+l)! =
= (A:+1)!-I+(A:+1)(A:+1)! = (A; + 2 ) ! - l .
5.8. 1-4 + 2-7 + 3- 10+ ... +^(3/t+l) + (/t+l)(3/t + 4) =
= k ( k + l f + i k+l ) {3k + 4) = {k+l ){k + 2)\
16* 243
5.9. Az = 1, 2, 3 termszetes szmokra az llts igaz, mert
(1 + 1 ) =1 + 1; ( 1 + 1 ) ( ^ 1 + = 2+1 = 3;
1\
' = 3 + 1 = 4 .
Feltve, hogy n = k rtkre igaz az sszefggs:
(1+1)1 i + f
/
1+ = k +\ .
bizonythat, hogy n = k +\ esetn is igaz. Ugyanis az elz egyenlsg mindkt
oldalt megszorozva (1 + ;-----Vgyel,
V k + \ J
(1 + 1)
n A: + 2
k+\
= k+2.
Az utols kifejezs ugyanolyan alak, mint a bizonytand llts jobb oldala.
2k
1 -
/
1
' - M -
1 -
k + 2
(A:+1)V 2(A:+1)
5.11.
1\ ^
I - -
3
V
k + 2
- M' - i
1
' 1 -
( k+i y
5.12.
I 1 1
--- + ----- + -----+
2 2-3 3-4
k 1
+ -
1
+
1
+
k(k+l ) ( k+l ) {k + 2)
k +l
5.13.
k +l i k+l ) {k + 2) k + 2
1 1 1
+ +
' 2-3 2- 3- 4 3- 4- 5
A:(:+3)
-+ ... +
1
+
1
+
A:(A:+l)(^ + 2) ( ^ + 1) (^ + 2) (A: + 3)
1 {k+\ ) {k + A)
4{k+\ ){k + 2) {k+\ ){k + 2){k + l>) 4(^ + 2)(^+3)
244
5.14.
1
+ + - + + -
2 - 3 - 4 3- 4- 5 4 - 5 - 6 ' (A:+l)(A: + 2)(A: + 3)
k +\ k{k+\ ) k +\
+
+ +
(A:+ 2) (A:+ 3) (A:+ 4) 4(A: + 2) (A: + 3) (A: + 2) (A; + 3) (A: + 4)
(A:+l)(A: + 2)
4(A: + 3) (A: + 4)'
2^ 3^
5.15. ^ ... +
1-3 3-5 5-7
+
+
i k + \ f
{ 2k- \ ) { 2k+\ ) (2A+l)(2A: + 3)
(A:+l)(^ + 2) k{k+ 1)
2(2A:+1) (2A:+l)(2A: + 3) 2(2A;+3)
5.16.
1 1 1
+ -----+ ------ + ... +
1
1-3 3-5 5-7
k 1
+
+
1
(2A:-1)(2A:+1) (2A:+1) (2A: + 3)
k +\
5.17.
2A:+1 (2A:-1)(2A:+1) 2A: + 3
1 1 1 1
+ -----+ ------- +
1-4 4-7 7-10
k 1
+
+ -
1
(3A:-2)(3A:+1) (3A:+1) (3A: + 4)
k +\
5.18.
3A:+1 (3A:+l)(3A: + 4) 3A: + 4 '
2-21 + 3-22 + 4- 23+ ... +A:-2' i + (A:+l)2' = (Ar-1)2*' + (A:+1)2'
= k-2' ^^K
5.19.
_ 1 1 1 3 1
~ 2 6 4 T 3^'
1
3
5
I I
n
7A
1
13
5.21.
5. i . Z . M . A .
2 12 5 10 2 r '
5.22. <
3 1 13 21 31 43
13 ^ I s 16 17 8
5.23.
245
, 1 5 13 3,
^ , 11 29 45 79 103 153
5.26.
, 3 7 15 31 63,
5.27. ^1; 2 4 ' J ' 32^'
, 3 11 25 137 147
5.28. 0 ; -
2 6 12 60 60
5.29. {- 1; 0; 1; 0; - 1 ; 0; 1}.
| / 2l / 2 |/2 1/2 1 /2 1 /2 - 1/2
5.30.
5.31.
2 2 2 2 2 2 2
1^ 3 31/3 9 91/3 27 21 ^ 81
2 4 8 16 32 64 128 256
3 3^ 3^" 3^
3 7 3 8
5.33.
22 2* 2 2 2^ 2^^ 2^^^ 2^
1 2 3 4 5 6 7 8
2 3 4 5 6 7 8 9
1 2 3 6 8
14 7 10 13 16 19 22 25
5.36. ^ 1; 1; 1; 1; 1; 2; 1; ^ l .
, 2 3 4 5 6 7 8 9
5.37. vS 2 4 5 6 7 8 9 ^^10
246
5.38. {1; 6; 18; 36; 54; 64,8; 64,8; 55,543; 41,657; 27,771}.
5.39. A sorozatot megad kplet a = ^ ^ . A sorozat szigoran nveked,
mert
3 + 2 _ 3 n - l ____
+l n (+l )
mivel n pozitv egsz. 2 = i <a2 < ..<a<3, teht a sorozat korltos.
Egy torldsi pontja van. Konvergens; a->3, ha n^oo.
5.40. a = - ; {a} szigoran cskken; korltos; ->0; n = 1, 2,... .
5.41. a = \ {a} szigoran cskken; korltos; a^0; = 1,2,
5.42. a = {a} szigoran nveked; nem korltos; a-> + oo; n = 1,2,....
n
5.43. a = fn\ {} szigoran cskken; korltos; a ^l ; = 1, 2,... .
5.44. a = ( 1)"^^; = 1, 2,... ; kt torldsi pontja van; nem konvergens.
5.45. a = 5 8; n = 1,2,... ; {a} szigoran cskken; nem korltos;
a^- co.
5.46. a = 1+ ^ ; = 1, 2,... ; {a} szigoran cskken; korltos; ->1.
1; = 0, 2, 4, . . . ,
(
I , - ) 5,...,
[ n+1
5.47. =
{a} nem monoton; korltos; kt torldsi pontja van.
5.48. { 1; 1; 1; 1;...}, a rekurzv formulval megadott {a} sorozat llan
d elem, teht konvergens, s lim a = l.
QO
5.49. {l ; 3; l l ; 37; 129; 443; . . . }; a^ + ^ .
5.50. {2; 3; 1 ; - 2 ; - 3 ; - 1; 2; 3;...}; {a} nem monoton; korltos; nem
konvergens.
247
5.51. ^ ^ ^ szigoran cskken; korltos; a^\.
{ 3 7 13 27 )
5.53. {1; 1; 2; 3; 5; 8; 13;...}; {a} szigoran nveked; nem korltos; -> + oo.
5.54. {4; 3,5; 2,875; 2,210; 1,648;...}.
A sorozat korltos (teljes indukcival igazolhat), monoton cskken. Az {a} soro
zat egyik als korltja 1, egyik fels korltja 5, teht konvergens, azaz elg nagy -tl
h^ + 5
hm a = lim a+i =h, gy h = -------, s a 6^+ 5 = 0 egyenlet gykei; /i = 5,
n-oc n-^cc 6
/?2= 1. A kt gyk kzl csak h= 1 lehet a hatrrtk, mivel az egyik fels korlt 5
s {a} monoton cskken.
5 41 2306
5.55. ( 1; 2; - ; ;
4 2 5 1681
vergens.
; . . . | ; {a} szigoran cskken; korltos; {a} kon-
5.56. {1; 0; - 1; 0; - 1; 0;...}; {a} nem konvergens.
f 1 1 3 5 11 1
6.57. torlatos; . . - 0 .
5.58. lim = 1.
n00
5.59. Belthat, hogy az a+1 = - ( a+ ) sorozat elg nagy -tl kezdve mono-
2 \ aJ
tn cskken. Az (n+I)-edik elemet megad kpletbl 2a +ia = al +a addik,
amibl viszont 1 = al 2a+ia + al^^ = ( + i n)^ ^ 0 kvetkezik. Teht
| a+il ^ l/a, ami azt jelenti, van olyan index, amelytl kezdve a sorozat elemeinek
als korltja fa. Legyen olyan rszsorozat, amelyben az elemek indexre k >N
fennll. Mivel al' ^a, gy----- = - H------ - ^ 1. A sorozat teht monoton cskken,
jt 2 2a,fc
alulrl korltos, gy konvergens is. Ebbl kvetkezik, hogy lim = lim a = h,
n-oo n-oo
vagyis Ebbl A= |/. Mivel a sorozat pozitv tag, ezrt
lim = Irt.
248
5.60. lim a = fa.
n-*00
5.61. lim a = 4.
5.62. lim a - 3.
n00
5.63. lim = 1.
n00
5.64. lim a = 2.
00
5.65. lim = 3.
00
5.66. lim = 6+1.
00
5.67. min a = a^, = = 120.
5.68. min a = Os = 10.
5.69. min a = 0 3 = = - 20.
5.70. min a = = 3 + i f = 6,464.
76
5.71. max a ^ = a-j = = 163,4.
6!
5.72. maxa = = 2-2l/6 = -2,89.
5.73. max a = ag = = 110.
5.74. max = 3 = 4.
5.75. - - < a < 1 ; nincs max a, min a elem.
-5]/2
5.76. 5^a< 1; nincs max a; min a = j = -0,1473.
^ D
(
27tai\
c o s ~ 1, 2, 3 , ... sorozat nhny eleme:
249
2 22 ^ ^ 1
A legkisebb elem ~ 2 ^ legnagyobb elem 1, amelyet az j = g = ... = 03^ elemek
esetn vesz fel, A: = 1, 2, 3,
5.78. - l ^ a ^ l ; max a = 03 + 121 = 1; A: = 0, 1, 2,...;
min a = 09 + 12, = - 1; / = 0, 1, 2, . . . .
5.79. {} szigoran cskken,
n+1 n - 2 n ^ - 2 n +\
< 0, mer. p . _ 4 + i , <
Korltos, mert a < 3 s > 0, teht az elbbiek alapjn konvergens s lim a = 0.
+ l
Az =10 ^ esetn - 0
22- l
tagtl kezdve a sorozat elemei
szmot.
< , ha n>\ , vagyis az = v(e) = 51 index
100
0^-nl nagyobb pontossgnl megkzeltik a 0
5.80. lim a = 1; n = v(e) = 70.
5.81. lim a = - ; = v(e) = 8.
n-*QO 4
5.82. lim = 1; n = v(e) = 25.
n-^co
5.83. lim a = - ; n = v(e) = 80.
n-^cc 5
5.84. lim = 0; n = v^e) = 25.
5.86. lim a = 0; n = v(e) = 13.
1
5.87. lim a = - ; = v(e) = 12.
n-QO 4
5.88. lim = 0; = v(e) = 436.
->oo
250
10" 10^'^ 10" ^ 10^
5.89. lim = lim-------------------- < lim------- ) = 0 ,
-oo n\ w-oo 1 0 ! 11 * 1 2 . ..fz -*oo 1 0 ! \ 1 1 /
JQlO /1A\10
-
mert lim = lland s lim
w-oo 10! n-^cc
5.90. 0.
5.91. 1.
5.92. 9.
5.93. 0, ha 0 <c <l ; ha c =l ; 0, ha c>l .
5.94. 00, divergens.
5.95. 1.
(1 + 5)(1 + 52)(1 + 53)...(1 + 5") . 1+ ... + 5'
, ^ ;l + 2+... + n
5.96. lim " ''X, " ^ = lim
->oo
n(n+ 1)
5 2n c2n
- > G 0 3
1+ ... + 5 2 / 1 ^3\
= lim-------- ------- = lim I + ... + 5 ) = 00, mert lim 5 = oo.
5 ->0O W-+00
5.97. 0.
1 2 1
5.98. 7 , ha c < 6; ---- , hac = ft; - , hac>A:
b b + c c
100
3
5.100.
2
5.101. 0.
5.102. - 00, divergens.
5.103. 00, divergens.
251
5.104.
/3
5.105. - j .
5.106. 00, divergens.
5.107. +1.
3^______ 3
5.108. a = |/ + 1fn;n = 1, 2, ... Szorozzuk meg, s osszuk is el a jobb oldali
3 3
kifejezst ([/(+ 1)^ + 1/(+ 1)+ |/^)-nal. talakts utn kapjuk, hogy
1
a =
" 3
; lim a = 0.
1/(m+ 1)2+!/(+ l)+|//^
5.109. 0 0 , divergens.
5.110. 9.
5.111
5.112. 3'.
5.113. *
61'
5.114. 1.
5.115. a =
1+ 2 + 3 + ... + n{n+\)
2n^
1
3
CO,
1
2
ha
ha
0,
ha
k = 2
i;
5.117.00, ha A: = 0, 1, 2, 3; - , ha k = 4; 0, ha k>4.
4
252
5.118.00, ha /: = 0, 1,2; ha ^ = 3; 0, ha k>7>.
5.119.00, ha A: = 0, 1, 2, 3; 1, ha k = A; 0, ha k>A.
5.120. 0 0 , ha A: = 0, 1, 2, 3, 4, 5; 1, ha A; = 6; 0, ha k>6.
5.121. 00, ha A: = 0, 1, 2; ha k = 3; 0, ha k>3.
5.122.00, ha A: = 0, 1, 2; 1, ha k=3; 0, ha k>3.
5.123.00, ha k = 0; 1, ha k =l ; 0, ha k>\ .
1 1 1
1+ - + - +. . . +
4
2 4 2"
5.124. a = ----- j-----j------------ ;n = 1, 2, 3,... . Alkalmazzuk a szmllban
1+ HH... + ---- -
3 9 3 i
s a nevezben a mrtani sor sszegkplett:
/ I V 1
( i ) - 5 - 4 ,
" i T v ------ 3
2 - ()-*
5.125. 1.
5.126. 0.
5.128.
2
5.129. 0 0 , divergens.
5.130.
3
5.131. Ha lim ([b-\) = a ltezik, akkor {a} = [b-\ konvergens. Ha b>\ ,
/7->00
253
akkor [/^ = 1+ rhat, ahol x > 0. A Bernoulli-egyenltlensget alkalmazva:
ft = ( f e " = ( l + x j " ^ l + x,
b - \ V b - \
teht x ^ . De a = p - 1 = 1+x - 1 = 0 < ^ -------- >0, azaz
n
a^0, vagyis |/ft 1, ha ^oo.
5.132. lim a = l.
n-^oc
5.133. lim a = e.
->oo
5.134. lim a = e.
->G0
5.135. lim a = In a.
/I-^OO
5.'136. lim = fbc.
#1
5.137. lim a =
n-*oD
5.138. 1.
5.139. 1.
5.140. y .
5.141. Az a = /i^([/+ 1-2|/n + |/- 1), = 1, 2, 3,... kifejezst szorozzuk meg
s osszuk el a (|/n+ 1+ 2|/ + |/w- 1) sszeggel, az talaktsokat hajtsuk vgre:
= ----- 7 = -----7 = ---- - F ---- = 2
1^+T+|/PT+21^ 1/+T+|^i^ + 2 ^ ^
majd szorozzuk meg s osszuk el a legutbbi kifejezst a - 1+ |/n) sszeggel:
^ __________ n h - l ) ___________^
i f i + l + ^ n ^ + 2 f n ) i ^ n ^ - \ + f i )
254
fn n 1
= 2 , .......;----- ----- pr- = - = = --- p=- > ---- , n->00.
(l/+l + + 2l/) + 1/ 4
f i
5.142.
5.143. - 1 .
5.144. -10.
5.145. 2][l.
5.146.
4
5.147. - - .
2
5.148. 0.
5.149. 0 0 , divergens.
5.150. -
5.151. 0.
5.152. - 0 0 , divergens.
5.153. - 2.
5.154. lim sin [n {Un^ + n + \ - W + 2n)] = lim sin , , .....=
;j^oo -*oo 1)/4w +2
. ( - 1) . ^
5.155. 1.
5.156. 1.
5.1 S7. 4 ( l / 2 - l ) .
255
5.158.
l^+j/5
8
5.159. l/6.
5.160. 0.
5.161. 00, divergens.
5.162. ^2.
/2/1 + 3\"~ ^
5.163. Hm ( - ---- - = lim / 1 + -
- oo\ 2 - 2 7 - oo I 2n- 2
2 n - 2
1 ' *
1+
1
- 4
2n- 2
5.164.
5.165. e
5.166. 0.
5.167. 00, divergens.
/n" + 2 V ' '*
5.168. lim
-*oo \^ + 3/
= lim
*00
1+
= e - l
n^ + 3
^T
2 + 3
-1
1+
n^ + 3
- 1
5.169.
5.170. 0.
5.171.
5.172. 00, divergens.
5.173. { aj konvergens, ha A^l ; ha A= 1; a-^0, ha /l< 1.
------ A- / i ) = 0, hozzuk kzs nevezre, kapjuk:
+1 /
256
Hm
//-+0O
= 0 akkor s csak akkor, ha 1/I = 0 s
+1
X- f i = 0, ebbl A= 1 s ^ = 1.
5.175. ha X = 4.
5.176. a^l , ha l<b.
5.177. {a} konvergens minden 1 esetben; ha A= 0; a->0,
ha X = 2,
5.178.
Hm
n-^oo _V2+i ;
= Hm
n^cc
1+
n^+ l
1 ^
n^+l
- 2
2 2 ^ '
n^+l
lim
n y oo
2]/n + n^+l
n'*' + fn + n^'^ / n ^ - f n l
5.179. 1.
5.180. - +1.
5.181. fe + 2.
5.182.
5.183. Hm
n-yco
1
+
. 1-2 2-3
jn rhat zrt alakban.
n(n+l )j
= Hm---- - = 1 kifejezs 5.12. alap-
n-yoon~^ 1
5.184.
2
17 Matematikai feladatok
257
5.185-7
4
77
5.186.
240
5.187. 00, divergens.
5.188. | / 2 - | / 3 .
5.189. Ig^.
5.190. 00, divergens.
5.191.
4
5.192. 2.
5.193. 0.
5.194. {a} konvergens, ha { x e R | x < - l s
5.195. {!} konvergens, ha {xeR| x ^ 1^1 < |/7;)/TI ^ x}.
5.196. {a} konvergens, ha { x e R | x ^ 0; 1 ^ x}.
, 1- / 17 i + yn-)
5.197. {a} konvergens, ha <xe R ^ - l ; 2 < x ^ - >
5.198.
5.199.
5.200. a2_i ^ ^ s 2= 1, {a} nem konvergens.
258
6.1. x g R\ { 1 , 3 } .
6.2. x g R \ { - 1 ; 0 ; 1}.
6.3. xe [ - 3; 3] ,
6.4. xe[ 0; 1[.
6.5. 0.
6.6. x e R \ { 0 } .
6.7. 1[.
6.8. X = 2kn, k e E.
6.9. XGR^.
6. fejezet.
5l/l + 2n + Skn
6.10. x = ---- ^---- r--------- , k = 0 , 1, 2, . . . .
6.11. x g R \ | x g R | x = ( 2f c+l ) ^+ 1 s k e Z
6.12. X G R \ {x G R| x = 2A:7z: s k eZ} .
6.13. Pros.
6.14. Pros.
6.15. Pratlan.
6.16. Se nem pros, se nem pratlan.
6.17. Pratlan.
6.18. Pros.
6.19. Pros.
17* 259
6.20. Pros.
6.21. Pratlan.
6.22. Periodikus, p = ;r.
6.23. Per i odi kus, =
6.24. Nem periodikus.
6.25. Periodikus,/>= 1.
6.26. Periodikus,/? = 2.
6.27. Periodikus,/? = 7t.
6.28. Nem periodikus.
6.29. Periodikus,/) = 27t.
3 - 7 j \ y ^ r 9
6.30. a) x = ; b ) x = ; c) x = logj ]^+ - ;
2y [' 3 4
3
d) X -
_ X 5 _
6.31. a) f{x) nem ltezik; b) f i x) = ---------2; c) f{x) nem ltezik;
In 9
1/ x + 1 _ _ f 1- |/x, ha X^ 0
d) Rx) = I ----- ; e) f (x) nem ltezik;/; f (x) = < ;
f X 1X, h a x < 0
1
g) f (x) = -Arcsinx.
y 2
6. 32. a) ^ ^ = - ~ ] , a = 0,b = 2 , A = l , B = - 3 ;
\ 1
- 3
260
b)
y+1 x - 2 \
- 1 V I
= 2 , b =
1 -
- h A = l , B = - l ;
1
1
1
0
Jo* 1*
'
/ f ' \
j y*=('A*/\
/ 1 \
/ ' \
/ jy=-x^+4;f-5\
261
c)
d)
e)
2
3
j , ^ = 0, . = ^ , ^ = l , 5 = - y ,
y - 0 ' / x - 2
- - 0\ -1
3 V I /
y + 2 _ / x - 3 \ ^
6 V - 1 y
, a = 2,b = 0, A=l , B=3;
, a=2>, b=- 2, A=- \ , B=6.
j - 0 f x 2\^
6.33. a; - 7- = ( ^1 ,z=2, 6= 0, - l , f i = l ;
1
7+1
- 1
' x - 2
, a = 2, b= - \ , A = l , B=2.
6.34. a) Xi = ~ +2kn, k eZ; X2 = -----\-2ln, l e Z;
71 ^ 5ti
b) X = - + k%, k e Z-, c) x = -----^2kn, k e Z ;
3 4
d) X e R;
e) xi X 16F34' + k 180, JC2 17032' + / 180, k, leZ.
1+ cos 2x sin^ X+ cos^ x + cos^ x - sin^ x
6.35. a)
2 2
b) Kvesse az alz bizonyts gondolatmenett!
= COS^ X.
c) cos X =
/ ^ I 2 ^ 2 ^
COS 2 " c o s ---- sm -
2 2 2
cos^ - - sin^ -
2 ^
2
1
X X 2 ^ , - 2 ^ '
cos^ - + sin^ - cos^ - + sin^ -
7 7 2 2
cos^
l + tg " r
d) Kvesse az elz bizonyts gondolatmenett!
262
6.36. a)
1
4
1
x + \ \
= cos I p 1, a
2
, , 1 1 1
= - 1 , 6 = , A = , = - ;
4 2 4
>^+1
= sin
^2
x - n
_ 1
2
j - 3 ( x- Qh
c ; r = - r
, <3= 0, Z) = 3, v4= l, 5 = -
y - \ f x+l ' ^
2
, a= - 1, 6= 1, - 2 , B = - .
y - l / x - l \ 1 1
6.37. a j ----^ = cos , a= l, b= - U A B = - - ;
y - l
b) = cos
2
( x + 2 \ 1 1
----- y a = - 2 , b=l , A = - - , B = - .
263
c) X
6.38. a) x ^ ^ \ - A x ^ \
X
e) X
] j \ - x^
4
x^ + 4
2 x - l
d) x
b) x ^ ]/ l -4x^;
|/4x2 - 4 x + 2
6.39. a) sin Arcsin x = x, sin ( - Arccos x ) =
= I sin - Icos Arccos x -
;r^
cos -
2/
sin Arccos x = x;
b) trendezs utn vegye mindkt oldal szinuszt!
c) tg arctg X = X, t g ---- arectg x = ctg arcctg x = x;
\2 /
d) Vegye mindkt oldal koszinuszt!
e) Vegye mindkt oldal kotangenst!
n 2n n
6.40. a ) - ; b ) j - c ) - ; d) 1,5.
3 ]/2
6.41. a) X = \ l%\ X = - ; c) Nincs megolds! d) x = .
y n
6.42. a) ~ = Arcsin
f x - \ \
_ 1
+ -
^ 2 x + l \ n 1
b) = Arccos y j , a = - l , b = - ^ = - 3 , 5 = ^
y -
c)
- 2
= Arctg
f x ^ 2 \
1
V 2 7
n
d)
2n
J -
= Arcctg
2n 1
, a = 0, b = - A 5=2.
V
264
d) x = A, 7=3; e) Xi = 16, j i = -28, ^2= 1, ;^2 = 2.
6.44. a ) y - \ = 2~ ^ , a = d=l , ^=1-
y l 1
= a = 2, b ^ l , A = - 2 , B = - .
y +l
c) ~ Y ^
X - 1 \
V 27
y+ 1
^ = lg
265
c) X
x+1
^x^ + 2x + 2
; d) X 2x]jx^ 1.
y + l x - l \ 1
6.46. ^ = sh ^ j , a= 1, - 1, - 2 , 5 = - ;
266
b)
y + 2 ' x - 1
- 2
= eh , a = l, b= - 2 , A = - 2 , 5=3;
v+1 x - 2 \ 1
c ; ^ = t h j , a = 2, Z>=-1, A = 2, B =
1 3^
d) ^ = cth
e)
1
2
y - l
= arsh
V - 2 7
, a = - l , 6=1, A ^ - , B=2;
y ~ 2 j
* = - 2 , ^ = - 2 , 5 =
v + 2
f ) j = arch
3
y - \ x + \ \
= a r t h j , a = - \ , b=\ , A=~3 , 5=2;
, a= - 1, b = 0, A = ^ , B = ^ .
) Z L
y
T '
= a r c t h
3
2
V ^ )
267
2 / X \ 1 1 1
6.47. aj j = eh / j -U a = 0, 6 = - - , A= - - , B = ~ ;
3
4
^ x - l \
\ ^ J
, a =l , =- l ,
6.48. a) 0 <\ x - l \ < - , ^ ^ 2 ^ ^ l-x+ll <e . S = e;
c) esetn \f{x)~ A\ =
X
- - 1
X
268
6, 49,. ; ^ = 2, ,5 = 4 ^ ; =
1306 4
6.50. a) ct(e) = ; b) (o{s) =
l g3'
6.51. a) x = 0-nl hzagpont (megszntethet szakads),
x= 1-nl pluspont (nem megszntethet szakads), ltezik hatrrtk;
b) x= 1-nl hzagpont (megszntethet szakads),
X = 2-nl pluspont (nem megszntethet szakads),
nem ltezik hatrrtk.
6.52. a) x = 0-nl nem megszntethet szakads,
x=3-nl megszntethet szakads;
b) x = 0-nl ugrs (nem megszntethet szakads);
c) x= nn-nh\ nem megszntethet szakads (n e N).
6.53. lim f{x) = - 1, lim /(x) = 1.
6.54. lim f { x ) = - 00, lim f {x ) = oo.
x^0~ x->0+
6.55. lim f { x ) = GO, lim f (x ) = - go.
6.56. lim f i x ) = 0, lim f(x ) = - 2 .
x^5~ x^5 +
6.57. lim f i x ) = 1, lim f i x ) = 1.
x->0-
6.58. lim f i x ) = 0, lim f i x ) nem ltezik.
x-*0~ x^0+
6.59. l i m / ( x ) = lim/(x) = ^ .
x - 0~ 2 a:^0+ 2
6.60. lim f i x ) = 0, lim f i x ) = oo.
x-l~ x-* +
6.61. -
6.62. 0.
6.63. 00.
269
6.64. 00.
6.65. 00.
6. 66. 0.
6.67. 00.
6.68. 0.
6.69. 0.
6.70. - - .
6.71. 00.
6.72. 100.
6.73. 0.
6.74. - i .
6.75. 6.
6.76. n.
6.77. 0.
6.78. -32.
6.79. 6.
6.80. T
6.81. - 1.
6.82. 00.
6.83. ]-.
2
270
6.84. 0.
6.85. - .
4
6.86. - 8.
6.87. 0.
6.90. - .
10
6.91. 4.
3
6.92. .
14
6.93. - .
6
6.94. 3.
6 . 9 6 . ^ .
6.96. ^
2
6.97. 3.
6.98. 00.
6.99. 0.
6.100. 1.
271
6.101.-^.
f i
a+b
6.102.-
2
6.103. 0.
6.104.
6.105.
6.106.
6.107.
6.108. 2.
6.109. 1.
hL
6.110. e".
1
6.111.
2
6.112. 00.
6.113. 0.
6.114. 0.
6.115. 2.
6.116. 1.
272
6.117.
6.118.
273
6.120
/
/
6.121.
274
6.123. 10.
6.124.
4
6.125. 1.
6.126. 0.
7
6.127.
4
6.128. - 9 .
6.129.
6.130.-.
2
6.131. 1.
6.132. - 2 .
6.133. 0.
9
6.134.-.
2
6.135. 2.
6.136.^.
6.137. l.
6.138. 1.
6.139. ( - 1 )- " - .
n
6.122. 2.
18* 275
]!x + h-][x ]jx + h-][x ]/x + h+][x
7.1. a) l i m-------------= l i m......................... -j=
h h^o h VX + h+vx
, {x + h) - X ,, 1 1
= hm p= = hm ^---- = = .
><^0h{p + h+pc) '--0 |/x + /! + |/5c 21/x
1 1
7. fejezet
2 + /! 2 2 - 2 - / ! - 1 1
b) hm----------- = hm----------- = hm---------- = -----
h^o h h^oh2{2 + h) h^o2{2 + h) 4
- 2 1
7.2./'(x) = + ^ .
7.3./ ' ( x ) = - 5x^ + 8x^ + 2x- 2.
4x^ + 4x + 3
7.5. / ' (x) = sin 2x 2x cos (x^).
7.6. f \ x ) = 2x arctg |/x +
2 ^ { \ + x)'
7.7. /X-^) = s ""(cos 2x - X cos 2 x - 2x sin 2x).
sin l/x
7.8./ ( x ) =
[/x cos^ j/x
7. 9. f ( x) =
arsh ( + 3)
7 A0 . f ( x ) =
7.11./'W =
^x^+12x + 40
l - x ^
x^ + 3x^+1
1
/ x ^ - l
276
7. 1 2 . / ( x ) =
7.13./ ' ( X) =
2|/x|/x 1arch ][x
1 - 2 sin 4x(a: ^ - 1 ) - 2x cos^ 2x
cos^ 2x ( x ^ - i y
ch^
x^- 1
7.14. /'(x) = 3 cth 3jc - 2 ctg x.
e~^il-2x)
7.16./'(x) = - 2x.
7.17. /'(x) = - 3tcx^ sin (2x^).
7.18./'(X) = sh^------------ ^ -------
7.19./'W =
7.20. f i x ) =
X X"
1 3*[(ln3)shx-2chx]
, 3" sh^x
cos^-T2
Xsin (1 x) + 3(x^+ 1) cos (1 x)
^x^+l si n^( l - x)
2 ^ 3
p 3 ^ X+ 2 | / ? sin X
7.21. f (x) = 2 *'" (In 2)
cos-* X
7.23./ ' ( X) =
7.24./'(X) =
XIn x^
- e"*sin (2 arctg
\+e^^
3(sh 3x) sin x 2(ch 3x) cos x
In 2(ch 3x) sin x
277
2 - - ^
-X ^shx+| / ^chx
7.25. f \ x ) = -------------------- .
f ? (sh x) In 2
2 sin 4x + (sin^ 2x) (In 2)x
7.26. f i x ) =
2^
\ nx + 2
7.27. f i x ) = -------- p ---------^ .
2(ln2)^G l+e^^
7.28. y\ x) = - 2 tg3 /.
7.29. / ( . V ) = ^ t g .
7.30. / ( . V ) = -(In 10)sin2 2/.
cos + sin
7.31. v'(x) =
cos / sin
3
7.32. / ( x ) = -(1 +COS t)^.
t ^+l
7.33. y'ix) = .
2 In t
7.34. r'(x) =

7.35. y'ix) =
- j/T- 1*
2 0 ^ ^
4
7.36./(.v) = -2{t.
7.37. y'ix) = ^ .
( s in / ) ' cos^ t
7.38. / ( . V) = ^ ^ --------------
' jtg r
278
7.39. y'{x)
7.40. y'{x)
7.41./ ( X )
7.42. y\ x)
7A3. j'W
7.44. / ( x )
7.45. j'W
7.46./(X)
7.47./'(X)
7.48./'W
7.49./'(X)
7.50./'(x)
7.51./'(x)
2 sin t
2* '(cos^ Oln 2
2(lg /)cos^ /
(In 10)e'
(In 3) ft 3'' In t
4
In 100
sh2/
( t - W
eh -----
V^+1
(/ + 1)^ (eh t) sh t
1
It arth t + 1
27
V 2 2>
i 1
(2x)" ( l - l n 2x).
1 - 1
eos^ -
1
X
In |/+ t g -
x j
1
279
7.52. f' {x) = ^ I n X-(cos x) \ n x + p . -
sin X
X
7.53./'(x) =
J
1
\
In - + 1
X /
(In x) In In X+ 2
7.54./'(x) = (Inx) ^
2|/x In X
7.55. f' (x) = (cos x)" ^ [cos x(ln cos x) - (tg x) sin x].
7.56. f i x ) =
7.57./ ( x ) =
/ 1 + x
,\ /
x j
In
1 +x
\
l - x
1-X 1-X2
(1+X)2
1- l n
1 + x
x^(ln x+ 1 )
7.58./'(X) = -(arctg x)^
( 1 + x^) arctg x_\
cos x 2 XV
7 . 5 9 . / = ^
7.60. / =
7.61. / =
y
x( l - 2xy)
( l +j 2) /
1-2xj^-2xj
3
3x^;; cos^j(j^ - In 4 4"^ In >>)
' 4^ cos^j +j - 2j ^ x^ cos^ j
^ 2x;;^3*^ln 3 2xy In y
7.63. y 2 tx2 I 2 ~'
_)^cos>' 2j 3 +x
7.64. /
1 + x^y^ + (In 2 ) (xy - y^)
1 + x^y^ - (In 2 ) (x^ - xy)
280
7.65. - = ^ { y + x / ) - e y - \ y ' - l ) = 0.
n - { x y Y
x^ + y^ + (lnl 0).y (x - * In 4
7.66. > - --------------------------------------- ^-----
x^ + y^ + (In 10)x (x - "In 4
-7C-7 ^ ~ 2 y f T ^ n { x y )
7.67. y\ x) = ;----- -------------- .
2x1/1 - y sin {xy)~ 1
y]/r+2y
7.68. / =
X
y^ cos^ I- xl/l + 2y
y sin l/x
7.69. / ------------- =!----------------.
COS [/x - X cth y
y^ sin^ x - > - ( l n cos \)y^ (cos 1)^ sin^ x
7.70. y' = ------------
l . l ^ . y '
1.12./
Po - -2-
Ctg X + ly^
3
7.73. j = - ^x + 4^^.
7.74. j = - 2 x + 5.
7.75. j = 30, 7 =| / 3 x+ ^ .
7.76. m = - 2 .
7.77. j = - 2 x + l .
7.78. >' = (l n2)x+l .
281
7.79. y = nx (n+ 1).
7.80. y = ~(ln3)x.
7.81. y = - 2 x +l .
1.82. y = x; y =~ x .
7.83. -2 ) , ^ ^ ( 1 ; - 2 ) ,
7.84. = x + 2 - n ; prhuzamos.
7.85. = - X - ^ = 90.
^ 2 2
7.86. (pi = (p2 = 9Q\
7.88. a) r =
5J/
c) rx4,3S, '0,23, O
51/10 V 3
'49 76\
= ( - 4 ; - ) ; >;r = /8, f = ^ , 0( - 2; 3) ;
16 16
o ; - ;
7 . 8 9 . / ' 0 = O, / - 0 > O .
7.90. a) Xi = minimumhely; ^2 = 0, Xa = - 2 a lehetsges inflexihelyek.
b) X = - maximumhely,/"(x')^O, vagyis inflexis pont nem lehet.
7.92. 2.
282
7.93. ^.
7.94.
1
7.95.
7.96. ]l2.
7.97. 2.
2
7.98. - .
7.99. 2.
7.101. 1.
7.102. 1.
7.103. 0.
7.104. 3.
7.105. 1.
7.106. 0.
7.107. e - l .
7.108. - 2 .
4
7.109.
In 2
7.110. 0.
283
7.111. 2.
7 .112. 2.
1
7.113.
e
7.114. 0.
7.115. 0.
7.116. 0.
7.117.
4 In 3
7.118. 0.
119. 00.
120. 0.
121. 0.
122. - 00.
123. 00.
1
7.125. 0.
1
7.126.
1
7.127.
e
7.128. e~k
7.129. e.
284
7.130. Zrushelyek: P^{0; 0), ^2(2; 0), lim f (x) = - oo, lim/(x)= oo,
- GO
3
2
- ; 2
3
]2;o)[
/(-V)
max.
/2
/ :
\'
32
mm.
f ( 2 ) = 0
7.131.
285
7.132.
7.133.
7.134.
286
7.135.
7.136.
4- 4x
-1
3 5
7.137.
287
7.138.
7.139.
7.140.
288
7.141.
7.142.
19 Matematikai feladatok
289
7.143.
7.144.
7.145.
290
7.146.
7.147. Pratlan fggvny, zrushelyek: /*i(1; 0), P j l 0)
lim f {x) = 00, Hm f ( x) = 0.
,v-*QO .v-0*
- oo; - -
e
- - ; 0
e
1
0; -
e
1
00
fix)
fix)
max. mm.
X ]-a^;0[ ]0;^[
f ix)
- 4-
fix)
n U
1 9 *
291
7.148.
7.149.
7.150.
292
7.151
7.152.
x-^e
i 1
2
7.153.
293
7.154.
7.155. lim /(x) = - 00, lim f(x) = - oo. Hm f (x) = oc, lim f(x) = 0.
x-*-ao x-*0~ jc0"^ x-oc
X oo; 1 -1
l -i ;0 (
10;a,( .V 1-00 ;0[ J0;oo[
f (x)
+
0 - - f ( x)
- +
fix) T
max.
i i
fix)
n u
294
7.156.
1 '
X
1
i 1
1 & 1 4
0 2 - e 2 2 * n
X
7.157.
7.158.
295
9(Z
L9LZ.
09 LZ.
6SL7.
8 4
7.162.6 s - 2; - s-
3 3
7.163. r =
1000 1000
3n
(=10,3) cm,
a 000
3n
( = 3433,55) cm^
7.164. A maximlis trfogat hasb alaple 6 dm, magassga 3 dm,
K_=108 dm^
7.165. = , a tglatest kocka.
7.166. w = ^ , r = -a.
)/3 y 3
7.167. A = = 1^2
3
8 a
7.168. a = b ------m 0,72 m, r = - .
8 + tt 2
7.169. A ngyzet; a = b =
7.170. A maximlis terlet trapz egyenl szr trapz, rvidebbik prhuzamos
oldala 4 cm, magassga (/2 cm.
m
7.171. M = - .
7.172. m = 8 egysg. Terlet = 40 terletegysg.
x + z y + 2x 3
7.173. = ~ ^ y = J +ay, mivel z = x + y s a = 2y + x.
T'iy) = a - 3 j .
T'(y) = 0, ha Ekkor x = ^ .
a
T'iy) = 3 < 0, teht a trapz terlete x y = - -nl maximlis.
^max . Ez a hromszg terletnek kb. 66,7%-a.
6
297
40
7.174. a= 12cm, m = cm.
i f i 4
7.175. r = " = 3^-
7.176. a = 271
7.177. sin 31 s 0,5151.
7.178. K1060cm\
7.179. JrKO,l cm.
7.180. Am S 16 Aa.
8. fejezet
\ \ k
8.1. X - - { \ - x f + { 2 x - \ f - - ( 3 + x ) - ^ + c.
1 !--------- T 10 / / x - l \ ^
8.2. X H. - [/^+ - l/(l + 2x f + - + c.
3 44
8.3. X^ 2 ) / x - 2 l / 2 l / x - 6 ^ ( 4 - x ) ^ - - |/ (2 x -1)^+c.
8.4. X ------ p= + - ^ + 1 0 V x + -x^ + 4x------- 31n|x| + c.
5 ^ ^ 2 X
2x j ^2-* ^:cln2
8.5. XH.---- +2e2* + 2e-*+ -e3^ + 3e*+x----------+ -------+c.
In 2 3 In a In 2 .
X 1
8.6. X!-3 sin x + 4 cos - + ^ t g x + actg (1 x) + r.
8.7. X!-ctgx t g x -cos2x+c.
298
1 X 5 3 - x
8.8. jc !-- eh x + 2 sh - - - th 2x + 25 cth +c.
2 2 4 5
8.9. X I-* - th x + c.
2
8.10. X H.-In !xl + l n | x - l | - - ^ l n | x + l | - - l n l 5 - 3 x | + c.
2 In 2 3
1 2 3 1
8.11. Xi- -p Arcsin l/2x H arsh 3x H Arctg x p arch x + c.
p. * ^ 3 4 ^ |/2
|/5 X ^ |/2^ X |/2 ^
8.12. X !- arch -7= - arsh l'2x+ Arcsin -7= + arsh l/2x + c.
15 [/5 ' 9 ][l 2 '
3 / 1\ 7T 3x+l
8.13. X ^ a r s h (2x+1) + Arcsin |^x- - j - ^arch^---- l-c.
1 2/3 2x + l
8.14. XHln lx +l|---- ( ^ + 1 ) --------Arctg l- 5 arth (x 2) + c.
2 3 i/3
8.15. ^ -cosx^ + cj-.
8.16. { - - s i n ( l + x)^ + c}-.
8.17. {2 tg(l+x)2 + c}.
8.18. <! - -cthx^ + c
8.19. {-2e-^" + c}.
8.20.
[in 2
8.21. {e^+c}.
299
8.22. { - e ^ + c y
8.23. {-ctg[/x + c}.
2 ^
8.25. -(x3 + 2)2 + c.
3 - 1 -
8.2S. - ( x2 + 2x- 3 ) 3 - - ( 1 - x3)2 + c.
8 3
8.28. - 2 ( x ^ +i y ^ + c.
8.29. -j/cosx+c.
8.30. - - sh^^2x + c.
6
1 . 3 1
8.31. - In'^x+ - n ^ - - +c.
4 4 In X
v8.32. - Arctg* x + c.
8.33. - In i2x'^-l| + c.
8.34. Hn(-2* + 9) + c.
8.35. In (1 + sin^ ;t) + c.
2
8.36. - I n |cos 3x| + c.
300
8.37. - In |sh 2x| + c.
4
8.38. 10 In larcth x| + c.
8.39. In |Arcsinx| + c.
8.40. In ish (1 x)|+ c.
8.41. - In Isin I+ c.
8.42. j/sin + c.
4 -
8.43. - ( l + sin2jc)^ + c.
8.44. - - In Icos jcI+ c.
6
8.45. COSX+ -cos^x+c.
8.46. 2 s h + +c.
2 3 2 5 2
8.47. -sin'^x---- sinx+c.
4 6
8.48. - ^cth jc + c.
8.49. - cos 2x + x sin + - cos 2x + c.
8.50. ( 2 x + l ) c h x - 2 s h x + c.
8.51. (x^ - I K - 3 x V + c.
8.52. - e *(cos x + sin x) + c.
301
8.53. ^2 cos ^ + sin ^ + c.
1
8.54. - e*(x sin X+ jc cos x - sin x) + c.
8.55. 2x + sin2x+c.
8.56. I l n ^ x - l n x +
i)""
8.58. XArccos jc - | / l - x ^ + c.
8.57. - ^ In |x| +
8.59. Arctg x - x + Arctg x + c.
8.60. - (1 x^)arth x +c.
3 6
8.61. sin x^ x^ cos x^ + c.
1 1 , 1
8.62. Arctg - + - x ^ ~ -In (x^+l) + c.
3 X 6 6
In 2
8.64. x2|/l + x2- ^|/(l + x)3 + c.
JC
8.65. - (sin In x cos In x) + c.
8.66. - (sh Xcos 2x + 2 eh X sin 2x) + c.
8.67. XArcsin x+ ^1 - x^ + c.
8.68. e - ^ \ x ^ - l ) + c.
302
1/ , . _ . 1 . . \
8.69. - ( sin 2x + x cos I x ---- sin 2x
4V 2
8.70. -
2
1 1
x -\ (1 x)sh2x+ - c h 2x
2 2 4
+ c.
8 . 7 3 . --------W + c
{\ + f xY
8.74. 2 cos |/x+ 1+ c.
8.75. j/2x - 1sin \ j l x- 1+ cos |/2jv:- 1+ c.
3 3 3 3 3
8.76. 3[ (|/x)^cos f x + 2fxs\n |/x+2cos j/x] + c.
8.77. - 2 [ l / 2 ^ s h | / 2 ^ - c h ^ / 2 ^ ] + c.
I
8.78. - (Arcsin x - - x f T l ? ) + c.
8.79. ^ + 4 ] / ^ + c.
8.80. )/jc^ + 1arcth ( |/x^ + 1) + c.
8.81. - + 3 ( | ^ ^ ) 2 + c.
8.83. 3
3
/[/j? 3
+ l/x + ln|l/x+li ]+c.
2
8.84. 2( )/x cos |/x+sin |/x) + c.
303
8.85. l n | 2j c - l | + - ~ ^ + c .
l - 3 x 2 x - l
8.86. - ^ \ n \ l - x ^ \ + ^ l n | x ^ - 2x + 5| + Arctg(x-l) + c.
3 x +l
8.87. In |jc^-2x + 2 | - Arctg(x- l) + ln |x^ + 2x+5|+ - Arctg^----\-c.
8.88. - I n | x + l | + 4 1 n |x + 2 | - 3 In |jc + 3 | + c.
8.89. I n i x l - ^l n{x^+l ) + c.
8.90. l n | x - l | + 51n| x + 3| + c.
8.91. ^ ( l n | x | - l n | l + x l - l n l l - x | ) + c .
8.92. In |x+ 11- - In (x^+ 1) + Arctgx + c.
8.93. - 9 In |x| + 101n | x - l | - l n |x + 2| + c.
1 1^ l ' x +l \
8. 94. - - - A r c . g ( ^ ] + c
9 1 jc
8.95.- +2x---- In + 4)-----Arctg - + c.
2 2 2 ^2
x^ 25
8.96 . x - 1 0 I n | x - 5 | + ------ +c.
3 x - 5
8.97. In IX- 11----- + In (x^ + 2x+ 2) - Arctg (x+ 1) + c.
X - 1
1 1
8.98. In |x| + ln | x + l | + ----------+c.
x 2x^
4]3 x+3
8.99. 2 In |x + 11In (x + 6x+ 12)H^ Arctg Hc.
304
8.100. - - I n \ 2 x - l \ + - I n \ x+l \ + c.
6 3
2 1
8.101. - In U - 1 I + - In | 2 x + l | + c.
8.102. ^ + | ^ + ^ l n | 3 x - 2 | + c.
8.103. y +5x+51n | x - 2 | - 5 1 n |x+2| + c.
x + 2
8.104. 2 In UI + 24 A r c t g ^ ----l-c.
1
8.105. 21n | x | - l n | x + 3 | ------- - +c.
x+3
8.106. 2|/2 Arctgp + ^ In + 3) + c.
2 3 X
8.107. In Ixl - - + -A r c t g - +c.
X 2 2
8.108. I n i x l - ^ - ln(x^ + 3) + 3/3 Arctg;^ +c.
8.109. x^ + 21n |x| - - + - In (x^ + l)+ Arctg x + c.
X 2
1
8.110. - (Arcsin \(lx + /3x )j \ -^x^) + c.
8.111. (x|/1+ x^ + arsh x) + c.
8.112. ^ - a r c h g + o.
20 Matematikai feladatok 305
8.113- - [Arcsin (jc + 5) + (x + 5)|/l - ( x + 5)^] + c.
f x + 2 i/(jc + 2)2 x + 2\
8.114. 2 - - 1- a r c h + c.
3 / 2 X + 1 i / ( 2 x + l ) 2 2 x + l \
8.115. - ( p / ---- ------- 1-1+ arsh p ) + c.
2 V l/6 y 6 |/6 /
/ :v+l x +l - , / 7 x+TV\
8.116. 2 ( ^ A r c s i n + ^ ) / l - ( ^ - y j j-^c.
/jc + 4 i / / x + 4\^ x + 4\
8.117. 2 ( ^ - y ] / ( ^ j + l + arsh j + c .
8.118.
j c - 2
+ 1+ arsh =I- c.
|/2 r 2 fi
t ) "
8.120. - Arcsin 2x + c.
4
8.121. 5 arch -p + c.
8.122. / 2 arsh|/2x + c.
2x - l
8.123. 2 Arcsin + c.
2
jc 2
8.124. arch - l- c.
8.125. arsh (2x + 1) + c.
8.126. ^ | / ? - j c + 21^-2In(fc+ l) + c.
306
8.127. 2(2|^+x+41n |2-|/x| ) + c.
8.128. 2|/2x+ 1- Arctg p x + \ + c.
1
8.129. - - ( x + l ) 2 - - ( x + l ) l / ^ + c .
^ .3
20
8.130. y |/(x+l) -4|/(x+l)* + c.
8.131. x+l+4l/j^+41n||/x+T-l| + c.
8.132. - ^ |/l-(2x+2)2 + ^ Arcsin (2x+ 2) + c.
8.133. + ' - t a r s h ^ + c .
8.134. - I n ( | / 2 x - l ) + l n H / 2 x - l - l | + c.
8.135 . - +X+C.
X
1 + t g -
8.136. Arctg ^2 tg ^ + c.
8.137.. ^ ^In tg.^ - 2 - In 2 tg ^ - I ^ + c.
8.138. x - t g ^ +c.
X
t g 2 -
8.139.4)12 arth ^ +c.
8.140. + ^tg^^ +tg^^ + c.
20* 307
1 / I
8.141. -p Arctg ^ tg X1+ c.
fS \ f 5
8.142. - - In
2
1
1- t g -
+ -In
1+ t g -
+ c.
8.143. In |tgx| + c.
2 1 4
8 . 1 4 4 . ---- X-----In | 2 - 2 c o s x - s i n x | + - In 1-
1
X
+ c.
1 ^ 3
8 . 1 4 5 . ---- sin'^x+ - x - s i n x + c.
8 2
1 X
8.146. - tg 3x 3 ctg - - 2 x + c.
5 5 5 ^ 1 . ,
8.147. X ------sin X cos x ------ sin^ x cos x ----- sm x cos x + c.
16 16 24 6
8.148. -
2
3 3 1
- X+ - sin 2x cos 2x H sin 2x cos 2x
\4 8 4
+ c.
/
8.149. 2
1 X X 4 X
8.150.
---- sin'^ - cos I- cos^--------cos - | + c.
5 2 2 15 2 5 2,
1
sin 3x---- sin^ 3x ) + c.
3 J
8.151.
1
+ ln |cos 2x|
8.152. 2
2 cos^ 2x
X X 1
- + ctg - - - ctg
+ c.
2 3
8.153. - sin^ X cos^ xH sin^ xcos x ------sin x cos x+ x + c.
6 8 16 16
1 2
8.154. - sin^ X cos^ ^ x + c.
D 1J
308
8.155. - sin X - sin X + c.
8.156. - sin X---- sin* xH----- sin^ x + c.
6 4 10
1 5 1 3
8 . 1 5 7 . ---- c o s - x -----cos- x + c.
5 2 3 2
8.158. sin 5x+ - sin x + c.
10 2
8.159. ^ln|2e^-l| + ^ ---- -
2 6(2^ + 2)3 4
- - A 5 - e T - ^ 1/(2" - 2 ) 3 + c.
1 1 1 , 1 1 1 7
8.160. - s h x + - s h 3 x + -sh^ 3x---- cth2x - sh8x - - s h 4 x - ~x + c.
2 3 9 2 64 8 8
X 2
2 In sh - ---- In sh 5x
+
2 5
1
~ i
4
1
2 '
1
12'
8.162. e *-ln(l + 0 + c-
1 1 1
8.163. - - x + + - I n \ e^-2\ + c.
4 2e^ 4
8.164. X-
1
In I 1j+ c.
e^- 1
8.165. X In (e"+ l) + c.
8.166. x - ^ l n (e2"+l) + Arctge*+c.
8.167. 2 Arctg + c.
8.168. - { - In + 1) + In | e" - 11+ In (e* + 1)] + c.
8.169. Alkalmazzuk a sh x = --------- , eh x = ---------- helyettestst, es integral-
309
2^^^ 14^^^ 2^^ 1
juk a fggvnyt a ^ ^ ^ ^ alakban az e^=t , dx = y dt helyettestssel. Ez
1 1 1 2 \ ^ .
dt integrlhoz az
2^ + 4 / - 2
-/ =
+
/
2( - i ) 2( / + i ) ( + i )2 ( t +i y
vezet. A feladat mskppen is megoldhat: Az integrland fggvnyt bvtjk
eh X eh X 1 1
(eh X- l)-gyel, s gy a fggvny r---- 1-----^--------;--------p - alakban integrlha-
sh^ X sh^ X sh^ X sh^ x
X
t. Az integrlst elvgezhetjk a = th - helyettestssel is. Az eredmny:
00 x^"'''^
8.170. y (- 1 )"--------------------+c.
^ (2 + l ) ( 2+l)!
00 jlH+l
8.171. y -------- ------ +C.
^ o ( 2 n + l ) \ 2 n + l
00 y2n +l
8.172. I ( - D " , , +c.
=o (2rt+l )-!
rtn2)" x^"'*'^
8.174. - y ^ ------r+C.
%(2n+i y
00
8.176. 27-12l^(= 10,03).
8.177. 0,72).
8.178. 7 + 7 s h 3 - 7COS2.
4 6 4
8.179. ^ t h 4 + Arctg2.
310
8.180. - - a r t h 0, 5.
6
8.181. -
e
8.182. 7-
4
8.183. ^ I n 3 ( = 0,55).
8.185. ^(8 l n 2 - 5 ) ( = 0,18).
8.186. a; b ) ^ .
8.187. a) e^~ - ; b) e^ - - ( = 7,02).
c e
8.188. a) = 2
28
8.189. y
' - ?
o " 5
b) [-cosx] = 2[-cosx]o = 2
8.190. ]/2.
8.191..
3
8.192. 20.
8.193. 6 In 2- 2.
8.194. 2.
311
8.195. ^In 2.
2 '
8.196. - In 3 + In 2.
4
71
8 . 1 9 7 . - .
8.198. 13,66.
8.199.
143-24 In 12
8.200 .
18
8.201. 2ab
a b n
Arcsin - Arcsin ;-r - -
+ a^ + b^ 2
8.202. 2.
8.205. 13,5.
64
8.206. j 16; b) A,5\ c) .
8.207. (3;r + 2 ) : { 9n- l ) = 0,43 : 1.
8.208. r^n.
8.209. 1271.
8.210. Q s h 2- 2 shl + l,5^ .
8.211. 67t.
312
8. 212.
2
8.213. ( p - 9 )7T= 0,746.
49
8.214. y = 5,44.
8.215'. 2 arth
tg
8.216. ^ (3 j/2 - /5) + ^ (arsh 3 - arsh ^/S).
8.217. - 7 + 2 a r t h ^ .
4 4
l + 2a^-3a*
8.218.-----
6a^
8.219. p + I - | / 2 - ^In
1 1
8.220. - - + 2 a r t h - .
2 2
8.221. - I n t g 7 -
6
8.222. +arsh l).
61
8 . 2 2 3 . - .
8.224. 6a.
8.225. ^ ( / 2 + a r s h 1).
| / ? + T + i
| / ? + T 1
1 l/2+ l
+ ----
2 |/2- l
313
8 /T- 1
8.226.
27 9
8.227. n.
8.228. 2|/2.
8.229. - rn^.
2
8.230. 2{e^-\).
8.231. - a l n l .
8.232. -sh^ 1.
2
8.233. Inchl .
8.234. f ia shl.
8.235. sh^ 1.
8.237.
16
8.238. 71.
2l7T
8.240.----
f
314
8.241. 7t(-2
8.242. ^(2 + sh 2).
8.243.
16
8.245. 5^3-2.
63
8.246. n.
4
8 . 2 4 7 . , ^
27
371 363n
8.248. 242 =
8.249.
10 5
2(2^-1)7t
9
8.250. - .
2
8.251. i2;r.
8.252. - I n - ^
2 e + 2
8.254.
8.255.
315
8.256.
56n
8.257.
15
n(2 In 2- 1)
8.258. 71
\
- +2 In 2
\2 /
8.259.
2
n ( 1^
e ----
V e j
8.260. 7t(3 In^ 3 - 6 In 3-4).
8 2gi
4
8.262.
4
8.263. Stt.
7r(e^ 2e) 471
8.264. F, = ^ ^ ^ K, = y .
lTT 2167T
8.265. F, = ; K, = .
8.266. F , - F, = 113,2-102 = 11,2.
l/21
8.267. a =y - J
/ch^ 4 ch^ 1
8.268. 7T---------eh 4--------- +ch 1
Stt
8 . 2 6 9 . - .
8.270.--------
316
8.271. 25St.
8.272. 3,2.
8.273. ^ ab^ n.
8.274. = V^ = n^- l n.
8.275. A,5n^.
8.276. 7Tsh2 + 2;r.
n{2 f i - 1)
8.277.
8.278. 367T.
\6n
8.279.
8.280. 27r(|/2 + arsh 1).
8.281. In th - arsh 2 + eh arsh 2
2
8.282.
1371
8.283.
27t(3|/3-l)
8.284. 47r2 + 47t.
8.285. y
8.286.
36rt
|/5
' / ^ |/5
+ Arcsin
36 6
317
8.288.
(5 1^- l)7t
8.289. fia^n.
2 |/27t(2e' + 1)
8.290.
8.291. T^.
8.292. Ar^n.
52a^n
8.293.
16
8.294. a) 4-
64
8.295. 4; y ;
256
8.296. ; 0;
b)
n 3
7 4
128 32 1312
21 5 105
64 256 4288
c )
c)
15 7 105
4096 128 1664
105 15 35
8.297. a) Az X tengelyre vonatkoz elsrend nyomatk az
1
6" , ,
{a^-x^)dx =
2 ^
a^x -
integrl rtke, az y tengelyre vonatkoz statikai nyomatk pedig
Cb b
- x ] / a^ - x ^ dx = -
a 2a
b \ 11
- I x ' ^ a ^ - x ^ d x = - { a ^ - x ^ f = 0.
3a
b) A. flellipszislemez terlete T = , gy a slypont abszcisszja Xj = -^ = 0,
Sx 2ab^ abn 4b
amely a szimmetribl is addik, ordintja pedig >>s = = r - : = .
T i 2 'in
318
c) Az X tengelyre vonatkoz msodrend nyomatk xzI^ = -
] r 3
- ( a ^ - x ^ y d x =
a
3 J
\ / /
dx integrl rtke, amely az - = sin t,
a
1- 1- ) = cos t,
.aj
dx = a cos dt helyettests utn
ab^
L =
ab^ r /l+cos2A^ ab^ C/3
cos'^ tdt = ---------- \ dt =
2 ) 12
- + 2 cos 2? + - cos At\dt =
2 2
ab^n
Az y tengelyre vonatkoz msodrend nyomatk.
b ,_____
/%
/ /
h =
- x ^ ^ a ^ - x ^ d x = b
a
x^
V - \
\
- 1 dx
kiszmtsnl is a fenti helyettestst alkalmazva:
ly = a^b
. - a^b
r , a^b C
sin t cos t dt = sin^ 2t dt =
4 .
8 .
(1 cos 4t) dt =
a^bn
Az origra vonatkoz polris msodrend nyomatk I = +1 =
abn(a + b )
8
8.298. a)
8.299. a)
e - 1 e^+1 / e^+l e ~2\ 6- 2e 2e^+l 2e^-6e+19
2 4
sh2 - 2 1
4 2
bj
e(ch 1- 1) 8(ch 1- l V
c h M - 3 c h l + 2
c j ---------------------; 3 eh 1 - 2 sh 1 - 2 ; 0,36.
8.300. ^ ^
l/3r l l r
8.301.
n lTT
''n 2 n
b)
2 8
2 n + 4 971 + 40
9 2 18
319
8.302. an,
8.303. 0;
6|/3
a) r^- r^-
2r 2r
n n
8.304.
8.305. a) s h2- 2; 4(sh 1- e h 1+ 1);
c)
r^n r^n r^n
/ 2(sh 1- c h 1+ 1) sh2 - 2
^ \ sh 1 2 sh 1
c) 8 sh 1+ - s hM; 24sh 1- 16eh-1; 32sh 1+ - s h M - 1 6 e h 1.
3
8.306. a) 2,78; 3,17; b) (1,52; 1,34);
8.307.
3 , 3 , ( 2 2 ^
b) [ - a , - a
c) 8,54; 5,54; 14,08.
8.308. a) - - a^\ - - a^\ b) - - a; - - a c) - -a^; - - a^\ - - a^
3
L
4
2304
8.309. a) 96; 12n\ b) (3;r; 4); c) ; 4327t^- 1228,8; 4327c^-768.
8.310
8.311. ay
'6 ^ 6\
n)
n{2 s h 2 - e h 2 + 3)
32 ^ 3 2 r 64
c; y 7 t - 8 | / 3 ; y7c + 8l/3; n.
8
/ 2 s h 2 - c h 2 + 3
<>[ 2(2. s h 2) - -"
71 / 3
c) ( 3 + - s h 2 + sh2 - ch^ 1
16 V 2
8.312. a) ; r ( l - l n 2); b) (2 + In 2; 0; 0);
c)
I n
48
8.313.
n
18'
320
8.314. a) 24,15n;
8.315. a) 4n;
8.316. a) 0;
8.317. a) - n;
8.3T8*. a) 71;
M45;r2+128)
b)
11 \
y ; o ;
b) Q ; 0; 0 ) ;
b) (0; 0; 0);
/4
b)
b)
c) 34,871.
128
c) 352,02.
c) - Ti-
8
8.319. a)
36
Ti
0; 0 ; c)
4_^
16
8.321. a) n\ nl \
8.322. a) I5n;
8.323. a) 18^^71;
8.324. a) 97,17;
8.325. a) 204,2;
32
9a 71
8.326. ;
8.327. a) 0;
21 Matematikai feladatok
5 = -
8
5 \
0; - ; 0)
/ = -
^ 22
b) (0; l n 2; 0);
b) (2,5; 0;0);
b) (2; 0; 0);
b) (1,66; 0; 0);
b) (0,21 ; 0; 0);
256
b) (0; 0; 0);
l7l
C) 7C.
81 7t
0 - ^ .
c) 388,72.
c) 5,84.
6a*n
c ) ----- .
^ 11
c) 2\6n.
321
8327T
8.328. a) ;
8.329. a) 8,09;
8 i/2^7t
8.330. a)
15
b) 0;0,53;0);
10247T
c) 32,26.
fa^n
8.331 c)47t; d) 2 arcth e) \n2; f ) - arcth g) 2;
h) - l ; i ) ^ ; j ) 6; k) 0; l) n-, m) n) 8; o) - 7 In 3.
2 9 4
8.332. a)
-480/3-1111
b)
-9601/3-233
18 480 27 720
8.333. 0,7462; j 0,7468; ej 0,7468; ?; 0,001; 0,0001; 0,0001.
8.334. 366;r.
3140
8.335.
8.336.
8.337. a) 7,91
cm^
18847T cm^.
kp
dm^
8.338. a) 1480 mm^
8.339. a) 470,58 g;
8.340.
8.341. a) (9; 3,100);
/ 79\
kp
b) 7 , 8 0 - ^ .
dm
1710
b)
b) 480,40 g.
"(41
b) (9; 3,125).
322
9.1. A mrtani sorozat sszegkplett alkalmazva, a szmsor a kvetkezkppen
rhat:
9. fejezet-.
00 l \ t n / l \ t 1
/ l y
v i ) - 1
mivel
3 -
n \
ha -oo. Teht a szmsor konvergens.
" 2* +3^ 3
9.2. lim y = - ; konvergens.
n - oo 6 2
" l + 3 + 5^ 23
9.3. hm 2, ---- 7Tk = ; konvergens.
n- 00 1j Zo
k'>"'' 8ens.
3*+
9.5. hm Y, ~TTk----divergens.
k=o 2
n 3(1+1
9.6^ hm Yj = 12; konvergens.
n^oo 2
^k+3 320
9.7. hm Z ( 1 ) * - ^ ~;r konvergens.
n-^oo jt = 0 ^ ^
9.8. hm y ;---- 1 = a +2; konvergens.
t=o \ a + 2/
9.9. A szmsor az 5.13. feladat alapjn zrt alakban rhat:
Qo 1 . " l
, ? i A:(/:+!)(/: + 2) ^ k( k+l ) ( k + 2) ^
n(n + 3) 1
= lim----------------- = - , teht konvergens.
-oo4(n+I)(n + 2) 4
21* 323
^ k 1
9.10. lim y - = - ; konvergens.
w-cx) 1^= 1(A:+1) (A: + 2) (A: + 3) 4

9.11. lim ^ - 00 ; divergens.


{ l k - \ ) { 2 k +\ )
" 1 1
9.12. lim Y. 77^7 = z ; konvergens.
n-oo 4k^ - 1 2
" 1 1
9.13. lim X ,ct/ = T konvergens.
00 {2,k - 2) (3A: + 1) 3
" 1 1
9.14. lim ^ ; konvergens.
"-> t=i + (<?+ ^ + 1) c+\
1 3
9.15. lim Y. T7rTT\ ^ konvergens,
n-oo A:(A: + 2) 4
" 3 11
9.16. hm 2, ; t = ; konvergens.
n-cx) j /r(/c: + 3) 6
9.17. lim y In ( 1- - 7 ) = In ; konvergens.
-^k=2 \ k J 2
" 6
9.18. lim X = 6; konvergens.
t=3 ( k - 2) (A:-1)
9.19. A szmsor mrtani sor. Ez konvergens, ha |sin2a| < 1, amelyet - +
4
n
+ kn < a < -^+ kn, ^ = 0, +, 2,... rtkek elgtenek ki. A szmsr sszege:
^ (sin l a f = lim ^ (sin 2af = ----- .
k=o 1~ sin 2a
3
9.20. (a) = ---- ha -^aeR
al
l O- b
9.21. 5(6) = : ^ , ha
101
- 10 < < y)-.
324
9.22. s{c) = ha { c e R | c < - 3 s 3<c}.
c+3
9d^
9.23.s(d) = , ha {dR\ - 9<d<9}.
4(v a)
9.24. Kpezzk |5+-5i -et, ahol S az -edik, S+ az ( + w)-edik rszletsz-
szeg. Kapjuk, hogy
1 1 1
+ ----- + . . . +
n+\ +2 n+m
1 1 1
+ ------ + . . . +
n + m n + m n + m
----- , ha w ^ 2. Akrmilyen nagy az n rtke, lim------- = 1, teht nem teljesl
n + m n ^ o o n + m
a Cauchy-fle konvergenciakritrium, ezrt a szmsor divergens.
9.25. = - p = -> 00, ha w-oo, a sor teht divergens.
p + l + ^n + m + l
9.26. ^ ^ 0, ha n^oo; a sor konvergens.
9.27. A szmsor lehet konvergens, mert a lim ^ = 0 szksges felttel fennll.
k-*co k
Tovbb &zf{k) = az [1; oo)-ben szigoran cskken s f{k) > 0, teht kvetkezik,
00
co 1 ^
hogy y -T konvergens, mert /x = lim
k = <0- 0.
l
- : d x = 1.
1 1
1 1 1
= - y
^ 00.
\ 2k 2 *t-i k
A ' 3 1 A 1
> - y - ^ 00, ha n^co
V *
" 1
<
fc=l AC
= 1, ha n^ oo.
9.31. A szmsor lehet konvergens, mert a lim ---- = 0 szksges felttel fenn-
*-00 5k^ + 2
00 J 1
ll. Tovbb a ^ ^---- szmsort majorlja a konvergens szmsor, ugyanis
n=i 5A: +2 k=k
konvergens.
325
9.32. Divergens, mert -7= > - es V - divergens.
k k=ik
1 1 , ^ 1 . .
9.33. Divergens, mert , > - es V - divergens.
/8A: + 7 k k= k
1
9.34. Konvergens, mert - < 7^ s X 7^ konvergens.
!c "I 3/r I 2 k
9.35. A szmsor konvergencijt a szmsorok konvergencijnak defincija alap
jn is belthatjuk.
oc 1 n I
t?2 A:2-l t?2 (A:-I)(A:+1)
Az------- ^--------= - ( ---------- ^ ) azonossgot felhasznlva,
(A:-1)(:+1) 2VA:-1 k + l j
1
\
1
= - lim
2 n-*oo
1 1 1 I 1 1 1
1---- + ------- + --------+ . . . + ----------------
3 2 4 3 5 n - l n+l
I n ^ i ' z K k - l k + l /
= - , ltezik vges hatrrtk, teht a szmsor konvergens.
4
" 3 11
9.36. Konvergens, mert lim V = .
n-*oo A:(fc-3) 6
2A:+1 1 * 1
9.37. Konvergens, mert 2 ^ ^ Ti konvergens.
rC (^rC Vj fC j rC
1 1 1
9.38. Divergens, mert - > - s > - divergens.
In k k= 2 k
9.39. Divergens, mert f (k) = , / a [2; oo)-ben cskken,
k i n k
/(A:)>Os -dx-^oo.
5/t + 3
9.40. Divergens, mert f {k) = ;---- az [1; oo)-ben cskken,
4^:^+ 1
326
/(^)>Os
5x+3
T-dx-^co.
4x^+1
00
9.41. Konvergens, mert f{k) = az [1; oo)-ben cskken,
k^+ 1
/(A:)>Os 7
\ +x 4
g ^ 2 _j_ j
9.42. Divergens, mert f{k) = ----;----- az [1; oo)-ben cskken,
k(k^+ 1)
f (k)>Os
8x^+1
x(x^+l )
00.
n J
9.43. Konvergens, mert [/^ = - < 1, ha n^co.
9.44. Konvergens, mert lim = - < 1.
a 4
9.45. A szmsor lehet konvergens, mert lim -:r = 0 szksges felttel fennll.
/ T 1
Tovbb lim = lim / - r = lim - = 0 < 1, a Cauchy-fle specilis gykkrit-
t-oo t-oo I' A:* k-oo k
rium is fennll, teht a szmsor konvergens.
n_
9.46. Konvergens, mert lim |/^ = ;? < 1.
9.47. Konvergens, mert lim = 0 < 1.
n-oo
9.48. Konvergens, mert lim = 0 < 1.
9.49. Konvergens, mert lim = 0 < 1.
n-oo a
327
9.50. Konvergens, mert lim = 0 < 1.
ri^_
9.51. Konvergens, mert Hm |/^ = 0 < 1.
n-oo
9.52. Divergens, mert lim a = + oo.
9.53. Konvergens, mert lim = - < 1.
-oo a 4
9.54. Divergens, mert lim a = + oo.
9.55. Konvergens, mert lim = 0 < 1.
k \
9.56. A szmsor lehet konvergens, mert lim = 0 szksges felttel fennll.
k-^co k
Tovbb a specilis Cauchy-fle hnyadoskritrium is fennll, mert
fc+i |. 1
= hm --------: lim = lim ------- ^ = - < 1, teht a szmsor konvergens.
v k-oo / 1 \ e
9.57. Konvergens, mert lim = - < 1.
a 2
9.58. Divergens, mert lim = e > l.
9.59. Konvergens, mert lim = - < 1.
"^oo a 3
d c
9.60. Konvergens, mert lim = - < 1.
a_ 3
9.61. Konvergens, mert lim l/j, = 0 < 1.
l/fc + 2 + | / F ^ 1 1
9.62. Divergens, mert---------^ ------> - s ^ - divergens.
3 |/fc k fc=2^
328
9.63. Konvergens, mert----- r < ,
l + 4* \4
s X ( ~ ) konvergens.
/ k = i \ 4 /
4
00 f^ 2
9.64. Divergens, mert a V ------szmsornl lim a = co konvergencia szks-
k=3 3 n-^>
ges felttele nem teljesl.
Cl
9.65. Konvergens, mert lim = - < 1.
9.66. Konvergens, mert lim l/^ = - < 1.
M->00 4
n 1
9.67. Konvergens, mert lim [/^ = " t 1*
M00 10
9.68. A szmsor nem konvergens, mert
^ 5k+l V^
lim
k-* 00
-
-1
lim
k-* 00
^1 +
1 A
5k + 2
- 1
1 ^
5k+2 5k + 2
-
V - 1 J
,
- 1 J
-2
= e~^- l # 0
szksges felttel sem ll fenn.
9.69. Konvergens, mert lim ^a = - < 1.
W00 3
k +l
9.70. Divergens, mert lim V - - oo.
n-oo t = i k(k + 2)
1 1 ^ 1 .
9.71. Konvergens, mert - < ^ es ^ ^ konvergens.
K "t" 4 K 1^=1 fc
k +l
9.72. Divergens, mert f (k) = - [ 2 ; oo)-ben cskken,
K I L
f ( k) >0 s
x+1
x^ + l
-dx-KX).
sin^ k 1 , 1
9.73. Konvergens, mert < ^ es 2^ konvergens.
k(k~^ I) k k= 1k
329
1 1 1 1
9.74. Divergens, mert----- j=> - s - V - divergens.
k + ]jk 2k 2k=i k
2 2 1 * 1
9.75. Divergens, mert V ------szmsornl - - > - es > - divergens.
k t ' z k - l k - l k
P + 6 1 1
9.76. Divergens, mert-------- > - s \ - divergens.
6 6 A : + 1 k it= 1 A:
9.77. Konvergens, mert lim = - < 1.
n-^co 3
9.78. Konvergens, mert lim = - < 1.
no} a e
9.79. Konvergens, mert f {k) = > 0 s f{k) cskken a [2; oo)-ben s
A:ln^ k
00
n____ y j
9.80. Konvergens, mert lim |/^ = - < 1.
ncxD 4
n J
9.81. Konvergens, mert lim (/^ = - < 1.
n-*QO 3
9.82. Konvergens, mert lim ^ Qjk + 2 2^A:+l + |/fc) = 1j/2.
"^0/c=l
n
9.83. Divergens, mert lim ^ {^k + 2 - |/A:+ 1) = oo.
n-a, = i
n
9.84. Konvergens, mert hm (2|/fc|/A: + 1 1) = l + '/2.
n
9.85. Divergens, mert lim ^ (|/A: + 3 - f k+ 1) = oo.
/k= l
330
9.86. A vltakoz eljel sor Leibniz ttele miatt konvergens, ugyanis
lim
k-^ oo
= 0 fennll.
2k+ 1
9.87. Konvergens a Leibniz-ttel alapjn.
9.88. Konvergens a Leibniz-ttel alapjn.
9.89. Konvergens a Leibniz-ttel alapjn.
9.90. Divergens, mert lim \a\ # 0 .
n-*QO
9.91. Konvergens, mert lim \a\ = 0.
nao
" 10
9.92. Konvergens, mert lim ^ ( - 1)'0,1*^^ = .
k =0 U
" 2k+l
9.93. Konvergens, mert lim Y ---- = - 1 .
"-00^=1 k^ + k
9.94. A vltakoz eljel sornl Leibniz ttele nem teljesl, ugyanis
k+l___
lim | ( - 1)' |/0,01 I = 1 # 0, teht a sor divergens.
n~*co
A ^ 10 '
9.95. Konvergens, mert lim ^ 3(-0,l)^ = .
n-^coj^^Q 13
< 1, azaz < 1, vagyis
9.96. A fggvnysor konvergens, ha -----
k X l
|x| < |x 11, amelyet a z x < O s a O < x < - rtkek elgtenek ki.
^ / 0 Y
Ha x = 0, akkor a ^ I ---- szmsort kapjuk; ez pedig konvergens.
k=\ V" 1/
1
Ha X= - , akkor a 1/ szmsort kapjuk; ez pedig divergens.
2 k=\
< / X 1
Teht a Y ------) fggvnysor konvergens, ha x < - .
2
Az sszegfggvny (x) = -----------= - x.
331
9.97. Konvergens, ha { x e R | x < 2 s O^x} s j(x) x + 1.
1
9.98. Konvergens, ha {xeR| x<0} s 5(x) = ^.
1+
9.99. Konvergens, ha { x e R\ { 0 } } s 5(x) = .
10
9.100. Konvergens, ha {x e R| - 10<x< 10} s s{x) =
9.101. Konvergens, ha { x e Rl - 2 ^ x ^ 0 } .
9.102. Konvergens, ha {xe R| 0^ x^ 2} .
9.103. Konvergens, ha {xeR| 2^x^0}.
9.104. Konvergens, ha | x e R - ^ x ^
9.105. Konvergens, ha | xgR 0 < x ^ ^ s l ^ x
9.106. Konvergens, ha {xeR| 0<x} s (x) =
e * - l
1 - 1
9.107. Konvergens, ha <xeR| |x| < - ( 4 m+ 1); me Z> s ^ 73------
l 4 J \ i gx
9.108. Konvergens, ha minden x e R s (x) = 2x.
3
9.109. Konvergens, ha minden x e R s S'(x) = .
1
9.110. Konvergens, ha -(xeR
1 1] - . X 1
------ < x < - > es i ( x ) =
3 3 l - 3 x
9.111. Konvergens, ha x = 0.
9.112. Konvergens, ha { x e R| 1<x< 1}.
332
9.113. Konvergens, ha <xeR
9.114. Konvergens, ha <xe R
1 L
- - < jc < ->.
2 - 2)
9.115. Konvergens, ha xe R.
9.116. Konvergens, ha xe R.
9.117. Konvergens, ha {xeR| - e + 1 ^ x ^ e + 1}.
9.118. A fggvnysor konvergens. ha
lim
k-^ ao
< 1, azaz
( i V Ul e 5 5
lim 1+ - = 7 Ix| < 1, vagyis---- < x < - .
fc-oo \ kJ 5 5 e e
5 / k \ ^ 1
Ha X= ---- , akkor a Y ( - l)*" ( - ) vltakoz eljel szmsort kapjuk, amely
e k=i \ e j k\
(
k V
- ] k \
e j
= 00, a sor
konvergens lehet, lim = 0 esetn; de mivel lim
f c CXD f c 00
divergens.
5 00 f k \ ^ 1
Ha X = - , akkor a Y - ] * pozitv tag sor konvergens lehet, lim aj, = 0 esetn
g t=i \ e j k\ k-^co
lim ( - ) ^! # 0, teht a sor divergens. gy a fggvnysor konvergenciatartomnya:
5 5
------< X < - .
e e
9.119. Konvergens, ha xe R.
9.120. Konvergens, ha xe R.
9.121. Konvergens, ha <x e R
9.122. Konvergens, ha {xeR| 1 < x < 1}.
9.123. Konvergens, ha {xeR| - 1 < x < 1).
9.124. Konvergens, ha { xe R| 0<x} .
333
9.125. Abszolt konvergens, h a { x G R \ { - l ; - 2; - 3; . . . ; - k } , k e N}.
9.126. Alkalmazzuk a specilis hnyadoskritriumot, majd algebrai talaktsok
utn kapjuk, hogy lim
k-^ oo
x e R
vnysort az albbi helyeken:
^H+l
X
t
3x+l
< 1. Az |jc| < |3x + 1| egyenltlensg az
x < - - u - - < x < - - u O< x } ^ halmazon igaz. Megvizsglva a fgg-
2 3 4
az jc = 0 helyen a sor konvergens;
1--------------- , A:
az X= ---- helyen a X ( 1------szmsor divergens;
4 )t=3 k 2
az X = - helyen nincs rtelmezve a sor;
1 k
az X = - - helyen a ^ -- szmsor divergens.
2 2
X < - u - < x < ---- u O < XV-ban abszo-
2 3 4 ~
Teht a fggvnysor az {x e R
lt konvergens.
.127. Abszolt konvergens, ha { x e R \ { - l ; l } } .
9.128. Abszolt konvergens, ha {xgR| x < - 1 s 1 < x}.
f 1 1 1
9.129. Abszolt konvergens, h a <x GR\ - { 1, - , t , . . . , ----
l [ 2 3
9.130. Abszolt konvergens, ha x g R.
9.131. sh2^ - . e R,
9.132. c h3 x= | ; ^ , x g R.
9.133. A 2"^ fggvny derivltjai: (2"')*"^ = 2*(ln 2)", n = I, 2, 3, Kiszmtva
ezen derivltak helyettestsi rtkt az x = 0 helyen, helyettests utn kapjuk, hogy
* (ln2)*
a fggvny Maclaurin-sora Y ------- x*. A specilis hnyadoskritriumot alkalmaz-
k% k\
va kapjuk, hogy ez a hatvnysor minden x-re konvergens.
334
Teht az egsz szmegyenesen rvnyes, hogy
r
k =0 k\
^ (n 2)^ ,
2 -
00 2^^ ^
9. 134. s i n^x= E x g R,
fc=i (2A:)!
00
9.135. cos^x = 1+ I ^e R.
k= (2a:)!
00 32fc-l
9.136. si n3x= ^
fc=i (2A:-1)!
9.137. cos 3x = f; x e R .
k =0 ylK)\
9.138. = X ( -1 ) ' = ^ ; x e R .
k =0 k
9.139. = ( - 1 ) ^ + 1 - ^ , x e R .
k =\ AJ!
9.140.^ = f; ( - l ) V , {xeRI - 1 < jc < 1}.
1+ x *=0
9 . 1 4 1 . {xgRI - 1 < X< 1}.
1X fc= o
1
9.142.------{xgRI - 1 < X< 1}.
1+ X t = o
9 . 1 4 3 . ^ = i {xgRI - 3 < x < 3}.
3 x k =i \ J /
^2k 1
9.145. ar ct gx= E ( - D*;rr-TT7 { x R \ - K x < l } .
t = l U!
335
ao
9.146. In ( 1 {x e R| 1 < X < 1}.
*=i
9.147.-'^= X (^+1)^*^; { x e R | - l < x < l } .
(1 X) jt = o
9.148. In(1+x) = {xeR| - 1 < jc < 1}.
t=i k
9.149. A z ------ fggvny els ngy derivltjt kiszmtva s vve e derivltak
cos X
helyettestsi rtkt az jc = 0 helyen, kapjuk, hogy a fggvny negyedfok Maclaurin-
1
polinomja:------ = 1 + ------1-----x'.
cos X 2 24
1 2
9.150. tgx = x+ +
1 2
9.151. thx - X- - x ^ + +
1 , 3 ,
9.152. arcsin x = x+ - x ^ + x^ + ....
6 40
9.153. arccos x = - - ( x+ -x^ + ...
2 \ 6 40 /
9.154. arth x = x+ - x ^ + -x+ -x"'+ -x+ ....
3 5 7 9
9.155. xln (x+ 1) = x ^ - - x ^ + - x ' ^ - -x+ - x-
2 3 4 5
x^ X* X 17
2 1 2
9.157. e^shx = x + x^+ - x^+ 15
x^^-i x^ x^ x^
9.158. L e g y e a j ; . . + y + (1)
k = i 2 k - l
336
hatvnysor konvergenciatartomnya K. Mivel az (1) hatvnysor a AT-ban konvergens,
ezrt a tagonknti differencilssal nyert
l +x" + x^+ ... = X (2)
ii=i
1
hatvnysor is ugyanott konvergens. A (2) hatvnysor sszegfggvnye AT-ban ^ ,
ennek konvergenciatartomnya pedig a ( 1; 1) nylt intervallum. A (2) sorbl az (1)
sort tagonknti integrlssal kapjuk. Az :v = 1s az x= 1helyen az (1) sor divergens
(ez az sszehasonlt kritrium segtsgvel knnyen belthat).
Teht, ha |xi < 1, akkor
1 l +x
i?i 2A;-1 2 1 - x '
TTTTTT " ( l - x ) l n ( l - x ) - x , {xgR| - 1 ^ x < 1}.
9.160. X ( - l ) ' J r - r = - arctgx, { x e R| - 1 ^ x ^ l}.
k=l 2k \
00 (x 2)^^^
9.161. X , , = - ln(3-x), ( xeRI 1 ^ x < 3}.
k =0 K+1
X^ . 7t
9.163. A sin X = X - harmadfok polinommal s a = 0,261 799 rtkkel
szmolva, sin ^ = 0,261 799 - 0,002 990 =. 0,258 809. A maradktag |i?5(x)| ^
^5 I
= 1,02 10 teht a kzelts legfeljebb 4 tizedesjegy pontossg.
X
5
9.164. cos-^ 0,978 147; < 10^
9.165. e 2,7180; |/?7(x)| < 10^
9.166. lnl, l 0,095 33; l/?4(x)1 < lO' ^.
9.167. tg 20 0,3632; i/?s(x)| < 10"^
22 Matematikai feladatok 337
9.168. c o s x 1 - y ; | / ? 4( x ) | < 10", a h o l 0 ^ x ^ 0 , l .
9.169. chx 1+ Y ; |i?4(x)| < 10-^ ahol - 0 , l ^ j c ^ 0 .
9.170. e* SS 1+ X+ |i?3(x)| < 10-^ ahol 0 ^ x ^ 0 , l .
9.171. x * - 3 x ^ + 2 x ^ - 5 = (x-1)'' + ( x - 1 ) 3 - ( a : - 1 ) 2 - ( x - 1 ) - 5 .
9.172. x * - x + l = (x-l)'^ + 4(x-l)3 + 6(x-l)2 + 3 ( x - l ) +l .
9.173. x ^ + A x ^ + l x + 2 = ( x + 2 y - 2 ( x + 2 f + 3 ( x + 2 ) - l 2 .
9.174. x^ + lx^+\%x+22 = (x+3)3-2(x+3)^ + 3(x + 3) + 4.
9.175. A sinx = x harmadfok Maclaurin-polinomjval kzeltsk. gy
6
6 ^x - + x^ = 2x^ algebrai egyenletet kapjuk, ennek kt gyke x^ = 0 s Xj = 3.
Az eredeti egyenletet csak az x=0 elgti ki.
9.176. xi,2 w 0,8243.
1
9.177. Xj1; Xj - - .
9.178. x l .
1
9.179. e"*'/x a 0,742 857.
9.180. - d x K 3,0529.
X
9.181. Az
sin u
du =
u* \ x^ X
1---- H----- ... I du = X ------- + ----------
3! 5! ) 3-3! 5-5!
338
Ha X= - , akkor /m= - - - = 0,493 05, teht elegend kt tag-
z 2 \ 2 j 3 - V.
0 ^
gal kzelteni, mert az els elhagyott tag ^ 5 lO""^.
9.182.
cosx
9.183.
*shx
/x 0,957 195.
0,1
0, 5
9.184.
arctg X
0,1
9.185. Mivel az /(x) = |x| pros, ezrt Fourier-sorban egy lland, s csak
koszinuszos tagok szerepelnek. A fggvny 2n szerint periodikus. Ezrt a Fourier-
egytthatk:
o =
1
2n ,
|x| dx s t = -
n
|x| -coskxdx.
00 = :: es = -
2 n
Mivel a fggvny az egsz szmegyenesen folytonos s szakaszonknt monoton, ezrt
Fourier-sora ellltja a fggvnyt:
^ n 4 * cos(2A:+l)x
/ W = r - - 1 , ..2
2 7t|k=o (2A:+lr
22* 339
^ ^ ^ , cos 3x cos 5x cos Ix
9.186./(x) = 7T+1- - l cos: v+ +. . . 1.
9.187./(x) = 7C^+12
cos 2x cos 3x cos 4x
3^
e- l
+
___ _ e ^ - l 2 [ ^ + \ e- l ^ e+l
9.188./(x) = -------- 1 I --------- cosxH--------- cos2x---------- cos3x +
n n \ 2 5 10
e^ - l
H------j y - C O S 4 x . . . ).
3 71 6 n f \ 1 1 ^
9.189./(x) = - s h I s h - l ------cosxH----- cos 2x------ cos3x+ ...
7T 3 7T 3 V 10 37 82
2 4
9.190./(x) = - + -
n n
cos X cos 2x cos 3x cos 4x
15 35 63
+ . . . .
9.191. Mivel/ pratlan, ezrt Fourier-sorban csak szinuszos tagok fordulnak el.
A fggvny In szerint periodikus. Ezrt a Fourier-egytthatk:
0
' 3 sin {kx) dx
= -
n
3 sin {kx) dx + , ahol k = 1, 2, 3, . . .
Az integrlst elvgezve kapjuk:
Teht /(x) Fourier-sora:
12 sin(2A;+l)x
/ W = - - 1 0^+1 ~
71 i( =Q ZK T1
y
0
0
Zn -in -n 0 n in 3/r Un X
-Jv
340
9.192. /(x)
9.193./(x)
9.194./(x)
9.195./(x)
_ ^ / s i n X s i sin 2x sin 3x sin 4x
+ ----------- + ... .
.4 18 7T/sinx 2 3 , .
sh - 1-------------sin 2x+ sm 3 x - sin 4x +
71 3 V 10 37 82 145
f i . 2 3 . , 4 N
- s i n x ----- sin2x+ sin 3 x - sm4x+. . .
15 35 63
n \ ( cos X cos 3x cos 5x \
1 /sin X si
+
sin 2x sin 3x sin 4x
+ --------------- + ... .
n
9.196. /(x) = Hsinx----
9.197./(x)
2 3 4
4 / cos 2x cos 4x cos 6x
15
+ + . . .
35 /
1 1 1 /
2 jiV
2 cos X 6 sin X 2 cos 2x ^ 8 sin 2x
9.198./(x) = - ( e * - j r - l ) + -
In n
15
e"+l
15
- cos X sin x+
e ^ - l e^- 2 e"+l e^+l .
+ ------ oos2xH---------sin2x-----^ cos 3x-----r:sin 3x+,
5
n
- + +
4 6
n
+ - ------- 5
10 10
2 + 2;t 47t + 7t^ 2n
cos XH------- j----- sin X + ^ cos 2x +
sin2x-
2 + 27T
_
- cos 3x +
n + TT
3 + ^ l s i n 3 x . . .
9.200. /(x) =
n
+
2 ( l - e ^ 3(1+
---------- sin 2x H------------- sin 3x +
8 13
4(1- e ^ . 5(1+e'') .
20
-sin 4x+
29
sin 5 x +...
9.201. f i x)
_ 5 4 /cos
~ 2 ~
nx cos 37TX cos 57tx
341
9.202.
1 4 cosnx COS 2nx cos 3>nx cos Anx
2^
(
cos nx cos 2nx cos 'inx
^ . 8/sinwA: sin3rtx sin Snx \
8 . 2 0 4 . / W - - ( +
^ ^ / nx \ , 2nx 1 3jc \
9.205.-f(x) = 4 ( sin-------- sm---- + - sin-------- ... .
\ 2 2 2 3 2 )
9.206. / W - ^ sh 1 ^ sin 2:+ ^ sin 3 -
9.207. f i x) = - - I ^cosrtx-sinjix+ -^sin27tA:+
4 n 2n
2 1 1 \
+ 7Cos3ji: - sin3wx+ sin4rx-... ).
9n^ 3re An )
9.208. f i x) = ^ +/^ ^ ^ costtjcH- ^^^cos 7tjc+...\
70. fejezet
10.1. j = In |a:| + c = In |x| + ln \k\ = In |A:x|; = In |x| + 2.
10.2. ;;=0; y --------
JC+ C
10.3. ;^= cx; yp=- 2x.
c 8
10.4. y p -------- .
X X
10.5. x^+y^ = c^; >p = ^20- x^.
10.6. 3x^ + 4( j +l ) 3 = c.
10.7. x^y = c.
342
10.8.y = {x^ + cf ; yp=x^; = (x^-2)^
10.9. y = (x + c)^; yp = (x-2)^.
10.10. j = c ( l + x ) - l .
10.11. j = c(x + 3) + 2.
10.12. / = c(x2-l).
10.13. y=ce*;
10.14./ == l - c ( l - x 2 ) .
10.15. / = - ^ + 1.
x^ - 1
10.16. (x-4)2(1 + 2j) = c.
10.17. x ^ + t ^ - 2 t = c .
WAB. y =
l - c x
^ 0^ 9. x*y^ = ce^.
10.20. (cos x) sin y = c.
10.21. 2^-2^ = c.
10.22. l(F+ 10* = c; ^ = lg( c - 10*).
10.23. + = 2c; yp= - ^(2-e"^*).
' i 0. 24. y = - ; Jp = ^ -
10.25. j = c( x+l ) e- \
10.26. = 2(x-c); In^ = 2(x-2).
10.27. y \ l + x^) = c!
343
10.28. y ^ + 3 y = \ n \ c x ^ \ .
1 1
10.29. >^= -------; Jp = 7: .
e x 1 +
10.30. y = sin (arsh x + c).
10.31. ( 4 - x ) y * = ex.
10.32. jcj = c ( x - !)(>-1); y = 0.
10.33./ ( 1 - c e * ' ) = 1; >^= 0.
10.34.
1

10.35. / = ce^ + 2.
10.36. x + j + l l n i x i - l n l j l = c.
10.37. = In ^
x - 2
10.38. y^ = 3 Arctge* + c; 4yl = 12 Arctg e*-3jr.
10.39. = cy^x*; y = 0.
W. 40. y ( c ] / l - x ^ - 2) = 1 ; y=0.
10.41. y^ = In^ | c x | - 1.
- 1 2x
10.42. y^ = - Arctg +c.
-1
|x^! /
10.43. I n : ^ = X - ^ +c; j=0.
10.44. y = ce^ ; j = 0.
10.45. j = c x ( x - 2) - l .
1 0 . 4 6 . / = c(x + 2)2+1, j = l .
344
10.47. y = c / l +x^; y^^ = 4(l + x^).
10.48. y + x = In (cjc(f+1 )|.
; c - l
10.49. sin^ y = c
10.50. (l + 2yy = c
x + 1
x 2
x + 1
10.51. 2e~>(y+l) = x^ + c; 2e>{y+l) = x^+l .
10.52. = 2 tg { c - 2 ^ \ - x ^ ) .
10.53. In Isin = 3 In lx| +- +c; In sin^ = In x^+ 2,386.
2 x X
1 1 1 1 1 1 . 1
10.54. In |_vl---- = ^ ----- sin 4x + c; In |;^p|------ = - sin 4 x - 1.
y 2 % jp 2
10.55. sin 7
2 _
( 5 - 2 x r '
10.56. j = 2-/c cos X, jp = 2-3cosx.
/
2
/
10.57. y = Arctg c---- ; y. = Arctg
V x j
10.58. Kin c| x^- I I ) = 1; y = ^
2
In |x^ - 11?
^ / 3 ^
10.59. j = V tg ~ l n 2 ( x - l ) + c
3 4
10.60. ( x + l f + ( y - 1)^ + 2 In ( x - l ) ( j ; + l ) = c.
10.61. cosj = c(e^+l); yp = Arccos (e^+1).
f2
4
345
1 0 . 6 3 . 16 In | l n ij + 111 = 2x^ + 2x s i n 2jc + cos 2x + c.
10.64. In Il-2j^l = 4(x cos X- sin x) + c.
10.65. tg^y = c-2l / 3-x^.
1
10.66. cos> = c(e^ + 2); cos jp = (e^ + 2).
10.67. = +c.
cos t
1 0 .6 8 . y = ce*'
10.69. y^ + \ny^ = 2x + c.
1 1
10.70. = c(2x+ 1)^ y^ = 5(2x+ 1)^
2ce^^-6
10.71 y =
10.72. ^
j - 2
10.73. y = cin x.
10.74. (j ^+ l)-c (x2+ l) = 0, OO.
10.75. y^ = 4 c(x^ + 9).
10.76. ny =
10.77. 3 ( x ^ - / ) + 2 ( x 3 - / ) = c; 3{x^-yl) + 2{x^-yl) = 4.
p>
1+e^
l + e
1 0 . 7 8 . / = In c(l + e")^ j ^ = l + 21n
1 0 . 7 9 . / = l - j ^ ; y , = L
X3
10.80. In 1;;+II = y - y + x - l n i x + 11+ c.
346
x-3
10.81. j + ln Ij - 11= + 4x + c.
10.82. y^ + 4y+8 = c(2x^ + 4).
1
10.83. I cos^y =
In c(cos X 1)
10.84. In
y
e^- 5
10.85. 6y \nc
e^+l
10.87. / = +
10.88. y^ = 2 Arctg ^3 tg + c.
10.89. 3 Arctg ^ = 2 (/(x + 3)^ - 6 + c.
1 v+1 ,
10.90. - Arctg----- = e^ + x + c.
2 2
+ 3 x
10.91. y = ce^ +2.
10.92. siny =
10.93. In Ijl = c e ' ^ .
10.94. - y + Arctg j + ctgx = c.
10.95. y ^ - 2 y + l n( y +i y = I nc
10.96. 2v- + 8 = c
2 t g +1
ei2x
( e ^ - i y ( e ^ - 0 , 5 f
347
10.97. cos (In y) = In
10.100. 7 =
dq
jelensget ler differencilegyenlet = - k q alak, ahol k az olddsi tnyez.
dt
10.101. Ha g jelenti a szilrd cukor pillanatnyi mennyisgt, t az idt, akkor a
alak, j
A ^(0) = 5 kezdeti felttelt kielgt megolds q=5e^'^. A megfigyelsnket felhasz
nlva (peremfelttel) A: = 3 In 2 = 2,08. Egy perc mlva = 4,38 gramm
cukor lesz a kvban oldott llapotban.
dy
10.102. A differencilegyenlet = - k y alak, ahol >>jelenti a rdium pillanatnyi
dt
In 2
tmegt s k = . 100 v alatt a rdium 4,3%-a bomlik el.
10.103. Ha jelli a ruhadarab hmrsklett, t pedig az idt, akkor a jelensget
ler differencilegyenlet;
dy
= - k{x- 20) .
dt
Ennek ltalnos megoldsa: y = cg'' + 20. A felttelt kielgt partikulris megl-
_ J _
ds: = 80 2 -I- 20. y{t) = 25, ha ?= 40 perc.
10.104. 60 perc alatt.
10.105. A test s a vz hmrskletnek a klnbsge:
/ 3 \ ' In 55
x{t) = 55 - ; x{t)= 1, ha / = , ^ 7,8 perc.
\ 5 / In 5- I n 3
10.106. Legyen az oldatban t perc mlva x kg s. Ekkor a koncentrci
X
k = ----kg/liter. A jelensget ler differencilegyenlet
dx 3x
dt 100
348
Ennek megoldsa: jc(0 = ce a kezdeti felttelt kielgt partikulris megoldsa
.v-p(0 = Ha = 60 perc, akkor x = 6 - e-^'^ = 0,9918 kg.
10.107. L. a 10.106. feladatmegoldst! x{t)= 10e^'; x(60)= 10e~^ = 0,5 kg.
10.108. A leveg szn-dioxid-tartalma: x(t) = 0,08 + 0,22e ^; x{t) = 0,1
ha ? = 10 In 11 = 24 perc.
10.109. Ha jo jelenti a kezdeti, y a pillanatnyi baktriumllomnyt, akkor a
cilegyer
dsa y = y^^K
differencilegyenlet = ky, ennek ltalnos megoldsa y = ce';partikulris megol-
a) Mivel 2yQ = yQe^'^, ezrt k = s gy y ^ 2 ^ ^ = >>0' 2^ = teht az
eredeti baktriumllomny nyolcszorosa.
10^
b) A = s 4 10"^ = egyenletrendszerbl A: = In 2,jo = = 1250
8
baktrium.
10.110. A. = ky differencilegyenlet megoldsa y = ^ felttelbl
dt
In 2 50 In 3
k = ---- . A lakossg t = ---------79,24 v alatt haromszorozodik meg.
50 ^ In 2
Megjegyzs: Ez a feladat (s mg nhny) megoldhat a mrtani sorozat sszefg
gseinek a segtsgvel is. Ha a lakosok szma kezdetben a, akkor 50 v alatt ez
In 2
aq 2a szmra nvekszik. Ebbl In o = ---- . Ha a lakosok szma x v alatt
50
50 In 3
nvekszik az eredeti ltszm hromszorosra, akkor aq^ = 3a; innen x = ^ , s
ez megegyezik az elbbi megolds eredmnyvel.
10.111. A jelensget ler differencilegyenlet:
d m
dt
= kL.
Ennek ltalnos megoldsa: L{t) = ce^', a kezdeti felttelt kielgt megoldsa:
L{t) = 1-0^'; a peremfelttelt is kielgt megoldsa:
/100 + A '
L{t) = Lo -------^ .
V 100 y
349
10.112. Legyen /?-a vztkrnek a tartly fenektl mrt (vltoz) tvolsga -
dh
a t id fggvnye. Ekkor a vztkr sllyedsnek sebessge---- -, a differencil
ni
egyenlet pedig
dt r^n
0,6 j / ^ R^n'
i f H
Ennek megoldsa, ha Tjelenti a kifolysi idt T = ----- msodperc, ahol a c lland,
c
c =
10.113. A kifoly vz trfogata egyrszt
h^n
dV = nr dh = n(h tg 30) dh = ---- dh.
msrszt
dV = 0,5 0,6 ]llgh dt.
Ebbl a differencilegyenlet
Q,'i][2gfhdt = - ^ dh.
2
a) Az ltalnos megolds: h{t) = ( c - 31,84?).
5 2
b) h{A) = (10^-127,36)5 ^ 8,137 cm.
5
c) m = 0,0314-102 9,93 s.
/ 5 5 \
d) t{A) = 0,0314|^ 10^-42 j 8,93 s.
101114. A hvesztesg
dT dT
= - k A = - I n k x - ,
dx dx
ebbl a differencilegyenlet
dx
2nkdT= - Q - .
x
a) A kezdeti feltteleket felhasznlva
200 10
f dx
dT= - Q .
J ^
30 16
350
340 n k cal
In 1,6 s
illetve Q = 245 000 cal mterenknt s rnknt.
/ 170 16\
10.115. A jelensget ler differencilegyenlet:
m dv mv^
----- = m---------
g dt 16
A kezdeti feltteleket figyelembe vve:
dv dt
16
3 0 0
1,6
Ebbl
v - 4 13 1 7-13e -^'
^ ~ 17+13^-5''
Ha t ^co, akkor v ->4, a sebessg lland.
10.116. A mozg test v(t) sebessgre felrhat differencilegyenlet
~ - e - k V .
ahol g a nehzsgi gyorsuls, pedig a mozg test anyagtl s alakjtl fgg
tnyez. d(0) = 0 kezdsebessg esetn a megolds
fg _ ^ fg
e +e
Mivel
9
lim v{t) = ,
^oo k
ezrt az llts igaz.
10.117. Ohm trvnye szerint
dl
Eo = RI +L- - .
dt
351
A megolds
R
10.118. a) y{t) = -------- . (Az y{t) fggvny neve: logisztikus fggvny.)
1+ ce
A
b) y{t) =
c) t =
In 9(^- 1)
Ac
10.119. A differencilegyenlet: y' = ax. Megoldsa: ^ + c. A grbk para
bolk.
10.120. A differencilegyenlet: y' = - . Megoldsa: y = a \ n x + c. A grbk lo-
X
garitmusgrbk.
10.121. A differencilegyenlet: y' = ay. Megoldsa: y = a'^' ^ c^. A grbk ex
ponencilis grbk.
10.122. A differencilegyenlet: y' = ~ . Megoldsa: y = i/2 {ax + c). A grbk
y
parabolk.
10.123. A differencilegyenlet:y dy = x dx. A megolds:y^ x^ c. A grbese
reg hiperbolasereg.
10.124. A felttel:
\ y dx = \ f i +y' ^ dx.
a a
Ebbl a differencilegyenlet:
y = y ^ - \ ,
megoldsa = eh (x + c). A grbk: lncgrbk.
10.125. Ha a grbe P{x,y) pontjnak s a P pontban hzott rintjnek az
tengelyig terjed szakasza vetlett az x tengelyen b jelli, akkor a szban forg
352
by y . ., y
hromszg terlete k = , tovbb y' = ---- . Ebbl a differencilegyenlet----- - ~
2 b y
2k 2k 4
= . Ennek megoldsa y = ----- . A felttelekbl c = 1, A: = 2. A grbe azy = ------
y x+c x +\
egyenlet hiperbola.
10.126. y = c e ^ - x - 1.
10.127. j; = ce^- 2x+l .
10.128. y - 4 x - 2 = ( y- 4y + 2)ce^\
l + j/3(2x + 3y) ^
10.129.
\ -f {l x+7>y)
10.130. j; = 2tg(2x + c ) - 4 x + l
1 0 . 1 3 1 . y = I n I X + y + 1 1+ c.
10.132. JC+ 2J + 2 = y
10.133. >' = xT]j c-2x.
10.134. y Arctg (x+j) +c.
10.135. 2 ^ y - 2 x + \n { y - 2 x - 2 f = JC+ 4+c.
10.136. ]lAx + 2 y - \ - 2 \ n j/4x+ 2;;- 1+ 2 = x + c.
10.137. - x l n\ c- x\ .
10.138. j; = In
2e
, 2x
c e
- 2 x
X.
y X
10.139. ctg ^ = x + c; y = x + 2 Arcctg (x + c).
10.140.
1+ tg^ 0 - 2x)
10.141. sin 2(x + j ) + 20; x) = c.
23 Matematikai feladatok
353
10.142. x+3y + 21n\x + y - l \ = c.
10.143. Azegyenlet/ = ( 2 x + 3 j - 1)^ alakra hozhat. Megoldsa: 3(2x+3_v1)
= )/6 tgl/6 (x + c).
10.144. y = xl n\ cx\ .
10.145. y = cx^- x.
10.146. x^ + xy + y^ = c.
10.147. y^ + 2xy + 2x^ = k.
10.148. x ^ - 2 x y - y ^ = c.
10.149. Arctg - = Inc ]/x^ + y^.
X
1.150. { ^ - x f ^ ' { ] / 2 y + x f ^ ' = c.
10.151. In icxl+ x = 0; y = 0.
10.152. = c>; y = Q.
10.153. 2 x V = x^ + c.
10.154. lnlx + ;;|+
x-ry
x + y
10.155. \nc\y^ + 2xy + 2x^\ = 4 Arctg-
10.156. y =
10.157. = j = 0.
1 0 . 1 5 8 . = 0; y = 0.
1 0 . 1 5 9 . / = 4x^ Inlcxl.
10.160. = j = 0.
354
10.161. j = ----- ; j = 0.
x-c
10.162. x^ + 2 x V = c; x^ + 2xYp = 3.
10.163. y ^ - x y + x^ = c(x + y); 3(x + yp) = y j - x y ^ + x^.
x^
1 0 . 1 6 4 . / = ;--------- ; 7 = 0.
In Iex I
10.165. x'^ + Axy^ = c.
10.166. 5 / x + 1 0 ; ; V + x5 = c; 5y*x + lOy^x^ + x^ = 1.
10.167. j; = XIn^ |/cx; y = 0.
10.168. j = x ( c | / x + 2)^; y = 0.
10.169. j = Xsin (In |cx|).
10.170. 2y = xsh(lnlcxl).
10.171. j = - X In IIn Iex 11.
I n -
X
; c = e; y = - x l n | l - l n x | .
10.173. y = 2x Arctg ex; yp = 2x Arctg x.
10.174. y = X Arcsin I In
/ I y \
10.175. In Iex 1= ctg I In -
\2 x j
10.176. = x(ee^-l).
V y ^ c
10.177. sin - + C O S - = e^ln .
X X X
10.176. y = X Arcsin ex.
23* 355
X
10.179. Most az - = / helyettests alkalmazsa a clszer. A megolds:jc + 2ye^ = c.
y
10.180. Legyen a keresett grbe egyenlete: y = y{x). A grbe tetszleges P(x;y)
pontjnak az origtl mrt tvolsga; ^x^+y^. Az rint egyenlete: Y - y = y' (X-x) .
Ez az ordinta tengelybl y - xy' hosszsg szakaszt metsz le. A differencilegyenlet:
]/x^ + y^ = y xy'.
A megolds: 2cy = c^- x^, a grbk parabolk.
10.181. j = ce"-l.
10.182. y = ce"-2x.
10.183. 7 = c e ^ - x ^ - 2 x - 4 .
10.184. y = ce^- 2e~\
10.185. y = ce^ + e ^ ^ - x - 2 .
1
10.186. j = ce"- (sin x + cos x).
1 , 1 , 2
y
10.187. y = ---- e = ce^ + ch2x-----e
2 6 3
10.190. y = (c + x)e^.
( x \ 1
10.191. y =
10.192. y = = c'^^+0,3cosx + 0,l sinx+2.
10.193. i = ce^- ^(3 sin 2 + cos 2).
356
10.194. y = Y+y = l ^ 2x_ 1 - 2x+ 2_^+ 1
^ 8 4 2
10.195. A jelensget ler differencilegyenlet:
dl
L ---- R = i7 sin (t.
dt
Az ltalnos megolds:
U
+ L^o
A felttelnek eleget tev megolds:
I = ce ^ {R sin cot coL cos cot).
- 1-t
I = ;-----r {(oLe ^ + i? sin ct/ - coL cos a>t).
10.196. y = - + 1 ; V = - + 1.
X ^ X
(A differencilegyenlet megoldhat a vltozk sztvlasztsval is.)
10.197. :>; = cx^ + x^.
10.198. y = ( x - 2 f + c(x-2).
10.199. 7 = 2 - ce (A differencilegyenlet megoldhat a vltozk sztvlasz
tsval is.)
10.200. y = x(ce~^-l ).
10.201. 7 = - ( c e "+3(x-I)).
X
10.202. y = ce^^ x ^ - 1.
10.203. j
10.204. >; = - I n - ; y ^ - - l n ~ .
X X ^ X X
10.205. ( x- l ) ^y = x - l n |x| + c.
357
10.206. y = ce ^ + 1; j p=l .
10.207. 2y = x^ + 6x^- 4xl n\ x\ + cx.
10.208. y = ( 2x +l ) ( l ni 2 x +l j + c) +l .
10.209.}' = "^(c + l n l x - l D + x - l .
X
10.210. >>= ( l + x 2)(c + Arctgx).
c 2 X r- r-
10.211.;; = + ^ V - - + ^ - l n | | ^ + l |
3 2
10.2 1 2 . y = e^(c+\nx).
+ c .
10.214. :f= - c( e^+l ) - ^(e^+l )-^.
10.215. >^= (In |x| + c)e
10.216. y ^ e^ + cx^.
10.217. y = ?*(^c + ln | x| + y
10.218. y = x^ +
X/
/
c 2 i~
10.220. y = - y ^ + -xl/^.
358
In X c
10.222. y = ---- + ; .
2 In X
10.223. y = c \ n ^ x - l n x .
C X l X
10.224. j = ------- In - + -
X 2 X 4
10.225. - +ln^ x - 2 l n x + 2.
X
10.226. y = cx^ + \ n x ^ - x + 1.
( ^
10.227. j ^ = ---- l -c)l nx; y ^ ^ - x ^ l n x .
\ 2 J 2
10.228. 3; = x ^ ^ + 1).
10.229. j = c sin Xcos x; yp = cosx.
10.230. y = c sin X 1.
10.231. y = x(sinx + c).
(x + c) cos X
10.232. jF= ^ ------ .
1+ sin X
10.233. j; = ce-^^ + t g x - l ] y^ = e - "+ t g x - l .
c + cos 2x
10.234. y = ---- ---------- x ctg x + 1.
sin X
c sin^ X
10.235. y = ----------- + tg X.
3 cos X
10.236. 2r sin^ 0 + sin'*^ 0 = c.
- i 2
10.237. y = c(sinx) -sinx.
c - cos X
10.238. y = --------+s mx.
1- X
359
10.239. y = -------- l-(l - x ) tg X- 1.
cos X
10.240. y = (x^ + ex) cos x.
10.241. y = x(sinx + c). Valamennyi integrlgrbe illeszkedik az origra.
c + 2x sin 2x + cos 2x
10.242. =
10.243. y =
4(1+x2)
c Xeh (1 - x ) sh (1 - x )
x^
1-
10.244. y sin x+ = c; y^ =
sm X
'1 1 ^
10.245. y = e~"^( - sin 2x+ - x + c
1
/
10.246. > = ce-^^'* + Arctgx-l.
10.247. y = (c + XArctg X In |/x^+ 1) (x+ 1)^.
10.248. y = [/x + X Arctg 3x - - In (1 + 9x^)^ .
10.249. Ha a differencilegyenletben a vltozkat egymssal felcserljk, akkor
az j egyenlet lineris. Ennek a megoldsa:
x^ . n ---------
y = CX+ , az eredeti egyenlet: x = cy + , vagy y = k fk + 2x.
10.250. Ha a differencilegyenletben a vltozkat egymssal felcserljk, az j
egyenlet lineris. Ennek megoldsa: y = (c - cos x) sin x. Az eredeti egyenlet:
X= (c cos y) sin y.
x^
10.251. y = cos X + C1X+ C2.
6
1 1
10.252. j +C1X + C2; jp = -(e^^ + 2x^ + ( 3- e^) x- l ) .
10.253. = X In Icjxl - x + Cj; yp = x In |x| - x + 1.
360
10.254. y = --------------hciX + C2 -
X
1 + t g -
2
10.255. y = (In x^ 3) + CiX + C2 -
= j ( l n x ^ - 3 ) + ----- In4 X------ +ln 4.
4 / 4
10.256. j = XArcsin x + |/l + CjX + C2;
yp = x Arcsin x + ]j\ x^ + - x + ;r - 1.
10.257. y = In--------hc,x + C2 -
x+ 1
In^ X In^x
10.258. +C1X+ C2; yp = ^ - + x .
qt^ qt^
10.259. = +C1 + C2; p = +t?o?+5o.
10.260. A testet mozgat Fr a test G slyereje lejt irny G sin a komponens
nek s az Fj, srldsi ernek a klnbsge. a G slyernek a lejtre merleges
G cos a komponensvel (a nyomervel) arnyos. gy a mozgst ler differencil
egyenlet
F = (j sin a kG cos a,
illetve
cPs
7 = G(sin a - ^ cos a)
at
alak. Ennek ltalnos megoldsa
s = (sina-A:cosa)^ + Ci+C2.
Mivel / = 0 pillanatban 5 = 0 s t) = 0, ezrt a kezdeti feltteleket kielgt megolds
s = (sin a cos a)/^.
361
Ebbl a feladat adataival:
t =
100
9,81(0,4226-0,3625)
13 msodperc.
10.261. Ebben s a kvetkez hsz feladatban az y'=p{x), y"=p' helyettests a
clszer. A megolds:
y = Cix^ + C2 .
c,x^
10.262.j = y + ^ + c ^ .
X x^ c,x^ 1
10.264. y = ^ (3x^ - 4x3+ 12cix2-24ciX + 24^2);
yp = ^ (3x^ - 4x^ - 36x^ + 72x + 8).
^0. 265. y = -
1
X----- In |CiX+ 11
+ C2\y = k i i y = +k2-
10.266. y = Ci(x-e~^) + C2 .
(Cj x)3
10.267. J = y = L
10.268. y = CjX^ + ln Ixl + Cj-
10.269.j = -l^(cix+ 1)3+ C2; y = x + k.
3ci
3 /
10.270. j = Cl ( + x +C2 ; yp = x^ + 3x+l .
1 0 . 2 7 1 . j = 2 ( x - I n i 1 -
10.272. y (x + Ci)e^ + C2 . (Ez a feladat a lineris inhomogn differencilegyen
letekre alkalmazhat mdszerekkel is megoldhat.)
10.273. j = 2ci |/x+C2; yp = 2 { ^ - \ ) .
362
10.274. 7 = (ciX + cf)e^'"Vc 2; y = k-, y=^- x^ + k^.
10.275. y = Arcsin x^-C2
10.276. y = + C2 cos X X sin X.
10.277. y = c j ( arsh - ^= + ]/1 + + ^2-
V 1^ |/2^ r 2c J
X
10.278.3^= iArcsin---- I-C2; J = c; jp = -Arcsinx+1.
10.279. >^= 1)In 11+^x1-A:x+c, ahol k = tgci.
10.280. Ha y'=p{x), akkor az egyenletbl
^ 1
^ cos x + c
Ha |c| < 1, akkor a megolds
ha lel > 1, akkor a megolds
X
hac =l , akkor = tg - + A:,
X
ha c= - 1, akkor = ctg - + A:.
Megolds mg y=K.
^0. 28^. y = yp = e^*^.
10.282. = (x + C2f + Ci; yl = x^+l .
363
10.284. = - ( x + C 2 ) ^ + - .
4 Cl
X+Ci
10.285.;^ = ^ , y = c.
X-hC2
10.286. c,y^ = [2c,{x + C2)? + \.
10.287. X= j+Ci In I7 I+ C2; y c.
y
10.288. In
10.289. In
j + 2
l + 2y
2^2(CiX +C2)
= 2(ciX + C2 ) vagy y = ^ 3^="^-
= 4(ciX + C2) vagy y = - ; :>^= C-
l - 2 y
10.290. 1) = C1X+ C2; y = c
1
10.291. y = tg(C1X+C2); y = c.
Cl
10.292. = Ci sh( 1/2X+C2).
10.293. Mivel a differencilegyenletbl mind az x, mind az y hinyzik, ezrt ktfle
helyettests is alkalmazhat. A megolds: y = Arcsin Cie* + C2-
10.294. Arcsiny + y ' ^ \ - y ^ = c^x + cx', y = c.
10.296. y = eh Ci(x + C2 ).
Cl
10.297. I n j = + 1.
C1X+ C2
10.298. X= 1 ^ - y In 2 | | ^ + Cil+C2; y = c.
10.299. j = In/ j = c.
1
364
10.300. A ktl egyenslyi alakjt ler differencilegyenlet:
alak, ahol y a ktl fajslya, V pedig a ktlre hat vzszintes irny (lland) er.
A differencilegyenlet ltalnos megoldsa
V y
y = - c h - ( x + c i ) + c2 .
y V
A V, Cl s C2 llandk a felfggesztsi pontok helyzetbl s a ktldarab hosszbl
hatrozhatk meg.
10.301. j = Ci + C2e^*.
10.302. y = +
10.303. y = C^e^^ + C2e~^^ s ez felrhat d0. y sh {Ix + k^} alakban is.
k
10.304. y = Cl cos 3x + C2 sin 3x. Ez felrhat 3; = ^ sin (x + ^2) alakban is.
10.305. y = Cie^^ + C2 e~^^.
10.306. y = Cie^^ + C2 e^^
1 +| 5 1 -fi
^O.ZO^.y = c^e ^ *+ C2 e ^ x.
10.308. y = (Ci + C2 x)e~^.
_ 4
10.309. y = (Ci + C2 x)e
10.310. y = cos 5x+C2 sin 5x).
10.311. j = cos j/sjc + cj sin i/5jc.).
10.312. y = cos 2x + C2'Sn 2x).
10.313. y cos J/Sjc + Cj sin
Vq
10.314. y{t) = Cl cos cot+ Cj sin cot; F() = sin cot.
0)
365
10.315. a) y{t) = +
b) Ha k>co, akkor
21/ F ^ ^
Ha k = co, akkor Y(t) = Vote~'.
VLtLk<0), akkor Y{t) = e~'"* sin t ^co^-k^.
^or - k^
10.316. y = Cie^^ + C2 e^^ + 2.
5
10.317. j = Cie-^^ + C2e-^^ + 2 x - - .
10.318. y = Cie^^ + C2e~^-x^ + x -
10.319. = (ci + c2x)e^* + x^ + 9x2 + 40,5x+81.
10.320. y = e~^*(ci cos Sx + Cj sin 5x)+ - 2 x + l.
10.321. y = Cl+ 2 6 - x ^ - x ^ + 4x.
6
10.322. = Ci^^ + Cje * + 2 cosx.
10.323. y - (Ci + C2x)e^' + 0,24(4 sin x + 3 cos x).
10.324. y = ci cos 2x + C2 sin 2a: + sin X - 2 cos x.
10.325. y = e^^(ci cos x + C2 sin x) + 2 cos 2x 3 sin 2x.
g
10.326. y = cos 4x + Cj sin 4x) - cos 4x+ - sin 4x.
l
10.327. y = Cl c o s x + C 2 s i n x + - x c o s x .
(66
10.328. y = Cl cos Sx + Cj sin 3x---- xcos 3x- .
6 9
1 1
10.329. y = Cl cos 2x + Cx sin 2x---- cos x+
6 8
2 16 . 10
10.330. y = cos4x + C2 sin4x) cos4x+ sin4x,
10.331. y = Ci6~^^ + C2 C~^^e^.
10.332. = (Ci+ 2x)e^* + 2e^".
10.333. y = (ci + 2x)e^^ + (c2 x)e^^^.
10.334. y = (ci + c2x-2x^)e^.
10.335. y = (ci + C2 x)e^ + e^^ + x^ + 4x+6.
10.336. y = (cj + CjX + + X +
10.337. y = e *(cj cos2jc + C2 sin2x) + 2e * + 2x
10.338. y = Cie-^^ + (c2 + x)e^^- -e-^\
10.339. y = (Ci + C2 X + 2x^)e^ + e-\
_ 5 3
10.340. >^= (ci + x)e^^ + c2e ^ e ~ ^ ^ - x + - .
2 j 2 2
/ 1 \ -
10.341. y = C1 + C2 X + - x^ e^ + x+4.
V 8 /
1 1 8
10.342. y = e^"(ci cos4x+C2 sin4x)- ^
10.343. ;; = ci + (c2 + 4x)e^^ + e~^^-8x.
367
10.344. y = Ci^ + [ Cj - j V ^
x\ , / x \ , 1
4
10.345. y = [ Ci + - e^"+ \ C2 ~ - ] e ~-
/
4 / 2
10.346. j = Cl +( C2+ -
x \ , 1 , X
------------- .
32 8 8
/
10. 347. y = {c, + c , x + j y ^ - ^ .
10.348. j (^C2+ )^" -
10.349. = cie* + (c2-x-2x^)e^*.
/
10.350.7= Ci + x + )e* + C2^ *
10 3 5 1 . j = (ci + c2x + ^ y ^
/ x\ . 1
10.352. 3; = Cl cos 2x+ C2---- )sin2x + x + - .
8/
10.353. y cos 2x+ ^C2+ ^ sin 2x+c*.
10.354. y = e^*(ci cos x + C2 sin x+ 1) + - (sin x + cos x).
8
. ( X X \ x l . x 32
10.355. ;; = c^ cos - +C2 sin - +10 1+ cos - - 2 * 2 Y7 '
10.356. V= c,e^* + C2e'^*H----- (5 sin X - 7 cos x).
74
10.357. j = f ^ + Ci 0,l sin X-0,3 cos X)e^*+C2e
368
10.358. j = ( Ci + C2x ) e * + sin x ( x 1) cos x.
10.359. y = + C2 e"-i (xcos x x sin x 2 sin x cos x).
V F
10.360. y = sin tH------- sin a>J.
0) m{(o^ - col)
A ltrejv mozgs kt lland amplitdj, de klnbz frekvencij rezgmozgs
eredje. Ezt a mozgst modullt rezgsnek szoks nevezni.
10.361. = x ^ - l ;
10.362.^2=
4
y = CiX + C2 (x^~ 1).
_ 1
y = CjX + CjX
10.363.3^2 =
10.364. 3;^ = - x ;
10.365. = - j - ;
4x
y = C1X+ C2 + 1.
y = CiX^ + CjX.
y = CjX^ + C2 - .
10.366. y2 = x + 1 - - - ln(x+l)^;
X X
1+ -
X /
+ (?2 1~ (^+ 1) In (x+ 1)^).
X
10.367.^2 = -
1
e^+1
y = c , ( e ^ - l ) +
e^+l
10.368. j2 = -
2x
10.369. y2 = sin^ x;
xy = Cie +C2 .
y = Cl sin x + C2 sin^ x.
10.370. :>;2 =
1+ sin X
In----- ------------sm X
cos X /
sm x;
, 1+ sin X
j = I Cl + C2 m-------------C2 sm X ) sm x.
cos X
24 Matematikai feladatok
369
10.371. }>2 = - s i n x ^ ; y ~ Cl cos + C2 sin x^.
10.372. = 1- x c t g x ; y = Ci ctgx + c j l - x c t g x ) .
10.373. 3^2 ~ 1- 3; = Cl Arcsinx + c2
10.374. j = - (Ci sin x+ C2 cos x).
X
y g-xHc,+C2X)
10.375. ;;2 = ;
10.376. Y = COS X+C2 sin x ; = sin x In
k2 = - sin Xcos x; y ^ cos x + C2sin x - (cos x) In
10.377. Y= 2 X6 ''; 7 = - ^ ^ l n ( l - x ) ; y = [c^ + C2 X \n{\ x)]e''.
10.378. F = (Cl + C2x y ; = (x Arcsin x + - x V * I y = y+yp-
10.379. r = cic^* + c2c";
1
yp
2 c* + 1
; y = Y+yp-
10.380. Y = e *(Ci cos 2x + C2 sin 2x);
1 _ 1
fi ^(^cos 2jc'>In lcos2xl + ^
yp = - e ^(cos 2x) In Icos 2x| + - xc "'sinx. y = Y+yp.
11. fejezet
11.1. 3612-flekppen.
11.2. 40 320-fle.
11.3. 63.
11.4. 10-fle.
11.5. 3432.
370
11.6. a; 26 730 600; ; 4 455 100.
11.7. x = 4.
11.8. 1061 520 150 601.
11.9. 20.
11.10. 1 391 280.
11.11. 465-flekppen.
11.12. 90-flekppen.
11.13. 52-flekppen.
11.14. 459.
11.15. 200-flekppen.
11.16. 9-fle.
11.17. 171.
11.18. 31 104.
11.19. 1800-flekppen.
11.20. P{A + E) = P{A) + P{E) - P{AB) alapjn, B helyre a 5 + C esemnyt rva:
P{A + B+Q = P{A) + P{B+Q-P{A{B+Q). Mivel P{B+C) = P{B) +P{Q-P{BC)
s P{A{B+C)) = P{AB + AQ = P{AB) + P{AQ- P{ABQ, P{A + B+Q-re a bizo
nytand lltst kapjuk.
804
11.21. .0,713,
850
11.22. . 0,694,
28
11.23 . 0,233.
120
11.24. a) 0,1512; b) 0,00 021; c) 0,005 005; d) 0,0108; e) 0,00 514.
24* 371
11.25. a) 84; j ^ ^ 0,119; cJ ^ 0,119.
12 o4
i m
57 344
11.27 . 0,285.
201 376
91
11.28 . S 0,421.
216
378
11.29 . 0,0461.
8192
1221 759 28 096 200
4 3 ^ 4 3 ^
81 900
- 0,000 0) 137; W * 0,0 OOO220.
11.33. kb. 0,294.
9
11.34. = 0,375.
24
360 216 1
11.36.a; 0 , 2 7 8 ; . = 0.167.
11.36. ^ = 0.1.
11.37. ^ = 0,65 625.
64
85
11.38. 0,350.
372
307
11.39. 0,600.
11.40. 0,0583.
720
112 112 64
S 0,156; b) 0,933; c) 0,533. d) Biztos esemny. P= 1.
^ 720 M20 ^ 120
11.42. 36-0,22-0,8 0,242.
11.43. 0,88^+10-0,12-0,88 0,658.
600
11.44. 0,300.
2002
11.45. 0,1008.
11. 46. 1 = 0.7.
11.47. = 0,875; b) = 0,64; c ) \ + Mn4 0,597.
8 25 4 4
11.48. - = 0,25.
4
11.49. P{B\A) K, 0,158. Ennek jelentst a kvetkezkppen lehetne szemlltetni.
A kiszemelt jtkos ltja, hogy 3 piros lapja van. rdekeln, vajon mennyi a valszi-
nsge annak, hogy az utna kvetkez jtkosnak is 3 piros lapja van? Ez ppen a
P{B IJ) valsznsg.
11.50. P(^) = ^, P(B) = ^, P(AB) = ^, P(A + B) = - ^.
11.51. 0,2646.
11.52. 1 = 0,1.
11.53. kb. 0,967.
373
11.54. F(x) =<
0, ha - 00 < X^ - 1;
^(x+l)^ h a - K x < 0 ;
l - ^ ( 2 x - 3 ) ^ h a 0 ^ x < ^ ;
3
1, ha - ^ X < 0 0 .
8- 1/8
11.55.---- ^ 0,739.
11.56. a) a =
b) M( 0 =
7T-3 |/3 + 6
4n-3]/3
1,521;
8 ( 7 1 - 3 1 / 3 + 6)
0,2335.
11.57. a) a =
11.58. a) a =
6e^ - 38
0,8418; b) MiO = ~ 1443.
2e^- 6 - 3
8 4e^ e^+l
---------- 0,9536; bj M( 0 = . . . . 2,177.
^2+ 1 - - 3^2+1
11.59. M(0 = 218,95; 0,8646.
11.60. M(<^) = 4. A valsznsgi vltoz szrsa nem ltezik.
11.61. D(0 =
374
0,6837.
11.62. ; P = -^ ,^ 0,583; b ) f { x ) = \ ^ ^ ' ^ 0 < x < 2;
0 egybknt.
11.63. P = H{a, b, c)-H(a, b, 0)-H(a, 0, c)-H(0, b, c) + H(a, 0, 0) + //(0, b, 0) +
+ H(0, 0, c)-H(0, 0, 0).
11.65./(xl j) = J
1_ _ Z
2 3 2
------ , h a O<j <3 s O < x < 2- - y i
1- -
\ 3/
2
0, ha 0 < j < 3 , de x <0 vagy x > 2- - y ;
nem rtelmezhet egybknt.
11.66. S 5,833.
6
49
11.67. 4,083.
64 880
11.68. M((^) = 380,- Ft, a feltteles vrhat rtk pedig Ft
175
370,74 Ft.
11.69./(xl) = j 2 5 0 ^ ^ ^ ^ ^ ha 75^x<100;
Lo egybknt.
= 85.
11.70. = ^ = 0,1875.
11.71. M([^ - M{0?) = M{e) - 3M(^)M(<^) + 2 M\ 0
0, ha X< 1;
I
11.72. F{x) ^ ( x - 1)^, ha l ^ x ^ 4 ;
1, ha 4 <x.
2 + 3]/2
3,121.
375
11.73. Valsznbb, hogy a mintban 2 db msodosztly termk lesz.
P(<^ = 2)^0,209, 1)^0,106.
11.74. Csekly mrtkben az a valsznbb, hogy a mintban legfeljebb egy srlt
termk lesz. P(<^ = 2) = 3 0,9^3, 1) = 3,06 0,9
11.75. kb. 2,07%-a.
11.76. O,38 + l/0U3 0,741.
11.77. 20%.
11.78. kb. 0,751.
11.79. 6.
11.80. 26.
11.81. 5.
61
11.82. a) K 0,151; b) 1- 2 4 4 , 6 0,394.
e*'
11.83. 3,6^-^' 0,267.
11.84. l - l , 25- e- ' 25 0,0265.
40
11.85. 0,238.
168
11.86. a) 45; b) 90.
i
11.87. J j 0,577; b) l - e ~ ^ 0,865; cj 20(1)-1 x 0,683.
11.88. A feltteles valsznsg defincija s x>0 alapjn
P(^ > X+Xo)
P(^ > X+Xol(^ > Xo) =
P(^>Xo)
Mivel az exponenciUs eloszls valsznsgi vltoz eloszlsfggvnye F{x) =
= 1 ha x>0, ezrt P{^>x) = 1-P(x) = ha x>0. Tekintve, hogy x>0
376
g-Mx +x,)
s Xq> 0, most mr csak azt kell beltnunk, hogy Ez pedig nyilvn
igaz.
11.89. kb. 18 420.
11.90. a)
2-]/2
100% 29,3%; b) kb. 23,2 v.
600
11.91. a) e ^0,0995; b) 260s-ln2 180s.
11.92. 0,045.
11.93. T3(2,5)0,456.
11.94. a) 1- 0( 2) 0,0228; b) 2<P(0,5) - 1 0,383.
11.95. 1-P(0,25) 0,4013.
11.96. 8 mm.
11.97. kb. 0,475.
11.98. a) 2^ ( 1,5)1 0,866; b) kb. 3,92 mm-es pontossg.
M\ i )
11.99. w = In
11.100. kb. 0,564; b) kb. 0,290.
11.101. Legfeljebb 0,16 valsznsggel.
11.102. Bernoulli ttelbl kvetkezik, hogy
1 +
M \ 0 \
- P
> e <
fin
000, n = 300 000, = 0,49. A javaslat elfogadshoz p>0,5 kellene, ehhez
n
viszont >0,01 szksges.
377
I
1 1
4e^n 4 3 10 120
teht 1%esly sincs arra, hogy a javaslatot megszavazzk.
1 2 . f e j e z e t
12,1.
' A i
30
0,01,
0.0A
0.03
a02
ObOl
15
r
21
J L

1
700 800 900 ^ 100 1200 ISOO 400 1500 1600 OOw|(Wh]
12.2.
fi
6
*.10']
200 300 400 900
378
12.3. = 99,71 g , 4 5 = 1 0 0 g .
90 95 100 105
12.4. <f-I,85, S 14,36, c0,3198.
12.5. kb. 0,42. (A Poisson-eloszlst kzeltse normlis eloszlssal.)
12.6. (243,5 cm^ 248,5 cm^).
12.7. kb. 286,5 .
12.8. kb. 70 ezer Ft
12.9. (0,240; 0,306).
12.10. 110 s 570 kztt
12.11. A 3-as szm esetben 99,8%-os, az 5-s szm esetben 99,2%-os megbzha
tsgi szinten.
12.12. (3,65 cm; 5,93 cm).
12.13. (4,60 cm; 7,46 cm).
12.14. 99%-os szigniikanciaszinten a hipotzist elvetjk.
12.15. a} Elfogadhat, b) /0,284. A prba ereje 0,716.
12.16. Legfeljebb 18,85 g eltrs esetn.
12.17. > Elfogadhat, b) Nem fogadhat el.
12.18. A feltevs elfogadhat.
379.
12.19. a) A szrsok 98%-os szinten megegyeznek, b) 95%-os szinten a mellker
letek vrhat rtke megegyezik.
12.20. A Welch-prba alapjn elfogadhat az a feltevs, hogy a ktfle mdszerrel
kapott mrsi eredmnyek vrhat rtke megegyezik.
12.21. Legalbb 16,5 g-os eltrs szksges.
12.22. a) kb. 0,0457; b) kb. 0,0486.
12.23. A hipotzist 95%-os szinten el kell vetnnk. (A kritikus rtk 112,0.)
12.24. A hipotzist 95%-os szinten el kell vetnnk.
Megjegyzs: Elfogadhat viszont 95%-os szinten az a hipotzis, hogy az adatok
olyan normlis eloszlst kvetnek, amelynek vrhat rtke 1120 kWh, szrsa pedig
145 kWh. Ellenrizze!
12.25. A hipotzis 95%-os szinten elfogadhat.
12.26. A hipotzis 95%-os szinten elfogadhat.
12.27. A testmagassg s a testtmeg a vizsglt csoportban 95%-os szinten fgget
len valsznsgi vltozknak tekinthetk.
12.28. 99%-os szinten elfogadhat az a feltevs, hogy a ktfle hiba elfordulsa
fggetlen egymstl.
12.29. Nem fogadhat el.
12.30. r0,1971.
12.31. >>= 0,6555x + 43,36.
12.32. V, = l + (3,57-l,779 + 0,52 9482)- 10-5, ahol t a C-ban mrt hmr
sklet, V, dm^-ben addik.
380
13. fejezet
13.1.
13.2. a) x^+y^ ^ l, teht az (x; y) skban egy orig kzppont, egysgnyi suga
r zrt krlap pontjai; b) x^+y^ > 1; c) x+y > 0; d)
e) 2kn ^ x^+y^ ^ (2k+ l)7r; A: = 0, 1, 2,... koncentrikus krgyrk.
13.3. aj x^+y^ + z^ ^ 1; bj x^+y^ < z^; c) P ( x ; j ; z ) e R ^
13.4. a) Kz x +y = 0 egyenlet egyenes pontjai, b) Azx=Os az >>= 0 egyenlet
egyenesek pontjai, c) Az x^+y^ ^ 4 zrt tartomny pontjai, d) Az x = 0',y = 0',z=Q
egyenesek pontjai.
13.5. a) x^+y^ (z=c^O); b) z )/5x; c) Kz = |/5xegyenesz tengely
krli megforgatsval keletkez forgskp.
13.6. a; 3 x + 2j -12 = 0; 2x + z- 8 = 0; 4j + 3z-24 = 0; 3x + 2 y - 9 = 0\
b) 4x^+y^ = 4; 4z^ + 4x^ = 4; 4z^ + j^^ = 4; 4x^+y^ = 2; cj x^+y^ = 4;
z = 4 - x ^ ; z = 4y^', x^-^y^ = 3.
13.7. a) \-,b) 0; c) 0.
13.8. a) lim ( lim ) = 1; Hm ( lim - ^ ) = - 1.
x ^ o \ y - ^ Q x y j y ^ o \ x ^ o x y j
y ( Y \
hm I hm sm-------
c-ao \y-oo 6x- ^yJ
b) lim
n x \
= 0; lim lim sin-------
y^co \x^co OX+ J /
1
2 '
13.9. a) lim Arcsin- = ArcsinO = 0\ b) lim Arctg- = Arctg 1 = - ;
/ 1 \ 1 2
z -
\
c) lim
p
z-l
pontokban.
Ix^+y'^
= 1- = - , azaz a fggvnyek folytonosak az adott
381
13.10. a) fj^x\y) = l x - \ 2x y , f ^ x , y ) == -6x^ + 3 / ; IJ/;(x;>-) = - }f p. ' , y) =
yc
= Inx; c) f:i x;y) = (cosx)!nj;+ fy{x\y) = ~ - (sin>) Snx; d) f . l x;y) =
X y
= = x^^', e) f ' x, y) * f' yix\y) = />* =
= 2 *(ln2)^;/y(.v;>') = 2 *(ln2)^- t = --(sin/^;^In>^;/;(x;>) -
= - (sin J W ; h) f x; y) = - -7- ; y) -
13.11. a) f j (x;y\ z) = Sx^-Sjz;//x;y;z)-= 3y^-3xz; f^(x;y;z) ==Sz^-S.yj^;
= > + z ; M x ; y ; z ) x + y ; c ) f j . x ; y ; z ) -
= ^ ; / ; ( ^ ; ; ' ; z ) = ^ ; . / l ( x ; . y ; z ) = ^ ; e ) f / x ; y ; z) = zx'~^y^; f ^ x ' , y , z ) ^
= z / V ; y;(x; >>; z) = (xj)^ In x>; /J /;(x; y; z) = z*> In z; f ^ x ; y; z) =
= z*^xlnz; f Xx\ y; z) = xjz*"^; /L(x;>';z) = -x* f y{x\ y;z) = ^-^--x*;
z z
vl nx
f Xx- , y, z ) --------
13.12. 2; 0; b ) A , - 1 ; c ) ^ - - ^ - .
13.13.^.
13.14. a) d z = p^ p ( - ^ d x + -4/f); b ) z = y ^ x ^ ^ d x + 2j>x^*(lnx)d x \
l ^ x - ' ^ y f \ p ^y /
JUL / I 1+ \
c) fe = 2* ^ In Tilxy dx + x^ </>); d) dz = ^ Ti a /
\ ( i - w ( i - x j r /
(^ln(x+>')+ (xln(x+>')+
= (2 sin X cos X + cos y ) d x - x(sin >) dy, g) dz = ^2x Arccos + jp=:^=r=|:^ dx+
e) dz = In (x+j)
; / > dz
x^
y 'jx^y'^ - 1
382
dy.
IS.IS. a) du ~ e^^^iyz dx+xz dy-^xy dz}',
i y X xy \ xy
b) dii f ' dx -I dv---- r /? 1 cos ; c) du
\ Z 2 2^ / Z
dx + dy+d2
x +y + z
13.1. a; - p = I ^ ^ 2 = ; b) 6*[3cos(/+/2)-(/ + 2/2)sin(/ + /2)];
dt | } - (3/-4/^)2 dt
I . 2,-4cos2,); J) f -
1 3 . 1 ? . = I 3/2 + 2 + COS {t^ + 2 / - I + f i) ;
dt \ 2 \ l /
' h i ' ^ s h i c h ^ ^ t \
^ dt / c h ^ - r + f * 1 \ s h ^ t e h * t ^ 1 - r * s h / e h r /
I i . 1 8 . &) " ' - = I - - - - - - - - I - -^-7 + . j s i n v ;
ilx \ v ^ w )
du ( w y X V Y
b) y - e ^ ' * ( - ~ ^ + r V " ~ ) ;
v \ x + j - - z /
dns
* 4. p V i n x ) ;
Sii
Ji
l a j s . *> =
2?w''"
--------------
X ' ^ y - z '
c my - z %mx
j s n t - c o s x '
ve
rf ; 4 - 2
.vz+xlisr
js2 te j V x ^
In (2x+j>')+
(2x +>')2 ln*(2x4->;)
c o s z - x s m >
> s i n z - c o s x
xe
-xy
e ^ - 2
yz In X+2 ^
yz In y +x y '
l n(2x+y)+l
(2.V+ v)* ln2 (2x+y) '
383
13.20. a) 2 - 2 ^ - b) - j
f c / 3 ^
+ l ; c) -
8+17e
lOe^
d) -
2 f i
3 + 41^ 25 1
3 In 10 16
13.21. aj - 2 |/2;
13.22. aj - ^ ;
13.23. aj - +l/3;
13.24. a; - 1 ;
(1;3).
(2; 0).
(5; 2).
13.25. aj
l/2(8r t - l )
13.26. a; z;
bj z;
dj K
24
Po
bj
1-1/3
4^^3-l
|/2 [/27r
T T T ; -
grad z
gradz
gradz
G4)^
_ j_ j4
~ 3~ 3~
Po ]! ][A
1 /l^;r
= - - l n 2 (
2 V 4
+ 1 ) ; gradz
io
71
= I - I n 2; - I n 2
e) <Ifo = +
2 + ^^3
/ ; <
gJ zi
/i; z;
Po
10
_ 1
7
gradzip, = (2e ; 2e);
gradz
gradz
. . - m
1 -2)/3'
Po
7 14
_j_
grad|p = (0; 1).
384
1 3 . 2 7 . = /;,!,= - 5 -
13.28. a) f% = /;^ = - e ^ s i n y - e ^ c o s x ;
b) /x. = / ; . =x^- Hl +3^1nx) ;
c) f l y = / ; . = 4x;;+ l 2 x Y cos (x^j^)+ 12xj; sin { x ^ ) .
13.29. - j c o s x j ; .
3y
13.30. -
8x
y l
13.31. - 4^*.
1 3 . 3 2 . r + / ; - ^ + ; ^ - o .
13.33. 0.
13.34. 0.
13.35. 0.
13.36. 0.
13.37. 0.
13.38. \An.
13.39. 0.
13.40. 1.
13.41.0.
13.42. Valban megoldsa a differencilegyenletnek.
1 3 . 4 3 . = 4:1.
13.44. - 3 .
25 Matematikai feladatok 385
13.45.
y{x-z)
^{x^+y^ + z ^ - 2 x z f
13.46. 1.
13.47. + 3xyz + 1).
13.48. A z / ; = -
/ ; = -
f ' L - -
/ z z = -
/ ; = -
]j{x^ + y^ + Z^Y '
derivlt fggvnyekbl az
^{x^ +
x^ + / + z^-3x^
^{x^ + y^ + z^Y
x^ + / + z^-3z^
> / ; = -
'^{x^ + y^ + z y
^{x^ + y^ + z ^ f
. ,/. / + f" 4- r ' = _ 3x^ + 3 / + 3 z ^ - 3 x ^ - 3 / - 3 z ^
tehat /j(x + f y y ~^f zz /, 2 I 2_|_ 2\5
f(x^ + y^ + z^)^
13.49. aj /(2; 1) = 1 loklis minimum;
b) / ( I ; 2) = 7 loklis maximum;
= 0.
1 1\ 4
c) / ( - ; - = - loklis minimum;
3 3/ 3
d) /(3; 3) = 1 loklis minimum; loklis maximum;
: i n 7n\ 3 . . .
/ ; = - - loklis mmimum; /
6 6 / 2
13.50. a = b = c = S.
Un ll7r\ 3 . .
---- ; ----- = ---- loklis minimum.
6 6 / 2
13.52. a = b = c = 5.
13.53. a) 3 ( x - 3 ) - 4 0 ; - 2 ) - z + l = 0;
j x = 3 + 3; y = 2- Af , z = \ - t .
386
13.54. a) 6(x- 2) + 2 ( y - 6 ) - z + n = 0;
b) X = 2 + 6; y = 6 + l f , z = 12.
13.55. z =l .
13.56. | JK| 10, 2m^
13.57. IJ e l 4, 5 cm.
\V\
\V\
0,13 = 13%.
13.58. a; b) c) 15; d) 18; ej 9.
13.59. aj 1; b) 10,25-,
c ) \ - .
4
d) I n- .
1 1
/
13.60. a) {x^ + y^)dy
-1
dx =
1 h-
c)
r / r \
l (
xe^ dx
* V /
dy =
3 - e
88
15
d)
b)
1 X
r
(x^ + y)dy
0 y
\
0 0
2
1 3 9 In 3 + I l i n 2- 12
13.61. a j - ; - ; cJ 3; d ) -----------------------
1 3 1 1 , 1
/ J - - s i n 3 + - s i n l ; g j - e ^ - - e ^ + -
1 4 1 4
AJ 5,4; +
2 5 2e 5e
1 2 1 571
yj - - cos 10+ - c o s 4 - -cos 2; k J .
3 3 3 6
r / r \ 1
xy sin (x^ + y^) dx j dy =
ej 27 In 3- 28 In 2 - 2 ;
13.62. aj 7,5; bj 8,93; cj 0,22; dj l n2; e j - - I n 2.
13.63. aj 17,5; bJ - 4 2 - ; c J - ^ + ^ l n 2; dJ^-{\b]2-5f5).
25*
387
16 8 81
13.64. a; 11,25; 13,5; ~ J j ~ T s M
/ ; - 7 2 : ^ ; 0 ^ 3 2 ^ ; Aj - 8, 1; i) j - 2 l n 2 +e* +e.
3
13.65. a ) - .
b) Mivel a vges T tartomny;
r = {(jc;j)gR2 I - 1 ^ jc i 1; x ^ - A ^ y ^ - x ^ - 2 ) ,
gy az integrl t
1 - x ^ - 2
c
(X+2JCOS x) dy dx = [2x2x^+ 12(:v^l)cos x] dx =
- * 2 - 4 _ i
x^
x^ + I2(x^ 3) sin x + 2x cos x
1
= 4 cos 148 sin 1.
-1
Megjegyzs: az (x^- 1) cos x szorzatfggvnyt parcilisn integrltuk, az (x) =
= 1; v'{x) = c o s x ; majd m(x) = x , v ' { x ) = sinx helyettestssel.
13.66. a) 18; b) 9; c) In 3.
13.67. 1170.
3rt
13. 68. .
5 3
13.69. 3 In------- .
2 5
In
13.70.------4.
4
1 l/3 \ 2 71 131^
1 5 7
e) (cos 1- s in 1) + - e + 2 sin 1---- ; .
2e 4 4 '
3 1 1 1 11 n
/ ; - 2 cos 1+ sm 1+ - - - ^ 2+ _ _ _ . g j 2 n - ;
388
i ) ^ ; j ) 181n2- 81n3- - - , k) -e^ + 8 In 2-41n 3 -
8 3 2 6
13.73. f
13.74. j
In y
J
2y
J In (:vj) dx
L
4
e +2
2 4
dy = J [xIn j 4-XIn jc dy = 3 I n 2- .
1 3
13.76.1
13.76. - s h 2- - s h 1.
4 2
13.77. a; 2,25; b) In^ 2.
13.78. -4,25.
71+3 2
13.79. a j -------------,b) - 2 si n4 - 5 cos4+13;
4 n
c) ^ch 1- 7 ; 21nch 1+ - J r 1;
4 2 2 4 4 eh 1
1 1 5 1 / 1\ 43
f ) - Tl n 2 - - I n - - -Arctg ~ - ] ;g) 8 1 n 2 - 4 1 n 3 -
4 8 4 4
1 125
/ij 4 +l n - 2 Arctg2 +2 Arctg( - 1);
18 32
n n 13/3 j/2- 1 n
13.80. a) - l n 2- ; b) - l n 2- ;
^2 16 ^ 2 16
c) H= { ( x ; j ; z ) e R 3 | x ^ 0 , 7 ^ 0 , z ^ 0 , x + : F + z ^ 1 } ; ?; ^ I n 2 - .
z lo
389
1
13.82.
144
13.83. a ) ^ - , b ) \ .
48 4
13.84
13.85. 4.
13.86. 0.
In 8
1 3 . 8 7 . - - - .
13.88. 8.
13.89. 8n.
13. 90. ^.
13.91
13.93. a) (2; 2; 2); b) g, = Qy ^ Q, = m.
13.94. a) (0; 0; 3); b) Q:^ = Qy = Q, = - m ; Q, = m.
/ l \ 32 256 ^ 5i:
13.95. a) 1; 2 ) . = e, = j m ; Qz = * = - ^
512
m.
390
13.96. a) 2- b ) ~- c ) 4.
4
14. fejezet
14.1. a) f(/) = (2 sin + c o s i)\ + 6e^'\ +
2 5 t
b) m = 7 ^ ^ +
k;
c; f(0 =
(l-O"" cos^S/' l + ^
l + 3^
2 |/z+?' V
l + 2\ /2 + 2< - l
i + Arcsin ,+ ^ j ) - [ (eh _
1
14.2. a) f(0 = 20/n + (9sh 3 0 j-
b) f(0 = 9(ln22)2"'i + (16cos40j-
4 [ / ^
k;
2 16/
cJ f(0 = ------- r i + -------- j + 48(2+3)^k.
^ (1 + /)^ (l+4/2)2' ^
14.3.
m-1)'3" 2/I-3)"
r<">(0 = (51n 3)"3^4 + ( ~ + 3^ J + ( ~ D"-- - y ~ d + 0
1 4 8
14.4. t = - - i - - j + -k.
9 9 J 9
2 (1-1
2 k.
14.5.
x+1 y - 5 z - 4
14.6. r(/*) = 4(l + *)i+ - ( 2 + *)j+ (2 + /*)k.
14.7. Egy ilyen pont van, &q = Qparamterrtkhez tartoz P(l; 1; 1) pont.
14.8. Pi(0; 0; 0); ^2 j ; ^3 (44 - ^ ; 2^.
/19 1 2\
14.9. Kt ilyen pont van, a Pi(3; - \ ;2) s &Pi i \ pont.
14.10. Vgtelen sok ilyen pont van, a t = kn, k e Z paramterrtkekhez tartoz
pontok.
14.11. <p= 60,2-
lA. M. a) x = 0 , y =l , z = l + t;
b) X == l + ^ t , y = ^ , z ^ ~ + l + t; (p=51, 5.
14.14. Az rint egyenletrendszere: X = 4 +/,_v = - + , z = 4 + 2; afnorm-
lis egyenletrendszere: x = 4 - 6 , j ^ + 6, z = - 4 ; a binomilis egyenletrendsze
re: X= 4 - , 7 = ^ - , z = - 4 + .
x+1 j 5 z+4
14.15. Az rint egyenletrendszere:------= -------= ------ ; a fonormalis egyenlet-
1 3 2
x + l z+4
rendszere; ^ = 5 >^ = ; a binomilis egyenletrendszere: x + 1 =
= = - ( z + 4).
14.16. A simulsik egyenlete: 3x + 3 j 3z 25 = 0; a normlsk egyenlete:
3x + 3;^ + 6z + 11 = 0; a rektifikl sk egyenlete: 3x 3j 11 = 0.
14.17. a ) x+>- z - 8 = 0; b) X - 37- 2Z + 8 = 0; c) 5x-_y + 4z + 26 = 0.
14.18. G =
14.19. G =
(= 0,0496); r = -
1 2 P 121
3 ^ ^ 1
- (= 0,1568); T = ~ .
169 ^ 3
(= -0,0496).
4
14.20. G = 0 , 1 3 6 8 ) ; 7 = - ( = 0 , 1 7 3 9 ) .
392
14. 21. G = ^ ; r = ^ ; igen.
16/5
14.22. G = 1,4311); r = 0 .
14.23.G = 1 ;
14.25. f elsfok, ir lland,'f =0, ezrt r = 0 , a grbk skgrbk.
14.25. r elsfok, f lland, r = 0 , ezrt T=0, a grbk skgrbk.
14.26. A grblet akkor egyenl a torzival, ha a = >.
1/2 1
14.27. G = (= 0,471); T = - ; az rint egyenlete:
r(t) = (1 + ?) + j + l ^ normlsk egyenlete: x + y + z = 2.
14.28. a) e{t) = ( 8 + 100i + ( 8+120j - ( l - ?) k; b) 1 0 x + 1 2 j - z - 177 = 0;
c) G =
'2484 1
(= 0,013); /; r = ------(= -0,0145).
14.29. Az rint egyenletrendszere: x = 2t, y = 3 +1, z = 3f, a. normlis egyenlet
rendszere: x = St, y = 3\\t, z = 9t; a binomilis egyenletrendszere: x=3t,
y = 3 3t , z =t ;a simulsik egyenlete: 3x 3y z +9 = 0,a normlsk egyenlete:
2x + y + 3z 3 = 0; a rektifikl sk egyenlete: 8 x + 11>9z 33 = 0; a grblet:
G =
19 3
(= 0,1664); a torzi: 7 = ------(= -0,1579).
2744' 19
14.30. Az rint, a fnormlis s a binormlis vektoregyenlete rendre:
r() = (1 + )i + (1 - Oj + k, r(0 = (1 + 3t){ + (1 + 3)j, r() = (1 - )i + (1 + Oj + 2?k;
a simul-, a norml- s a rektifikl sk egyenlete rendre: y x+2z = 0,
1/2 1
x - y + z = 0, x + y - 2 = 0\ ^ = 7 = - - .
14.31. 5 = 8 egysg.
14.32. 5 = 129 egysg.
393
14.33. = 47tl/5 (= 28,1) egysg.
14.34. 5 = 1) ^ (= 33,06) egysg.
14.35. 5 = Stt(= 25,13) egysg.
14.36. = ^l n9 + 48 (= 48,55) egysg.
14.37. a) s= 100 egysg; b) s = c egysg.
14.38. r(M,t?) = (2 + 2-i;)i + (3 + 2M)j + (5-lM-3i;)k.
14.39. r(w, v) = (2 cos )i + (2 sin w)j + k.
14.40. r(, u) = (2 eh M+ 4i;)i + (2 sh u + 2v)\ + 5t;k.
14.41. r ( u , v ) = (wt) 2 + 2)i + (2i; (/m+ i;+l)j + (3 3i;)k.
14.42. r(u, v) = [{a+b cos u ) cos r]i + [{a + b cos u) sin rU + {b sin )k.
14.43. Ha u-i vlasztjuk paramternek, akkor r() = (M+ 2^)i-M^j + 4M^k,
ha v-t, akkor r(t)) = ( ^ + 2d) - rj + 4k.
14.44. r(0 = (2 + 2^'3)i + 3j + (2l^-20k.
14.45. 4jc+17;;+13z + 20 = 0.
14.46. 3jc-32j +128z+16 = 0.
14.47. 3|^x-3j; + 4z-47r = 0.
14.48. rintsk e pontban nincs.
14.49. grad u = 15i-3j.
14.50. a) grad u = 6xyH + (9x^y^ 2z^)} 4yzk.
chx \ , 2s hx \
b) grad M= I 2e^"zcosy+ ------ i - e^^zsinv------------ rij +
V 2y + z J \ (2y + z y j
394
c) grad u = (2x(j^ + 2^)i + 2j ( z " - x 2) j - 2z(x2+ 3;^)k).
2
14.51. a) grad m= i+ - j + 2k; Aj grad u = 6i + 3j-k;
c) grad u = (2i + 2j + k).
14.52. aj gradw = - 2 i + 3j-k; ftjgradw = d + (7t +l ) j - k(= 2,72i + 4,14j-k);
ej grad M= (3i-4j + k).
26
i r r ..................................|r|2
14.54. grad m(0) = 3i-2j-6k; Igrad m(0)| = 7;
grad u{P) = 7i; Igrad (/*)! = 7; grad m= 0 az A( 2; 1; 1) pontban.
14.55. Derkszget.
14.56. aj div y = 3; b) div y = 2; c) div v = ye"' + ze^^ + xe'^.
14.57. a) div \ = 6; b) div v = ; c) div v = 26.
6
14.58. j div V= ; b ) divv = 4 |r|; c) div v = 6 |r|^.
|r|
14.59. Igen, mert div v = 0.
14.60. Nem, mert div v = 3 # 0.
14.61. a) rt V = 2(y x ) l + 2(z x)j + 2(:v>^)k;
b)rot V= j + k; c) rt v = (ye'i + ze^^j + xe^^k).
2 1
14.62. a) rt V= 4j + 2k; rt v = -i +3jH k', c) rt v = 4i + 4j + 6k,
9 4
14.63. a) rt \ = 0', b) rt v = 0; cJ = rot v = 0.
14.64. Igen, mert rt v = 0.
395
14.65. Igen, mert rt v = 0.
14.66. div V= 3; rt V= 2(z- >)i- 2xj- 7k.
14.67. div V= 6xyz; rt v = +
14.68. div V= rt v =
y z,
1 1 \ / I 1
i _ } \
y )
14.69. div V= Ixyz (cos x^yz + cos xy'^z + cos xyz^),
rt V= x(z^cos xyz^ _v^cos xy^z)\ + ^.^^cos x^yz z^cos xyz^)\ +
+ z(y^cos xy'^z x^cos x^yz)k.
1
14.70. div V= 2; rt v = - i.
14.71. div V= ; rt v = 0.
14
14.72. divv = -
^16 \
+6
, . , ( =- 7 , 6 2 ) ;
( ) A \
rotv = 2 i - 4 j - 8+ - k ( = 2i- 4j- 9,27k).
V n)
14.73. rt rt V= - ( y^ + z^)eH - (x^ + z^)e"^i - (x^ + y^)e"^k.
14.74. rt V= - i - j + 3k; rt rt v = 3i + k; div rt v = 0.
14.75. div grad u = 14.
14.76. grad div v = 5i+j + 2k.
14.77. div grad = 6; rt grad = 0.
14. 78. 4.
1 1
14.79. a; 0; b) O; cj 0; dj 0; e j - n 2; f j - - l n 2 ; g) 0.
14.80. 9.
14.81. : ^ ( = 0,4095).
396
14.82. 17.
14.83. - ^ ( = 11,83).
6
14.84. Mivel a vektorfggvnynek van potencilfggvnye, ezrt a zrt grbe
mentn szmtott integrl rtke 0.
143
14.85. - (= -35,75).
4
14.86. ^ - i n 4 ( = -1,11).
14.87. In.
14.88. 12 + l n2 - l n 3 (= 11,6).
14.89. 16( e' - l n2) - e( = 1,4218-108).
-|Q(0; -1; 1)
14.90. / = xy + xz->ryz = 0.
F(0; 1; 1)
n i f i 1\
1 4 . 9 2 . - ( ^ Y-
14.93. 2(|/3-l/2) - ^ +l n^ + -0,2392).
14.94. v(r) = In j.- i + (^xf ~^+ j ; 3 egysg.
14.95. V = o + z)2- ^ i + (2x(y + z ) - j + (^2x0 + z) + 7=12.
14.96. V= -
, 2
i + - ' f ^ + 1 = 0 .
X 2 ' " 2
397
14.97. V = I - vsi nxvH----- ) i + (zcos jz- x sin :Cj)j+
cos^xz/
+ \ y cosyz-\------r ) k; 7=0.
\ cos^xz/
14.98. a) Van. u = x^y xz^ + yz^ xyz + K.
b) Van. u = Arccos +K.
xy
14.99. a) Igen, u = x^y\ nz + e^^ + z^ + K\ b) igen,
u = y ^ - ^ x][y + {x+y)fz + K\ c) nem.
14.100./? = 8; u = 3x^ yz + 4xy^z^ + K.
Nemzeti Tanknyvkiad Rt.
A kiadsrt felel: dr. brahm Istvn vezrigazgat
Raktri szm: 42320
Felels fszerkeszt: Palojtay Mria
Felels szerkeszt: Olh Judit
Utnnyomsra elksztette: Zsadonn dr. Szilasi Mria
Mszaki szerkeszt: Grg Istvnn
Terjedelem: 35,75 (A/5) v
tdik kiads, 1996
Nyomtatta s kttte a Dabas-Jegyzet Kft.
Felels vezet: Marosi Gyrgy gyvezet igazgat
Munkaszm: 96-0297
8 7 3 , - F t + F A
42320
RSES
9 " 7 8 9 6 3 1 ' 8 7 4 2 4 2

You might also like